Printable Version of Topic
Click here to view this topic in its original format
თბილისის ფორუმი > ჯანმრთელობა და მედიცინა > რა არის ეს?


Posted by: LUKA-BRAZI 16 Jul 2009, 22:28
ესეც ასე!

"რა არის ეს?" თემამ უკვე მეორე ნაწილიც შეავსო 100 გვერდამდე და დროა მესამე ნაწილიც გაიხსნას. თუ ვინმემ არ იცის ამ თემის დანიშნულება, ვეტყვი, რომ ესაა თემა სადაც იდება კლინიკური შემთხვევები (ქეისები, როგორც ხშირად ვუწოდებთ ხოლმე smile.gif) და ხდება განხილვა, გამოცნობა, დიაგნოზის დასმა.

იდეის ავტორია Guardian, რომელმაც 2007 წლის 14 მარტს გახსნა პირველი თემა. მადლობა გარდიანს მშვენიერი იდეისათვის და საინტერესო ქეისებისათვის. მართალია ცოტა გვაწვალებდა ხოლმე გამოცნობაზე, მაგრამ givi.gif

თემის პირველი ნაწილი: http://forum.ge/?f=43&showtopic=33720157&st=0
თემის მეორე ნაწილი: http://forum.ge/?f=43&showtopic=33902549&st=0

It is wrong for a man to say that he is certain of the objective truth of any proposition unless he can produce evidence which logically justifies that certainty.
- Thomas H. Huxley

smile.gif

ამხელა შესავალი იმიტომ გავუკეთე, რომ Guardian გარკვეული მიზეზების გამო, გაურკვეველი დროით, აღარ დაპოსტავს ამიერიდან ფორუმზე და მინდოდა მისი დამსახურება აღმენიშნა. smile.gif რადგანაც ბევრისთვის ამ თემას როგორც გასართობი, ისე სასწავლო დანიშნულებაც ქონდა smile.gif

ჰო, წინა თემის აქტიური იუზერები:
Blind_Torture_Kill
LUKA-BRAZI
vano_t
basa-ttt
Cousteau
Guardian
Thandrus
mtvareuli
donvaso
zviadcardio
mika9
Tornike Alashvili
Solveig
Romina
LULA_QABABI
irakli222
anarxisti
Freedoctor
SOULFUL
DrJohnes
roo
lgogokhia
Cor-toni
DICOM


შემოდით ხალხო და დადეთ ქეისები, თორემ მინელდა რაღაც აქტიურობა smile.gif სხვა თუ არაფერი, პირველი იქნებით ვინც მესამე ნაწილში დაპოსტავთ ახალ ქეისს smile.gif

Posted by: Blind_Torture_Kill 16 Jul 2009, 23:52
LUKA-BRAZI
ჯიგრული შესავალი გამოვიდა smile.gif




* * *
28 წლის ქალბატონი უჩივის ძლიერ ტკივილს მუხლის სახსრებში და მარჯვენა მხრის სახსარში. ტკივილების დაეწყებამდე ერთი თვით ადრე მას ქონდა დიარეა რომელიც გაუგრძელდა სამი დღე. რამოდენიმე დღის შემდეგ განუვითარდა გამონადენი საშოდან და წვის შეგრძნება შარდვისას.
ფიზიკური გამოკვლევისას : სახსრები შესიებული და შეწითლებული, აღინიშნება პერიანალური მიდამოს და საშოს შესავალის ერითემა + საშოდან სქელი მოთეთრო ფერის გამონადენით, ასევე მცირე ზომის მომრგვალო ფორმის წყლულები.
T-39c, Hb-11.3g/dl, ლეიკოციტები-7.600/მმ3, ESR-120mm
პაციენტი სქესობრივი ცხოვრებით ცხოვრობს და ყავს ერთი უცვლელი პარტნიორი.

აჰა პირველი ქეისიც გაჩუქეთ

Posted by: Cousteau 17 Jul 2009, 14:26
QUOTE (Blind_Torture_Kill @ 16 Jul 2009, 23:52 )
და ყავს ერთი უცვლელი პარტნიორი.

მაგ ერთ უცლვლელ პარტრნიორს რამდენი ცვალებადი პარტნიორი ყავს კიდევ?


რიევიჰ შეიძლება რეიტერის სინდრომია (a.k.a. Reactive Arthritis) ქლამიდიისგან, ან გონორეააქ (ცუდ ართრიტს იწვევს medical emergency მაგრამ პოლიართრიტია თუ არა არ მახსოვს, მუხლის სახსარს რო აზიანებს ნაღდად მახსოვს) შეიძლება ორივეში ვცდები ეხლა 20 წუთში RNA ვირუსები მაქ გასამეორებელი და ჯობია იმას მივხედო : ))

თუ ჩამეთვალა აგერ თქვენ ადვილი თვითშეფასების ასამაღლებლად : )
user posted image

Posted by: Blind_Torture_Kill 17 Jul 2009, 14:36
QUOTE
რიევიჰ შეიძლება რეიტერის სინდრომია


იეს

Posted by: Cousteau 17 Jul 2009, 14:38
QUOTE (Blind_Torture_Kill @ 17 Jul 2009, 14:36 )
QUOTE
რიევიჰ შეიძლება რეიტერის სინდრომია


იეს

ქუუულ : ))))
წავედი მე
:წიგნი: :წიგნი: :წიგნი:

Posted by: Thandrus 17 Jul 2009, 16:51
მომილოცავს ახალი თემა!

Blind_Torture_Kill

წინა თემის ბოლოში რომ გულის ჰისტოლოგიური სურათი დადე, იმ ქეისის პასუხი რა იყო?

user.gif

Posted by: Blind_Torture_Kill 17 Jul 2009, 19:01
QUOTE
წინა თემის ბოლოში რომ გულის ჰისტოლოგიური სურათი დადე, იმ ქეისის პასუხი რა იყო?


contraction bands smile.gif

Posted by: Solveig 17 Jul 2009, 21:10
Cousteau
ბუსუსუჯრედოვანი ლეიკემიაა ეგ.

(HCL hairy cell leukemia)

Posted by: Cousteau 17 Jul 2009, 22:56
QUOTE (Solveig @ 17 Jul 2009, 21:10 )
Cousteau
ბუსუსუჯრედოვანი ლეიკემიაა ეგ.

(HCL hairy cell leukemia)

yes.gif yes.gif yes.gif

Posted by: basa-ttt 17 Jul 2009, 23:18
LUKA-BRAZI
სიამოვნებით ვკიხულობდი და ვმონაწილეობდი ამ თემაში -
მაგრამ ზოგიერთი აქ დაწერილ პოსტებს
მერე სხვა მიზნებისთვის იყენებს ხოლმე და ერთმანეთს ქულებს უთვლის -
ვინ რა იცის და რა არა ანუ პირადი შურისძიებისათვის იყენებს.
არაეთიკურია ეს -
მაგრამ მაინც აკეთებს.
ამიტომ ბევრი უკვე იკავებს თავს ამ თემაში მონაწილეობაში მიღებისგან.

Posted by: Cousteau 18 Jul 2009, 00:07
QUOTE (basa-ttt @ 17 Jul 2009, 23:18 )

ამიტომ ბევრი უკვე იკავებს თავს ამ თემაში მონაწილეობაში მიღებისგან.

yes.gif შეცდომების დაშვება ძალიან ბევრ ხალხს სცხვენია....


user posted image


Posted by: Blind_Torture_Kill 18 Jul 2009, 00:43
Cousteau დღეს ჰემატოლოგიის დღეა ?

Adult T cell leukemia

Posted by: Cousteau 18 Jul 2009, 10:05
QUOTE (Blind_Torture_Kill @ 18 Jul 2009, 00:43 )

Adult T cell leukemia

იეს იეს yes.gif yes.gif yes.gif

QUOTE
Cousteau დღეს ჰემატოლოგიის დღეა ?

ნეა, მიკროსი : )

Posted by: LUKA-BRAZI 19 Jul 2009, 12:59
უყურე ამათ რა დღეში არიან biggrin.gif
ესეც მიკრო სამყაროს "ბლატნოი" ქეისი:

user posted image

user posted image

ვინც გამოიცნობს, საჩუქრად გადაეცემა ეს:
High-Resolution Transmission Electric Microscope Hitachi : H-1500
user posted image

smile.gif

Posted by: Blind_Torture_Kill 19 Jul 2009, 13:03
LUKA-BRAZI

ნეირონებია ?
ბიჯო რამე პატარა კლინიკა დაადე

Posted by: LUKA-BRAZI 19 Jul 2009, 13:22
Blind_Torture_Kill
QUOTE
ნეირონებია ?
ბიჯო რამე პატარა კლინიკა დაადე

ნეირონებია yes.gif მოკლედ უკურნებელი დაავადებაა. იღუპებიან ნეირონები. დანარჩნი იცი: ნეირონი აღარ არის, აღარ არის რეფლექსური რკალი, მეხსიერება etc. ეტიოლოგიას ვრ ვიტყვი, სრაზუ გამოიცნობთ smile.gif

დათო გამოტყდი, რომელი ხარ აქედან? smile.gif

user posted image

Posted by: Blind_Torture_Kill 19 Jul 2009, 13:25
LUKA-BRAZI

ეს სურათი შენ საიდან smile.gif
მე არ მაქ ჯერ

მეორე რიგში მარჯვნიდან მეორე

Creutzfeldt–Jakob disease ?

Posted by: LUKA-BRAZI 19 Jul 2009, 13:27
Blind_Torture_Kill
ჰა ჰააააააააააააა !!!! ეგრეც ვიფიქრე! biggrin.gif ნუ აი მაინც ტვინის 3D გრაფიკა რა givi.gif

http://tsmu.edu/index.php?fullid=285&Id_pg=1&key=full


* * *
QUOTE
Creutzfeldt–Jakob disease ?

არა, აშკარად სხვა "ბლატნოი" ქეისი უნდა მოვიფიქრო.
up.gif

რამ მიგანიშნა პრიონებზე? smile.gif

Posted by: Blind_Torture_Kill 19 Jul 2009, 13:34
LUKA-BRAZI

შენ ეგ დაავადება მაგარი გისწორდება smile.gif
სლაიდზე არი spongiform encephalopathy

ხოდა თან სურათსაც სეივი რო მივეცი პრიონები ეწერა biggrin.gif

Posted by: LUKA-BRAZI 19 Jul 2009, 13:45
Blind_Torture_Kill
QUOTE
შენ ეგ დაავადება მაგარი გისწორდება

biggrin.gif
ჰო, მახოვს ადრე დავდე და ბევრი გაწრუწუნეთ, ბოლოს Thandrus თუ Cousteau-მ გამოიცნო smile.gif

QUOTE
ხოდა თან სურათსაც სეივი რო მივეცი პრიონები ეწერა 

შე უსინდისო givi.gif

Posted by: Thandrus 19 Jul 2009, 14:18
ვააახ, ჯერ ვერცერთ ქეისს ვერ მოვუსწარი რა biggrin.gif

LUKA-BRAZI

პრიონებზე მეც ვაფრენ, შენი დადებული სურათი რომ დავინახე ახლა, გული ამიჩქარდა biggrin.gif

QUOTE
ჰო, მახოვს ადრე დავდე და ბევრი გაწრუწუნეთ, ბოლოს Thandrus თუ Cousteau-მ გამოიცნო


რავიცი, ჩემი ამ თემაში პოსტვის დროს ეგეთი ქეისი არ მახსოვს, ასე რომ ალბათ კუსტომ გამოიცნო biggrin.gif

* * *
აბა, ჯამაათნო, ეს მითხარით რა დაავადებაა smile.gif

http://up.jeje.ge//download.php?id=B09E5BE433

ვიდეოა, გადმოწერეთ და ნახეთ.

Posted by: LUKA-BRAZI 19 Jul 2009, 18:40
Thandrus
რამე ნევროლოგიურია?

პრიონებზე რატო აფრენ კაცო? smile.gif

ისე კი, მართლაც ძალიან საინტერესო პროტეინები არიან. მეც მიყვარს ეგეთი მისტიური ელემენტები smile.gif

Posted by: Thandrus 20 Jul 2009, 01:05
LUKA-BRAZI

QUOTE
რამე ნევროლოგიურია?


კი, ნევროლოგიურია smile.gif

QUOTE
პრიონებზე რატო აფრენ კაცო?


იმიტომ, ყველასაგან განსხვავებული პათოგენეზი ახასიათებთ...

Posted by: basa-ttt 20 Jul 2009, 15:24
ეს თემა რისთვის არსებობს?
რომ მერე ერთმანეთს შეურაცხყოფები აყენონ
და თითით უშვირონ ერთმანეთზე?

ვისაც ეხება -
კარგად გაიგებს.

Posted by: LUKA-BRAZI 20 Jul 2009, 17:35
basa-ttt
მისმინე, შენ წინა გვერდზეც დაწერე ეგეთი პოსტი და ვერ ვხვდები რატომ .... ამ თემის II ნაწილი კაი ხანი 5 გვერდზე იყო, მერე ამოვწიეთ და ახლა მესამე ნაწილია. ჰოდა ვერ ვხვდები რატომ იწყებ ისევ თავიდან, ამ ამ ნაწილში ყოველშემთხვევაში, არავის არავისთვის შეურაცხყოფა არ მიუყენები. თუ თვლი რომ ვინმე არასწორად იქცევა, ნუ მიაქცევ ყურადღებას, მიაქციოს მოდერატორმა, მაგრამ ჰო გეუბნები, ვერც კი ვხვდები რამ გამოიწვია შენი გეთი პოსტები. თუ გაქვს რამე საინტერესო ქეისი დადე, თუ არამაშინ სხვის დადებულზე ვცადოთ გამოცნობა. აი რისთვის დაპოსტე ეხლა ეგეთი რაღაცეები? რა საჭირო იყო? ხომ ხვდები რომ იძაბება სიტუაცია მასეთი პოსტებით.

Thandrus
ჰოო? მაშინ ცოტა გამიჭირდება. ხო იცი თსსუ-ში კერძო ნევროლოგია 10 დღე ისწავლება :| საინტერესო რამე ჩანს, ერთი მხარეს თითბი გაშლილი აქვს, მეორე მხარეს მოხრილი, მუშტად შეკრული. საწყალი ადამიანი. ჯერ არც კი ვიცი იდეაში რა უნდა იყოს.

Posted by: Romina 20 Jul 2009, 19:13
Thandrus
LUKA-BRAZI

პრიონულ დაავადებსასთან დაკავშირებით მაქვს ერთი შეკითხვა. კროიცფელტ-ჯაკობის დაავადებაა თუ კროიცფელტ-იაკობის? როგორ არის სწორი?
smile.gif

Posted by: Solveig 20 Jul 2009, 20:29
QUOTE
ბევრი უკვე იკავებს თავს ამ თემაში მონაწილეობაში მიღებისგან.

QUOTE
შეცდომების დაშვება ძალიან ბევრ ხალხს სცხვენია....

ვითომ რატომ..თემა სასწავლო-შემეცნებითი მიზნით გაიხსნა და რა არის სამარცხვინო.

სამარცხვინო ისაა, უცოდინრობის გამო გამოცდაზე რომ ჩავარდები.

მერე კიდევ-დავიჯერო, ეგ იყო ამ თემაში აქტიურად მწერლების გაქრობის მიზეზი?
მეეჭვება.
Romina
ჩემი აზრით, იაკობი უნდა იყოს, იმიტომ, რომ ადამიანი, რომელმაც ეგ დაავადება აღწერა, გერმანელი იყო და გერმანულში, მოგეხსენებათ, "ჯ" არაა.

Posted by: Romina 20 Jul 2009, 20:36
Solveig
დიდი მადლობა smile.gif

Posted by: basa-ttt 20 Jul 2009, 21:46
LUKA-BRAZI
წაიკითხა უკვე იმან, ვისაც ეხებოდა ეს
ძალიან კარგად მიხვდა რატომაც დავწერე.

აგერ თქვენ კეისი

user posted image

user posted image

წარსულში მეტად გავრცელებული დაავადება იყო
ახლა მხოლოდ აფრიკის და აზიის ზოგ ქვეყნებშია.

Posted by: mika9 20 Jul 2009, 21:47
basa-ttt

ალბათ ბასა ჰომეოპათი რომ ხარ არასერიოზულად გიყურებენ, იმიჯის საქმეა, და განწყობის . . .
* * *
ფავუსი

სოკო

...............................................

Posted by: LUKA-BRAZI 20 Jul 2009, 22:24
basa-ttt
ჰოდა ამოიწურა smile.gif კარგია ქომ ქეისი დადე, ოღონ ჯერ Thandrus-ის ქეისია გამოსაცნობი smile.gif კაი, ორივე ერთად გამოვიცნოთ.... უი, მიკას უკვე აქვს სწორი პასუხი smile.gif

Posted by: basa-ttt 20 Jul 2009, 22:26
QUOTE
ფავუსი

სწორია.

სოკო Trichophyton

QUOTE
ალბათ ბასა ჰომეოპათი რომ ხარ არასერიოზულად გიყურებენ

რომ იცოდნენ ჰომეოპათია რაა,
ტაშს დაგვიკრავდნენ.

* * *
QUOTE
რავიცი, ჩემი ამ თემაში პოსტვის დროს ეგეთი ქეისი არ მახსოვს, ასე რომ ალბათ კუსტომ გამოიცნო

კურუ მახსოვს მე გამოვიცანი...
gigi.gif

Posted by: LUKA-BRAZI 21 Jul 2009, 00:34
basa-ttt
QUOTE
კურუ მახსოვს მე გამოვიცანი...

up.gif



smile.gif

Posted by: Cousteau 21 Jul 2009, 01:49
user posted image
+
user posted image


და
user posted image
+
user posted image

ადვილი ქეისი (ნა სმეკალკუ):

დაწერეთ დიაგნოზი რომელიც ამ ადამიანს რამოდენიმე წლის წინ დაესვა (ვთქვათ 10 წლის წინ)

Posted by: Blind_Torture_Kill 21 Jul 2009, 02:12
Cousteau

QUOTE
დაწერეთ დიაგნოზი რომელიც ამ ადამიანს რამოდენიმე წლის წინ დაესვა (ვთქვათ 10 წლის წინ)


HIV

Posted by: Cousteau 21 Jul 2009, 02:16
QUOTE (Blind_Torture_Kill @ 21 Jul 2009, 02:12 )
Cousteau

QUOTE
დაწერეთ დიაგნოზი რომელიც ამ ადამიანს რამოდენიმე წლის წინ დაესვა (ვთქვათ 10 წლის წინ)


HIV

: )
yes.gif


ბარემ თქვით სლაიდებზე რა არის

Posted by: Blind_Torture_Kill 21 Jul 2009, 02:21
პნევმოცისტის კარინი
და კაპოში სარკომა

Posted by: Professor Xachikian 21 Jul 2009, 05:45
42 წლის ჯანმრთელი მამაკაცი, მძიმე ჰიპოგლიკემიური ეპიზოდებით. დაეწყო რამდენომე თვის წინ. ჰიპოგლიკემიური შეტევები ხდება როგორც უზმოზე, ასევე ღამით და საკვების მიღების შემგეგ. ანამნეზში დიაბეტი არ აქვს, და არანაირ პრეპარატს არ იღებს. ტოქსიკოლოგიური სკრინინგი ანტიდიაბეტურ პრეპარატებზე (1-ლი და მე-2 თაობის სულფონილურეა) უარყოფითია. ინსულინს არ იკეთებს - სისხლში ერთდროულად გაზომილი C-პეპტიდის და ინსულინის დონე ერთდროულად დაბალია: გამოირიცხა ეგზოგენური და ენდოგენური ჰიპერინსულინემია. სიმსივნური დაავადებები, ღვიძლის, თირკმლის პრობლემები და თირკმელზედა ჯირკვლის უკმარისობა გამოირიცხა. ალკოჰოლს არ იღებს.
პაციენტს დაემართა ასთმის მწვავე შეტევა, დაენიშნა პრედნიზონის მოკლე კურსი. მკურნალობის ამ პეროოდში ჰიპოგლიკემია არ აღნიშნულა. ამის შემდეგ ისევ გაუმეორდა ჰიპოგლიკემიური შეტევები, რამაც სპონტანურად გაიარა 2-3 თვის შემგედ.

დიაგნიზი:

Posted by: mika9 21 Jul 2009, 11:17
მოიცა და ასთმა როდის დაემართა, ???ჰიპოგლიკემიური ეპიზოდების ფონზე თუ როგორ??

Posted by: LUKA-BRAZI 21 Jul 2009, 11:19
Professor Xachikian
საინტერესოა. მაგრამ ვერ გავიგ ცოტა, ინსულინის დონე და გლუკოზის დონე როგორი აქვს სისხლში? შეტევის დროსაც გაზომილი ეს მონაცემები საინტერესო იქნებოდა. ანუ როგორც გავიგე ამ კაცს არც ჰიპერინსულინიზმი აქვს და არც ჰიპო?

მაშინ ვთქვათ, აქვს მალაბსორბციის სინდრომი, ნახშირწ###ბს ვერ იწოვს გასტროენტერალური ტრაქტი, ამას უშველა პრედინიზოლონმა, რადგან ჰორმონოთერაპიის დროს კარგად იყო და შეტევები აღარ ქონდა....

მეორე ვარიანტია ქსოვილოვანი ინსულინრეზისტენტობა, სისხლში არის გლუკოზა, მაგრამ ქსოვილბში ვერ შედის, შედეგად ჰიპოგლიკემიის კლინიკური სურათი; ან კიდევ ტოლერანტობის დარღვევა.... რეავიცი smile.gif მაინც სისხლის ანალიზები მაინტერესებს, უზმოზზე გლუკოზა და ინსულინი, შემდეგ გლუკოზით დატვირთვის ტესტი.

Posted by: mika9 21 Jul 2009, 12:54
LUKA-BRAZI

შენ სად ბანაობ smile.gifsmile.gifsmile.gif .......ეს ბოლო აბზაცი წაიკითხე რა დაწერე და დაფიქრდი

თუ ასთმა ქონდა და მერე პრედნიზონის მკურნალობის შენდეგ დაეწყო ეს ყველაფერი მაშინ ყველაფერი ნათელია მერადი მერადი ჰყპოკორტიკოლიზმი ტუ რაც ქვია ქართულად ...

ხოდა თუ ასთმა მერე დამერთა მაშინ უნდა დავპიქრდე............................

Posted by: LUKA-BRAZI 21 Jul 2009, 13:50
ჰოდა ცოტა ვერ გავიგე რა უწერია.
მაგას უწერია რომ ჰიპოგლიკემირი შეტევები ქონდა, ასთმა მერე დაემართა და უმკურნალეს პრედნიზოლონით, ამ პერიოდში ჰიპოგლიკემიური შეტევები არ ქონიაო. მეორადი ჰიპოკორტიზლიზმს კი ახასიათებს ჰიპოგლიკემია yes.gif

Posted by: Professor Xachikian 21 Jul 2009, 16:41
mika9
LUKA-BRAZI
mika9
LUKA-BRAZI
ასთმის შეტევა არ არის კავშირში ჰიპოგლიკემიასთან. ასთმა თავისით დაემართა, და უმკურნალეს პრედნიზონით. პრედნიზონით მკურნალობის დროს არც ერთი ჰიპოგლიკემიური ეპიზოდი არ ქონია.
QUOTE
მეორადი ჰიპოკორტიზლიზმს კი ახასიათებს ჰიპოგლიკემია 

თირკლემზედა ჯირკვლის უკმარისობა (პირველადი და მეორადი ჰიპოკორტიზოლიზმი), და სხვა დაავადებები გამოირიცხა (პირველი პოსტი)
LUKA-BRAZI
ჰიპოგლიკემიური შეტევები სპონტანურად ემართება, დროის ნებისმიერ მონაკვეთში და არა მარტო უზმოზე ან ძილის დროს (პირველი პოსტი). ჰიპოგლიკემიის დროს გლუკოზა არის 30 მგ/დლ ფარგლებში. ამ დროს ერთდროულად გაზომილი C-peptide და ინსულინი არის დაბალი, ანუ ჰიპერინსულინემია ეგზოგენური (ჩუმად ინსულინს იკეთებს) და ენდოგენური (ინსულინომა) გამოირიცხა
სხვა ყველაფერი ნორმაში აქვს. კახექსიური არ არის, და ქრონიკული ან მწვავე მალაბსორფცია არა აქვს.

დააკვიდით: ჰიპოგლიკემიურმა დაავადებამ რამდენიმე თვეში თავისით გაიარა (პირველი პოსტი)


Posted by: basa-ttt 21 Jul 2009, 17:17
QUOTE
ამ დროს ერთდროულად გაზომილი C-peptide და ინსულინი არის დაბალი

ეს საინტერესო დეტალია
ანუ ენდოგენური ინსულინი დაბალია
და თან ჰიპოგლიკემია აქვს?

ცენტრალური რეგულაციის მოშლის გამო ხომ არაა?
ასთმამ მეც დამაბნია.
თან პრედნიზოლონის ფონზე...

Posted by: LUKA-BRAZI 21 Jul 2009, 17:32
Professor Xachikian
ვაჰ smile.gif

მაშინ იდიოპათური ჰიპოგლიკემიური კრიზები givi.gif

ვიფიქრებ კიდე.....

Posted by: basa-ttt 21 Jul 2009, 17:56
QUOTE
იდიოპათური ჰიპოგლიკემიური კრიზები

ალბათ.

ვიფიქრებდი რომ შიმშილობდა რაღაცის გამო
ან გლუკოზით ღარიბ საკვებს ღებულობდა.

ტკბილეული არ უყვარს ეტყობა...
gigi.gif


Posted by: mika9 21 Jul 2009, 18:02
basa-ttt

ინსულინომა, რავი აბა ,

Posted by: basa-ttt 21 Jul 2009, 18:25
QUOTE
ინსულინომა

ინსულინომას დროს C პეპტიდი მომატებულია,
ანუ ენდოგენური ინსულინი მაღალია და არა დაბალი

საინტერესო საიტს მივაგენი
http://metaboly.ru/INDUTSIROVANNAYA-GIPOGLIKEMIYA1.html
აქ საუბრობენ ლეიცინის როლზე ჰიპოგლიკემიაში
თუმცა არ წერია სხვა დეტალები
врожденными дефектами метаболизма или необъяснимой чувствительностью, которые придают определенным ингредиентам диеты способность вызывать гипогликемию (например, врожденное нарушение толерантности к фруктозе, гипогликемия, индуцируемая лейцином).
ანუ ამ ტიპის ჰიპოგლიკემია ვითარდებაო ლეიცინით მდიდარი საკვების მიღების შემდეგო,
და ხშირად თავისით გაივლისო....
ზოგჯერ სტეროიდები ეფექტურიაო.
http://metaboly.ru/CHUVSTVITELNOST-K-LEYTSINU1.html

კიდევ ერთი შესაძლებელი მიზეზი შეიძლება იყოს გლუკაგონის დეფიციტი-
თუმცა იზოლირებულად მისი დეფიციტი თუ გვხვდება რაიმე დაავადებისას -
არ ვიცი ნამდვილად.

Posted by: mika9 21 Jul 2009, 21:05
basa-ttt
მაგ კი ვიცი მაგრამ,,,,,

ვინ იცის როგორ უნდა განასხვაო ინსულინომა, სულფონილურეას ზედმეტი მოხმარებისგან ???

Posted by: basa-ttt 21 Jul 2009, 21:22
QUOTE
ვინ იცის როგორ უნდა განასხვაო ინსულინომა, სულფონილურეას ზედმეტი მოხმარებისგან ???

ინსულინომა ზოგჯერ ჩანს კტ და ექოზე.
გარდა ამისა თუ C პეპტიდი ძალზე მომატებულია ინსულინთან შედარებით (თუ არ ვცდები 20% ზე მეტად ინსულინზე) ეს უფრო ინსულინომაზე მიუთითებს.

Posted by: mika9 21 Jul 2009, 22:32
basa-ttt

არა, ანუ ორივეს დროს არის მომატებული ინსულინი და ც პეპტიდი ნუ ეხლა ცტ და ექოს კი აქვს მოგნი სადგაც ნორმალური სენსიტივობა მაგრამ ზაან ზვირიანი მეთოდია და ეგ რომ გაუკეტო კარგად მიგშტრაფავს სადაზგვეო, ისე მარტივი ანალიზია სისიხლსჰი რაგაც უნდა გაზომო......
მოზებნე აბა ინტერნეტში და უკეთესად დაგამახსოვრდება
* * *
იმ ქეისში აშკარად რაგაც ჰიპოკორტიკოლიზმია , რა ქვია კორტიზოლის ნაკლებობას ????

Posted by: LUKA-BRAZI 21 Jul 2009, 23:37
mika9
QUOTE
რა ქვია კორტიზოლის ნაკლებობას

ადისონის დაავადება.

Posted by: basa-ttt 21 Jul 2009, 23:49
QUOTE
ისე მარტივი ანალიზია სისიხლსჰი რაგაც უნდა გაზომო......

ასე ტოქსიკოლოგიური ანალიზი და მისი ჯანი

QUOTE
სიმსივნური დაავადებები, ღვიძლის, თირკმლის პრობლემები და თირკმელზედა ჯირკვლის უკმარისობა გამოირიცხა

...
QUOTE
თირკლემზედა ჯირკვლის უკმარისობა (პირველადი და მეორადი ჰიპოკორტიზოლიზმი), და სხვა დაავადებები გამოირიცხა (პირველი პოსტი)

არაო - ეგ არააო
ეწერა მაგაზე წინა გვერდზე.

Posted by: LUKA-BRAZI 22 Jul 2009, 01:27
ყველას გემშვიდობებით 3 კვირით. გავდივარ ქალაქიდან. არ მინდა, მაგრამ იძულებული ვარ :| თქვენ იცით, თემას ნუ ჩააგდებთ და წესიერად მოეპყარით ერთმანეთს smile.gif

დროებით smile.gif

See U soon dudes biggrin.gif

თუ რამეა - PM smile.gif

Posted by: mika9 22 Jul 2009, 01:48
basa-ttt
პროინსულინი ინსულინომის დროს მაგალია,, და წამლებით ჰიპერინსულინემიისდ როს ნორმალური...

LUKA-BRAZI
ადისონი კი არადა კორტიზონის ნაკლებობას ანუ ჰიპოკორტიკოლიზმი ტუ როგორ უნდა ვთქვა ჰიპოკორტიციზმი თუ როგორ ,,,,,,

Posted by: Professor Xachikian 22 Jul 2009, 04:06
პასუხი:

აუტოიმუნური ჰიპოგლიკემია.
იშვიათი დაავადებაა, ძირითადად ემართებათ აზიური წარმოშობის პაციენტებს.
ამ დროს ხდება დიდი რაოდენობით ინსულინის ანტისხეულების გამომუშავება, უკავშირდება ინსულინს, და ანტიგენ-ანტისხეულის კომპლექსი სისხლში ცირკულირებს. გარკვეულლი პერიოდის შემდეგ ხდება ანტიგენ-ანტისხეულის დისოციაცია და ინსულინის ერთბაშად გამონთავისუფლება, რაც იწვევს ჰიპოგლიკემიას.
მკურნალობისთვის გამოიყენება პრედნიზონი. პლაზმაფერეზი ამ დროს უეფექტოა. შემთხვევაში აღწერილ პაციენტს რომ დაენიშნა პრედნიზონი ასთმის გამო, ამიტომაც მოეხსნა ჰიპოგლიკემიის შეტევები.

საინტერესო დისკუსია გამოვიდა. გმადლობთ.

Posted by: basa-ttt 22 Jul 2009, 10:22
QUOTE
პერიოდის შემდეგ ხდება ანტიგენ-ანტისხეულის დისოციაცია და ინსულინის ერთბაშად გამონთავისუფლება, რაც იწვევს ჰიპოგლიკემიას.

გამარჯობა შენი!
აბა ინსულინი და C პეპტიდი დაბალიაო!
თუ ინსულინს გადმოისვრის და ამის გამოა ჰიპოგლიკემია,
მაშინ ინსულინიც მაღალი უნდა იყოს და C პეპტიდიც
შენ კი გვიწერდი ორივე დაბალიაო.
ეს იყო პარადოქსი სწორედ.
ასეთი სიტუაცია (როცა ინსულინიც დაბალია, C პეპტიდიც და ამაც დროს
ჰიპოგლიკემიაც გაქვს) შესაძლებელია შიმშილის დროს
ან როცა მართლა ცენტრალური რეგულაციაა მოშლილი შაქრის , ან დეფექტი მეტაბოლურ სისტემაში.

mika9
QUOTE
პროინსულინი ინსულინომის დროს მაგალია,, და წამლებით ჰიპერინსულინემიისდ როს ნორმალური...

მერე და მაგი ხომ მეწერა ზემოთ?
20 % ზე მეტადაა მომატებული თქო პროინსულინი ინსულინთან შედარებით

Posted by: mika9 22 Jul 2009, 13:17
basa-ttt

ვერ წავიკითხე სად გიწერია, ც პეპტიდი და პროინსულინი სხვადასხვა რამეა

Posted by: basa-ttt 22 Jul 2009, 14:52
mika9
მართლია - მე ც პეპტიდზე ვწერდი.
მაგრამ აზრობრივად ერთსა დაიმავეს უჩვენებს ორივე
თუ მომატებულია პროინსულინი - ც პეპტიდიც მომატებული იქნება,
ც პეპტიდი პროინსულინის მონაკვეთია, რომლის მოშორების შემდეგ წარმოიქნება ინსულინი.
ამიტომ ინსულინომის დროს ის ასევე მომატებულია.

Posted by: Professor Xachikian 22 Jul 2009, 16:44
basa-ttt
[quote]გამარჯობა შენი!აბა ინსულინი და C პეპტიდი დაბალიაო!
თუ ინსულინს გადმოისვრის და ამის გამოა ჰიპოგლიკემია,
შენ კი გვიწერდი ორივე დაბალიაო.[/quote]
ასეა.
quote]ან როცა მართლა ცენტრალური რეგულაციაა მოშლილი შაქრის [/quote]
რას ნიშვანს? მეტი კონკრეტიკა.
[quote]დეფექტი მეტაბოლურ სისტემაში.[/quote]
რომელი დეფექტი?

რაც შეეხება აღწერილ შემთხვევას.
C-პეპტიდი გამომუშავდება პანკრეასიდან. როდესაც პაციენტს აქვს ჰიპოგლიკემია (არა ინსულინომით, ანუ არა ენდოგენური სეკრეციით გამოწვეული), მაშინ ნორმალური პანკრეასი ამაზე რეაგირებს და კომპენსატორულად ამცირებს ინსულინის (და ამასთან C-პეპტიდის) სეკრეციას. ასე რომ, შიმშილის, ადისონის დაავადების, სიმსივნეების და ა.შ. სხვა სახის ჰიპოგლიკემიების დროს პანკრეასი ამცირებს ინსულინის და C-პეპტიდის სეკრეციას. იგივე ხდება როდესაც პაციენტები ჩუმად იკეთებენ ინსულინს (ასეთ შემთხვევაში C-პეპტიდი არის ძალიან დაბალი, ხოლო ინსულინი - მაღალი)
ანუ: ნორმალური რეაქცია ჰიპოგლიკემიაზე (ეხება არა-ენდოგენურ ჰიპერინსულინემიას) არის ინსულინის გამოყოფის, და შესაბამისად, C-დაქვეითება.

რაც შეეხება ინსულინის დონეს, შეიძლება მომატებული იყოს მხოლოდ იმ შემთხვევაში, თუ ინსულინი გაიზომა მაინც და მაინც ანტიგენ-ანტისხეულის დისოციაციის მომენტში. ამის ალბათობა კი ნაკლებია. ჩვეულებრივი ინსულინის ანალიზი კი ანტიგენ-ანტისხეულის კომპლექსს ერთდროულად ვერ ზომავს (ანუ ვერ "აღიქვამს"), და ამიტომ ჭარბი, მოცირკულირე ინსულინი ანალიზზე არ ჩანს.

Posted by: basa-ttt 22 Jul 2009, 21:12
QUOTE
ასე რომ, შიმშილის, ადისონის დაავადების, სიმსივნეების და ა.შ. სხვა სახის ჰიპოგლიკემიების დროს პანკრეასი ამცირებს ინსულინის და C-პეპტიდის სეკრეციას.

მეც მაგას ვამბობ
მაგრამ დაბალი რომ იყოს ინსულინიც, შაქარიც და ც პეპტიდიც
ეს მხოლოდ შიმშილის დროსაა შესაძლებელი და ისიც თეორიულად.

QUOTE
ჩვეულებრივი ინსულინის ანალიზი კი ანტიგენ-ანტისხეულის კომპლექსს ერთდროულად ვერ ზომავს (ანუ ვერ "აღიქვამს"), და ამიტომ ჭარბი, მოცირკულირე ინსულინი ანალიზზე არ ჩანს.

მაგრამ ეს არ ეხება ც პეპტიდს.
ის არ შედის ამ იმუნურ რეაქციაში და წესით ის ასეთ სიტუაციაში დაბალი კი არა მაღალი უნდა იყოს
თქვენ კი გვიწერდით ც პეპტიდი დაბალიაო...

QUOTE
რომელი დეფექტი?

დავწერე უკვე ზემოთ -
საინტერესო შემთხვევებია აღწერილი
ლეიცინი მოვიყვანე მაგალითად-
როცა ადამიანი ღებულობს ლეიცინით მდიდარ საკვებ
და მისი მეტაბოლიზმია დარღვეული
ვითარდება ჰიპოგლიკემია.

QUOTE
რას ნიშვანს? მეტი კონკრეტიკა.

მეტი კონკრეტიკა -
ალბათ ძნელი სათქმელია,
უბრალოდ გავიფიქრე
რომ შეიძლება არსებობდეს
ადისონის მსგავსი სინდრომები,
როცა ჰიპოფიზი არაადექვატურ ბრძანებას იძლევა

Posted by: mika9 22 Jul 2009, 21:29
basa-ttt
ძაან გასაგებად დაწერა (ექიმმა ქაჩიკიანმა) რა როგორ რატო ეკამათები, უფრო თუ გინდა დეტალებში მაშინ ჰარისონში ძაან დეტალურადაა ახსნილი...

ც პეპტიდი, ინსულინი, პროინსულინი, და ის რომ გაიგო იყო სულფონილურეას ოვერუსე თუ ინსულინომა გაარჩევ მარტო პროინსულინის დონით
ინსულინომის დროს მომატებულია და სულფონილურეაოვერუსი დროს კიდე არაა არაა..........................

Posted by: basa-ttt 22 Jul 2009, 22:02
mika9
მე მგონი არ ვეკამათები
და მეც გასაგებად დავწერე რაც დავწერე.

QUOTE
ინსულინომა გაარჩევ მარტო პროინსულინის დონით
ინსულინომის დროს მომატებულია და სულფონილურეაოვერუსი დროს კიდე არაა

კაი ერთი.....
gigi.gif

ამ სიაშია ჰიპოგლიკემიის გამომწვევი მიზეზები
მათ შორის რამდენიმე მეტაბოლური (ფერმენტოპათიები)

Основные причины гипогликемии натощак

Недостаточная продукция глюкозы
А. Дефицит гормонов
1. Гипопитуитаризм
2. Недостаточность надпочечников
3. Дефицит катехоламинов
4. Дефицит глюкагона

Б. Дефекты ферментов
1. Глюкозо-6-фосфатаза
2. Печеночная фосфорилаза
3. Пируваткарбоксилаза
4. Фосфоэнолпируват карбоксикиназа
5. Фруктозе-1,6-дифосфатаза
6. Гликогенсинтетаза

В. Дефицит субстрата
1. Кетотическая гипогликемия грудных детей
2. Тяжелое нарушение питания, мышечное истощение
3. Поздняя беременность

Г. Приобретенные заболевания печени
1. Застойная печень
2. Тяжелый гепатит
3. Цирроз
4. Уремия (возможно, сложного механизма)
5. Гипотермия

Д. Медикаменты и наркотики
1. Алкоголь
2. Пропранолол
3. Салицилаты

Повышенная утилизация глюкозы
А. Гиперинсулинизм
1. Инсулинома
2. Экзогенный инсулин
3. Сульфонилмочевина
4. Иммунное заболевание с антителами к инсулиновым рецепторам
5. Медикаменты: хинин при malaria falciparum, дизопирамид, пентамидин
6. Эндотоксический шок

Б. Адекватные уровни инсулина
1. Внепанкреатические опухоли
2. Системный дефицит карнитина
3. Недостаточность ферментов, окисляющих жиры
4. Недостаточность 3-гидрокси-З-метилглютарил-КоА-лиазы
5. Кахексия с истощением жировых запасов

Posted by: Blind_Torture_Kill 23 Jul 2009, 00:14
Physical examination reveals jugular venous disten-
tion in the neck of a 50-year-old male, even when he is
sitting up. He also has an enlarged and tender liver that
can be felt 10 cm below the right costal margin. Pitting
edema is observed on his lower extremities. A chest radio-
graph reveals large pleural effusions. Thoracentesis on the
right yields 500 mL of clear fluid with few cells. The most
likely cause of this clinical picture is

A) Tricuspid valve stenosis
B) Acute myocardial infarction
C) Pulmonary valve stenosis
D) Chronic obstructive lung disease
E) Primary pulmonary hypertension

Posted by: mika9 23 Jul 2009, 00:33
ე ალბათ.......................
ამის ეხპლანეისჰენი დადე რა მერე

Posted by: Blind_Torture_Kill 23 Jul 2009, 00:39
mika9

ძაან ნაგავი კითხვაა იმენა ერთი სამი წუთი ვიაზრებდი ამ კითხვას smile.gif
აბა სხვები რას ფიქრობთ ?

Posted by: basa-ttt 23 Jul 2009, 00:40
Blind_Torture_Kill
ღვიძის გადიდება და ქვემო კიდურების შეშუპება მიუთითებს
შეგუბებაზე დიდ წრეში.

Acute myocardial infarction - თუ არის მაშინ გულის
უკმარისობის ფონი უნდა იყოს


QUOTE
Tricuspid valve stenosis

ტრიკუსპიდალურის სტენოზი არ ატარებს სისხლს მარჯვენა წინაგულიდან პარკუჭში
ანუ შეგუბება ვითარდება დიდ წრეში
ეს შეიძლება იყოს

QUOTE
Puhnonary valve stenosis

ამასაც შეუძლია მოგვცეს ეს კლინიკა
ტექსტი ქართულად გადათარგმნე
რაღაც დეტალები ვერ გავიგე

QUOTE
Chronic obstructive lung disease

ეს არ იძლება შეგუბებას დიდ წრეში

QUOTE
Primary pulmonary hypertension

ესეც ნაკლებად -
ამას უფრო ახასიათებს მცირე წრეში ჰიპერტენზიის ნიშნები
და გამოხატული ქოშინი.

Posted by: Blind_Torture_Kill 23 Jul 2009, 00:44
QUOTE
Chronic obstructive lung disease

ეს არ იძლება შეგუბებას დიდ წრეში


რატო არ იძლევა ?
ფილტვის ჰიპერტენზიას იწვევს და მერე გულის უკმარისობა შეიძლება განვითარდეს

QUOTE
Primary pulmonary hypertension

ესეც ნაკლებად - ამას უფრო ახასიათებს მცირე წრეში ჰიპერტენზიის ნიშნებიდა გამოხატული ქოშინი.


ესეც იგივე მექანიზმით მოქმედებს

Posted by: basa-ttt 23 Jul 2009, 00:48
QUOTE
ფილტვის ჰიპერტენზიას იწვევს და მერე გულის უკმარისობა შეიძლება განვითარდეს

QUOTE
ესეც იგივე მექანიზმით მოქმედებს

თეორიულად კი -
მაგრამ ასაკი 50 წელი ცოტა ნაადრევია ასეთი ღრმა გ/უკმარისობისათვის
ტექსტი დადე ერთი ქართულად.

აიერსის სინდრომისათვის 50 წელი ცოტა არა
მარა კლინიკაში წამყვანი უნდა იყოს
ფილტვის პრობლემები -
ანუ მკვეთრად გამოხატული ქოშინი

Posted by: Blind_Torture_Kill 23 Jul 2009, 00:54
ფიზიკურად გამოკვლეული მამაკაცი არის 50 წლის
გადიდებული (გაფართოებული) აქვს საუღლე ვენები (რომელიც წამოდგომით არ ქრება)
ასევე აღენიშნება გადიდებული და მტკივნეული ღვიძლი რომელიც პალპაციით ლოკალიზდება 10სმ ქვემოთ ნეკნთა რკალიდან + ქვედა კიდურების შეშუპება და გულმკერდის რენტგენოგრამით ვლინდება სითხე პლევრაში (მარჯვენა გულმკერდიდან თორაქოსენთეზის შედეგად მიღებული სითხე სუფთაა ერთეული უჯრედებით)

Posted by: basa-ttt 23 Jul 2009, 01:04
QUOTE
გადიდებული (გაფართოებული) აქვს საუღლე ვენები

ჰიპერტენზია დიდ წრეში ( კავა სუპერიორში ?)

QUOTE
გადიდებული და მტკივნეული ღვიძლი რომელიც პალპაციით ლოკალიზდება 10სმ ქვემოთ ნეკნთა რკალიდან

მკვეთრი ჰიპერტენზია დიდ წრეში

QUOTE
ქვედა კიდურების შეშუპება

დიდი წრე

QUOTE
გულმკერდის რენტგენოგრამით ვლინდება სითხე პლევრაში

პლევრაში ექსუდატი შესაძლებელია მცირე წრის შეგუბების დროსაც, მაგრამ
მცირე წრეში რომ იყოს შეგუბება
ფილტვებში იქნებოდა ხიხინი და ექსუდაცია
რენტგენოლოგიურადაც იცვლება ფილტვის ქსოვილი.
* * *
QUOTE
A) Tricuspid valve stenosis

ჩემი აზრით ეს უნდა იყოს.
იზოლირებული ტრიკუსპიდალურის სტენოზი
ცოტა იშვიათობაა მაგრამ შესაძლებელია
სწრაფად ვითარდება ხოლმე დეკომპენსაცია
და ხშირად მისი მიზეზი რევმატიზმია.


Posted by: Professor Xachikian 23 Jul 2009, 04:20
basa-ttt
QUOTE
ეს მხოლოდ შიმშილის დროსაა შესაძლებელი და ისიც თეორიულად.

no.gif არა მხოლოდ შიმშილის დროს. ნებისმიერ დროს, როდესაც პანკრეასი ნორმაშია და შაქარი დაბალია - შიმშილის იქნება თუ სხვა მიზეზი, პანკრეასი წყვეტს(ან ძალიან ამცირებს) ინსულინის, და შესაბამისად, C-პეპტიდის სეკრეციას (დრებით, თავდაცვის მექანიზმით). კიდევ ვიმეორებ, ეს არ ეხება ინსულინომებს და სულფონილურეით მოწამვლას. ჰიპოგლიკემიის დროს პანკრეასმა რომ გააგრძელოს ინსულილის გამოყოფა (რაც ხდება ინსულინომების დროს), კარგად მოგეხსენებათ ამას რა შედეგი მოყვება. ამიტომ, როცა პანკრეასი წყვეტს (ან ამცირებს) ინსულინის გამოყოფას, მასთან ერთად C-პეპტიდიც ძალიან ქვეითდება, რაც აისახება ანალიზზე.
QUOTE
დავწერე უკვე ზემოთ -
საინტერესო შემთხვევებია აღწერილი
ლეიცინი მოვიყვანე მაგალითად-

გმადლობთ კონკრეტიკისათვის. ეს კიდევ ერთი მაგალითია პრეპარატით/ნივთიერებით გამოწვეული ჰიპოგლიკემიის.
QUOTE
ადისონის მსგავსი სინდრომები,
როცა ჰიპოფიზი არაადექვატურ ბრძანებას იძლევა

კიდევ ერთხელ მადლობა. ადისონის დაავადება თირკლემზედა ჯირკვლის პირველადი უკმარისობაა, ხოლო როცა
QUOTE
ჰიპოფიზი არაადექვატურ ბრძანებას იძლევა
, ანუ როცა პაციენტს ჰიპოპიტუიტარიზმი აქვს (ყველა ფუნქცია ან ფუნქციების ნაწილია დაქვეითებული), მაშინ ვითარდება თირკმელზედა კირკვლის მეორადი უკმარისობა. ეს დაავადებები ასე კლასიფიცირდება, და ეს ვიგულისხმე მეტ კონკრეტულობაში.
QUOTE
ამ სიაშია ჰიპოგლიკემიის გამომწვევი მიზეზები
მათ შორის რამდენიმე მეტაბოლური (ფერმენტოპათიები)

საქმეც მაგაშია - ჰიპოგლიკემიის გამომწვევი მიზეზების ძალიან კონკრეტული და დეტალური სია არსებობს,და აუხსნელი "რამე მეტაბოლური დაავადება" გამორიცხულია. ამიტომაცაა ჰიპოგლიკემიის დიფერენციალური დიაგნოსტიკა ადვილი (რაც შეეხება ფერმენტოპათიებს, ეს პრობლემები ძირითადად ბანშვობის ასაკში ვლინდება)
ამიტომ, უხშირესად პაციენტის პირველი ვიზიტის დროს, ანამნეზის შეკრებისთანავე დაგნოზი ნათელია, და ლაბორატორიული ტესტებით ხდება მისი დადასტურება.
როცა ჰიპოგლიკემიის დიფ-დიაგნოზი ნათელი არ არის, და ლაბორატორიული ტეტსებიც კონკრეტულად არაფერზე არ მიუთითებს, მაშინ ხდება პაციენტის ჰოსპიტალიზაცია, და ე.წ. 72-საათიანი შიმშილის ტესტის გაკეთება. შემთხვევაში აღწერილი პაციენტიც ამ ტესტისათვის იყო გამზადებული, როცა საბედნიეროდ (თუ საუბედუროდ) ასთმის შეტევის გამო პრედნიზონი დაენიშნა, რამაც ჰიპოგლიკემია მოხსნა. ეს იყო ამ შემთხვევის "გასაღები". პაციენტს გაეზომა ინსულინის ანტისხეულიბი, რისი ტიტრიც საკმაოდ მაღალი აღმოჩნდა. ეს მონაცემი რომ შემთხვევის დასაწყისში დამეწერა, მაშინ საერთოდ აზრი ეკარგებოდა შემთხვევის ამ განყოფილებაში აღწერას.
კიდევ მაქვს ჰიპოგლიკემიის (და არა მარტო) საინტერესო შემთხვევები და შეძლებისდაგვარად დავწერ.

საერთოდ, ეს თემა ძალიან კარგი წამოწყებაა. მადლობა თების ავტორს და ყველა მონაწილეს.

P.S. ყველა მოყვანილი შემთხვევა/პაციენტი ნანახია ჩვენი ჯგუფის მიერ, და შემთხვევები არსაიდან არ არის გადმოწერილი.
* * *
Blind_Torture_Kill
QUOTE
ძაან ნაგავი კითხვაა იმენა ერთი სამი წუთი ვიაზრებდი ამ კითხვას

მართალია, მაგარი პროვოკაციული კითხვაა. ფილტვისმიერი გულის კლასიკური სურათა აღწერილი და ყველა პასუხი შეიძლება მართალი იყოს.
ამას ადასტურებს პლევრიტული სითხის ანალიზიც - ტრანსუდატია და არა ექსუდატი.
არ არის ნახსენები შემთხვევა მწვავედ განვითარდა (პასუხი B, რაც მოხდებოდა მარჯვენა პარკუჭის მწვავე ინფარქტის დროს), თუ ქრონიკულად (პასუხები A, C, D, E).
ალბათ ამ კითხვის პასუხი მდგომარეობს იმაში, თუ რა სიხშირით გვხვდება ეს დაავადებები.
QUOTE
A) Tricuspid valve stenosis

QUOTE
C) Pulmonary valve stenosis

QUOTE
E) Primary pulmonary hypertension

ეს დაავადებები ძალიან იშვიათია




QUOTE
B) Acute myocardial infarction

ეს კი უხშირესად მარცხენა პარკუჭს მოიცავს.

ამიტომ პასუხი მგონია
QUOTE
D) Chronic obstructive lung disease

Posted by: mika9 23 Jul 2009, 10:55
ძაან ტრიკი კითხვაა ..................... დ იმიტომ არ დავწერე რომ არანაირი ნამიოკი არ არის ცოპდ აქვს, სიგარეტს მაინც ეწეოდეს ეს უპატრონო smile.gif




Posted by: basa-ttt 23 Jul 2009, 13:39
Professor Xachikian
გეთანხმებით იმაში რომ ყველა ჩამოთვლილ დავადებას შეუძლია გამოიწვისო ასეთი კლინიკა,
მაგრამ შევხედოთ ცოტა რეალისტურად.
კეისებში ხშირად წარმოდეგნილია ხოლმე
იდეალური სიტუაციები,
რაც სინამდვილეში სულ სხვაგვარადაა ხოლმე.

ფილტვისმიერმმა გულმა, რომ ჯერ მცირე წრეში გამოიწვიოს შეგუბება
და მერე დიდ წრეში -
თან გაქვს ეს -
QUOTE
ასევე აღენიშნება გადიდებული და მტკივნეული ღვიძლი რომელიც პალპაციით ლოკალიზდება
10სმ ქვემოთ ნეკნთა რკალიდან + ქვედა კიდურების შეშუპება


10 სმ გამოსვლა ნეკნთა რკალიდან არის უზარმაზარი ღვიძლი.

ამხელა შეგუბება სანამ განვითარდება დიდ წრეში
მანამდე მცირე წრეში შეგუბება და ფილტვის შეშუპება მოკლავს კაცს.
საერთოდ კი კეისებში ხშირად გამოტოვებულია მთავარი სიმპტომი -
მაგ აქ ეკგ და გულსი ექსოკოპია დოპლერით,
და ამის გარეშე ასეთ კითხვებზე პასუხის გაცემა
რთულია ხოლმე.\
ამავ დროს ტრიკუსპიდალურის იზოლირებული დაზიანებაც ასევე იშვიათობაა.

QUOTE
Primary pulmonary hypertension

ეს არც ისე იშვიათობაა.
მაგრამ არ წერია გამოხატული ქოშინი კლინიკაში.
კლინიკაც და გამოკვლევებიც არასრულია.
და პრინციპში ყველა პასუხი სწორია მეტ ნაკლებად.
* * *
QUOTE
არა მხოლოდ შიმშილის დროს. ნებისმიერ დროს, როდესაც პანკრეასი ნორმაშია და შაქარი დაბალია - შიმშილის იქნება თუ სხვა მიზეზი, პანკრეასი წყვეტს(ან ძალიან ამცირებს) ინსულინის, და შესაბამისად, C-პეპტიდის სეკრეციას (დრებით, თავდაცვის მექანიზმით). კიდევ ვიმეორებ, ეს არ ეხება ინსულინომებს და სულფონილურეით მოწამვლას. ჰიპოგლიკემიის დროს პანკრეასმა რომ გააგრძელოს ინსულილის გამოყოფა (რაც ხდება ინსულინომების დროს), კარგად მოგეხსენებათ ამას რა შედეგი მოყვება. ამიტომ, როცა პანკრეასი წყვეტს (ან ამცირებს) ინსულინის გამოყოფას, მასთან ერთად C-პეპტიდიც ძალიან ქვეითდება, რაც აისახება ანალიზზე.

ერთი საინტერესო სიტუაცია მინდა შემოგთავაზოთ
დავუშვათ ც პეპტიდი მომატებულია
და პროინსულინი და ინსულინი კი დაბალია
თქვენი აზრით - ეს რას უნდა ნიშნავდეს?

Posted by: mika9 23 Jul 2009, 20:52
basa-ttt

ც პეპტიდით მკურნალობს ალბათ,,,,,,,,,,

Posted by: Blind_Torture_Kill 24 Jul 2009, 12:10
His findings point to a pure fight-sided conges-
tive heart failure. This can be caused by right-sided valvu-
lar lesions such as tricuspid or pulmonic stenosis, but these
are rare. Much more common is pulmonary hypertension
resulting from obstructive lung diseases such as emphy-
sema. Primary pulmonary hypertension can also cause
right-sided heart failure, but it is a much less common
cause than are lung diseases. Because acute MI usually
affects the left ventricle, left-sided heart failure is more
common in these patients. Chronic left heart failure can
eventually lead to right-sided heart failure.

(D) აჰა პასუხი

Posted by: basa-ttt 24 Jul 2009, 12:26
Blind_Torture_Kill
ანუ რაც აქ დაიწერა
ყველაფერი სწორია.

ყოველ შემთხვევაში ანამნეზში უნდა იყოს მითითებული
დამატებითი მონაცემები,
რაცა საბოლოოდ გადაწყვეტს დიაგნოს-
მაგ რევმატიზმის არსებობა, ეკგ, ქოშინი, ფილტვის ფიზიკალური მონაცემები,
და ა.შ....
რეალურ ცხოვრებაში ეს სიტუაცია
ცოტა სხვაგავარად იქნებოდა წარმოდგენილი

Posted by: mika9 24 Jul 2009, 14:23
basa-ttt

მაგ ქეისი იმაზე იმას ამოწმებს რამდენად იცი რა არის მოსტ კომმონ, და არა იმას რომ გამოიცნო ცოპდ,

ინგლისურად მეც დავდებ ცასებს თუ ნებას მომცემთ

Posted by: Blind_Torture_Kill 24 Jul 2009, 15:57
mika9
QUOTE
ინგლისურად მეც დავდებ ცასებს თუ ნებას მომცემთ


მიდი

Posted by: mika9 24 Jul 2009, 18:23
A 65-year-old woman with long-standing hypertension has dyspnea associated with the classic symptoms
and physical findings of CHF. Her chest x-ray shows signs of pulmonary edema. Her echocardiogram,
however, shows slightly thickened myocardium and a normal left ventricular ejection fraction. A
diagnosis of diastolic dysfunction is made.
Which of the following would improve this patient's symptoms?
❏ A. Digoxin
❏ B. Furosemide
❏ C. Enalapril
❏ D. Metoprolol
❏ E. None of the above

Posted by: Blind_Torture_Kill 24 Jul 2009, 18:37
QUOTE
❏ B. Furosemide


ალბათ

ისე ენალაპრილსაც მივცემდი ჰიპერტენსიისთვის

Posted by: mika9 24 Jul 2009, 19:05
სწორია.....

A 36-year-old man comes to your clinic complaining of lack of energy. He was diagnosed with diabetes
2 years ago. Review of systems is positive for decreased libido and energy for the past several months. He
has been married for 3 years. He and his wife have been trying to conceive a child for the past year.
Physical examination shows decreased pubic and axillary hair; his testicular volume is 15 ml. Total
testosterone levels are low; LH and follicle-stimulating hormone (FSH) are in the low-normal range. The
prolactin level is normal.
Which of the following would be the most appropriate test in the evаluation of this patient?
❑ A. Testicular biopsy
❑ B. Head MRI
❑ C. Testicular ultrasound
❑ D. Karyotype

Posted by: basa-ttt 24 Jul 2009, 19:51
mika9
ქართული თარგმანიც მიაწერეთ -
გვერდზე.....

Posted by: Blind_Torture_Kill 24 Jul 2009, 20:28

QUOTE
❑ C. Testicular ultrasound


არაინვაზიური მეთოდით დავიწყოთ biggrin.gif

Posted by: mika9 24 Jul 2009, 21:27
basa-ttt
იმედია რუსულიც წავა

А 36-летний мужчина приходит к вашей клиники с жалобами на отсутствие энергии. Он был диагностирован диабет
2 года назад. Обзор систем, является положительным для уменьшилось либидо и энергии на протяжении последних нескольких месяцев. Он
был женат в течение 3 лет. Он и его жена были пытались зачать ребенка в течение года.
Физическое обследование показывает, сократилось лобке и подмышечные волосы, его тестикулярной объемом 15 мл. Всего
уровень тестостерона низок; LH и фолликул-стимулирующий гормон (ФСГ) находятся в нормальном низким диапазоном. Определенный артикль
уровень пролактина в норме.
Какие из следующих будет наиболее соответствующих испытаний в evаluation этого пациента?

Posted by: basa-ttt 24 Jul 2009, 23:19
QUOTE
уровень тестостерона низок

სახეზეა ტესტოსტერონის დაქვეითების კლინიკა
ანუ მამაკაცის ჰიპოგონადიზმი.

ამ ჰორმონის დაქვეითების მიზეზებია:
- თავად სათესლე ჯირკვლების ანომალია
- ცენტრალური წარმოშბის (მაგ ჰიპოფიზის ადენომა)
- ქრომოსომული ანომალია
- სხავ სისტემის (ძირითადი ) დაავადების ფონზე განვითარებული სხვათაშორის შაქრიანი დიაბეტიც იწვევს მეორად ჰიპოგონადიზმს

LH и фолликул-стимулирующий гормон (ФСГ) находятся в нормальном низким диапазоном.
ანუ ცენტრალური წარმოშობის ჰიპოგონადიზმი თითქოს გამოირიცხა?

ქრომოსომული სავარაუდოდ უფრო ადრე გამოვლინდებოდა
წესით ყველა ეს გამოკვლევა სჭირდება
თავსი ტვინის კტ ყველაზე ბოლო რიგში



Posted by: mika9 25 Jul 2009, 00:15
Male hypogonadism can occur as a consequence of a disease of the testes (primary hypogonadism)
or as a consequence of a disease of the pituitary or hypothalamus (secondary
hypogonadism). The clinical findings of hypogonadism result from either decreased spermatogenesis
or decreased testosterone secretion. The sole clinical finding of decreased spermatogenesis
is infertility. In contrast, decreased testosterone secretion causes a wide variety
of clinical findings; specific findings depend on the stage of life in which the deficiency
occurs. In adults, common manifestations are decreases in energy, libido, sexual hair,
muscle mass, and bone mineral density, as well as the presence of anemia. Once the diagnosis
of hypogonadism is suspected on the basis of symptoms and physical examination,
the diagnosis must be confirmed by documenting decreased production of sperm or testosterone.
If hypogonadism is confirmed, the next step is to measure LH and FSH levels.
Elevated serum concentrations of LH and FSH indicate primary hypogonadism, whereas
subnormal or normal values indicate secondary hypogonadism. In patients with secondary
hypogonadism, MRI of the sellar region is indicated. This patient has secondary hypogonadism,
so testicular biopsy and ultrasound are not indicated. Furthermore, testicular
biopsy usually provides no more information about spermatogenesis than does sperm
analysis. Karyotype should be considered in the evаluation of some congenital disorders,
such as Klinefelter syndrome; however, this disorder causes primary hypogonadism.
(Answer: B—Head MRI)

Мужчины Гипогонадизм может произойти вследствие болезни, из яичек (первичный Гипогонадизм)
или вследствие болезни, из гипофиза или гипоталамуса (вторичный
Гипогонадизм). Клинические результаты Гипогонадизм результатом либо сократилось сперматогенез
или снижение секреции тестостерона. Единственный вывод клинических снижения сперматогенеза
является бесплодие. В отличие от этого, уменьшился секрецию тестостерона приводит к широкому кругу
клинических результатов; конкретные выводы зависят от стадии жизни, в котором дефицит
происходит. У взрослых, общие проявления являются уменьшение энергии, либидо, сексуальная волос
мышечной массы и костной минеральной плотности, а также наличие анемии. После того, как диагноз
Гипогонадизм из подозреваемых на основании симптомов и физическое обследование,
диагноз должен быть подтвержден документально сократилось производство спермы и тестостерона.
Если Гипогонадизм подтверждается, то следующий шаг заключается в том, чтобы мера LH и FSH уровнях.
Повышенные концентрации сыворотки LH и FSH указать начальное Гипогонадизм, тогда
субнормальных или нормальные значения указывают среднее Гипогонадизм. У пациентов, имеющих среднее
Гипогонадизм, МРТ в регионе sellar указывается. Этот пациент имеет среднее Гипогонадизм,
так тестикулярной биопсии и ультразвук не указано. Кроме того, тестикулярной
биопсии, как правило, не дает никакой дополнительной информации о сперматогенез, чем сперматозоиды
анализа. Кариотип должны быть рассмотрены при оценке некоторых врожденных заболеваний,
такие, как синдром Klinefelter, однако причины этого расстройства начальной Гипогонадизм.
(Ответ: B-Head МРТ)
* * *
basa-ttt

მომატებულია , ხოგჯერ მნიშვნელოვანია იცოდე რამდენად
დაკლებულია თუ ნორმალურია ერთიდაიგივეა დიაგნოსტიკური თვალსაზრისით
* * *
A 55-year-old man presents to your clinic complaining of swollen breasts. His symptoms started 3 or 4
months ago, when he noticed tenderness and swelling in both breasts. His medical history includes congestive
heart failure and hypertension. His medications are benazepril, metoprolol, furosemide, and
spironolactone. Review of systems is positive only for occasional dyspnea on exertion. Physical examination
shows bilateral gynecomastia in the periareolar area, with some tenderness to palpation.
Testicular examination is normal.
Which of the following would be the best step to take next in the evаluation and management of
this patient?
❑ A. Liver ultrasound
❑ B. Mammography
❑ C. Testicular ultrasound
❑ D. Cessation of spironolactone

А 55-летний мужчина представляет вашу клинику с жалобами на опухшие груди. Его симптомы начали 3 или 4
месяцев назад, когда он заметил, опухание и болезненность в груди. Его медицинская история включает застойной
сердечной недостаточности и артериальной гипертензии. Его препараты benazepril, metoprolol, фуросемид, и
спиронолактон. Обзор систем является положительным только иногда одышка от нагрузки. Физическое обследование
показывает двусторонние gynecomastia в periareolar области, с некоторой нежностью к пальпации.
Тестикулярной экспертизы является нормальным.
Какие из следующих бы наилучшим шагом в следующей оценке и управлению
этот больной?
❑ А. Печень ультразвуком
❑ B. Маммография
❑ C. тестикулярной ультразвуком
❑ D. Прекращение спиронолактон

Posted by: basa-ttt 25 Jul 2009, 00:48
QUOTE
Прекращение спиронолактон

სხვათაშორის ვეროშპირონი იწვევს ასეთ კლინიკას.
გააჩნია რამდენი ხანია და რა დოზით ხმარობს ამ პრეპარატს.
პირველი უნდა ჰკითხოთ პაციენტს სწორედ ეს.

Posted by: mika9 25 Jul 2009, 00:56
basa-ttt
სვამს და რა უნდა კიტხო ანუ შეუწყვიტო, არის ზოგიერთი წამალი რომელიც გაძლევს რაგაც გვერდით ეფექტს და არ უნა სჰეუწყვიტო მიუხედავად გვერდითი ეფექტისა,, ზოგიერთ შემთხვევაში უნდა შუწყვიტო და მერე მომავალში თუ დაჭირდა ეს წამალი შეუძლია განუახლო, ხოლო არის ისეთი რამელბი რომლებმაც თუ გამოიწვიეს გვერდითი ეფექტი უნდა შეუწყვიტო და საერტოდ არ დალევინო

ეს ის შემთხვევა როცა უნდა შუწყვიტო, ხოლო თU შედეგი არ მოგცა მერე სხვა რამეზე უნდა იპიქრო
* * *
A 22-year-old woman presents to the emergency department with nausea, vomiting, and abdominal
pain of 4 days' duration. Her fluid balance profile is as follows: Na+, 145; K+, 5.0; Cl-, 105; HCO3
-, 15;
BUN, 37; Cr, 1.6; glucose, 780; UA, 4+ ketones.
What is the best initial treatment of this patient's acid-base disorder?
❑ A. Free water
❑ B. Normal saline
❑ C. Normal saline, sodium bicarbonate, and insulin
❑ D. Half-normal saline and insulin
❑ E. Normal saline and insulin


А 22-летняя женщина представляет чрезвычайную отдела с тошнотой, рвотой и брюшной
более 4 дней. Ее баланс жидкости профиль выглядит следующим образом: Na + 145 + 5.0 K, Cl-, 105; HCO3
-, 15;
BUN, 37; Cr, 1.6; глюкоза, 780; АС 4 + кетоны.
Что является лучшим первоначального лечения этого пациента, кислотно-щелочной беспорядка?
❑ А. Свободная вода
❑ B. Нормальный соленых
❑ C. Нормальный соленых, бикарбонат натрия, и инсулин
❑ D. Половина нормальной соленых и инсулин
❑ E. Нормальный соленых и инсулин

Posted by: basa-ttt 25 Jul 2009, 01:03
QUOTE
ეს ის შემთხვევა როცა უნდა შუწყვიტო, ხოლო თU შედეგი არ მოგცა მერე სხვა რამეზე უნდა იპიქრო

ეგ რასაკვირველია, ცხადზე ცხადია
მაგრამ თუ გითხრა 2 კვირაა რაც ვსვამო ვეროშპირონს -
ეგრევე სხვა მიზეზს ეძებ...

Posted by: Blind_Torture_Kill 25 Jul 2009, 01:03
QUOTE
Male hypogonadism can occur as a consequence of a disease of the testes (primary hypogonadism) or as a consequence of a disease of the pituitary or hypothalamus (secondary hypogonadism). The clinical findings of hypogonadism result from either decreased spermatogenesis or decreased testosterone secretion. The sole clinical finding of decreased spermatogenesis is infertility. In contrast, decreased testosterone secretion causes a wide variety of clinical findings; specific findings depend on the stage of life in which the deficiency occurs. In adults, common manifestations are decreases in energy, libido, sexual hair, muscle mass, and bone mineral density, as well as the presence of anemia. Once the diagnosis of hypogonadism is suspected on the basis of symptoms and physical examination, the diagnosis must be confirmed by documenting decreased production of sperm or testosterone. If hypogonadism is confirmed, the next step is to measure LH and FSH levels. Elevated serum concentrations of LH and FSH indicate primary hypogonadism, whereas subnormal or normal values indicate secondary hypogonadism. In patients with secondary hypogonadism, MRI of the sellar region is indicated. This patient has secondary hypogonadism, so testicular biopsy and ultrasound are not indicated. Furthermore, testicular biopsy usually provides no more information about spermatogenesis than does sperm analysis. Karyotype should be considered in the evаluation of some congenital disorders, such as Klinefelter syndrome; however, this disorder causes primary hypogonadism. (Answer: B—Head MRI)


იცი რა არ მესმის ეხლა პრობლემა რომ ქონოდა ჰიპოფიზზე მაშინ პროლაქტინის კონცენტრაციაც არ უნდა დაცემულიყო ?
თან LH da FSH ნორმის ქვედა ზღვარზე იყო და მაშინ დაბალი უნდა ყოფილიყო

Posted by: mika9 25 Jul 2009, 01:05
Blind_Torture_Kill

არა ნორმა არ უნდა ყოპილიყო ნორმა რომ არის მაგ მიუთითებს რომ არანორმალურია, ნორმალური მაგ შემთხვევაში იქნებოდა მმომატებული რომ იყოს...

არ ვიცი ბრმად ვდებ ამ MჩQ მიხედვით ვარჩევ, პასუხსაც მერე ვნახულობ



Posted by: Blind_Torture_Kill 25 Jul 2009, 01:12
QUOTE
❑ E. Normal saline and insulin


ესაა მგონი

პროლაქტინსაც არ უნდა დაეკლო ?

Posted by: Cousteau 25 Jul 2009, 01:12
D. Half-normal saline and insulin ?
ნეზნაუ user.gif

Posted by: mika9 25 Jul 2009, 01:15
პასუხი ხვალ,,,,, .................................................................

Posted by: Cousteau 25 Jul 2009, 01:17
QUOTE (mika9 @ 25 Jul 2009, 01:15 )
პასუხი ხვალ,,,,, .................................................................

: D

აბა დავიძინე მაშინ eek.gif

Posted by: Blind_Torture_Kill 25 Jul 2009, 01:24
QUOTE
პასუხი ხვალ,,,,, .................................................................


ხვალე გავდივარ და 4 დღეში ჩამოვალ
მანამდე ამოსახსნელი დატოვეთ რამე ქეისები

Posted by: TIALATA 25 Jul 2009, 12:32
ხალხო მაინტერესებს
2 ტუ 3 წლის წინ გავცვდი ბრონქიალური ასთმა მქონდა ეხლა კარგად ვარ არაფერს არ ვისუნთქავ წამალს და ვარ ჩვეულებრივად და ექიმმა მითხრა რო ზღვაზე არ შეიძლება ბანაობაო და აღარასდროს აღარ უნდა ვიცურაო ზღვაში თუ როგორაა ჩემი საქმე ზღვაზე ვაპირებ წასვლას და მაინტერესებს შეიძლება თU არა

Posted by: mika9 25 Jul 2009, 13:25
TIALATA

შენ ექიმს "შ ქონია" სხვა ვერანაირი ახსნა.....

შავ ზღვაზე თუ გაქვს ალერგია მაშინ ხმელთაშუაზე წადი
* * *
Metabolic acidosis can be classified into two types: that associated with an elevation in
the anion gap, and that in which the anion gap is normal. A calculation of the anion
gap in this patient reveals a gap of 25. Among the causes of acidosis associated with an
elevated anion gap are alcoholic ketoacidosis, lactic acidosis, starvation, ingestion of
alcohols, ingestion of salicylates, and diabetic ketoacidosis. In patients with diabetic
ketoacidosis (such as this patient), optimal initial treatment includes fluid replacement
with normal saline to promote ketonuria and insulin to facilitate glucose transport.
Bicarbonate therapy is not usually indicated unless the acidosis is severe or severe
hyperkalemia is present. (Answer: E—Normal saline and insulin)

ბასა რორრყ იქნებ http://translate.google.com/translate_t# აქ შეხვიდე და ცოპი პასიტიტ თვითონგადარარგმნო

Posted by: basa-ttt 25 Jul 2009, 13:35
TIALATA
საუბარია იმაზე, რომ ასთმა ხშირად ვითარდება ნესტიან გარემოში -
ანუ პროცენტულად ასთმიანები დასავლეთ საქართველოში მეტია,
ვიდრე აღმოსავლეთში.
თუ თქვენ ნესტზე ასთმის გამწვავება არ გემართებათ -
შეგიძლიათ წახვიდეთ ზღვაზე.
ზოგიერთი პირიქით უკეთესად გრძნობს თავს ზღვაზე.

QUOTE
ბასა რორრყ იქნებ http://translate.google.com/translate_t# აქ შეხვიდე და ცოპი პასიტიტ თვითონგადარარგმნო

ვიცი ეს თაგრმანი, მაგრამ ზოგიერთ დეტალებს უაზროდ თარგმნის ხოლმე.

Posted by: mika9 25 Jul 2009, 13:38


A 42-year-old woman presents with nausea, vomiting, and left flank pain with radiation to the groin;
these symptoms have persisted for 3 days. A helical CT scan reveals a stone in the left ureter. On the basis
of urinalysis and serum chemistries, a diagnosis of type 1 renal tubular acidosis (RTA) is made.
Which of the following is NOT consistent with type 1 RTA?
❑ A. Normal-anion-gap metabolic acidosis
❑ B. Urine pH < 5.3
❑ C. Hypokalemia
❑ D. Urinary calcium phosphate crystals
❑ E. Sjögren syndrome


А 42-летняя женщина подарки с тошнотой, рвотой, и левый фланг боли с радиацией в пах;
эти симптомы сохраняются в течение 3 дней. В спиральных КТ сканирование показывает, камень в левом мочеточнике. На основе
в анализ мочи и сыворотки химии, диагноз типа 1 почечной трубчатых ацидоз (RTA) производится.
Какие из перечисленных ниже, не соответствует типу 1 RTA?
❑ А. Нормальная-анион-разрыв метаболический ацидоз
❑ B. мочи рН <5,3
❑ C. Hypokalemia
❑ D. фосфата кальция в моче кристаллов
❑ E. Sjögren синдром

Posted by: Professor Xachikian 26 Jul 2009, 05:43
basa-ttt
QUOTE
ერთი საინტერესო სიტუაცია მინდა შემოგთავაზოთ
დავუშვათ ც პეპტიდი მომატებულია
და პროინსულინი და ინსულინი კი დაბალია
თქვენი აზრით - ეს რას უნდა ნიშნავდეს?

უკაცრავად დაგვიანებული პასუხისათვის.
ასეთ სუტუაციაში... არ ვიცი. ალბათ ვიფიქრებდი, რომ ლაბორატორიული შეცდომაა. ისე, სიმართლე გითხრათ, ამაზე არ მიფიქრია.
ბევრჯერ, როცა 72-საათიანი შიმშილის ტესტი გაგვიკეთებია და ინსულინომა დადასტურებულა, ც-პეპტიდიც, პროინსულინიც და ინსულინიც ერთდროულად მომატებულია და მათი კონცენტრაცია ერთმანეთის "პარალელურია", ანუ დაახლოებით ერთნაირი თანაფარდობით ცვალებადობს ც-პეპტიდი, ინსულინი და პროინსულინი (მათი გაზომვა სერიულად ხდება 72-საათის განმავლობაში, დროის გარკვეული ინტერვალით). ეს ტესტი ისედაც იშვიათად კეთდება, და ვერ ვიხსენებ შემთხვევას, როცა ც-პეპტიდის იზოლირებული მომატება ყოფილიყოს.
ჩანს, რომ ჰიპოგლიკემიაზე მუშაობის კარგი გამოცდილება გაქვთ, და თქვენ თუ პრაკტიკაში გქონიათ ასეთი შემთხვევა, მგონი ძალიან კარგი სადისკუსიო "ქეისი" გამოვა.
მეც მაქვს ჰიპოგლიკემიის კიდევ ერთი-ორი კარგი შემთხვევა და შეძლებისდაგვარად დავწერ (ისე ეხლა სხვა "ქეისს" ვამზადებ - ჰიპერკალცემიაზე. მგონი ასე ჯობია მრავლფეროვნებისათვის)
* * *
mika9
კლასიკური რენალური ტუბულარული აციდოზი, იგივე Type 1 RTA, ხასიათდება ჰიპერქლორემიით და თირკმლის დისტალური ტუბულების მიერ H+ იონების გენერაციის დაქვეითებით. ამის გამო, მიუხედავად აციდოზისა, შარდის PH არის მომატებული. იგივე მიზეზის გამო (ანუ არასაკმარისი H+ იონების გამო) შარდში გროვდება ფოსფატი და კრისტალდება კალციუმთან (პასუხი D). სისხლში აღინიშნება ჰიპოკალემია(პასუხი C). ანიონური სხვაობა (anion gap) ტუბულარული აციდოზების დროს მომატებული არ არის(პასუხი A). Sjogren-ის დაავადების დროს ვითარდება RTA 1. (პასუხი E)
ამ მდგომარეობის დროს თირკმელს დაკარგული აქვს შარდის "შემჟავების" უნარი. ამიტომ RTA Type 1-ის დროს რაც არ ხდება (სწორი პასუხი) მგონია
QUOTE
❑ B. Urine pH < 5.3


Posted by: Professor Xachikian 29 Jul 2009, 05:59
55 წლის მამაკაცი შემოვიდა მიმღებში საერთო სისუსტით, მუცლის არეში დისკომფორტით, გულისრევის შეგრძნებით და კუნთების სისუსტით. სისხლის ანალიზმა აჩვენა ჰიპერკალცემია: Ca 13.4 mg/dl (ნორმა 8.5-10.5 mg/dl).პაციენტი მიღებული იქნა საავადმყოფოში ჰიპერკალცემიის დიაგნოზით, სათანადო მკურნალობისათვის და შემდგომი გამოკვლევებისათვის.
ანამნეზში აქვს კარგად კონტროლირებული ჰიპერტენზია. ღებულობს მეტოპროლოლს 50 მგ 2-ჯერ დღეში და ასპირინს 81 მგ დღეში ერთხელ. კალციუმის პრეპარატებს (ანტაციდებს და ა.შ.) და ვიტამინ D-ს პრეპარატებს არ იღებს.

ლაბორატორიულმა ანალიზებმა აჩვენა:
სისხლის საერთო ანალიზი – ნორმა

ბიოქიმიური ანალიზები:
ელექტროლიტები (კალციუმის გარდა) – ნორმა
კრეატინინი 1.6 mg/dl (ნორმა 0.7-1.5 mg/dl). გლუკოზა უზმოზე 89 mg/dl
ღვიძლის ფუნქციური ტესტები - უმნიშვნელოდ მომატებული.
პარათჰორმონი (PTH) – 11 pg/ml (ნორმა 12-65),პარათჰორმონ-დაკავშირებული პეპტიდი (Parathyroid-related peptide, PTH rP) – უარყოფითი, ჰიდროქსივიტამინ D (25(OH)Vit.D) – 17.8 mkg/l (ნორმა 8.9-46.7 mkg/l), დიჰიდროქსივიტამინ D (1,25(OH)2 Vit.D) – 107 ng/l (ნორმა 12.9-55.6)
სისხლის და შარდის პროტეინის ელექტროფორეზი- ნორმა.

1. დიფერენციალური დიაგნოზი
2. საბოლოო დიაგნოზი

Posted by: Blind_Torture_Kill 30 Jul 2009, 12:01
ეხლა ჩამოვედი
იბიო მატ ვკითხულობ და თავში ვერ შედის sad.gif
გამოვიძინებ და მერე წავიკითხავ

Posted by: Tornike Alashvili 30 Jul 2009, 17:55
Professor Xachikian
ეს მოწამვლაა დიჰიდროგენის მონოოქსიდით
დიფერენსიაციისთვის შარდის ანალიზი უნდა გაკეთდეს ამ ქიმიკატის არსებობაზე
ფრთხილად maxati.gif
-დაიბანეთ ხელები და არ ისუნთქოთ 2 წუთით

user posted image
და თუ პასუხი დადებითია- სარკოიდოზია (ან სხვა გრანულემატოზი)
ვინაიდა მის დროსაა მომატებული 1,25(OH)2D3
და PTH rP რადღანც არაა -სოლიდური სიმსივნეები
ირიცხება ამით და სხვაც არაფერია არც ანამნეზში და არც ანალიზებში

გადავარჩინოთ ბიოსფერა დიჰიდროგენის მონოოქსიდიდან !!!!!
* * *
http://www.youtube.com/watch?v=MawlljmaS1w&hl=ru
* * *
Is Dihydrogen Monoxide (DHMO) contaminating our water?

Posted by: Professor Xachikian 31 Jul 2009, 03:13
Tornike Alashvili
QUOTE
სარკოიდოზია (ან სხვა გრანულემატოზი)

up.gif

მართალია, ამ პაციენტს სარკოიდოზი აღმოაჩნდა. 1,25-დიჰირდოქსივიტამინ D-ს ენდოგენურ მომატებას იწვევს გრანოლომატოზური დაავადებები ან ლიმფომა.
თუმცა PTHrP უარყოფითი მოვიდა, ძვლის მეტასტაზური დაავადება მხოლოდ ამ ტესტით ვერ გამოირიცხებოდა. ამიტომ გაკეთდა ძვლის სკანირება ტექნეციუმით. შედეგი - უარყოფითი.
გულმკერდის რენტგენმა აჩვენა ფილტვის ინტერსტიციული ინფილტრატები (რეტიკულური სახის)
მთელი სხეულის კომპიუტერულმა ტომოგრაფიამ ასევე აჩვენა ფილტვის ინფილტრატები, და ჰილარული ლიმფადენოპათია.
გაკეთდა ბრონქოსკოპია და ტრანსბრონქიალური ბიოფსია.
პათანატომიური დასკვნა: არაკაზეოზური გრანულომა.
საბოლოო დიაგნოზი: სარკოიდოზი.


Posted by: basa-ttt 31 Jul 2009, 15:42
QUOTE
უკაცრავად დაგვიანებული პასუხისათვის.
ასეთ სუტუაციაში... არ ვიცი. ალბათ ვიფიქრებდი, რომ ლაბორატორიული შეცდომაა. ისე, სიმართლე გითხრათ, ამაზე არ მიფიქრია.
ბევრჯერ, როცა 72-საათიანი შიმშილის ტესტი გაგვიკეთებია და ინსულინომა დადასტურებულა, ც-პეპტიდიც, პროინსულინიც და ინსულინიც ერთდროულად მომატებულია და მათი კონცენტრაცია ერთმანეთის "პარალელურია", ანუ დაახლოებით ერთნაირი თანაფარდობით ცვალებადობს ც-პეპტიდი, ინსულინი და პროინსულინი (მათი გაზომვა სერიულად ხდება 72-საათის განმავლობაში, დროის გარკვეული ინტერვალით). ეს ტესტი ისედაც იშვიათად კეთდება, და ვერ ვიხსენებ შემთხვევას, როცა ც-პეპტიდის იზოლირებული მომატება ყოფილიყოს.
ჩანს, რომ ჰიპოგლიკემიაზე მუშაობის კარგი გამოცდილება გაქვთ, და თქვენ თუ პრაკტიკაში გქონიათ ასეთი შემთხვევა, მგონი ძალიან კარგი სადისკუსიო "ქეისი" გამოვა.
მეც მაქვს ჰიპოგლიკემიის კიდევ ერთი-ორი კარგი შემთხვევა და შეძლებისდაგვარად დავწერ (ისე ეხლა სხვა "ქეისს" ვამზადებ - ჰიპერკალცემიაზე. მგონი ასე ჯობია მრავლფეროვნებისათვის)

მოვედი-
ეს შემთხვევა ჩემი პრაქტიკიდან არაა -
მაგრამ საინტერესოა სუფთა თეორიული თვალსაზრისით -
ანუ აღწერილია ზოგჯერ ე .წ. ცე პეპტიდის ცრუ მატება -
როცა პროინსულინი და ინსულინი დაბალია.
ეს ხდება მაშინ, როცა
პაცინეტს აქვს ანტისხეულები ინსულინის მიმართ -
ამ დროს ხდება პროინსულინის შებოჭვა ამ ანტისხეულებით
და მისი დონე ამ დროს ლაბორატ გამოკვლევებით
დაბალია
ამავე დროს კი ეს ანტისხელები არ ურთიერთქმედებენ ცე პეპტიდთან და
ამის გამო ის რჩება მაღალი სისხლში.
აი ასეთ სიტუაციაში გვაქვს ცე პეპტიდი მაღალი და პროინსულინი და ინსულინი დაბალი.
თუ რომელი დაავადებების დროს შეიძლება ეს იყოს? -
სავარაუდოდ აუტოიმუნური პროცესებისას და I ტიპის დიაბეტის დროსაც .


У больных диабетом, имеющих антитела к инсулину, связывающие проинсулин, иногда наюлюдаются ложноповышенные уровни С-пептида за счет перекрестно реагирующих с проинсулином антител

Posted by: Tornike Alashvili 31 Jul 2009, 16:07
Professor Xachikian
დიაგნოზი კარგია მაგრამ მკურნალობა ?
უნდა გაიჭყიპოს კორტიკოსტეროიდებით
მერე მაინც ურჩიეთ ჰომეოპათია აღუდგენს იმუნიტეტს და
გაწოვს ბოლომდე გრანულომებს

Posted by: Professor Xachikian 1 Aug 2009, 03:09
basa-ttt
გეთანხმებით, საინტერესო ფენომენია.
Tornike Alashvili
მართალი ხართ, როგორც კი სარკოიდოზის დიაგნოზი დადგინდა, პაციენტი გაგზავლინი იქნა პულმონოლოგებთან, და რამდენადაც ვიცი, გლუკოკორტიკოიდებით მკურნალობა უტარდებოდა.

Posted by: Blind_Torture_Kill 2 Aug 2009, 02:51
25 წლის ქალბატონის გვამის გაკვეთისას ინახა :

საავადმყოფოში მოთავსებისას მას აღენიშნებოდა პანციტოპენია + შარდში ცილა. გულმკერდის რენტგენოგრამაზე პლევრის ღრუებში სითხე და აუსკულტაციით ისმინებოდა - friction rub (ქართულად არ ვიცი როგორ ითქმება - ხახუნი რა მოკლედ) + ქონდა გახანგრძლივებული PTT.
ორსულობა ორი - ორივე with spontaneous abortion (ნაყოფის მოშორებით) დასრულდა

რა არის სურათზე ?

დაადიაგნოზეთ და სურათიც აღწერეთ

Posted by: Professor Xachikian 2 Aug 2009, 07:37
Blind_Torture_Kill
კლინიკური პრეზენტაცია სისტემურ წითელ მგლურას გავს.
სურათზე ალბათ ე.წ. marantic endocarditis, და ამ დროს წარმოქმნილი ვეგეტაცია არის ნაჩვენები.

Posted by: Romina 2 Aug 2009, 08:52
Professor Xachikian
ალბათ smile.gif შეიძლება ანტიფოსფოლიპიდური სინდრომი ერთო თან.

Posted by: Blind_Torture_Kill 2 Aug 2009, 09:20
Professor Xachikian
QUOTE
კლინიკური პრეზენტაცია სისტემურ წითელ მგლურას გავს.


Romina
QUOTE
ალბათშეიძლება ანტიფოსფოლიპიდური სინდრომი ერთო თან.


ხოო ეგრეა

სურათზე ლიბმან საქსის ენდოკარდიტია ნაჩვენები
marantic endocarditis - უფრო მცირე ზომისაა

Posted by: svani67 2 Aug 2009, 10:58
basa-ttt
QUOTE
Прекращение спиронолактон

სხვათაშორის ვეროშპირონი იწვევს ასეთ კლინიკას.



იქნებ ამიხსნათ< რას ნიშნავს რუსული სიტყვები, ანუ სპირონოლაქტონი?

მე ვსვამ ვეროშპირონს პერიოდულად, 50 მგ დღეში ერთხელ 1-2 კვირის განმავლობაში, მერე 1-2 კვირა აღარ ვსვამ. მუცელში რომ სითხე არ დამიგროვდეს. მაქვს ც ჰეპატიტი და ციროზი.

მე ისეთი დასკვნა გამოვიტანე, რომ ვეროშპირონი არ უნდა იწვევდეს მაინც და მაინც კარგ შედეგებს. ჩემს შემთხვევაში რა უარყოფითI შეიძლება იყოს? იქნებ სხვა შარდმდენით შემეცვალა?

ძალიან გთხოვთ მიპასუხოთ

Posted by: Solveig 2 Aug 2009, 17:04
Tornike Alashvili
QUOTE
გადავარჩინოთ ბიოსფერა დიჰიდროგენის მონოოქსიდიდან !!!!!

არადა, თქვენ ძალიან ხშირად იყენებთ მაგ ნივთიერებას.

svani67
QUOTE
სპირონოლაქტონი

სპირონოლაქტონი რუსული სიტყვა არ არის. ეგ არის ნივთიერება, რომლის შემცველ წამლებსაც სხვადასხვა დასახელება აქვს, ერთ-ერთია ვეროშპირონი. დიურეტიკია (შარდმდენი), რომელიც აკავებს კალიუმს ორგანიზმში და სამაგიეროდ, ხელს უწყობს ნატრიუმის გამოდევნას. ანტიანდროგენული მოქმედებაც აქვს..
შენს შემთხვევაში პირდაპირი ჩვენების გამო დაგინიშნავდა ექიმი (რომ სითხის დაგროვება არ მოხდეს)...თუ რაიმე უარყოფით გავლენას ახდენს შენზე, ექიმს უნდა დაელაპარაკო მაშინ.

Posted by: Professor Xachikian 2 Aug 2009, 18:16
20 წლის ქალბატონი , ანამნეზში ჰიპოკალცემიით, შემოვიდა კლინიკაში შემდგომი მკურნალობისათვის.
სიმპტომური ჰიპოკალცემია დაეწყო ~12 წლის ასაკში, რაც გამოიხატებოდა მსუბუქ ტეტანიებში. სიმპტომების დროს მშობლებს მიყავდათ ადგილობრივ, სოფლის საავადმყოფოში, სადაც უტარდებოდა კალციუმის ინტრავენური გადასხმა, რაც სიმპტომებს ხსნიდა. დაახლოებით 14 წლის ასაკში სიმპტომები იმდენად გახშირდა და გამწვავდა, რომ მშობლებმა წაიყვანეს ექიმთან დიდ ქალაქში. პაციენტს დაენიშნა კალციუმის პრეპაეატები და ვიტამინ D რეგულარულად, რამაც საგრძლობლად გამოასწორა მისი მდგომარეობა. 15 წლის ასაკში დაეწყო ამენორეა (მანამდე ციკლი ნორმაში ქონდა, მენარქე - 13 წლის ასაკში).
პირველი ვიზიტის შემდეგ (ანუ 20 წლის ასაკში) დაენიშნა ციკლური კონტრაცეპტივები (კონკრეტულად ორთო-ტრიციკლენი), რამაც განაპირობა ციკლის აღდგენა.
პაციენტი სტაბილურად იყო ~2 წლის განმავლობაში, და როგორც მოსალოდნელია, ზოგჯერ ჭირდებოდა კალციუმის და ვიტამინ D-ს დოზის ცვლილება, თუმცა ამ დროის განმავლობაში არც ერთი მძილე ჰიპოკალცემიური ეპიზოდი არ ქონია.

პირველი ვიზიტიდან 2 წლის შემდეგ (~22 წლის ასაკში) პაციენტი მოხვდა საავადმყოფოში შემდეგი სიმპტომებით, რაც დაეწყო რამდენიმე კვირით ადრე და პროგრესულად გაუარესდა: ძლიერი საერთო სისუსტე, თავბრუხვევა, გულისრევა, ღებინება, წონაში კლება (დაახლოებით 3 კგ).
მიმღებში წნევა ქონდა 85/55, გულისცემა 110. აღენიშნებოდა ორთოსტატიული ჰიპოტენზია. სხეულის ტემპერატურა 37.1
გულმკერდის რენტგენი - ფილტვები სუფთა. შარდის ანალიზი - ნორმა. სისხლის ბაქტეროლოგიური ანალიზი - უარყოფითი (სისხლი გაიგზავნა ბაქტერიოლოგიურ კულტურაზე)
სისხლის საერთო ანალიზი - მცირედ გამოხატული ნორმოციტული ანემია.
სხვა ანალიზები:
Na – 135 (135-145)
K – 5.4 (3.5-5.5)
Cl-97 (93-105)
HCO3 – 25 (20-28)
შარდოვანა – 22 (4-22)
კრეატინინი 1.7 (0.6-1.3)
Ca – 8.3 (8.5-10.2)
ალბუმინი 3.8
გლუკოზა 63 მგ/დლ
TSH - 1.67 mcu/mL (ნორმა)
FT4 - 1.2 ნგ/დლ (ნორმა)

სავარაუდო დიაგნოზი, საბოლოო დიაგნოზი

Posted by: Tornike Alashvili 2 Aug 2009, 18:26
Solveig
QUOTE
არადა, თქვენ ძალიან ხშირად იყენებთ მაგ ნივთიერებას.

ჩვენ ყველა შხამებს კი ვიყენებთ –ოღონდ უმცირეს დოზაში wink.gif

Posted by: Blind_Torture_Kill 2 Aug 2009, 22:39
Professor Xachikian

ფოსფორის დონეც გვითხარი

Posted by: PHOBOSS 2 Aug 2009, 22:59
საწყისი სიმპტომები ძალიან გავს ჰიპოპარატირეოზს. მერე უკვე თითქოს ადისონის დაავადება იკვეთება, ოღონდ კანის ფერი არ არი ნახსენები sad.gif ჰოსპიტალიზაციისას სიმპტომები ადისონის კრიზს გავს. ამ ორივე პათოლოგიის გაერთიანება მარტო ერთი მომდის თავში smile.gif პოლიგლანდულარული ავტოიმუნური დაავადება ტიპი I. იმედია ზალიან კონსმოსში არ წავედი smile.gif

Posted by: Professor Xachikian 3 Aug 2009, 00:29
QUOTE
ფოსფორის დონეც გვითხარი

ფოსფორი ჰოსპიტალიზაციის დროს არ გაკეთებულა sad.gif
PHOBOSS
QUOTE
იმედია ზალიან კონსმოსში არ წავედი smile.gif

lol.gif lol.gif lol.gif
QUOTE
smile.gif პოლიგლანდულარული ავტოიმუნური დაავადება ტიპი I

up.gif up.gif up.gif

ჩვენი დიაგნოზიც ეგ იყო - პოლიგლანდულარული აუტოიმუნური სინდრომი, I ტიპი. თუმცა აქვე უნდა ითქვას, რომ პაციენტს ერთ-ერთი პირველი ნიშანი - კანის და ლორწოვანის კანდიდიაზი - არ ქონია, და შედარებით გვიან ასაკში დაეწყო ეს დაავადება.

ეს "ქეისი" ფაქტიურად, "მოვარგე" ამ თემის ფორმატს, მაგრამ რეალურად ცოტა სხვაგვარად იყო საქმე:
პაციენტის პირველივე ვიზიტიდან ცხადი იყო რომ პოლიგლანდულარულ უკმარისობასთან გვქონდა საქმე, ამიტომ პაციენტს მაშინვე გაეზომა ელექტროლიტები და კორტიზოლი სხვა ანალიზებთან ერთად. ასევე გაკეთდა ანალიზი ანტი-არდენალურ ანტისხეულებზე, რაც მოვიდა უმნიშვნელოდ მომატებული. მიუხედავად ამისა, თირკმელზედა ჯირკვლის ფუნქცია ჯერ კიდევ ნორმაში იყო.
პაციენტი მრავალჯერ იქნა გაფრთხილებული მოსალოდბელი თირკმელზედა ჯირკვლის უკმარისობის (და ადისონის კრიზის) შესახებ. ასე რომ პაციენტმა თეორიულად კარგად იცოდა თირკმელზედა ჯირკვლის უკმარისობის სმპტომები, და ამის გამო, საავადმყოფოში წასვლა არ დაუგვიანია.
პაციენტს გასინჯვით ტიპიური ჰიპერპიგმენტაცია არასდროს არ ქონია. საავადმყოფოში გაზომილი კორტიზოლის დონე სისხლში იყო ძალიან დაბალი, <4 მგკ/დლ. შესაბამისად, აკტჰ (ACTH) იყო ~800 პგ/მლ (ნორმა 9-52).
ეხლა მდგომარეობა სტაბილურია, მკურნალობა უტარდება კორტეფით და ფლურდოკორტიზონით. ასევე აგრძელებს აგრძელებს კალციუმით და ვიტამინ D მკურნალობას.

Posted by: Romina 3 Aug 2009, 07:39
Professor Xachikian
ძალიან საინტერესო კლინიკური შემთხვევა იყო
smile.gif
PHOBOSS
ყოჩაღ შენ
smile.gif

Posted by: PHOBOSS 3 Aug 2009, 09:44
QUOTE (Romina @ 3 Aug 2009, 07:39 )
PHOBOSS
ყოჩაღ შენ
smile.gif

დიდი მადლობა 2kiss.gif

Posted by: Cousteau 3 Aug 2009, 10:46
QUOTE
ჩვენი დიაგნოზიც ეგ იყო -

სად მუშაობ ეგეთი რაღაცეების დიაგნოზს რო სვავთ?
ნუ მაინც რამე რო იყოს მეცოდინება : D

Posted by: Thandrus 4 Aug 2009, 06:39
Professor Xachikian

მართლაც საინტერესო შემთხვევა იყო... კაკ რაზ დღეს გადავხედე ამ დაავადებაზე რაღაცეებს smile.gif

აბა ამას თუ გამოიცნობთ, რა არის... smile.gif

Posted by: Professor Xachikian 4 Aug 2009, 06:46
Cousteau
QUOTE
სად მუშაობ ეგეთი რაღაცეების დიაგნოზს რო სვავთ? ნუ მაინც რამე რო იყოს მეცოდინება : D

დიაგნოზისათვის კლინიკის ადგილმდებარეობას არა აქვს მნიშვნელობა. ყველგან შეიძლება ამ დიაგნოზის დასმა, თუ სადმე გადაკარგულში არ ხარ, და ნორმალური ლაბორატორია ახლოს გაქვს.
ეს შემთხვევა სადაც ვნეხე, პმ-ში გწერ. იმედია კონფიდენციალურობას დაიცავ.

Thandrus
მადლობა
2kiss.gif
* * *
Romina
თქვენც მადლობა
2kiss.gif

Posted by: PHOBOSS 4 Aug 2009, 11:59
Thandrus

V1-V2 ძაალიან გავს ბრუგადას სინდრომს. დანარჩენი განხრები - "ჩვეულებრივ" ნაადრევი რეპოლარიზაციას.
კლინიკურად რამე გამოვლინება ქონდა პაციენტს? სპორცმენი ხო არაა ან გადაციებული? smile.gif

Posted by: Thandrus 4 Aug 2009, 14:47
PHOBOSS

QUOTE
გადაციებული?


ეგააა smile.gif ჰიპოთერმია smile.gif Osborne J-waves.

up.gif up.gif up.gif

Posted by: PHOBOSS 4 Aug 2009, 15:10
[/QUOTE]
ეგააა ჰიპოთერმია Osborne J-waves.[QUOTE]



smile.gif ძლივს არ ვნახე ცოცხალი ოსბორნის ტალღები smile.gif ბევრგან წამიკითხია მაგრამ ნანახი არ მქონდა smile.gif

Posted by: Romina 7 Aug 2009, 22:25
PHOBOSS

შენ მე ხეზე ცოცვა არ მასწავლე :-) არ მახსოვს რაღაც ოსბორნის ტალღები რომ გეხსენებინა, მეცადინეობების პერიოდში :-)

Posted by: PHOBOSS 7 Aug 2009, 22:32
QUOTE (Romina @ 7 Aug 2009, 22:25 )
PHOBOSS

შენ მე ხეზე ცოცვა არ მასწავლე :-) არ მახსოვს რაღაც ოსბორნის ტალღები რომ გეხსენებინა, მეცადინეობების პერიოდში :-)

ყველაფერს თავისი დრო აქვს smile.gif ჯერ "სიარული" გასწავლე, ცოტა ხანში "ირბენ" და "ხეზე ცოცვაზე" მერე გადავალთ smile.gif

Posted by: Romina 7 Aug 2009, 22:35
PHOBOSS
სიარულიც ფეხის წამოკვრით ვიცი სამწუხაროდ :-) კარგი, ვნახოთ მომავალში. ეხლა გავჩუმდეთ თორემ თემიდან გადახვევისთვის დაგვსჯიან :-)

Posted by: PHOBOSS 7 Aug 2009, 22:50
QUOTE (Romina @ 7 Aug 2009, 22:35 )
PHOBOSS
სიარულიც ფეხის წამოკვრით ვიცი სამწუხაროდ :-) კარგი, ვნახოთ მომავალში. ეხლა გავჩუმდეთ თორემ თემიდან გადახვევისთვის დაგვსჯიან :-)

რაღაც ძალიან მიძინებული გახდა ეს განყოფილება sad.gif იქნებ რამე შემთხვევა დადო თორე მგონი ყველანი დასასვენებლად არიან წასულები help.gif smile.gif შენთან უფრო ხშირადაა ეგეთი პაციენტები, ჩემთან ხო იცი 2+2=4 baby.gif

Posted by: Romina 8 Aug 2009, 09:01
b]PHOBOSS[/b]
კარგი დავდოთ ერთი ადვილი ქეისი :-)

პაციენტი, 45 წლის მამაკაცი, უჩივის: ცხელებას (38-39), ძლიერ საერთო სისუსტეს, კუნთების ტკივილს, ძლიერი ინტენსივობის ტკივილს წელის არეში, ოფლიანობას (უფრო მეტად ღამით), მარჯვენა მხარეს სათესლე ჯირკვლის შესიებას და ტკივილს. ავადაა დაახლოებით 5 თვეა. დაავადება დაიწყო ცხელებით და მზარდი საერთო სისუსტით, ცხელებას ტალღისებური ხასიათი ჰქონდა. შემდეგ დაემატა ზემოთ აღნიშნული სიმპტომები.

Posted by: Romina 8 Aug 2009, 09:06
ეს მეორე სურათი. არც ისე კარგად ჩანს და მე თვითონ ავხსნი. პირველზე სპონდილიტის სურათია, მეორეზე ცალმხრივი საკროილეიტი. აბა რა დაავადებაა? :-)
* * *
უკაცრავად პოსტი უაზროდ გავანაწილე :-(

Posted by: PHOBOSS 8 Aug 2009, 10:41
QUOTE (Romina @ 8 Aug 2009, 09:06 )
ეს მეორე სურათი. არც ისე კარგად ჩანს და მე თვითონ ავხსნი. პირველზე სპონდილიტის სურათია, მეორეზე ცალმხრივი საკროილეიტი. აბა რა დაავადებაა? :-)
* * *
უკაცრავად პოსტი უაზროდ გავანაწილე :-(

მაინც არ გადაუხვიე ხო შენი რეალური მოწოდებისგან? smile.gif smile.gif

Posted by: Romina 8 Aug 2009, 12:25
PHOBOSS
დაწერე პასუხი თუ იცი :-)

Posted by: Professor Xachikian 8 Aug 2009, 18:25
Romina
შემთხვევა ქრონიკულ ინფექციას გავს. ამ მონაცემებით რამე კონკრეტულის თქმა ძნელია.
კარგი იქნებოდა სისხლის საერთო და ბიოქიმიური ანალიზის, სისხლის და შარდის მიკრობიოლოგიური ანალიზის, სკროტუმის ულტრასაუნდის ცოდნა.
პაციენტის ცხელების ტალღისებური ხასიათით, საკროილიით (ალბათ ინფექციური) და სხვა სიმპტომებით, შეიძლება ვიფიქროთ ბრუცელოზზე (წინასწარი დიაგნოზი smile.gif)
* * *
P.S. მცდარი პასუხისათვის ინფექციონისტები ნუ ჩამქოლავთ biggrin.gif

Posted by: Romina 8 Aug 2009, 18:31
Professor Xachikian
bis.gif ეს შენ :-) ბრუცელოზია! ტალღისებრი ცხელება, ცალმხრივი საკროილეიტი და ცალმხრივი ეპიდიდიმო-ორქიტი ბრუცელოზის თითქმის კარდინალური ნიშანია :-) პაციენტი მწყემსი იყო, აზერბაიჯანელი. კიდე მეტალის გემოც ჰქონდა პირში და ოფლსაც დამახასიათებელი "ობის სუნი" ჰქონდა. მაგრამ ამის დაწერას უკვე დიაგნოზის დაწერა ჯობდა :-) ყოჩაღ!


* * *
PHOBOSS
QUOTE
მაინც არ გადაუხვიე ხო შენი რეალური მოწოდებისგან?  


რევმატოლოგია ჩემი პროფესიაა, უნფექციური დაავადებები გატაცება
biggrin.gif biggrin.gif
ვიცი რომ გამოიცანი. მოყოლილი მექნებოდა შენთვის ეს შემთხვევა :-)

Posted by: Professor Xachikian 8 Aug 2009, 18:36
Romina
კარგი ქეისი იყო. მადლობა
2kiss.gif

Posted by: Thandrus 9 Aug 2009, 00:21
მოკლედ, ვეღარცერთ ქეისს ვეღარ მოვუსწარი ამ ბოლო დროს რა biggrin.gif

Posted by: Professor Xachikian 15 Aug 2009, 17:49
32 წლის ქალბატონი მოვიდა ამბულატორიულ კლინიკაში შემდეგი სიმპტომებით: წონაში მატება დაახლოებით 12 კგ უკანასკნელი 1 წლის მანძილზე, სახეზე მცირედ გამოხატული ჭარბთმიანობა, კუნთების სისუსტე (გასინჯვით სისუსტე განსაკუთრებით გამოხატული იყო კიდურების პროქსიმალურ კუნთებში), სხეულის ფორმის შეცვლა – წონის მატება ძირითადად ტორსის არეში და შედარებით თხელი კიდურები, კანის გათხელება და მოწითალო სტრიები კანზე, მუცლის არეში. ბოლო დროს შეამჩნია კანის ადვილად დალურჯება უმნიშვნელო ტრავმების ფონზე.
ანამნეზში აქვს კარგად კონტროლირებული ჰიპერტენზია (ჰიდროქლოროთიაზიდზე) და დეპრესია (ფლუოქსეტინზე). ბოლო პერიოდში ჰიპერტენზიის მკურნალობისათვის დაჭირდა მეორე პრეპარატის შემატება, ასევე გაუმწვავდა დეპრესია. ეგზოგენურ სტეროიდებს (ანუ სტეროიდულ პრეპარატებს) არ იღებს.

დაახლოებით 3 წლის წინ ავტოავარიის გამო მსუბუქი დაზიანებებით პაციენტი მოხვდა საავადმყოფოს მომღებში, სადაც გაუკეთდა გულმკერდის და მუცლის კომპიუტერული ტომოგრაფია. ამ კვლევამ სერიოზული დაზიანებები არ გამოავლინა, მაგრამ აჩვენა მარცხენა თირკლელზადა ჯირკვლის 3.3 სმ ზომის სიმსივნე, ადენომის ტიპიური შესახედაობით (ერთგვაროვანი, კტ სიმკვრივე 5 ჰაუნსფილდის ერთეული). ამ პერიოდისათვის თირკლემზედა ჯირკვლის ჰორმონული ანალიზები ნორმაში იყო, სიმსივნეს ადენომის შესახედაობა ქონდა, და ხელმეორედ გაკეთებულ ტომოგრაფიაზე 3 თვის და 1 წლის შემდეგ, სიმსივნის (ადენომის) ზომა სტაბილური დარჩა (3.3 სმ). ამიტომ მისი ამოკვეთა არ მოხდა.

ზემოთ აღნიშნული სიმპტომების გამო მოხდა პაციენტის ხელახლა გამოკვლევა.
სისხლის საერთო და ბიოქიმიური ანალიზები (ელექტროლიტები, ღვიძლის ფუნქციური ტესტები და თირკმლის ფუნქცია) – ნორმა. ფარისებრი ჯირკვლის ფუნქციური ტესტები – ნორმა. ტესტოსტერონი – ნორმა, დიჰიდროეპიანდროსტერონ-სულფატი – ნორმის ზედა ზღვარი.

24-საათიანი შარდის ანალიზი კორტიზოლზე (გაზომილი HPLC მეთოდით)
1-ლი ანალიზი: 158 მკგ/24 სთ (ნორმა 5-50 მკგ/24სთ)
მე-2 ანალიზი: 78 მკგ/24 სთ
მე-3 ანალიზი: 167მკგ/24 სთ

სისხლის ანალიზი კორტიზოლზე და აკტჰ-ზე.
1-ლი ანალიზი:
კორტიზოლი 18 მკგ/დლ (ნორმა 3-20 მკგ/დლ), ადრენოკორტიკოტროპული ჰორმონი (აკტჰ) – 32 პგ/მლ (ნორმა 9-52 პგ/მლ)
მე-2 ანალიზი:
კორტიზოლი 25 მკგ/დლ, აკტჰ 21 პგ/დლ
მე-3 ანალიზი:
კორტიზოლი 23 მკგ/დლ, აკტჰ – 34 პგ/დლ

1. დიაგნოზი
2. მკურნალობა

Posted by: irakli222 15 Aug 2009, 18:51
1.კორტიკოსტერომა, ჰიპერკორტიციზმი
2.ადრენალექტომია

Posted by: Professor Xachikian 15 Aug 2009, 21:39
ცოტა კიდევ დაველოდოთ, დაიპოსტოს სხვა მოსაზრებებიც

Posted by: Romina 15 Aug 2009, 22:03
sad.gif sad.gif არ ვიცი. კლინიკურად გასაგებია რომ კუშინგის სინდრომი თუ დაავადებაა აღწერილი. მაგრამ ელექტროლიტები რომ ნორმის ფარგლებშია და აკტჰ, ეს მაბნევს. თუ არის ჰიპოფიზის სიმსივნე ან აკტჰ-ს სხვა ექტოპიური კერა, რატომ არ არის აკტჰ-ს დონე მაღალი? თუ თირკმელზედას ადენომაა რატომ არ არის დაბალი? smile.gif smile.gif მოკლედ ავიბლანდე საკუთარ აზრებში და პასუხი ძალიან მაინტერესებს smile.gif

Posted by: Professor Xachikian 15 Aug 2009, 22:10
QUOTE
1.კორტიკოსტერომა, ჰიპერკორტიციზმი 2.ადრენალექტომია

no.gif

Posted by: Romina 15 Aug 2009, 22:17
ხალხო აქ თითქმის ყველა დასასვენებლად არის წასული. მხოლოდ საკუთარ ბედზე ხელჩაქნეული, სამსახურს გადაყოლილი საწყალი ექიმები დარჩნენ biggrin.gif biggrin.gif ხოდა დიდი ხანი უნდა ველოდოთ კიდე სწორ დიაგნოზს?

Posted by: Professor Xachikian 15 Aug 2009, 22:22
Romina
QUOTE
კუშინგის სინდრომი თუ დაავადებაა აღწერილი

ნამდვილად ასეა, და ეს ბიოქიმიურადაც არაერთხელ დადასტურდა (შარდში მომატებული კორტიზოლი)
QUOTE
თუ თირკმელზედას ადენომაა რატომ არ არის დაბალი?

ზუსტად მაგაშია საქმე, და აქედან დაიწყო ამ შემთხვევის "ამოხსნა"
თითქოს ყველაფერი ნათელია: თირკმელზედა ჯირკვლის ადენომა, და კუშინგის სინდრომი.
მაგრამ სისხლის ბიოქიმიური ანალიზი არ ადასტურებს კლინიკურ დიაგნოზს: კორტიზოლ-გამომმუშავებელი თურკმელზედა ადენომების დროს აკტჰ, როგორც წესი, არის <10, უხშირესად კი <5. ამ პაციენტის შემთხვევაში ეს ასე არ იყო, და კვლევა სხვა მომართულებით წავიდა.
ბიოქიმიური ანალიზი უფრო შეესაბამება აკტჰ-დამოკიდებულ კუშინგის სინდრომს, კერძოდ კი კუშინგის დაავადებას (ჰიპოფიზით გამოწვეულ კუშინგის სინდრომს). ამ დროს აკტჰ ან ნორმაშია, ან შეიძშლება იყოს მომატებული.
ექტოპიური აკტჰ სეკრეციის დროს (ძირითადად არა-ჰიპოფიზის სიმსივნეებიდან) აკტჰ, როგორც წესი, არის ძალიან მაღალი.

ამიტომ პაციენტს გაუკეთდა თავის ტვინის MRI, ჰიპოფიზის პროტოკოლით.
კვლევამ გამოევლინა " 2მმ ზომის ჰიპოფიზის შესაძლო ადენომა ჰიპოფიზის მარცხენა წილში, თუმცა ზომის სიმცირის გამო დანამდვილებით ამის თქმა ძნელია"

გამოკვლევის შემდეგი ეტაპი:
* * *
QUOTE
ხალხო აქ თითქმის ყველა დასასვენებლად არის წასული. მხოლოდ საკუთარ ბედზე ხელჩაქნეული, სამსახურს გადაყოლილი საწყალი ექიმები დარჩნენ 

მარტო მე და თქვენ დავრჩით? lol.gif lol.gif lol.gif

Posted by: Thandrus 15 Aug 2009, 22:30
Professor Xachikian

QUOTE
ბიოქიმიური ანალიზი უფრო შეესაბამება აკტჰ-დამოკიდებულ კუშინგის სინდრომს, კერძოდ კი კუშინგის დაავადებას (ჰიპოფიზით გამოწვეულ კუშინგის სინდრომს). ამ დროს აკტჰ ან ნორმაშია, ან შეიძშლება იყოს მომატებული.


კაკ რაზ ვაპირებდი ამის დაწერას.


QUOTE
გამოკვლევის შემდეგი ეტაპი:


High-Dose Dexamethasone Suppression Test

Posted by: Romina 15 Aug 2009, 22:35
Professor Xachikian
გასაგებია smile.gif smile.gif მე მეგონა ჰიპოფიზით გამოწვეულ კუშინგის სინდრომს ახასიათებდა აკტჰ-ს მხოლოდ მაღალი დონე და არა ნორმა. საერთოდ თემას ძალიან კარგი საგანმანათლებლო დატვირთვა აქვს smile.gif

QUOTE
მარტო მე და თქვენ დავრჩით?   

ჩვენ და კიდევ ერთი-ორი ჩვენნაირი "თავგანწირული"
biggrin.gif biggrin.gif

Posted by: Professor Xachikian 15 Aug 2009, 22:35
Thandrus
QUOTE
High Dexamethasone Suppression Test

მართალია, ეგ ტესტი გამოიყენება კუშინგის დიფ-დიაგნოზისათვის, ანუ რომ გაარჩიო კუშინგის დაავადება და ექტოპიური აკტჰ სეკრეცია ერთმანეთისაგან.
აკტჰ ნორმაზე ზემოთ რომ ყოფილიყო, ალბათ ამ ტესტს გავაკეთებდით, მაგრამ ვინაიდან აკტჰ ნორმაში იყო, თანაც პაციენტს კლასიკური პრეზენტაცია ქონდა (ახალგაზდრა ქალი, კუშინგის ყველა სიმპტომით და ნიშნით), ამიტომ ეს ტესტი აღარ გაკეთდა

შემდეგი ტესტი:
* * *
QUOTE
ჩვენ და კიდევ ერთი-ორი ჩვენნაირი "თავგანწირული"
 

lol.gif lol.gif
QUOTE
საერთოდ თემას ძალიან კარგი საგანმანათლებლო დატვირთვა აქვს 

ნამდვილად. მეც აღვნიშნე ადრე

Posted by: Thandrus 15 Aug 2009, 22:43
Professor Xachikian

QUOTE
მართალია, ეგ ტესტი გამოიყენება კუშინგის დიფ-დიაგნოზისათვის, ანუ რომ გაარჩიო კუშინგის დაავადება და ექტოპიური აკტჰ სეკრეცია ერთმანეთისაგან.აკტჰ ნორმაზე ზემოთ რომ ყოფილიყო, ალბათ ამ ტესტს გავაკეთებდით, მაგრამ ვინაიდან აკტჰ ნორმაში იყო, თანაც პაციენტს კლასიკური პრეზენტაცია ქონდა (ახალგაზდრა ქალი, კუშინგის ყველა სიმპტომით და ნიშნით), ამიტომ ეს ტესტი აღარ გაკეთდა


რავიცი, ჰარისონის მიხედვით, ეს ტესტი საკმაოდ სპეციფიკურია ჰიპოფიზის მიკროადენომის მიმართ smile.gif

Posted by: Professor Xachikian 15 Aug 2009, 22:56
Thandrus
არა, ეს ტესტი გეხმარება დიაგნოზისათვის, მაგრამ ძალიან სპეციფიური არ არის. ამ ტესტის საერთო აკურატულობა (diagnostic accuracy) არ აღემატება 70-80%.
როცა კუშინგის ტიპიური შემთხვევაა, კუშინგის დაავადების (ანუ ჰიპოფიზის ადენომის) ალბათობა არის ~85-90%. ანუ: ამ ტესტის (High-dose Dexamethasone suppression test) pre-test probability არის კუშინდის დაავადებაზე ნაკლები. ამდენად, ბევრი ექსპერტი მიიჩნევს, ამ ტესტის გაკეთება საჭირო აღარაა.
თუმცა ძველი თაობის ენდოკრინოლოგები ამ ტესტს ტრადიციულად მაინც აკეთებენ.
ასეა თუ ისე, ეს ტესტი ამ პაციენტს არ გაუკეთდა

Posted by: Thandrus 15 Aug 2009, 22:58
ნუ, ამის მეტი Corticotropin-Releasing Hormone Infusion Test-ი და ბოლოს Petrosal Sinus Sampling-ი, სხვა მეტი რა...

მაგრამ მგონი დექსამეთაზონი უნდა ეყოს ამას biggrin.gif

Posted by: Professor Xachikian 15 Aug 2009, 23:05
Thandrus
QUOTE
Petrosal Sinus Sampling

up.gif
ფაქტიურად, ეს ტესტი უკვე ითვლება ე.წ. "gold standard" კუშინგის დაავადების დიაგნოზის დადასტურებისათვის.
პაციენტი გაიგზავნა ამ გამოკვლევაზე, რამაც დაადასტურა დიაგნოზი (მკვეთრი ლატერალიზაცია, აკტჰ გაცილებით მაღალი იყო მარცხენა მხარეს, და ბევრად უფრო მეტი, ვიდრა პლაზმაში გაზომილი აკტჰ)

პაციენტს გაუკეთდა ჰიპოფიზის ტრანს-სფენოიდური რეზექცია (მარცხენა წილის). საბოლოო, ჰისტოქიმიურმა გამოკვლევამ დაადასტურა:
აკტჰ-მაპროდუცირებელი ჰიპოფიზის ადენომა.
* * *
მგონი შეკითხვები აღარ დარჩა და შემთხვევა ამოიწურა
irakli222
Romina
Thandrus
მადლობა საინტერესო დისკუსიისათვის smile.gif

Posted by: LUKA-BRAZI 16 Aug 2009, 15:13
სალამი ყველას!

smile.gif

როგორ ხართ და რამე? biggrin.gif

Posted by: Romina 16 Aug 2009, 15:23
LUKA-BRAZI

კარგად, საწყლად რამდენიმე კაცი ვიყავით დარჩენილი და ამ თემას ვყარაულობდით, ეშმაკები რომ არ დაპატრონებოდნენ
biggrin.gif

Posted by: LUKA-BRAZI 16 Aug 2009, 15:37
QUOTE (Romina @ 16 Aug 2009, 15:23 )
LUKA-BRAZI

კარგად, საწყლად რამდენიმე კაცი ვიყავით დარჩენილი და ამ თემას ვყარაულობდით, ეშმაკები რომ არ დაპატრონებოდნენ
biggrin.gif

lol.gif lol.gif

ჰა, ჰა, ჰა ! მაგრად გამეცინა! smile.gif

კარგი გიქნიათ !

მოვალ აზრზე და მეც უფრო აქტიურად დავპოსტავ რამდენიმე დღეში smile.gif

Posted by: Professor Xachikian 16 Aug 2009, 17:43
65 წლის მამაკაცი შემოვიდა საავადმყოფოში მწვავე პანკრეატიტის დიაგნოზით. ანამნეზში აქვს დისლიპიდემია, ჰიპერტენზია და კარგად კონტროლირებული დიაბეტი.

საავადმყოფოში გაკეთებულმა ანალიზებმა აჩვენა მკვეთრად მომატებული ტრიგლიცერიდები (~900 მგ/დლ) დაბალი HDL და მაღალი LDL. სხვა ანალიზებმა აჩვენა ზომიერი ჰიპერკალცემია, კალციუმის დონით 10.7-11.8 მგ/დლ (ნორმა 8.5-10.5) ჰოსპიტალიზაციის განმავლობაში. ეს დადასტურდა გადამეორებული ანალიზებით, და პაციენტს ჩაუტარდა შემდგომი გამოკვლევები: PTH, მომატებული 70-110 პგ/დლ ფარგლებში (ნორმა 12-65). 24 საათიანი შარდის ანალიზი კალციუმზე – Ca-26 მგ/24 საათში (ნორმა 100-300მგ/24სთ), ვიტამინ D – ნორმაშია. სიმსივნური დაავადებები გამოირიცხა სათანადო ტესტებით.
პაციენტმა გაიხსენა რომ 5-6 წლის წინ ქონდა მწვავე პანკრეატიტის შეტევა მსუბუქი ფორმით. ახსოვს რომ კალციუმზე მაშინაც იყო რაღაც პრობლემები, მაგრამ სხვა დეტალები არ იცის.

დიეტის და მკურნალობის ფონზე პაციენტის მდგომარეობა საგძნობლად გაუმჯობესდა, ლიპიდების ცვლა საგრძნობლად გამოსწორდა და სტაბილური მდგომარეობით გაეწერა საავადმყოფოდან. ღებულობს შემდეგ პეპარატებს: ასპირინი, ატორვასტატინი, ფენოფიბრატი, მეტფორმინი, ენალაპრილი.

პაციენტი დაბრუნდა ამბულატორიულ კლინიკაში ჰიპერკალცემიის შემდგომი გამოკვლევისათვის. ყველა ანალიზი რამდენჯერმე გადამეორდა, და შედეგები მსგავსი იყო საავადმყოფოში გაკეთებული ანალიზებისა.
სავარაუდო დიაგნოზი

Posted by: Thandrus 16 Aug 2009, 23:30
პირველადი ჰიპერპარათიროიდიზმი გამორიცხულია? თუ ეს გამორიცხეთ (ანუ არც ადენომა და არც სხვა სიმსივნე არ იყო), მაშინ Familial Hypocalciuric Hypercalcemia შეიძლება იყოს... მართალია აქ ბავშვობიდან ეწყებათ ჰიპერკალცემია, მაგრამ აქაც არ იყო დაკოკრეტებული, რა ასაკიდან აწუხებს ეს ამ კაცს smile.gif

Posted by: Professor Xachikian 17 Aug 2009, 00:38
Thandrus
QUOTE
Familial Hypocalciuric Hypercalcemia

მართალია, ესაა yes.gif

* * *
კიდევ ერთი შემთხვევა ჰიპერკალცემიაზე:

54 წლის ქალბატონი მოვიდა ამბულატორიულ კლინიკაში შემდგომი გამოკვლევისათვის ჰიპერკალცემიაზე. კალციუმი სისხლში ზომიერადაა მომატებული 10.4-11.6 მგ/დლ ფარგლებში. შემდგომმა ანალიზებმა აჩვენა მომატებული PTH ~70-120 მგ/დლ ფარგლებში (ნორმა 12-65). ვიტამინი D, სისხლის ბიოქიმიური ანალზი, სისხლის საერთო ანალიზი – ნორმაშია. 24-საათიანი შარდის გამოკვლევამ აჩვენა Ca-18მგ/24სთ. (ნორმა 100-300მგ/დლ). ეს ანალიზები გადამეორდა, რამაც მსგავსი შედეგები აჩვენა.
პაციენტს ანამნეზში აქვს კარგად კონტროლირებული ჰიპერტენზია, დიაბეტი, დისლიპიდემია და სიმსუქნე. ღებულობს ატორვასტატინს, ლიზინოპრილს, ჰიდროქლოროთიაზიდს, მეტფორმინს და ასპირინს.
პაციენტი თავს კარგად გრძნობს, არაფერი არ აწუხებს, და ჰიპერკალცემია ნანახი იქნა ყოველწლიური რუტინული შემოწმების დროს.
სავარაუდო დიაგნოზი

Posted by: Thandrus 17 Aug 2009, 01:07
Professor Xachikian

QUOTE
ჰიდროქლოროთიაზიდს


ამის გამო ხომ არ არის? თიაზიდების მიღებას შეუძლია ჰიპერკალცემიის გამოწვევა...

Posted by: Professor Xachikian 17 Aug 2009, 01:15
Thandrus
bis.gif გნებდები!
ეგ არის. ამ ანალიზებმა კინაღამ შეცდომაში შეგვიყვანა. გავაჩერეთ ჰიდროქლირითიაზიდი, და ~1 კვირის შემდეგ გავიმეორეთ შარდის ანალიზი. შარდში კალციუმი ~200მგ/24სთ აღმოჩნდა.
პაციენტს ქონდა სუსტად გამოხატული ოსტეოპოროზი (T-score ბარძაყის ყელზე -2.7). ამიტომ გაიგზავნა ქირურგიულ კლინიაში. ოპერაციამდე გაკეთებულმა სცინტიგრაფიამ (ტექნეციუმის სკანირება) აჩვენა მარჯვენა/ქვედა პარაფარისებრი ადენომა. პაციენტს გაუკეთდა მინიმალურად-ინვაზიური პარათიროიდექტომია. პათანატომიური დიაგნოზი: ჰიპერცელულარული პარაფარისებრი ჯირკვალი, პარაფარისებრი ადენომა.

Posted by: Blind_Torture_Kill 18 Aug 2009, 17:10
სალამი ხალხო

ხოო
ამ ქეისებმა რაღაც ჩამაფიქრა
ცოდნას აფდეითი თუ არ გავუკეთე დავიძირები

Posted by: LUKA-BRAZI 18 Aug 2009, 17:59
Blind_Torture_Kill
სალამ! smile.gif

მეც ეგ გავიფიქრე ამ გვერდს რომ გადავხედე და ცუდ ხასიათზე დამაყენა.... რა ვქნათ? :|

Posted by: Blind_Torture_Kill 18 Aug 2009, 19:38
LUKA-BRAZI

თერაპიის NMS უნდა გადავხედო ყოველდღე

war.gif

Posted by: Thandrus 19 Aug 2009, 01:59
Blind_Torture_Kill
LUKA-BRAZI

ვაააჰ, სალამი smile.gif

როგორ ხართ, როგორ დაისვენეთ?

smile.gif

Posted by: Blind_Torture_Kill 19 Aug 2009, 09:02
Thandrus
ნორმალურად smile.gif
ახალ ახალ ქეისებს ველით

Posted by: LUKA-BRAZI 19 Aug 2009, 11:42
Blind_Torture_Kill
QUOTE
თერაპიის NMS უნდა გადავხედო ყოველდღე

კარგი იქნება! მარა მე რა ჯანდაბას გადავხედო?! ტატიშვილს? :| ისე ჩემთვის, დამოუკიდებლად რომ დავიწყო USMLE-ის კეთება, ვერ შვძლებ? მაინც და მაინც მასწავლებელი მჭირდება? რას ფიქრობ დათო? Thandrus?

Thandrus
გაუმარჯოს მეგობარო! smile.gif

Posted by: Thandrus 19 Aug 2009, 16:16
LUKA-BRAZI

QUOTE
ისე ჩემთვის, დამოუკიდებლად რომ დავიწყო USMLE-ის კეთება, ვერ შვძლებ? მაინც და მაინც მასწავლებელი მჭირდება? რას ფიქრობ დათო? Thandrus?


რავიცი, გააჩნია როგორი ტიპი ხარ, ზოგადად... თუ შეგიძლია მარტო მეცადინეობა და პროგრესი, თუ დამხმარე გჭირდება... ზარმაცი ხარ თუ პირიქით, მიზანმიმართული. მე მაინც გირჩევდი რომ ვიღაც გყოლოდა დამხმარედ, თუმცა ეგ ადამიანზე არის დამოკიდებული.

Posted by: Cousteau 19 Aug 2009, 18:00
QUOTE (LUKA-BRAZI @ 19 Aug 2009, 11:42 )
Blind_Torture_Kill
QUOTE
თერაპიის NMS უნდა გადავხედო ყოველდღე

კარგი იქნება! მარა მე რა ჯანდაბას გადავხედო?! ტატიშვილს? :| ისე ჩემთვის, დამოუკიდებლად რომ დავიწყო USMLE-ის კეთება, ვერ შვძლებ? მაინც და მაინც მასწავლებელი მჭირდება? რას ფიქრობ დათო? Thandrus?


when there is a will there is a way yes.gif

Posted by: LUKA-BRAZI 19 Aug 2009, 18:46
გილოცავთ ყველას ფერიცვალების ბრწინვალე დღესასწაულს! სულით და გულით გისურვებთ ყველას ყოველივე კარგს და საუკეთესოს!

Cousteau
QUOTE
when there is a will there is a way 

აბსოლიტურად სწორი ხარ, მაგრამ ჩემში სწორედ მაგ will-ის პრობლემაა გიგო; მაშინ რაც ვთქვი გარდიანთან რომ შევიკრიბეთ..... ისევ ისეთ გაურკვევლობაში ვარ!!!!!!!!!!!!!!!!!!!!!!!! weep.gif weep.gif თან მინდა, თან არ მინდა, თან ვფიქრობ შევძლებ თუ ვერ შევძლებ, ვმერყეობ......... უფრო მინდა, ვიდრე არ მინდა, მაგრამ ეს "არ მინდა"-ც რომ ხშირად დიდ ადგილს იკავებს? smile.gif საკუთარ თავზე მეცინება smile.gif) დასაცინი ვარ აბა რა smile.gif) კიდე კარგი გიო აღარ პოსტავს ფორუმზე და ამ პოსტს არ წაიკითხავს :|

შენ რას შვები სტეპების პონტში და რრზიდენტურაზე? PM-ში მომწერე თუ გინდა (ეგრეც ჯობია)

smile.gif

Posted by: Cousteau 19 Aug 2009, 19:18
QUOTE (LUKA-BRAZI @ 19 Aug 2009, 18:46 )
Cousteau
QUOTE
when there is a will there is a way 

აბსოლიტურად სწორი ხარ, მაგრამ ჩემში სწორედ მაგ will-ის პრობლემაა გიგო; მაშინ რაც ვთქვი გარდიანთან რომ შევიკრიბეთ..... ისევ ისეთ გაურკვევლობაში ვარ!!!!!!!!!!!!!!!!!!!!!!!! weep.gif weep.gif თან მინდა, თან არ მინდა, თან ვფიქრობ შევძლებ თუ ვერ შევძლებ, ვმერყეობ......... უფრო მინდა, ვიდრე არ მინდა, მაგრამ ეს "არ მინდა"-ც რომ ხშირად დიდ ადგილს იკავებს? smile.gif საკუთარ თავზე მეცინება smile.gif) დასაცინი ვარ აბა რა smile.gif) კიდე კარგი გიო აღარ პოსტავს ფორუმზე და ამ პოსტს არ წაიკითხავს :|

შენ რას შვები სტეპების პონტში და რრზიდენტურაზე? PM-ში მომწერე თუ გინდა (ეგრეც ჯობია)

smile.gif

წაიკითხავს : ) ათვალიერებს ხოლმე yes.gif

რაც გინდა ეგ უნდა გააკეთო yes.gif
თუ გინდა მაშინ შეძლებ, თუ არ გინდა მაშინ უფრო რო არ შეძლებ (თუმცა შეიძლება მაინც შეძლო)
თუ არ იცი რა გინდა, მოითმინე... ოღონდ ეს ''მოითმინეს'' დრო ტყუილად არ დახარჯო - იმეცადინე, პრაქტიკულად შეხედე იმას რა და როგორ მუშაობს,რა პროფესიას რა სპეციფიკა აქვს, სანამ რეალურად არ შეხედავ იქამდე წარმოდგენა არ გექნება რა რა არის.

ჩემი ერთ-ერთი ყველაზე დიდი შეცდომა იყო რომ ავირჩიე დარგი რომელზეც როგორც ბოლოს აღმოჩნდა საკმაოდ აბსტრაქტული წარმოდგენა მქონდა, ის რომ ვინმე ნაცნობი თუ გყავს საავადმყოფოში თხოვე რომ ყველა განყოფილებაში თითო კვირა მაინც გაგატარებინოს
ქირურგიაში, ტრავმაში, IM-ში, ED-ში, ICU-ში, ეს აუცილებლად უნდა გააკეთო რომ რეალური წარმოდგენა შეგექმნას რად მუშაობა ღირს და რად არა. ეს რათქმაუნდა იმ შემთხვევაში თუ პირველი კურსიდან რაღაც არ გინდოდა (თუ შენ თავს რაღაც კონკრეტულში ხედავ, მაშინ ამ ''კონკრეტულს'' მიეჩვიე)

რაც სტეპს ეხება, კიდევ გამოცდის ჩაბარება (და მითუმეტეს მაღალ ქულაზე) ძნელია (თან ძალიან-ძალიან ძნელი) მაგრამ შეუძლებელი არ არის, ყველაზე მთავარი არის დიდი მონდომება.

ნათიასგან ვიცი რომ აკადემიურობის მხვრივ მისაბაძი სტუდენტიხარ : ) ისერო go ahead, რასაც მოინდომებ ყველაფერი გამოგივა yes.gif

Posted by: LUKA-BRAZI 19 Aug 2009, 22:30
Cousteau
QUOTE
წაიკითხავს : ) ათვალიერებს ხოლმე 

გამარჯობა გარდიან! smile.gif
QUOTE
თუ გინდა მაშინ შეძლებ, თუ არ გინდა მაშინ უფრო რო არ შეძლებ (თუმცა შეიძლება მაინც შეძლო)
თუ არ იცი რა გინდა, მოითმინე... ოღონდ ეს ''მოითმინეს'' დრო ტყუილად არ დახარჯო - იმეცადინე

მართალი ხარ ყველანაირად. დავიწყებ და გზადაგზა გამოიკვეთება რაღაც-რაღაცეები.
QUOTE
რაც სტეპს ეხება, კიდევ გამოცდის ჩაბარება (და მითუმეტეს მაღალ ქულაზე) ძნელია (თან ძალიან-ძალიან ძნელი) მაგრამ შეუძლებელი არ არის, ყველაზე მთავარი არის დიდი მონდომება.

All the luck to you! smile.gif
QUOTE
ნათიასგან ვიცი რომ აკადემიურობის მხვრივ მისაბაძი სტუდენტიხარ : ) ისერო go ahead, რასაც მოინდომებ ყველაფერი გამოგივა

ნათიაზეც იგივეს ვიტყვი ყოველგვარი გადაჭარბების გარეშე.
QUOTE
ED

ეს რას ნიშნავს კუსტო?
smile.gif

Posted by: basa-ttt 20 Aug 2009, 00:06
QUOTE
თან მინდა, თან არ მინდა, თან ვფიქრობ შევძლებ თუ ვერ შევძლებ, ვმერყეობ......... უფრო მინდა, ვიდრე არ მინდა, მაგრამ ეს "არ მინდა"-ც რომ ხშირად დიდ ადგილს იკავებს?  საკუთარ თავზე მეცინება ) დასაცინი ვარ აბა რა


ესეც ახალი ქეისი
lol.gif

Posted by: LUKA-BRAZI 20 Aug 2009, 00:42
lol.gif

აბა ვინ გამოიცნობს?!

მოდერატორი რომ არ გაგვიწყრეს დავდებ ქეისს (სახელდახელოდ) biggrin.gif

პაციენტს აღენიშნება სიცხე, გამონადენი ცხვირიდან, ხახა შეწითლებულია, აღენიშნება ფხაჭნის შეგრძნება ყლაპვის დროს, სუნთქვა გაძნელებულია, აქვს თავის ტკივილი, უჩივის შრომის უნდარის დაქვეითებას.

დიაგნოზი.

smile.gif

Posted by: Thandrus 20 Aug 2009, 00:50
ვირუსული ინფექცია smile.gif

...........

Posted by: Cousteau 20 Aug 2009, 01:38
QUOTE (LUKA-BRAZI @ 19 Aug 2009, 22:30 )

QUOTE
ED

ეს რას ნიშნავს კუსტო?
smile.gif

Emergency Deparment


QUOTE
ესეც ახალი ქეისი

Role Confusion - Erik Eriksson Development stage 5 problem : D : P (ხუმრობით რათქმაუნდა)

Posted by: Thandrus 20 Aug 2009, 03:06
მოკლედ, აი ერთი ქეისი smile.gif

14 წლის გოგო მოიყვანეს საავადმყოფოში. მას 2 კვირის განმავლობაში სტკიოდა მარცხენა ყური და ჰქონდა ყურიდან ჩირქოვანი გამონადენი. საავადმყოფოში მოყვანამდე 10 დღის განმავლობაში სვამდა ანტიბიოტიკს (უცნობია), რომელიც მას ექიმმა გამოუწერა ყურიდან სისხლჩირქოვანი გამონადენის, თავის, საფეთქლის მიდამოს და თავის ტკვილის გამო. ბავშვს სმენა არ აქვს დაქვეითებული და ყურში არ არის უცხო სხეული. გაკეთებული აქვს ყველა სტანდარტული აცრა და ალერგიული არ არის.

საავადმყოფოში შემოწმებისას მას აღენიშნება: ტემპერატურა 36,6 'C, გულისცემის სიხშირე 86bpm, სუნთქვის სიხშირე 16 br/min. წნევა 110 / 80. სისხლჩირქოვან გამონადენთან ერთადა, აღენიშნება ტკივილი და შესიება ყურის მიდამოში. მას არ შეუძლია შუბლის მარცხენა ნაწილის შეჭმუხვნა და მარცხენა თვალის დახუჭვა. Nasolabia fold-ი სწორია და პირი მარჯვნივაა "მოქცეული". გამოკვლევის დანარჩენი ნაწილი ნორმალურია.

თავდაპირველი ლაბორატორიული ანალიზები (CBC და basic metabolic panel) ნორმალურია. CT-მ დაადგინა მარცხენამხრივი მასტოიდიტი, მაგრამ ინტრაკრანიალური წნევა აწეული არ არის. CSF-ში არ არის აღმოჩენილი სისხლის თეთრი ან წითელი უჯრედების მნიშვნელოვანი რაოდენობა და არც გლუკოზისა და ცილის კონცენტრაციაა ნორმიდან დიდად გადახრილი. CSF გაიგზავნაა დასათესად.

პაციენტის თავდაპირველი მკურნალობა იყო IV ხსნარი, ტკივილგამაყუჩებლები, ტიმპანოსტომია (დრენაჟისა და კულტურისთვის), ინტრავენული მეროპენემი.

საავადმყოფოში მისვლიდან მეორე დღეს ბავშვის სიმპტომებს დაემატა: მარცხენა ორბიტალური ტკივილი, დიპლოპია და მარცხენა თვალის აბდუქციის უნარის დაკარგვა. გადაიღეს MRI.

დაწერეთ თავდაპირველი დიაგნოზი და ჰოსპიტალში მიყვანის მეორე დღეს განვითარებული გართულების (სინდრომის) სახელწოდება

აი, MRI-ც, თუ გამოგადგებათ:

http://images.beef.ge

http://images.beef.ge

Posted by: Professor Xachikian 20 Aug 2009, 05:32
თავდაპირველი დიაგნოზი – რაც კტ-ზე გამოჩნდა, ალბათ ის, ანუ მარცხენა მასტოიდიტი. როგორც წესი, იწყება შუა ყურის ანთებით.

გართულება – ლატერალური სინუსის სეპტიური თრომბოზი, ასეთი კლინიკური მანიფესტაციით aka გრადენიგო-ს სინდრომი.

ტემპერატურა, ლეიკოციტები და თავ-ზურგტვინის სითხის ანალიზი (თითქმის) ნორმაში – ალბათ ჰოსპიტალიზაციამდე ანტიბიოტიკით მკურნალობის გამო.

Posted by: Thandrus 20 Aug 2009, 05:47
Professor Xachikian

აბსოლუტურად მართალია! up.gif up.gif up.gif

თავიდან ოტიტის მედია, შემდეგ, არასწორი ანტიბიოტიკით მკურნალობის ფონზე გართულება - გრადენიგოს სინდრომი ( =აბდუცენს ნერვის პარალიზი, რეტროორბიტალური ტკივილი, ოტიტის მედია)

კულტურამ შემდეგ Stenotrophomonas maltophilia ამოთესა, რომელიც ტრიმეტოპრიმ-სულფომეტოქსაზოლისადმი იყო სენსიტიური. ეს წამალი (ინტრავენულად) ექვსი კვირის განმავლობაში დაენიშნა + ფიზიოთერაპია აბდუცენსის პარალიზის მოსახსნელად. რამდენიმე თვეში ბავშვი სრულიად გამოჯანმრთელდა.

RESPECT!

Posted by: Professor Xachikian 20 Aug 2009, 05:51
Thandrus
QUOTE
RESPECT!

დიდი მადლობა smile.gif

Posted by: LUKA-BRAZI 20 Aug 2009, 09:43
Thandrus
QUOTE
ვირუსული ინფექცია

ჰო, გრიპი biggrin.gif
Cousteau
QUOTE
Role Confusion - Erik Eriksson Development stage 5 problem

lol.gif მარა მაგ ასაკში რომ აღარ ვარ? smile.gif (12-18 yy)
Professor Xachikian
well done smile.gif

Posted by: Professor Xachikian 22 Aug 2009, 03:30
23 წლის ჯანმრთელი მამკაცი, სამხედრო მოსამსახურე, სამედიცინო პრობლემების გარეშე. წამლებს არ იღებს.
დაახლოებით 4 თვის წინ დაეწყო საერთო სისუსტე, გულის ფრიალი, ცხელი ტემპერატურის აუტანლობა და ჭარბი ოფლიანობა. სპონტანურად დაიკლო ~12 კგ, დიეტის შეცვლის გარეშე.
ბოლო თვის განმავლობაში შეამჩნია კუნთების ძლიერი სისუსტე, რაც ხელს უშლის ყოველდღიური მუშაობის შესრულებაში. ერთ დღეს, გაღვიძების შემდეგ, შეამჩნია რომ მარჯვენა ქვედა კუდურს ვერ ამოძრავებს. ცოტა ხანში ძლიერი სისუსტე განვითადრა ორივე მხარეს, ქვედა და ზედა კიდურებზე. ამოძრავებს მხოლოდ თითებს. გონება და მეტყველების უნარი არ დაუკარგავს. მხედველობის გაორებას არ აღნიშნავს. მეუღლემ გამიოძახა სასწრაფო და პაციენტი გადაყვანილი იქნა საავადმყოფოში.
სავარაუდო დიაგნოზი:

Posted by: Thandrus 22 Aug 2009, 12:33
Professor Xachikian

კალიუმის კონცენტრაცია როგორია, დაბალი ხომ არ აქვს?

მოცემული სიმპტომები თირეოტოკსიკოზს შეესაბამება. თუ პარალიზი მასთანაა ასოცირებული, შეიძლება Hypokalemic (Thyrotoxic) periodic paralysis იყოს.

Posted by: Professor Xachikian 22 Aug 2009, 16:15
Thandrus
up.gif up.gif
საავადმყოფოში მიყვანის დროს K-1.9 მლექ/ლ (ნორმა 3.5-5.5). TSH <0.001 (ნორმა 0.4-4.5) FT4 - 4.2 ნგ/დლ (ნორმა 0.8-1.8) FT3 - ~1000 (ნორმა 200-420). ფარისებრი ჯირჯვალი მკვრივი და ოდნავ გადიდებული; მცირედ გამოხატული ეგზოფთალმი.
დიაგნოზი:
1.
QUOTE
Hypokalemic (Thyrotoxic) periodic paralysis

2. გრეივსის დაავადება, თიროტოქსიკოზი.

საავადმყოფოში ინტრავენურად გადაესხა კალიუმის ქლორიდი (40 მლექ) და დაენიშნა პერ-ორალური კალიუმის კლორიდი 10 მლექ ორჯერ დღეში.
ამბულატორიულ კლინიკაში პაციენტს დაენიშნა პროპრანოლოლი 20 მგ 2-ჯერ დღეში და თიროზოლი (მეთიმაზოლი) 30 მგ ერთხელ დღეში. კუნთების სისუსტე მთლიანად მოიხსნა, კალიუმიც ნორმაშია შენარჩუნბული.
პაციენტმა მიიღო გადაწყვეტილება რომ იმკურნალოს რადიოაქტიური იოდით, როცა მდგომარეობა დასტაბილურდება.
* * *




ეს შემთხვევა "ასეთებიც ხდება"-ს კატეგორიას მიეკუთვნება. საინტერესოა იმდენად, რამდენადაც ასეთი პაციენტები არცთუ ისე ხშირად, მაგრამ მაინც გვხვდება კლინიკურ პრაქტიკაში.


67 წლის მამაკაცი შემოვიდა მიმღებში ჰიპოგლიკემიის ტიპიური სიმპტომებით. სიმპტომები დაეწყო 2-3 დღის წინ, ძირითადად დღის საათებში. სისხლში გლუკოზა 26 მგ/დლ. სხვა ანალიზები – ნორმაში. გლიკოჰემოგლობინი – 5.4%. სისხლი გაიგზავნა სულფონილურეაზე (ტოქსიკოლოგიური სკრინინგი), და ასევე ც-პეპტიდზე, ინსულინზე და პროინსულინზე. ანამნეზში აქვს ართრიტი და ეპილეფსია, ღებულობს აცეტამინოფენს (1-2-ჯერ დღეში) და კარბამაზეპინს (3-ჯერ დღეში). პაციენტი ანტიდიაბეტურ პრეპარატებს არ იღებს, და სამედიცინო სფეროსთან არანაირი შეხება არა აქვს (პროფესიით – გადამდგარი ბუღალტერი)

პაციენტი მიღებული იქნა საავადმყოფოში, და დაენიშნა სათანადო მკურნალობა. თირკლემზედა ჯირკვლის უკმარისობა და ფარისებრი ჯირკვლის პრობლემები გამოირიცხა. სიმსივნური დაავადებები, თირკმლის და ღვიძლის პრობლემები არა აქვს. ჰიპოგიკემია მალე გამოსწორდა, და 24 საათში პაციენტი სახლში გაეწერა სტაბილური მდგომარეობით.

რამდენიმე დღეში პაციენტი ისევ შემოვიდა მიმღებში მსგავსი კლინიკური პრეზენტაციით. ამჯერად გლუკოზა იყო 32მგ/დლ. სხვა ანალიზები – ნორმაში. ამ დროისათვის უკვე მოვიდა წინა ჰისპიტალიზაციის დროს გაკეთებული სისხლის ანალიზის პასუხები. ც-პეპტიდი, ინსულინი და პროინსულინი მომატებული (დაახლოებით 2-3-ჯერ ნორმაზე მეტი, ჰიპოგლიკემიის ფონზე).
პაციენტი ისევ მიღებული იქნა საავადმყოფოში. ჰიპოგლიკემია მოიხსნა, და მეორე დღეს გაეწერა სახლში. ამ ჰოსპიტალიზაციის დროსაც განმეორდა ტოქსიკოლოგიური სკრინინგი და ანალიზები ც-პეპტიდზე, ინსულინზე და პროინსულინზე.

რამდენიმე დღეში განმეორდა იგივე კლინიკა. ამ დროს პაციენტი იმყოფებოდა სხვა ქალაქში, და მიღებული იქნა ადგილობრივ საავადმყოფოში. წინა ჰოსპიტალიზაციის დროს მიღებული ანალიზის პასუხები დაახლოებით იგივე იყო: ც-პეპტიდი, ინსულინი და პროინსულინი მომატებული ჰიპოგლიკემიის ფონზე. 1-ლი და მე-2 თაობის სულფონილურეაზე ტოქსიკოლოგიური სკრინინგი უარყოფითია.

პაციენტის ჰიპოგლიკემიური ისტორიის გათვალისწინებით, პაცინეტი დარჩა საავადმყოფოში 72-საათიანი შიმშილის ტესტის გასაკეთებლად. ამ ტესტის დროს არცერთი ჰიპოგლიკემიური ეპიზოდი არ აღნიშნულა.
სავარაუდო დიაგნოზი:

Posted by: Mrs_Zum 22 Aug 2009, 18:02
ინსულომა ვითომ?

...........................

Posted by: Thandrus 23 Aug 2009, 00:49
უცნაური შემთხვევაა... ც-პეპტიდი, ინსულინი და პროინსულინი მომატებულია... ეს მართლაც ინსულინომაზე მეტყველებს. მაგრამ, თუ სიმსივნური დაავადებები მართლაც გამოირიცხა და პაციენტს ჰიპოგლიკემიური შეტევები საავადმყოფოში აღარ უვითარდება, ესეიგი ის თვითონ იმართავს ამ ჰიპოგლიკემიაას...

როგორც ვიცი, ეგზოგენური ინსულინით გამოწვეული ჰიპოგლიკემიის დროს ც-პეპტიდი დაბალია ხოლმე. რადგან ინსულინის პრეპარატებში ც-პეპტიდი არაა. რავიცი, ხომ არ იღებს ეს პაციენტი უხარისხო, არაგასუფთავებულ ინსულინს, სადაც ც-პეპტიდი არაა მოცილებული? სხვა ახსნა ვერ მოვუნახე givi.gif

Posted by: Mrs_Zum 23 Aug 2009, 01:56
ისა და შეიძლება აბსურდია, მაგრამ celiac disease ხომ არ აქვს?

Posted by: Thandrus 23 Aug 2009, 02:26
მოიცა და... ტოქსიკოლოგიური ტესტი მხოლოდ სულფონილურეებზე ჩატარდა? user.gif იქნებ და სხვა კლასის წამლით იყო გამოწვეული (მაგ. მეგლიტინიდების)? კიდევ ერთ ვერსიას დავწერ: რეპაგლინიდით გამოწვეული ჰიპოგლიკემია. (ჩემს წინა შიზოიდურ ვერსიაზე უკეთესი ალბათ არის lol.gif)

Posted by: Professor Xachikian 23 Aug 2009, 03:05
Thandrus
QUOTE
იქნებ და სხვა კლასის წამლით იყო გამოწვეული (მაგ. მეგლიტინიდების)?

up.gif up.gif up.gif
მართალია. ჰიპოგლიკემია გამოწვეული იყო რეპაგლინიდით (იგივე პრანდინი, ნოვონორმი)

პაციენტის კლინიკური პრეზენტაციიდან და ანალიზებიდან ცხადი იყო, რომ ჰიპოგლიკემია ვითარდებოდა მხოლოდ სახლში ყოფნის დროს, ჰოსპიტალიზაციისას კი ყველაფერი წესრიგდებოდა. ანუ: პაციენტი სახლში იღებდა რაღაც ისეთს, რაც იწვევდა ჰიპოგლიკემიას. ერთი დეტალი, რაც შეგნებულად გამოვტოვე იყო ის, რომ პაციენტის მეუღლე იყო დიაბეტიკი, და ღებულობდა გლიბენკლამიდს (იგივე გლიბურიდი). ამ ინფორმაციაზე დაყრდნობით, როცა ეს შემთხვევა მოხსენდა, კათედრის ყველა თანამშრომლის დიაგნოზი, უფრო სწორად კი ვერდიქტი, იყო ერთი: "ამას ნაღდად ცოლი წამლავს" .

ცოტა დამაბნეველი გამოდგა სულფონილურეაზე ტოქსიკოლოგიური ანალიზის უარყოფითი პასუხი. კათედრის ერთსულოვანი "დიაგნოზი" არ გამართლდა. ამიტომ, პაციენტის ვთხოვეთ მოეტანა წამლები, რომელსაც სახლში იღებდა.

როცა კარბამაზეპინი შეიძინა, პაციენტი მიხვდა რომ სხვანაირი ტაბლეტები ეყარა კარბამაზეპინის ბოთლში, მაგრამ შეცდომა ვერანაირად ვერ დაუშვა: უბრალოდ იფიქრა რომ სხვა ბრენდის კარბამაზეპინი მისცეს, რაც ხშირად ხდება, ადრეც ქონია ასეთი შემთხვევა, და ამისათვის დიდი ყურადღება არ მიუქცევია.
ტაბლეტის ანალიზი მოხდა საანადმყოფოს აფთიაქში. ყველა ტაბლეტს აქვს კოდი - ასოების და ციფრების კომბინაცია - რითაც შესაძლებელია მათი იდენტიფიკაცია. აღმოჩნდა, რომ კარბამაზეპინის ბოთლში იყო რეპაგლინიდი: აფთიაქის შეცდომა.
ეს შემთხვევა 6 წლის წინანდელია, მაშინ მეგლიტინიდები ჯერ კიდევ "ახალი ხილი" იყო, და ტოქსიკოლოგიურ სკრინში რუტინულად არ მოწმდებოდა. ეხლა ე.წ. ჰიპოგლიკემიურ პანელში ლაბორატორიების უმრავლესობა ზომავს სულფონილურეას და მეგლიტინიდებს.

Posted by: Thandrus 23 Aug 2009, 04:25
საინტერესოა ნამდვილად! "ჰაუსის" სერიადაც კი წავიდოდა ეს ქეისი biggrin.gif ისე, თუ ამდენი ხანი კარბამაზეპინის მაგივრად რეპაგლინიდს სვამდა, კრუნჩხვები არ ემართებოდა?

Posted by: Professor Xachikian 23 Aug 2009, 04:33
QUOTE
თუ ამდენი ხანი კარბამაზეპინის მაგივრად რეპაგლინიდს სვამდა, კრუნჩხვები არ ემართებოდა

კარგი შეკითხვაა. ამ პაციენტს ეპილეფსია ქონდა ძალიან დიდი ხნის განმავლობაში, და წლების მანძილზე სტაბილური იყო კარბამაზეპინით მკურნალობის ფონზე. იმ დაახლოებით ორი კვირის განმავლობაში, სანამ ჰიპოგლიკემიის მიზეზი გაირკვეოდა, არც ერთი კრუნჩხვა არ ქონია.
* * *
კიდევ ერთი ჰიპოგლიკემია:

13 წლის გოგონა, 1-ლი ტიპის დიაბეტით 3 წლის ასაკიდან. ღებულობს ინსულინ NPH-ს (ინსულატარდი) და ხანმოკლე მოქმედების ინსულინს – Regular (აქტრაპიდი) დღეში 2-ჯერ - საუზმის წინ და ვახშმის წინ.
ბოლო რამდენიმე თვის განმავლობაში ბავშვს დაეწყო მძიმე ჰიპოგლიკემიური შეტევები, ძირითადად ღამით ან ადრეული დილის საათებში, საკვების მიღებამდე. ჰიპოგლიკემური ეპიზიდები სპორადულია და ყოველდღე არ ხდება. რამდენჯერმე მშობლებს დაჭირდათ სასწრაფოს გამოძახება. ჰიპოგლიკემიური ეპიზოდების დრის გლუკოზა 20-35 მგ/დლ ფარგლებში. დღის განმავლობაში გლუკოზა ~100-300 მგ/დლ ფარგლებში.
ოჯახის ექიმმა პაციენტი გამოიკვლია, და ურჩია რომ მთლიანად გაჩერებინა ინსულინი. პაცინეტს დიაბეტის ქრონიკული გართულებები არა აქვს. გამოირიცხა თირკმლის, ღვიძლის, ფარისებრი და თირკმელზედა ჯირკვლის პრობლემები.
რჩევის მიხედვით, პაციენტი ინსულინს აღარ იღებს. ინსულინის გაჩერების შემდეგ ჰიპოგლიკემიური ეპიზოდები გაგრძელდა. პაციენტს გაუკეთდა ც-პეპტიდი, რაც აღმოჩნდა <0.5 ნგ/მლ. პროინსულინი არ გაკეთებულა. ინსულინი - ~3-ჯერ მეტი ნორმაზე. გლიკოჰემოგლიბინი – 9.1%. ამ ანალიზების საფუძველზე მშობლებს ეთქვათ რომ ბავშვს ალბათ აქვს “ძალიან იშვიათი სიმსივნე” (პაციენტის მშობლები ვერ აკონკრეტებენ), და შემდგომი გამოკვლევისათვის პაციენტი გაიგზავნა ენდოკრინოლოგიურ კლინიკაში.
სანარაუდო დიაგნოზი:



* * *
------------------------------------------------------------------------------------------------------------------------------------------------

Posted by: Dru 23 Aug 2009, 18:24
ვითომ ინსულინომა შეიძლება დაემართოს 13 წლის ბავშვს?

Posted by: Thandrus 23 Aug 2009, 20:34
Professor Xachikian

ჰმ... ანუ სიმსივნეები უნდა ვეძებოთ? ინსულინომას არ ჰგავს, ეს რომ იყოს, ც-პეპტიდიც მომატებული იქნებოდა. მაშინ რჩება ან ინსულინის ექტოპიური წარმოქმნა (რაც უიშვიათესია). მაგრამ მაშინ ც-პეპტიდს არ ვიცი, რა მოსდის ან ჰიპოგლიკემია როგორც პარანეოპლაზიური სინდრომი...


Posted by: Professor Xachikian 23 Aug 2009, 21:02
QUOTE
ექტოპიური წარმოქმნა (რაც უიშვიათესია)

yes.gif
QUOTE
ინსულინომას არ ჰგავს, ეს რომ იყოს, ც-პეპტიდიც მომატებული იქნებოდა

yes.gif
QUOTE
ანუ სიმსივნეები უნდა ვეძებოთ?

სიმსივნეები (ხშირად მეზენქიმური წარმოშობის) იწვევს ჰიპოგლიკემიას. ასეთ შემთხვევებში ქრონიკული ჰიპოგლიკემიაა ხოლმე, რასაც ეწინააღმდეგება ამ პაციენტის შემთხვევაში მომატებული გლიკოჰემოგლობინი (Hgb A1c), და ჰიპოგლიკემიის შემთხვევითობა მხოლოდ ღამით.
სიმსივნით გამოწვეული ჰიპოგლიკემია ძირითადად ხდება ტერმინალურ სტადიაში. ბავშივი კი თავს ძირითადად კარგად გრძნობდა. მიუხედავად ამისა, ოჯახის ექიმმა პაციენტს გაუკეთა სხეულის კტ. სიმსივნე ნანახი არ იქნა. ენდოკრინოლოგიურ კლინკაში გაკეთდა IGF-II (insulin-like growth factor II) რაც უარყოფითი აღმოჩნდა.

Posted by: Thandrus 23 Aug 2009, 21:08
Professor Xachikian

ანუ, ესეიგი, ექიმების თავდაპირველი დიაგნოზი მცდარი იყო და სიმსივნე არ არის?

Posted by: Professor Xachikian 23 Aug 2009, 21:12
Thandrus
ფაქტიურად, ოჯახის ექიმმა რაც ივარაუდა "ძალიან იშვიათი სიმსივნე" - ნაწილობრივ თვითონვე გამორიცხა: გააკეთა სხეულის კტ. ინსულინომა კტ-ზე ხშირად არ ჩანს, მაგრამ ინსულინომას გამორისხავს ბიოქიმიური ტესტი: ც-პეპტიდი <0.5. პარანეოპლასტიური სინდრომიც არ არის: უარყოფითი IGF-II

Posted by: Thandrus 23 Aug 2009, 21:18
აჰა... ნუ მაშინ გასაგებია, სხვა დიაგნოზია საძებნი smile.gif

ინსულინის რეცეპტორის მიმართ ხომ არაა ანტისხეულები? ამ დროს ინსულინის მეტაბოლიზმი შეიძლება შემცირდეს, რაც მოგვცემს ჰიპერინსულინემიას. ჰიპერინსულინემია კი ინსულინის წარმოქმნას დაწევს დაბლა, რაც ც-პეპტიდის შემცირებას მოგვცემს.

Posted by: Professor Xachikian 23 Aug 2009, 21:31
Thandrus
ანტისხეულები პაციენტების უმრავლესობას უნვითარდება, ვინც ინსულნის იღებს. ამიტომ ანტისხეულები არ გაზომილა. როგორც წესი, ეს ანტისხეულები პრობლემას არ ქმნის, განსაკუთრებით მას შემდეგ, რაც ადამიანის რეკომბინირებული ინსულინები შემოვიდა ფართო ხმარებაში.

მართალი განცხადებაა, და ამაზე ამ თემის წინა გვედრებზეც იყო საუბარი: თუ ჰიპოგლიკემიის ფონზე გაიზომა ც-პეპტიდი, მაშინ ც-პეპტიდი დაბალია (არ იგულისხმება ინსულინომა და სულფონილურეათი მოწამვლა). ცხადია, ამ პაციენტს კიდევ გაუკეთდა ეს ანალიზი, ჰიპერგლკემიის ფონზე (გლუკოზა ~200) რამაც ერთი და იგივე შედები მოგვცა: ც-პეპტიდი <0.5.
ჰიპერინსულინემიამ თავისთავად შეიძლება დათრგუნოს ც-პეპტიდის გამომუშავება, მაგრამ იმ შემთხვევაში, თუ გლუკოზა ნორმაშია ან დაბალია. მაგრამ ამ პაციენტს 1-ლი ტიპის დიაბეტი კარგა ხანია აქვს, და ამიტომაა ც-პეპტიდი ძალიან დაბალი.

Posted by: Blind_Torture_Kill 23 Aug 2009, 22:53
QUOTE
13 წლის გოგონა, 1-ლი ტიპის დიაბეტით 3 წლის ასაკიდან. ღებულობს ინსულინ NPH-ს (ინსულატარდი) და ხანმოკლე მოქმედების ინსულინს – Regular (აქტრაპიდი) დღეში 2-ჯერ - საუზმის წინ და ვახშმის წინ.ბოლო რამდენიმე თვის განმავლობაში ბავშვს დაეწყო მძიმე ჰიპოგლიკემიური შეტევები, ძირითადად ღამით ან ადრეული დილის საათებში, საკვების მიღებამდე. ჰიპოგლიკემური ეპიზიდები სპორადულია და ყოველდღე არ ხდება. რამდენჯერმე მშობლებს დაჭირდათ სასწრაფოს გამოძახება. ჰიპოგლიკემიური ეპიზოდების დრის გლუკოზა 20-35 მგ/დლ ფარგლებში. დღის განმავლობაში გლუკოზა ~100-300 მგ/დლ ფარგლებში.ოჯახის ექიმმა პაციენტი გამოიკვლია, და ურჩია რომ მთლიანად გაჩერებინა ინსულინი. პაცინეტს დიაბეტის ქრონიკული გართულებები არა აქვს. გამოირიცხა თირკმლის, ღვიძლის, ფარისებრი და თირკმელზედა ჯირკვლის პრობლემები.რჩევის მიხედვით, პაციენტი ინსულინს აღარ იღებს. ინსულინის გაჩერების შემდეგ ჰიპოგლიკემიური ეპიზოდები გაგრძელდა. პაციენტს გაუკეთდა ც-პეპტიდი, რაც აღმოჩნდა <0.5 ნგ/მლ. პროინსულინი არ გაკეთებულა. ინსულინი - ~3-ჯერ მეტი ნორმაზე. გლიკოჰემოგლიბინი – 9.1%. ამ ანალიზების საფუძველზე მშობლებს ეთქვათ რომ ბავშვს ალბათ აქვს “ძალიან იშვიათი სიმსივნე” (პაციენტის მშობლები ვერ აკონკრეტებენ), და შემდგომი გამოკვლევისათვის პაციენტი გაიგზავნა ენდოკრინოლოგიურ კლინიკაში.სანარაუდო დიაგნოზი:


reactive hypoglycemia of diabetes თუ ინსულინომა არაა

Posted by: Thandrus 23 Aug 2009, 23:32
Blind_Torture_Kill

QUOTE
reactive hypoglycemia of diabetes


კი მაგრამ ეს ჭამიდან დაახლოებით 2-4 საათში არ ვითარდება? თან ც-პეპტიდი რატომ უნდა იყოს ამ შემთხვევაში დაბალი?

Posted by: Professor Xachikian 23 Aug 2009, 23:45
Thandrus
Blind_Torture_Kill
Reactive hypoglycemia, იგივე Functional alimentary hypoglycemia ვითარდება პრე-დიაბეტის, ან მე-2 ტიპის დიაბეტის დროს. ამ დროს აუცილებელი პირობაა ენდოგენური ინსულინის გამოყოფა, ანუ პანკრეასი უნდა მუშაობდეს. ასეთ შემთხვევებში პანკრეასი უშვებს შეცდომას და ტკბილი/ნახშირწყლოვანი საკვების მიღების შემდეგ (Thandrus მართალია, ~2-4 საათის შემდეგ) ხდება ჭარბი რაოდენობით ინსულინის გამოყოფა, რაც იწვევს რეაქტიულ, იგივე ფუნქციურ ჰიპოგლიკემიას.
ამ პაციენტის შემთხევაში კი პანკრეასი არ მუშაობს (1-ლი ტიპის დიაბეტი, ~10 წელი).

Posted by: Thandrus 23 Aug 2009, 23:54
მოდი, მაშინ ასე მაინც დავალაგოთ: დიაბეტი დაახლოებით 10 წელია რაც ჩამოყალიბდა, ასე რომ, პანკრეასის ინსულინი აქ არაფერ შუაშია. ინსულინის ექტოპიურ წარმოქმნაზე თქვი, რომ ეს არ იქნება. ეგზოგენურად მიღებული ინსულინიც გამორიცხულია. მაშინ რავიცი, რაღაც ბრძნულ-ჩახლართული მექანიზმი: ხომ არ შეიძლება, რომ ადრე გაკეთებულ ინსულინს არ მიეღწია "დანიშნულების ადგილამდე" და მაგალითად, ცხიმოვან ქსოვილში ჩარჩენილიყო (ცილებთან ან სხვა რამესთან ბმული) და ახლა პერიოდულად ამ ინსულინის გამოთავისუფლება ხდება... რაც იწვევს ეპიზოდურ ჰიპოგლიკემიურ შეტევებს, ც-პეპტიდის მოუმატებლად.

Posted by: Professor Xachikian 23 Aug 2009, 23:55
Thandrus
QUOTE
ეგზოგენურად მიღებული ინსულინიც გამორიცხულია

რატომ?

Posted by: Thandrus 23 Aug 2009, 23:58
Professor Xachikian

QUOTE
რატომ?


ხომ თქვი, რომ პაციენტი ინსულინს აღარ იღებს? მაშინ ერთადერთი გზა, ეგზოგენური ინსულინი რომ მოხვედრილიყო, ისაა, რომ ნელნელა გამოთავისუფლებულიყო ქსოვილებიდან...

Posted by: Blind_Torture_Kill 23 Aug 2009, 23:59
მაშინ პაციენტი თითონ იჩხერს ინსულინს
baby.gif



Posted by: Thandrus 24 Aug 2009, 00:04
QUOTE
მაშინ პაციენტი თითონ იჩხერს ინსულინს


თუ ეს ასეა, მაშინ გამოსაცნობიც არაა ბევრი არაფერი, მაგრამ...

QUOTE
რჩევის მიხედვით, პაციენტი ინსულინს აღარ იღებს


... რავიცი... აქ წერია, რომ პაციენტი არ იღებს... იქნებ პაციენტის დედა / მამა / დეიდა უკეთებს...

Munhausen By Proxy?

Posted by: Professor Xachikian 24 Aug 2009, 00:06
მოკლედ, კლინიკური სურათი და პაციენტის მშობლების მონაყოლი დიამეტრალურად ეწინააღმდეგებოდა ფაქტებს.
არადა კლასიკური სურათია: ძალიან დაბალი ც-პეპტიდი, მაღალი ინსულინი და ჰიპოგლიკემია. მიუხედავად მშობლების მტკიცებისა რომ "პაციენტი ინსულინს არ იღებს", პაციენტს არ განვიტარებია არც ერთი დიაბეტური კეტოაციდოზი, რაც წარმოუდგენელია ინსულინის მიღების გარეშე 1-ლი ტიპის დიაბეტიკში (<24 საათია საჭირო ინსულინის შეწყვეტიდან კეტოაციდოზის განვითარებამდე). გარდა ამისა, პაციენტ ქონდა მაGალი გლიკოჰემოგლობინი, რაც ქრონიკულ ჰიპოგლიკემიას გამორიცხავს.

ამ მონაცემებზე, და კლინიკურ სურათზე დაყრდნობით, პაციენტის მშობლებს ეთქვათ, რომ ბავშვს ვირაცა უკეთებს ინსულინს.
საშინლად შეურაცხყოფილმა მშუბლბმა დატოვეს კლინია.
ერთი თვის შემდეგ პაციენტს ისევ დაემართა ძალიან მძიმე ჰიპოგლიკემია - მშობლებმა უგონოდ იპოვეს ლოგინში. როდესაც სასწრაფო გამოიძახეს და უტარდებოდა შესაბამისი მკურნალობა, შემთხვევით ბავშვის ლოგინში, (და შემდეგ ლოგინის ქვეშ) იპოვეს ინსულინის ფლაკონები.
დიაგნოზი: მშობლების უყურადღებობა. ბავშვი თვითონ იკეთებდა ინსულინს, ღამის საათებში. ფსიქოლოგიური მომენტი: ბავშვს არ უნდოდა რომ დიაბეტი ქონოდა, და pretended რომ ინსულინი აღარ ჭირდებოდა.

Posted by: Thandrus 24 Aug 2009, 00:09
QUOTE
მოკლედ, კლინიკური სურათი და პაციენტის მშოვლების მონაყოლი დიამეტრალურად ეწინააღმდეგებოდა ფაქტებს.არადა კლასიკური სურათია: ძალიან დაბალი ც-პეპტიდი, მაღალი ინსულინი და ჰიპოგლიკემია. მიუხედავად მშობლების მტკიცებისა რომ "პაციენტი ინსულინს არ იღებს", პაციენტს არ განვიტარებია არც ერთი დიაბეტური კეტოაციდოზი, რაც წარმოუდგენელია ინსულინის მიღების გარეშე 1-ლი ტიპის დიაბეტიკში (<24 საათია საჭირო ინსულინის შეწყვეტიდან კეტოაციდოზის განვითარებამდე). გარდა ამისა, პაციენტ ქონდა მაGალი გლიკოჰემოგლობინი, რაც ქრონიკულ ჰიპოგლიკემიას გამორიცხავს.

ამ მონაცემებზე, და კლინიკურ სურათზე დაყრდნობით, პაციენტის მშობლებს ეთქვათ, რომ ბავშვს ვირაცა უკეთებს ინსულინს.საშინლად შეურაცხყოფილმა მშუბლბმა დატოვეს კლინია. ერთი თვის შემდეგ პაციენტს ისევ დაემართა ძალიან მძიმე ჰიპოგლიკემია - მშობლეგმა უგონოდ იპოვეს ლოგინში. როდესაც სასწრაფო გამოიძახეს და უტარდებოდა შესაბამისი მკურნალობა, შემთხვევით ბავშვის ლოგინში, (და შემდეგ ლოგინის ქვეშ) იპოვეს ინსულინის ფლაკონები.დიაგნოზი: მშობლების უყურადღებობა. ბავშვი თვითონ იკეთებდა ინსულინს, ღამის საათებში. ფსიქოლოგიური მომენტი: ბავშვს არ უნდოდა რომ დიაბეტი ქონოდა, და pretended რომ ინსულინი აღარ ჭირდებოდა.


lol.gif lol.gif lol.gif

აუ... მართლა ასე ყოფილა biggrin.gif

მე კიდევ მივედე და მოვედე tongue.gif

Posted by: Professor Xachikian 24 Aug 2009, 00:11
Blind_Torture_Kill
Thandrus
გამოსაცნობი იმდენად იყო ეს შემთხვევა, რომ პაციენტი, მშობლები და ოჯახის ექიმი ერთს ამტკიცებენ, გამოკვლევა და ანალიზები კი - საწინააღმდეგოს. ზოგჯერ პაცინეტის მონაყოლი და რეალობა განსხვავდება ერთმანეთისაგან. ასეთ სიტუაციაში ძალიან ძნელია პაციენტის კონფრონტაცია, თუ არ გაქვს უტყუარი მტკიცებულებები.

Posted by: Thandrus 24 Aug 2009, 00:13
Professor Xachikian

რა მაინტერესებს, მალე დაადგინეთ ეს ამბავი? თუ თქვენც ათასგვარი ვარიანტები იფიქრეთ?

Posted by: Blind_Torture_Kill 24 Aug 2009, 00:14
რამე არაენდოკრინული ქეისს თუ დადებთ ოღონდ კაი ტვინის საჩიჩქნს კაი იქნება

Posted by: Professor Xachikian 24 Aug 2009, 00:19
Thandrus
ეჭვი თავიდანვე იყო: თუ 1-ლი ტიპის დიაბეტი გაქვს და ინსულინის გაკეთებას შეწყვეტ, მაშინ გემართება კეტოაციდოზი, რაც ამ ბავშვს არ დამართნია. ყველაზე უცნაური სწორედ ეს იყო ამ შემთხვევაში. აქედან დასკვნა: ე.ი. ინსულინის მიღება არ შეწყვეტილა. ფაქტიურად, ლაბორატორიულმა ანალიზებმა ეს დაადასტურა.

"იშვიათ სიმსივნესთან" რომ არ გვქონდა საქმე, ამას მოწმობდა ქრონიკული ჰიპოგლიკემიის არარსებობა (მაღალი Hgb A1c) და უარყოფითი IGF-II
* * *
Blind_Torture_Kill
მართალია, ნამეტანი ბევრი ენდოკრინოლოგია გამოგვივიდა.
დაველოდოთ არანედოკრინულ ქეისებს.

Posted by: LUKA-BRAZI 24 Aug 2009, 20:51
ისე, ყოჩაღ Thandrus, "ბეზ პიწი მინუტ" ენდოკრინოლოგი ხარ. კარგად ხსნიდი ქეისებს.

smile.gif

Posted by: Thandrus 25 Aug 2009, 02:30
LUKA-BRAZI

ეგრე რომ იყოს, კარგი იქნებოდა...

აბა, სანამ ვინმე ტვინის საჭ###ტს დადებდეს, შედარებით ადვილი ქეისი:

როდესაც ექიმი მორიგეობდა ჩინეთის ერთ-ერთ ღარიბ რაიონში, მასთან მივიდა ქალი და თხოვა ბავშვის გასინჯვა. დედამ თქვა, რომ ბავშვი ამ ბოლო დროს "მოუხერხებელია და თავს ცუდად გრძნობს". ამავე დროს, ბავშვს ხშირად აქვს დიარეის შეტევები და ფილტვების ანთება. დედა ამბობს, რომ ბავშვი "განსაკუთრებით მოუხერხებელია მაშინ, როდესაც საღამოობით საყიდლებზე ვაგზავნი". ექიმმა გასინჯა ბავშვი, შენიშნა კონიუქტივის სიმშრალე და სკლერაზე კერატინიზირებული ეპითელიუმის ფოლაქები. რა დიაგნოზი დასვა ექიმმა?

Posted by: Professor Xachikian 25 Aug 2009, 04:21
Thandrus
QUOTE
მოუხერხებელია

იგივე restless?

Posted by: LUKA-BRAZI 25 Aug 2009, 10:02
QUOTE
"მოუხერხებელია და თავს ცუდად გრძნობს".

QUOTE
"განსაკუთრებით მოუხერხებელია მაშინ, როდესაც საღამოობით საყიდლებზე ვაგზავნი".

მოუხერხებელი რაღას ნიშნავს?
QUOTE
restless

მოუსვენარი?

Posted by: Thandrus 25 Aug 2009, 11:24
Professor Xachikian
LUKA-BRAZI

QUOTE
მოუხერხებელია


არა. იგივე "clumsy". - დედამ ასე თქვა biggrin.gif (მოუხერხებელი ბრჭყალებში ჯობდა რომ ჩამესვა)

Posted by: LUKA-BRAZI 25 Aug 2009, 11:36
QUOTE
"clumsy".

ასევე მოუქნელი, ტლანქი biggrin.gif
QUOTE
"განსაკუთრებით მოუხერხებელია მაშინ, როდესაც საღამოობით საყიდლებზე ვაგზავნი".

ე.ი. საღამოობით რაღაც ემართება ხოლმე? რამე ისეთი ხდება მაგ დროს რაც ბავშვს არ მოსწონს, მაგრამ არც ამბობს და ამავდროულად დიარეის, პნევმონიის და კონიუნქტივიტის მიზეზი შეიძლება იყოს? eek.gif
იმ ბავშვის არ იყოს ინსულინს რომ იკეთებდა ღამე, აქაც რამე ეგეთი odd thing ხომ არ არის? biggrin.gif

Posted by: Thandrus 25 Aug 2009, 11:46
LUKA-BRAZI

QUOTE
ე.ი. საღამოობით რაღაც ემართება ხოლმე?


yes.gif

QUOTE
რამე ისეთი ხდება მაგ დროს რაც ბავშვს არ მოსწონს, მაგრამ არც ამბობს და ამავდროულად დიარეის, პნევმონიის და კონიუნქტივიტის მიზეზი შეიძლება იყოს?


არა, მოუხერხებლობას / ტლანქობას დიარეა და კონიუქტივიტი არ გამოუწვევია. smile.gif

QUOTE
იმ ბავშვის არ იყოს ინსულინს რომ იკეთებდა ღამე, აქაც რამე ეგეთი odd thing ხომ არ არის?


არა, ჩინელები დამჯერი ხალხია biggrin.gif

Posted by: Blind_Torture_Kill 25 Aug 2009, 11:52
QUOTE
რა დიაგნოზი დასვა ექიმმა?


ა ვიტამინის დეფიციტი აქვს

Posted by: LUKA-BRAZI 25 Aug 2009, 11:54
biggrin.gif
მოკლედ ბავშვი მიდის მაღაზიაში, გამყიდველს კი რაიმე ვირუსული დაავადება ჭირს და ახველებს ხოლმე biggrin.gif
ჰოდა ამ ბავშვს არ მოსწონს რომ გამყიდველი სულ ახველებს და აცემინებს, მაგიტომაც არ უნდა მაღაზიაში წასვლა ხოლმე biggrin.gif
ეს ვირუსი კი ხვდება კონიუქტივაზეც, ფილტვებშიც და გასტროინტესტინურ ტრაქტსაც აფორიაქებს.

ჰა, აბა, როგორი ფანტაზიის უნარი მაქვს? gigi.gif

Posted by: Thandrus 25 Aug 2009, 11:57
Blind_Torture_Kill

QUOTE
ა ვიტამინის დეფიციტი აქვს


საღოლ, აბსოლუტურად მართალია!

up.gif up.gif up.gif

"მოუქნელობის" მიზეზი ღამეული სიბრმავე იყო, ხშირი ინფექციების მიზეზი A ვიტამინის ნაკლებობით გამოწვეული იმუნოდეფიციტი, პლუს ქსეროფთალმია და სკლერაზე თეთრი ფოლაქები - Bitot's Spots.

Posted by: LUKA-BRAZI 25 Aug 2009, 12:04
lol.gif

მე სადმე სცენარისტად უნდა ვიმუშავო....

ახალი ქეისი (პაციენტი ვარ მე smile.gif): მოკლედ 2 დღეა სულ ვამთქნარებ, განსაკუთრებით მაშინ როცა ამაზე ვფიქრობ. გარდა მაგისა 2 კვირაა სულ მეჩვენება რომ ჰაერის უკმარისობა მაქვს, სულ მინდა რომ ღრმად ვისუნთქო. სიარულის დროსაც მასე მემართება, ოღონდ პირველივე ნაბიჯიდან და არა ისე რომ თანდათან გამივითარდეს ჰაერის უკმარისობა. დაღლით არ ვიღლები. 2 კვირის წინ მთაში ვიყავი და ~2500 მეტრიან მთაზე დავაწექით ექსკურსიაზე. მაშინაც მასე იყო. სიარულის დროს სულ ვგრძნობდი რომ თითქოს ჰაერი არ მყოფნიდა, თუმცა მთაზე მაინც ავედი, ანუ გულის უკმარისობა არ მაქვს biggrin.gif
რა უნდა იყოს? ტვინის ჰიპოქსია? ანემია? რამე ჰემოდინამიკური დარღვევა? თუ საქმე არ მაქვს და სისულელეებს ვლაპარაკობ? givi.gif არადა ეხლაც სულ მინდა რომ დავამთქნარო ან რრმად ჩავისუნთქო..... what's up dudes?

smile.gif

Posted by: Thandrus 25 Aug 2009, 22:56
LUKA-BRAZI

QUOTE
თუ საქმე არ მაქვს და სისულელეებს ვლაპარაკობ?


იმედია ესაა gigi.gif

ისე, მართლა, სულ პირველ გვერდზე რომ დავდე ქეისი (ვიდეო რომ იყო) ის ამოხსენით ვინმემ?

Posted by: LUKA-BRAZI 26 Aug 2009, 11:22
Thandrus
QUOTE
იმედია ესაა

უკვე მესამე დღეა (კვირაა) მეც მაგის იმედი მაქვს :|

QUOTE
ისე, მართლა, სულ პირველ გვერდზე რომ დავდე ქეისი (ვიდეო რომ იყო) ის ამოხსენით ვინმემ?

მოიცა ეგ ჯერ კიდევ არ არის ამოხსნილი?
მე მაინც ვერ ვხვდები რა არის. თსსუ-ში ხომ ნევროლოგიას არ ასწავლიან user.gif
პრიონები ჭირს givi.gif

Posted by: Thandrus 26 Aug 2009, 11:31
LUKA-BRAZI

QUOTE
მოიცა ეგ ჯერ კიდევ არ არის ამოხსნილი? მე მაინც ვერ ვხვდები რა არის. თსსუ-ში ხომ ნევროლოგიას არ ასწავლიან პრიონები ჭირს


ხო, პრიონებია. ვიდეოზე პაპუასები იყვნენ, იმ კაცს კიდევ საშინელი ატაქსია ჰქონდა... ანუ ავადმყოფს კურუ სჭირდა.

აბა, მე რამდენიმე დღით უნდა დაგტოვოთ... მოუარეთ თემას smile.gif

Posted by: LUKA-BRAZI 26 Aug 2009, 11:35
Thandrus
QUOTE
აბა, მე რამდენიმე დღით უნდა დაგტოვოთ

დროებით !

smile.gif

Posted by: Blind_Torture_Kill 29 Aug 2009, 17:09
კარგით
მაშინ ჩემებურ ქეისს დავდებ

47 წლის კახექსიური მამაკაცის გვამი ნაპოვნია სადარბაზოს პადვალში. გვამის გარეგნული დათვალიერებით-აღენიშნება ნაჩხვლეტი ჭრილობები ზომით 0.1 დიამეტრი ორივე იდაყვის წინა ზედაპირზე + ირგვლივმდებარე უსწორო ფორმის მოწითალო/მოლურჯო ფერის სისხლნაჟღენთებით. ფიზიკური დაზიანების სხვა რაიმე ნიშანი არა აქვს. გვამი გადასვენდა მორგში და გაიკვეთა.
გაკვეთისას მარჯვენა წინაგული დილატირებული. სამკარიან სარქველზე აღინიშნება ნადებები

საიდან არის ეს ჰისტოლოგიური პრეპარატი აღებული და რა არის მასზე ?
* * *
http://img269.imageshack.us/i/97917715.jpg/

მეორე სურათი გულია და პირველი სურათის მაკრო პრეპარატს არ წარმოადგენს

Posted by: vano_t 31 Aug 2009, 02:08
QUOTE (Blind_Torture_Kill @ 29 Aug 2009, 17:09 )
კარგით
მაშინ ჩემებურ ქეისს დავდებ

47 წლის კახექსიური მამაკაცის გვამი ნაპოვნია სადარბაზოს პადვალში. გვამის გარეგნული დათვალიერებით-აღენიშნება ნაჩხვლეტი ჭრილობები ზომით 0.1 დიამეტრი ორივე იდაყვის წინა ზედაპირზე + ირგვლივმდებარე უსწორო ფორმის მოწითალო/მოლურჯო ფერის სისხლნაჟღენთებით. ფიზიკური დაზიანების სხვა რაიმე ნიშანი არა აქვს. გვამი გადასვენდა მორგში და გაიკვეთა.
გაკვეთისას მარჯვენა წინაგული დილატირებული. სამკარიან სარქველზე აღინიშნება ნადებები

საიდან არის ეს ჰისტოლოგიური პრეპარატი აღებული და რა არის მასზე ?
* * *
http://img269.imageshack.us/i/97917715.jpg/

მეორე სურათი გულია და პირველი სურათის მაკრო პრეპარატს არ წარმოადგენს

IV drug user-ია. მაკროზე სამსაგდულიანი სარქვლის ვეგეტაციებია. უფრო ხშირად მაგათ ემართებათ S. Aureus-ით გამოწვეული ინფექციები. სიკვდილის მიზეზი სეპსისია ხშირად. ემბოლიები შეიძლება მოგცეს ფილტვებში და ფილტვის აბსცესები გამოიწვიოს ან ფულმინანტური პნევმონია (მაქვს ასეთი შემთხევა ნანახი). ხოდა ალბათ ფილტვებია მაგ პრეპარატზე თავისი სეპტიკური ემბოლიებით (ან აბსცესი კიდევ ან პნევმონია).

Posted by: BadbadGirl 31 Aug 2009, 02:25
vano_t
გთხოვ თერაპევტების ტემაში შეიხედე და მიპასუხე რა ფლიზ smile.gif
მადლობა smile.gif

Posted by: vano_t 31 Aug 2009, 02:46
82 წლის ავადმყოფი, რომელიც გუშინწინ მოვიდა კლინიკაში შემდეგი ჩივილებით: თავის ტკივილები მწვავე დასაწყისით, რომელიც განსაკუთრებით ძლიერდება ფეხზე ადგომისას; მხედველობის დაბინდვა და თავბრუსხვევა, რომელიც ასევე ძლიერდება ფეხზე ადგმოისას. პაციეტს ასევე აქვს სისუსტე. სხვა სისტემების მიმოხილვა უარყოფითია!

წარსულის ისტორია: მეორე ტიპის დიაბეტი; არაჰოჯკინისეული ლიმფომა; ჰიპერტენზია; ჰიპერლიპიდემია; წინაგულების ფიბრილაცია; რაღაც ქირირგიული ჩარევა გაუკეთდა წარსულში პანკრეასზე და ნაწლავებზე, რომელიც ზუსტად არ იცის ავადმყოფმა;

ალერგიულია მორფინზე და დიაზეპამზე;

იღებს სტატინს, ACE (აგფ) ინჰიბიტორს; კუმადინს; დიგოქსინს; ასპირინს; მეტოპროლოლს; ყველა წამალი დიდი ხნის დაწყებულია და არცერთი არ არის ახალი.

არ ეწევა, არ სვამს, არ კაიფობს (რაზეც სასტიკად ვუსაყვედურე ავადმყოფს).

უარყოფს ტრავმას ამ ბოლო დროს; უარყოფს რაიმე ტოქსინებთან ექსპოზიციას; უარყოფს ურეცეპტო წამლების მოხმარებას ამ დღეებში. აცრები up-to-date არის.

ფიზიკური გამოკვლევისას ყველაფერი ნორმაშია, გარდა იმისა, რომ პაციენტს აღენიშნება სისუსტე და ვერ დგება; ადგომა ასევე აძლიერებს თავის ტკვილს, მხედველობის დაბინდვას და თავბრუსხვევას. მცირედი ბრადიკარდია (HR=50) არარეგულარული გულისტონებით და პულსით; გულმკერდის წინა კედელზე მცირე ვენები გაფართოებულია-ავადმყოფის ცოლი ამბობს, რომ ეს ახალი რამ არის და ადრე არ ქონდა.

კლინიკაში ავადმყოფს გაუკეთდა CBC (ერითროციტები, ლეიკოციტები, თრომბოციტები, ჰემოგლობინი ნორმაშია); Chem16 (შარდოვანა, კრეატინინი მცირედ აწეული-32 და 1.7, მაგრამ baseline-ზე მაღლა არ არის, უმნიშვნელო ჰიპონატრემია-135, ნორმალური კალიუმი, კალციუმი, ქლორი, ბიკარბონატი, ღვიძლის ენზიმები, LDH, AlkPhos, მთლიანი და პირდაპირი ბილირუბინი; შაქარი ცოტათი მომატებულია 220 და დიაბეტი კარგ კონტროლში აქვს ძირითადად); INR იყო 1.8; ECG-მ აჩვენა წინაგულების ფიბრილაცია, 2 ერთმანეთს მიყოლებული ფართო QRS კომპლექსები (PVC); EKG-ზე ძირითადი QRS კომპლექსები და ST-T იგივე იყო, რაც 1 წლის წინა EKG-ზე.

ახლა წარმოიდგინეთ ხართ სოფლის კლინიკაში და 2 საათის სავალზე არის დიდი საავადმყოფო. რას გაუკეთებს ავამდყოფს? რა დიაგნოზებზე იფიქრებთ? როგორ management-ს მიყვებით? თუ დამატებითი ინფორმაცია დაგჭირდათ ისტორიიდან და ფიზიკური გამოკვლევებიდან (აგრეთვე სხვა ჩატარებული ტესტებიდან), იკითხეთ და თუ გაკეთებული იქნა გიპასუხებთ.

Posted by: Professor Xachikian 31 Aug 2009, 04:18
vano_t
1. ორთოსტატიული წნევის და გლულისცემის გაზომვა
2. შარდის ანალიზი.
3. თუ ტემპერატურა მომატებულია და ინფექციაზეა ეჭვი - შარდის და სისხლის ბაქტერიოლოგიური კულტურა
4. TSH
5.თავის ტვინის CT კონტრასტის გარეშე და კონტრასტით
6. გლულმკერდის არეში მცირე ვენების გაფართოება - შეიძლება ასევე გულმკერდის CT გაკეთება გულმკერდის/შუასაყრის მასის გამოსარიცხად (ლიმფომის რეციდივი?)

Posted by: vano_t 31 Aug 2009, 04:52
Professor Xachikian
QUOTE
1. ორთოსტატიული წნევის და გლულისცემის გაზომვა
2. შარდის ანალიზი.
3. თუ ტემპერატურა მომატებულია და ინფექციაზეა ეჭვი - შარდის და სისხლის ბაქტერიოლოგიური კულტურა
4. TSH
5.თავის ტვინის CT კონტრასტის გარეშე და კონტრასტით

1) ორთოსტატიკა გაიზომა მარტო დაწოლილ და დამჯდარ მდგომარეობაში. წნევის ვარდნა და პულსის გახშირება არ მოხდა. დამდგარ მდგომარეობაში შეუძლებელი იყო.
2) შარდი ნორმალურია.
3) ტემპერატურა ნორმალური იყო და ფიზიკური გამოკვევით და ანამნეზით ინფექციაზე ეჭვი არ არსებობდა. თუმცა, მაინც შესაძლებელია ალბათ ინფექციის არსებობა. სისხლი არ დათესილა. გულმკერდის რენტგენი არ გაკეთებულა, თუმცა CT გაკეთდა გულმკერდზე, რომ გამორიცხულიყო SVC სინდრომი (გაგანიერებული გულმკერდის ვენები, ლიმფომის ისტორია და მხედველობის დაბინდვა, თავის ტკივილი). ეს ყველაფერი გაკეთდა რეგიონალურ ჰოსპიტალში. უშუალოდ რეზულტატები ჯერ არ მოსულა ჩვენთან, მაგრამ ავადმყოფი ER-დან გამოუშვეს გამოკვლევების შემდეგ და მეუღლეც ადასტურებს, რომ გამოკვლევები ნორმალური იყო.
4) TSH იმ დღეს არ გაკეთებულა. როცა ავადმყოფი გაგზავნილი იქნა რეგიონალურ ჰოსპიტალში და შემდეგ უკან გამოუშვეს, გუშინ ავადმყოფი დაბრუნდა ჩვენს ER-ში იგივე ჩივილებით. თან ავადმყოფი შეეცადა რომ ამდგარიყო და წაიქცა, რის შედეგადაც შუბლზე ქონდა რამოდენიმე პატარა აბრაზია და laceration. ახალი სიმპტომები არ ქონია წაქცევის შემდეგ. TSH გაეზომა გუშინ და იყო ნორმაში-2.5.
5) იგივე ითქმის CT-ზეც. როგორც ავადმყოფის მეუღლე აღწერს, CT არ უნდა ყოფილიყო, არამედ MRI, რომელიც ასევე გადმოცემით ნორმალური იყო. ყოველ შემთხვევაში ავადმყოფს რომ სერიოზული პრობლემა ქონოდა MRI-ზე, არ გამოუშვებდნენ ER-დან, ან გამოუშვებდნენ და რეზულტატს შემატყობინებდნენ შემდეგი follow-up-ისათვის.

როგორც ვთქვი ავადმყოფი დაბრუნდა უკან და გუშინ ისევ მოვიდა ER-ში. სავადმყოფოში იქნა დაწოლილი დაკვირვების მიზნით და დამატებითი ლაბორატორიები იქნა გაკეთებული სოფლის საავადმყოფოს შესაძლებლობის ფარგლებში smile.gif ლაბორატორიების შედეგებს მოთხოვნის მიხედვით მოგახსენებთ. დავამატებ, რომ ავადმყოფს ახლა მიღებისას მცირედ შესიებული ქონდა სახის მარჯვენა მხარე და კისრის მარჯვენა მხარე, რისთვისაც მიიღო სოლუმედროლის 1 დოზა 125 მგ. (სტრიდორი და wheezing არ ქონდა ამ დროს) დაახლოებით 3-4 საათში სახის შეშუპება გენერალიზებული გახდა; მთელი კისერიც გასივდა ორივე მხარეს-უფრომ მარჯვნივ. შეშუპება გამოხატული იყო პერიორბიტალურადაც. ენა ნორმალური ზომის იყო და არ იყო შესიებული. ისევ, ანამნეზში და გამეორებით ფიზიკურ გამოკვლევაში არ იყო ალერგიული რეაქცია გამოხატული. თუმცა, ისტორიაში 1 შესაძლო მიზეზია ამისა. მაგრამ, ძნელია იმის თქმა ეს არის თუ არა მიზეზი და ამის დადგენა ვერასდროს ვერ მოხდება დანამდვილებით, თუ ანგიოედემის მიზეზი მკვეთრად გამოხატული რაიმე მწვავე დაავადება არ არის.

სახე/კისრის შესიებასთან ერთად დაეწყო დისფაგია. საერთოდ ვეღარ ყლაპავდა წყალს. impending respiratory compromise-ის (თუმცა, ჯერ არ ქონდა ავადმყოფს არც სტრიდორი, არც wheezing და პულს ოქსიმეტრია აჩვენებდა 95%-ს) გამო ავადმყოფი ისევ გადაყვანილი იქნა ICU-ში (ინტენსიური თერაპიის განყოფილება) რეგიონალურ ჰოსპიტალში. დღეს დარეკილი იქნა იმ ჰოსპიტალში, რათა გაირკვეს მიზეზები მისი პრეზენტაციის. ჯერ-ჯერობით მკვეთრად გამოხატული მიზეზი ვერ უპოვიათ შეშუპების. თუმცა, სხვა სიმპტომების სავარაურო მიზეზი გარკვეულ ლაბაორატორიაში გამოჩნდა უკვე სოფლის ჰოსპიტალში.

Posted by: Professor Xachikian 31 Aug 2009, 08:01
vano_t
1. ანგიოედემა რამ გამოიწვია ძნელი სათქმელია, და ისიც საკითხავია, საერთოდ არის თუ არა რამე კავშირში პაციენტის საწყის სიმპტომებთან. ანგიოედემა ალერგიული რომ იყოს, სლოუმედროლის ფონზე არ გაუარესდებოდა. ყოველ შემთხვევაში, ACE-ინჰიბიტორს გავაჩერებდი.

2. საწყისი სიმპტომების (თავის ტკივილი, თავბრუხვევა და ა.შ.) სტრუქტურული მიზეზი, ფაქტიურად, არ არის (თუ, რა თქმა უნდა, ინფორმაცია სწორეა და თავის ტვინის MRI მართლა უარყოფითია). თუ სიმპტომატიკა გრძელდება, მაშინ საინტერესოა თავ-ზურგტვინის სითხის (CSF) ანალიზი - შეიძლება სუბარაქნოიდული სისხლდენა იყოს, რაც რადიოლოგიურ კვლევაზე შეიძლება არ გამოჩნდეს. გარდა ამისა, CSF-ის ანალიზით სხვა მიზეზებიც გამოირიცხება ან გამოვლინდება.
თუმცა CSF-ის ანალიზის გაკეთება ამ ეტაპზე ალბათ შეუძლებელია, სანამ კუმადინი არ გაჩერდება (ან შეიცვლება ჰეპარინით) და სანამ INR-ი არ ჩამოვა ნორმამდე.

Posted by: Blind_Torture_Kill 31 Aug 2009, 08:19
vano_t
QUOTE
IV drug user-ია. მაკროზე სამსაგდულიანი სარქვლის ვეგეტაციებია.

კი ეგაა

QUOTE
სიკვდილის მიზეზი სეპსისია ხშირად.
კი

მარა აი ფილტვში რა არის

Posted by: vano_t 31 Aug 2009, 09:20
Blind_Torture_Kill
QUOTE
QUOTE
IV drug user-ია. მაკროზე სამსაგდულიანი სარქვლის ვეგეტაციებია.

კი ეგაა

QUOTE
სიკვდილის მიზეზი სეპსისია ხშირად.
კი

მარა აი ფილტვში რა არის

როცა არ შემხვედრია მსგავსი შემთხვევები და მიკროპრეპარატს ვერ ვკითხულობ, ძნელია თქმა რა არის ფილტვებში. თუმცა შევეცდები რამოდენიმე თეორიულად შესაძლო ვარიანტები ჩამოვთვალო. ე.ი. სეპტიური ემბოლიები არ არის. მაშინ შეიძლება მრავლობითი აეროვანი ემბოლიები ან ცხიმოვანი ემბოლიები იყოს-ის თეთრი სტრუქტურები არ იღებს საღებავს და ჰაერი ან ცხიმი (თუ სპეციალური შეღებვას არ წარმოადგენს ეგ პრეპარატი ცხიმისათვის) შეიძლება იყოს. თუმცა ჯერ ვერ ვნახე ასეთი case report-ები ნარკომანებში. აიროვანი ემბოლია თეორიულად შესაძლებელია, მაგრამ პატარა ვენებში მცირე ჰაერის შეპარვით? სინამდვილეში გამორიცხული არაფერია. შეიძლება საერთოდ მთელი შპრიცი საერით გაავსო და ის შეუშვა ვენაში, ან კაიფში დავარდა, ფეხი (ბარძაყის ძვალი) მოტყდა და ცხიმოვანი ემბოლიები დაერთო ყველაფერს თან. მოკლედ, ვარიანტები ამის გარდაც შეიძლება ბევრი იყოს.

Professor Xachikian
QUOTE
1. ანგიოედემა რამ გამოიწვია ძნელი სათქმელია, და ისიც საკითხავია, საერთოდ არის თუ არა რამე კავშირში პაციენტის საწყის სიმპტომებთან. ანგიოედემა ალერგიული რომ იყოს, სლოუმედროლის ფონზე არ გაუარესდებოდა. ყოველ შემთხვევაში, ACE-ინჰიბიტორს გავაჩერებდი.
მეც მასე ვფიქრობ. ალერგიული არ უნდა იყოს და აგფ ინფიბიტორი უნდა შეუწყდეს, იმიტომ რომ ამათ შეუძლიათ გამოიწვიონ ანგიოედემა დიდი ხნის მოხმარების შემდეგაც. ანგიოედემა სერიოზული ლეტალური შეიძლება აღმოჩნდეს და მეც აღარ გავრისკავდი აგფ-ს ინჰიბიტორის გაგრძელებას.

QUOTE
2. საწყისი სიმპტომების (თავის ტკივილი, თავბრუხვევა და ა.შ.) სტრუქტურული მიზეზი, ფაქტიურად, არ არის (თუ, რა თქმა უნდა, ინფორმაცია სწორეა და თავის ტვინის MRI მართლა უარყოფითია). თუ სიმპტომატიკა გრძელდება, მაშინ საინტერესოა თავ-ზურგტვინის სითხის (CSF) ანალიზი - შეიძლება სუბარაქნოიდული სისხლდენა იყოს, რაც რადიოლოგიურ კვლევაზე შეიძლება არ გამოჩნდეს. გარდა ამისა, CSF-ის ანალიზით სხვა მიზეზებიც გამოირიცხება ან გამოვლინდება.
თუმცა CSF-ის ანალიზის გაკეთება ამ ეტაპზე ალბათ შეუძლებელია, სანამ კუმადინი არ გაჩერდება (ან შეიცვლება ჰეპარინით) და სანამ INR-ი არ ჩამოვა ნორმამდე.
შესაძლებელია ასე იყოს. მაგ არ მიფიქრია, მაგრამ რაც უფრო ფართოა დიფერენციალური დიაგნოზები, მით უფრო მეტი შანსი იქნება მიზეზი იპოვო. მე თვითონ ამას არ გადავწყვიტავდი ეგ შესაძლო მიზეზი რომ დამედგინა. ნევროლოგის კონსულტაციას ავიღებდი ამის თაობაზე და შეიძლება ღირდეს კიდეც. ოღონდ, ლაბორატორიაში (სისხლის ანალიზში) გამოჩნდა კიდევ აშკარა შესაძლო (მე არ ვამბობ, რომ მაინცდმაინც ეს მიზეზია მიზეზი) მიზეზი, რომლის მკურნალობაც უფრო მარტივად შეიძლება და შეიძლება ავადმყოფი გადარჩეს კიდევ უფრო ინვაზიურ ჩარევას. ადრე მყავდა ავადმყოფი მხედველობის შეცვლით, ძლიერი სისუსტით, უმადობით და ადვილი დაღლილობით. იქაც მსგავსი პრობლემა პრობლემა იყო, რომელიც რამოდენიმე დღეში მოიხსნა. თუ ზოგად შესაძლო პათოლოგიურ მიზეზებზე იფიქრებ (მაგ. ტრავმა, ტოქსინებთან ექსპოზიცია, თერმული დაზიანებები, recent იატროგენული ჩარევები და ა.შ.), მაშინ ამ შესაძლო მიზეზსაც დაადგენ. კიდევ ერთ რამეს მივამატებ. ეს პრობლემა შესაძლოა თუ არა იყოს ამის მიზეზი, არ აქვს მნიშვნელობა. მაინც უნდა გასინჯო ამაზე და მაინც უნდა უმკურნალო.

Posted by: Blind_Torture_Kill 31 Aug 2009, 12:36
vano_t

ვასკულიტზე თუ ნახე ?
* * *
.............................................................................................
წნევა რამდენი იყო ?

Posted by: vano_t 31 Aug 2009, 21:56
Blind_Torture_Kill
QUOTE
ვასკულიტზე თუ ნახე ?
* * *
.............................................................................................
წნევა რამდენი იყო ?

ვასკულიტზეც შეიძლება იფიქრო, მაგრამ მანამდე უფრო აშკარა პრობლემა უნდა გამორიცხო.

წნევა მომატებულია, მაგრამ ქრონიკულად. ავადმყოფს ჰიპერტენზია აქვს. წნევის ბრალი არ არის ამ შემთხვევაში ეს სიმპტომები.

გარდიანმა იცის სწორი პასუხი შენს კეისზე, ჩემი აზრით.

Posted by: Professor Xachikian 1 Sep 2009, 03:09
vano_t
ამ ინფორმაციით, კიდევ ვიფიქრებდი დიგოქსინის დოზის გადამეტებას (კრეატინინი მომატებული აქვს, თან მოხუცია და ალბათ გლომერულური ფილტრაცია დაქვეითებული აქვს). მოკლედ, გავზომავდი დიგოქსინის დონეს სისხლში. B12 და ფოლატის გაზომვაც შეიძლება, მოხუცებში ამ ვიტამინების ნაკლებობა ხშირია (თუმცა სიმპტომატიკა არ გავს)
ზამთარი რომ იყოს (სოფელში) ვიფიქრებდი CO ინტოქსიკაციაზე. თუმცა თუ ამაზეა ეჭვი, სეზონის მიუხედავად, გავზომავდი კარბოქსიჰემოგლობინს.
ამ ეტაპზე მეტი სხვა იდეა არ მაქვს.
ეტყობა, ახალი შემთხევაა; საინტერესოა როგორ ვითარდება სუტუაცია, და სხვა რამე "clue" თუ არის.

Posted by: Blind_Torture_Kill 1 Sep 2009, 07:02
vano_t
QUOTE
გარდიანმა იცის სწორი პასუხი შენს კეისზე, ჩემი აზრით.

დაწეროს მერე

შენ ქეისზე წოტა კიდე მიგვანიშნე აბა


Posted by: vano_t 1 Sep 2009, 07:45
Professor Xachikian
QUOTE
ამ ინფორმაციით, კიდევ ვიფიქრებდი დიგოქსინის დოზის გადამეტებას (კრეატინინი მომატებული აქვს, თან მოხუცია და ალბათ გლომერულური ფილტრაცია დაქვეითებული აქვს). მოკლედ, გავზომავდი დიგოქსინის დონეს სისხლში. B12 და ფოლატის გაზომვაც შეიძლება, მოხუცებში ამ ვიტამინების ნაკლებობა ხშირია (თუმცა სიმპტომატიკა არ გავს)
ზამთარი რომ იყოს (სოფელში) ვიფიქრებდი CO ინტოქსიკაციაზე. თუმცა თუ ამაზეა ეჭვი, სეზონის მიუხედავად, გავზომავდი კარბოქსიჰემოგლობინს.
ამ ეტაპზე მეტი სხვა იდეა არ მაქვს.
ეტყობა, ახალი შემთხევაა; საინტერესოა როგორ ვითარდება სუტუაცია, და სხვა რამე "clue" თუ არის.
მართალია, დიგოქსინის დონე უნდა გაიზომოს ყველა ვარიანტში. პრინციპში, ნებსიმიერი სიმპტომებით შემოსული ავადმყოფში, რომელიც ისეთ წამალზეა რასაც ვიწრო თერაპიული ინტერვალი გააჩნია, უნდა გაიზომოს ასეთი წამლის დონე. ამ ავადმყოფში დიგონქსინი იყო 2.5, რაც აშკარად მაღალია. დიგოქსინი შეუწყდა, მაგრამ ავადმყოფი იმ დღეს სახე/კისრის შეშუპების გამო გადაყვანილი იქნა რეგიონალური საავადმყოფოს ICU-ში. დიგოქსინით მოწამვლა სახის შეშუპებას ვერ ახსნის ალბათ, თუმცა თეორიულად შესაძლებელია. სხვა სიმპტომები არის დაკავშირებული დიგოქსინით მოწამლვასთან.

CO-თი მოწამვლაზე არ მიფიქრია, მაგრამ გეთანხმები-თუ მოგივა ეს აზრი, ავადმყოფის ისტორიაც შეგიძლია შეამოწმო ალბათ: რა სახის ენერგიას იყენებს სახლში და თუ არის CO-ს წარმომშობი ხელსაწყოები. რა თქმა უნდა, ნახშირჟანგის დონის შემოწმება უფრო ზუსტია.

მოკლედ, გამოწერენ ამ ავადმყოფს მალე. გადავუმოწმებ დიგოქსინის დონეს და სიმპტომების არსებობა/არარსებობას; ეკგ-საც შევამოწმებ, ბრადიკარდია არის თუ არა კიდევ. თუ სიმპტომები გაგრძელდა, მერე თქვენს მიერ მოწოდებულ რჩევებსაც გავითვალისწინებ აუცილებლად.

Blind_Torture_Kill
QUOTE
QUOTE
გარდიანმა იცის სწორი პასუხი შენს კეისზე, ჩემი აზრით.
დაწეროს მერე
ერიდება ალბათ.

Posted by: LUKA-BRAZI 1 Sep 2009, 13:10
vano_t
Professor Xachikian
ჩემი მოკრძალებული ცოდნით რომ ჩავერიო თქვენს საუბარში ხომ შეიძლება? smile.gif
შეშუპება რომ განუვითარდა პაციენტს, ხომ შეიძლება ამის მიზეზი ზემო ღრუ ვენის სისტემის შემადგენელი ვენების, რომელირაც დონეზე იყოს მექანიკური კომპრესია? სტაზი ან ლიმფურ სადინრებზე იყოს ზეწოლა? ვთქვათ რამე მასეთი კომპრესისს გამომწვევი (სიმსივნე, თან ავადმყოფს აქვს ონკოლოგიური ანამნეზი) რომ იყოს საუღლე ვენასა და საერთო საძილე არტერიას შორის და ორივეზე ახდენდეს ზეწოლას, მაშინ შეშუპება (აიხსნებოდა ადგილობრივი კომპრსიით), ხოლო თავბრუსხვევა და მხედველობის დაბინდვა აიხსნებოდა ცერებრული სისხლის მიმოქცევის მოშლით, ოღონდ იმ შემთხვევაში თუ ავადმყოფს არ აქვს arteria communicans ან ასაკობრივი თავისებურებებიდან გამომდინარე სკლეროზირებულია და ცუდად ატარებს სისხლს.

მაგრამ რახან CT გაკეთდა და არაფერი გამოჩნდა, თან შეშუპება 3-4 საატში გენერალიზებული გახდა, ჩემ ვარიანტს გამორიცხავს და უფრო ვაზომოტორული მოშლილობისკენ უნდა გადავიხაროთ?

P.S. გარდიანი აღარ პოსტავს.....

Posted by: Blind_Torture_Kill 1 Sep 2009, 22:25
vano_t

მე კიდე მომეჩვენა რომ კუშინგის რეფლექსი ქონდა და მიზეზი თავში უნდა ყოფილიყო

Posted by: Solveig 2 Sep 2009, 19:35
გამოცდილო და განათლებულო, მაგრამ გამოუცდელო ექიმებო smile.gif

თქვენი აზრი როგორი იქნებოდა ამ შემთხვევაზე?

http://forum.ge/?f=43&showtopic=34007113

Posted by: Professor Xachikian 3 Sep 2009, 09:33
QUOTE
მაგრამ გამოუცდელო ექიმებო

sad.gif

Posted by: Thandrus 4 Sep 2009, 04:03
vano_t

რა დაუდგინეს ბოლოსდაბოლოს ამ კაცს? user.gif

Posted by: Blind_Torture_Kill 4 Sep 2009, 14:36
მოდი ჩემ ქეისს იმას დავამატებ რომ მიკროსკოპული სურათი ესე გამოიყურება პოლარიზებული სინათლის ქვეშ

ხვალე
მორიგეობიდან რომ მოვალ დავწერ პასუხს მანამდე თქვენთვის მომინდია

Posted by: Mrs_Zum 5 Sep 2009, 00:51
QUOTE (Blind_Torture_Kill @ 4 Sep 2009, 14:36 )
მოდი ჩემ ქეისს იმას დავამატებ რომ მიკროსკოპული სურათი ესე გამოიყურება პოლარიზებული სინათლის ქვეშ

ხვალე
მორიგეობიდან რომ მოვალ დავწერ პასუხს მანამდე თქვენთვის მომინდია

მიკროსკოპულ სურათს ვერ ვხედავ, ისე ვარაუდი მაქვს პნევმოცისტური პნევმონია ვითომ? baby.gif spy.gif

Posted by: ინფინიტი 5 Sep 2009, 19:33
ფეხის ცერა თითი მომყვა კარებში, გასიებულია, გაწითლებული და ფრჩხილი არის ჩაშავებული, ძალიან მტკივა.
რა მეშველება? შეიძლება გართულდეს? ფრჩხილი არ მძვრება და შეიძლება მომძვრეს?
გთხოვთ, მიპასუხოთ ვინმემ.

Posted by: Blind_Torture_Kill 5 Sep 2009, 21:44
QUOTE
მიკროსკოპულ სურათს ვერ ვხედავ, ისე ვარაუდი მაქვს პნევმოცისტური პნევმონია ვითომ?


არაა პნევმონია

Posted by: mtvareuli 9 Sep 2009, 18:07
AnEeEeE=]]
QUOTE
თქვენ რას იტყვით?

http://forum.ge/?act=ST&f=43&t=33710084

infinity-infinity
QUOTE
რა მეშველება?

ნუ გეშინია, გაგივლის თავისით


ასეთი კითხვები სხვა თემებში დასვით თუ შეიძლება, აქ სხვა რაღაცეები ირჩევა

Posted by: Cousteau 10 Sep 2009, 02:23
QUOTE (mtvareuli @ 9 Sep 2009, 18:07 )

http://www.facebook.com/ntvare

დაგამატე yes.gif


Posted by: Thandrus 11 Sep 2009, 01:45
Blind_Torture_Kill

ბოლოსდაბოლოს, რა არის შენი ქეისის პასუხი? smile.gif არ მინდა რაღაცეები გარტყმაზე ვწერო user.gif

ისე, კარგი იქნებოდა, თუ ერთ თემას გახსნიდი პათანატომიურ სურათებზე, დადებდი ხოლმე სურათებს და ახსნიდი, როგორ უნდა ამათუიმ სურათის ინტერპრეტაცია. ეს ბევრ ექიმსაც კი გამოადგებოდა, ვისაც აქტიური შეხება არ აქვთ ასეთ სურათებთაბ. smile.gif

Posted by: Blind_Torture_Kill 12 Sep 2009, 00:12
Thandrus

QUOTE
ბოლოსდაბოლოს, რა არის შენი ქეისის პასუხი?


Talc-ი

Posted by: Thandrus 12 Sep 2009, 02:42
Blind_Torture_Kill

QUOTE
Talc-ი


ვაჰ... ეს იდეა არც მომსვლია თავში biggrin.gif და იქ ტალკს რა უნდოდა?

მეც დავდებ მაშინ ერთ ქეისს...

33 წლის უშვილო ქალი მოიყვანეს Emergency Department-ში უეცრად განვითარებული სუნთქვის უკმარისობის გამო, რომელიც მას ინ ვიტრო განაყოფიერებიდან (IVF) ერთ კვირის შემდეგ დაეწყო. სუნთქვის უკმარისობა ED-ში მიყვანის დღეს, დილით დაეწყო, მანამდე კი, რამდენიმე დღის განმავლობაში პაციენტს მუცლის შებერილობა, გულისრევის შეგრძნება და ზოგჯერ ღებინება (უსისხლო, უნაღველო) აღენიშნებოდა. მუცლის ტკივილი ღამით უფრო მძიმდებოდა. პაციენტი ამბობს, რომ ღრმად ჩასუნთქვის დროს მას მუცელი ძალიან ტკივა. მას არ შეუძლია მწოლიარედ ყოფნა, რადგან ეს ტკივილს აძლიერებს და სუნთქვას ართულებს.

პაციენტს ისტორიაში პოლიკისტოზური საკვერცხის სინდრომი აქვს (Policystic Ovary Syndrome) და ამიტომაც ის რამდენიმე წლის განმავლობაში ვერ დაორსულდა. ED-ში მოყვანამდე სამი კვირით ადრე მას საკვერცხის კონტროლირებული სტიმულაცია (გონადოტროპინებით) ჩაუტარდა. სულ მიღებული იყო 25 ემბრიონი. საკვერცის სტიმულაციის დროს ესტრადიოლის კონცენტრაციის მწვერვალი 4638 pg/mL (17026 pmol/L) იყო. ამის შემდეგ მას ჩაუტარდა transvaginal ultrasound-guided oocyte retrievаl, შემდეგ კი, ED-ში მოყვანამდე ერთი კვირით ადრე მას ჩაუსვეს განაყოფიერებული ემბრიონი.

ისტორიის დანარჩენი ნაწილი არაფრით იყო გამორჩეული. ახლადჩატარებული ჰორმონ-თერაპიის გარდა ის ყოველდღიურად მხოლოდ მულტივიტამინებს იღებდა. იგი ალერგიული არ არის, არ ეწევა, არ სვამს.

გამოკვლევის დროს იგი ფხიზელი და ორიენტებულია. ის ცოტათი შეწუხებული ჩანს. პულსი - 108 bpm, წნევა - 110/60. მას ხშირი და ზედაპირული სუნთქვა აქვს, 26 br/min სიხშირით. ოქსიმეტრია აჩვენებს 95 % გაჯერებას ოთახის ჰაერის სუნთქვისას და 98%-ს 2 L/min ჟანგბადის დამატებისას. პაციენტი 90 კგ-ს იწონის. თავისა და კისრის გამოკვლევა ნორმალურია. ფილტვების გამოკვლევისას შეინიშნება ეგოფონია, breath sound-ების ორმხრივად შემცირება და tactile fremitus-ის დაკარგვა. მისი გულის ცემა ტაქიკარდიული, მაგრამ რეგულარულია. შუილები არ არის. მუცელი შებერილია, მაგრამ ნორმალური ნაწლავური ხმები ისმის. მუცლის პალპაციის დროს არ შეინიშნება რამე მასა ან ორგანომეგალია, მაგრამ მთელი მუცლის მანძილზე პალპაციისადმი მგრძნობელობა და ტკივილი ცოტათი მატებულია და არის positive fluid waves. პელვისის გამოკვლევისას საშოდან სისხლდენა ან რაიმე გამონადენი არ სჩანს. ბიმანუალური პელვისის გამოკვლევა არ ჩატარებულა.

ლაბორატორიული ტესტების პასუხები:

WBC = 12.2 × 103/µL (12.2 × 109/L)
ჰემოგლობინი = 16.2 g/dL (162 g/L)
ჰემატოკრიტი = 48.7% (0.487)
ნატრიუმი = 130 mEq/L (130 mmol/L)
კალიუმი = 5.2 mEq/L (5.2 mmol/L)
BUN = 10 mg/dL (3.57 mmol/L)
კრეატინინი = 1 mg/dL (88.4 µmol/L)

ღვიძლის ბიოქიმიური და სხვა ანალიზები ნორმალურია.

Abdomen-shielded chest radiograph-მა აჩვენა ორმხივი პლევრალური ეფუზიები.

ასევე გაკეთდა ტრანსვაგინალური ულტრაბგერითი გამოკვლევა:

http://images.beef.ge

http://images.beef.ge

დასვით დიაგნოზი!

Posted by: Professor Xachikian 12 Sep 2009, 06:17
Thandrus
საკვერცხის ჰიპერსტიმულაციის სინდრომი, მე-3 ხარისხი (grade III, საკვერცხის ზომა >12სმ)

Posted by: Thandrus 12 Sep 2009, 07:07
Professor Xachikian

QUOTE
საკვერცხის ჰიპერსტიმულაციის სინდრომი, მე-3 ხარისხი


მართალია!

up.gif up.gif up.gif

რამე ახალი, please smile.gif

Posted by: Professor Xachikian 12 Sep 2009, 08:03
ეს შემთხვევა დიაგნოსტიკური თვალსაზრისით იმდენად თავსატეხი არ არის, რამდენადაც კლინიკური პრეზენტაციაა საინტერესო. vano_t-მ საინტერესო სტილი შემოგვთავაზა, და მთელი შემთხვევის აღწერას ეტაპობრივად მივყვეთ პაციენტის პრეზენტაციის განვითარებას.
თუ თვლით რომ ასეთი სტილი საინტერესო არაა, მაშინ მთლიანად დავწერ შემთხვევას.

44 წლის მამაკაცი სასწრაფო დახმარებამ მოიყვანა მიმღებში მწვავე დელირიუმით.
პაციენტს ახლო ოჯახი არ ყავს, და მეზობლემბა გამოიძახეს სასწრაფო დახმადება. პაციენტის ადრეული დაავადებების ანამნეზი, ოჯახის ანამნეზი და მედიკამენტები ცნობილი არაა. მეზობლის თქმით, პაციენტი მუშაობს მშენებლობაზე, იყო ჯანმრთელი, არაფერს არ უჩიოდა და წამლებს არ იღებდა. ასევე არ ყოფილა შემჩნეული ალკოგოლის ან რამე არალეგალური პრეპარატების მიღებაში.
მიმღებში პაციენტი იყო დელირიუმში: საკმაოდ აგზნებული, მთლიანად დიზორიენრიტებული – ვერ ასახელებდა თარიღს, ადგილსამყობელს და საკუთარ სახელს. კითხვებზე ფაქტიურად ვერ პასუხობდა და ამბობდა გაურკვეველ ფრაზებს. ასევე აღენიშნებოდა უმიზნო მოძრაობები.
გასინჯვით წვევა 88/53, გულისცემა 110, სუნთქვა 20, ტემპერატურa 36.9. გასინჯვით – ტაქიკარდია და მშრალი ლორწოვანი გარსები. (სხვა მხრივ გასინჯვით -გული, ფილტვები, მუცელი - ნორმაშია). გუგები და გუგის რეფლექსები – ნორმალური. თვალების მოძრაობა – სპონტანური, მაგრამ მოტორული დეფიციტის გარეშე. კარდიოგრამაზე – სინუსური ტაქიკარდია. გულმკერდის რენტგენოგრამა - ნორმა.

ვიმეორებ იგივე შეკითხვებს:
QUOTE
რას გაუკეთებდით ავამდყოფს? რა დიაგნოზებზე იფიქრებთ? როგორ management-ს მიყვებით? თუ დამატებითი ინფორმაცია დაგჭირდათ ისტორიიდან და ფიზიკური გამოკვლევებიდან (აგრეთვე სხვა ჩატარებული ტესტებიდან), იკითხეთ და თუ გაკეთებული იქნა გიპასუხებთ


Posted by: Thandrus 12 Sep 2009, 08:23
საინტერესოა...

Professor Xachikian

ლაბორატორიული და ტოქსინებზე ანალიზები როგორია?

Posted by: Professor Xachikian 12 Sep 2009, 08:34
1. შარდის ანალიზი ტოქსინებზე უარყოფითი.
2. შადრში გლუკოზა 4+, კეტონები 1+
3. გლუკომერტით გაზომილი გლუკოზა (სისხლში) >600
4. ბიოქიმიური პანელი: გლუკოზა 2400 მგ/დლ (ორი ათას ოთხასი), Na-126, K-5.8, HCO3 -16, Cl - 95, ანიონური სხვაობა - 15, კრეატინინი - 2.1 მგ/დლ ღვიძლის ტესტები - ნორმაში.

შემდეგი ეტაპი:

Posted by: Thandrus 12 Sep 2009, 08:54
ამ მაჩვენებლებით თუ ვიმსჯელებთ, ამ კაცს Hyperglycemic Hyperosmolar State აქვს და კიდევ დროზე მოსულა, სანამ გონება მთლად არ დაუკარგავს. შემდეგი ეტაპი ალბათ უკვე მკურნალობა უნდა იყოს: რეჰიდრატაცია + ინსულინი.

Posted by: Professor Xachikian 12 Sep 2009, 09:09
QUOTE
Hyperglycemic Hyperosmolar State

QUOTE
რეჰიდრატაცია + ინსულინი.

კი, ასეა.

პაციენტი იმდენად იყო დეჰიდრატირებული, რომ სუბკლავიის კათეტერის ჩადგმა ვერ მოხერხდა. დავაყენეთ ბარძაყის ვენური კათეტერი და დავიწყეთ ინტრავენური ინსულინი და ფიზიოლოგიური ზსნარი.
რადგან შემთხვევა სტანდარტული არ იყო, ამიტომ ფიზიოლოგიური ხსნარი მალე შეიცვალა ნახევრად-ფიზიულოგიური, გლუკოზის 5%-იანი სხნარით (იმისათვის, რომ გლუკოზა სწრაფად არ ჩამოსულიყო ნორმაში).
24 საათის შემდეგ გლუკოზა შევინარჩუნეთ ~200 მგ/დლ ფარგლებში. ადექვატური ჰიდრატაციის და მოცულობის აღდგენის შემდეგ კრეატინინი გამოსწორდა და გახდა ~1.0 მგ/დლ
მიუხედავად კალიუმის აგრესიული გადასხმისა (40 მილიექვივალენტი ყოველ 1 ლ სითხეზე), კალიუმი ერთ მომენტში დავარდა 1.8-მდე, რაც რამდენიმე საათში გამოსწორდა კალიუმის გადასხმის შემდეგ. სხვა ელექრტოლიტებიც მოწესრიგდა.
პაციენტის გონებრივი სტატუსი გამოსწორდა, და დელირიუმი მთლიანად მოიხსნა.
დაახლოებით მე-3 დღეს კრეატინინმა დაიწყო მატება და ასე, მე-6-მე-7 დღეს კრეატინინი იყო ~8 მგ/დლ, შარდოვანა ~90 მგ/დლ.
პაციენტს განუვითარდა ოლიგურია. პაციენტის მოცულობით სტატუსი არ შეცვლილა: ისევ ესხმებოდა სითხეები, არტერიული და ცენტრალური ვენური წნევა ქონდა ნორმაში.
თირკმლის უკმარისობის სავარაუდო მიზეზი:

Posted by: irakli222 12 Sep 2009, 12:09
Professor Xachikian

თირკმლის უკმარისობის მიზეზზე ვერაფერს ვიტყვი. მე უფრო ამ ფრაზამ დამაინტერესა
QUOTE
პაციენტი იმდენად იყო დეჰიდრატირებული, რომ სუბკლავიის კათეტერის ჩადგმა ვერ მოხერხდა. დავაყენეთ ბარძაყის ვენური კათეტერი

ესეთი არ გამიგია, პირიქით ვიცოდი, რომ ლავიწქვეშა ვენა არასოდეს იჩუტება

Posted by: Professor Xachikian 12 Sep 2009, 16:50
irakli222
QUOTE
ესეთი არ გამიგია, პირიქით ვიცოდი, რომ ლავიწქვეშა ვენა არასოდეს იჩუტება

ზოგადად ასეა, მაგრამ ისიც გაითვალისწინეთ, რომ ეს შემთხვევა არასტანდარტულია: გლუკოზა 2400, ამის გამო ოსმოლალობის მომატების და დეჰიდრატაციის ხარისხი
2-3 დღეში თავისუფლად მოხერხდა სუბკლავიის კათეტრის ჩადგმა (ბარძაყის ვენის კათეტერი სასურველია არ გაკეთდეს ინფექციის მაღალი რისკის გამო, და თუ გაკეთდა, ჯობია რაც შეიძლება მალე შეიცვალოს, როგორც კი ამის საშუალება იქნება).

Posted by: Thandrus 12 Sep 2009, 18:16
Professor Xachikian

QUOTE
თირკმლის უკმარისობის სავარაუდო მიზეზი:


მოკლედ, იმედია, ძალიან სისულეებს არ დავწერ gigi.gif

ალბათ, თუ ასეთ მგომარეობაში მოვიდა ავადმყოფი, ესეიგი დიაბეტი ტიპი 2-იც დიდი ხანია აქვს, თან არაკონროლირებული. ამიტომ, შეიძლება, რომ უკვე დიაბეტური ნეფროპათია, სისხლძარღვების დაზიანება და ათეროსკლეროზიც ჰქონდეს. ასეთ სიტუაციაში, 88/53 წნევას უკვე შეიძლება ტუბულების ნეკროზი გამოეწვია.

მეორე ვარიანტი - ჰიპეროსმოლური მგომარეობის გამოსწორების შემდეგ სითხის ინტრავასკულარულმა მოცულობამ ბევრად მოიმატა, ამას კი დიაბეტით დაზიანებულმა თირკმელებმა (სისხლძარღვებმა და ა.შ) ვერ გაუძლეს. ინტრაგლომერულურ ჰიპერტენზიას მოყვა თირკმლის კიდევ უფრო მეტი დაზიანება.

Posted by: Professor Xachikian 12 Sep 2009, 18:23
არა, დიაბეტი ანამნეზში არ ქონდა. რა თქმა უნდა, დიაბეტი სულ მცირე, რამდენიმე თვის წინ დაეწყო.
ჰიპოვოლემიამ თირკმლის უკმარისობა თავიდან გამოიწვია, მაგრამ მოცულობის შევსების შემდეგ თირკმლის ფუნქცია გამოსწორდა. ამის შემდეგ თირკმლის ფუნქცია საგრძნობლად გაუარესდა.
რა გამოკვლევებს გააკეთებდით?

Posted by: Dru 12 Sep 2009, 18:41
გამორიცხეთ თუ არა შარდსაწვეთების ობსტრუქცია?

Posted by: Professor Xachikian 12 Sep 2009, 18:45
Dru
კარგი იდეაა. ობსტრუქცია ყველა შემთხვევაში უნდა გამოირიცხოს. შარდსადენის ობსტრუქცია გამორიცხულია - პაციენტს ჩადგმული აქვს ფოლის კათეტერი, და სითხე თავისუფლად მოძრაობს (ფიზიოლოგიური ხსნარი).
შარდსაწვეთების ორმხრივი, ერთდროული ობსტრუქცია თუმცა იშვიათია, მაგრამ შესაძლებელი. თირკმლის და შარდსაწვეთების ულტრასაუნდმა ობსტრუქცია არ აჩვენა.

Posted by: Thandrus 12 Sep 2009, 18:49
Professor Xachikian

ურინალიზი თუ გაკეთდა, რა შედეგები აჩვენა?

Posted by: Professor Xachikian 12 Sep 2009, 18:52
Thandrus
QUOTE
ურინალიზი თუ გაკეთდა

კი. შარდი - წითელი ფერის. მიკროსკოპული ანალიზი - ჰემატურია არ არის.

Posted by: Dru 12 Sep 2009, 18:53
რა იყო შარდის ანალიზით? დასჭირდა თუ არა ჰემოდიალიზი? თუ კი რატომ? უშედეგო იყო კონსერვატული თთერაპია?


Posted by: Professor Xachikian 12 Sep 2009, 18:55
Dru
QUOTE
რა იყო შარდის ანალიზით?

ამის პასუხი ზემო პოსტში

Posted by: Thandrus 12 Sep 2009, 18:59
Professor Xachikian

QUOTE
შარდი - წითელი ფერის. მიკროსკოპული ანალიზი - ჰემატურია არ არის.


ანუ, ეს ფერი იმის გამოა, რომ ჰემოლიზი ხდება...

Posted by: Professor Xachikian 12 Sep 2009, 18:59
QUOTE
დასჭირდა თუ არა ჰემოდიალიზი? თუ კი რატომ? უშედეგო იყო კონსერვატული თთერაპია?

ჯერ მიზეზი დავადგინოთ. ამაზე პასუხი - მოგვიანებით

Posted by: Dru 12 Sep 2009, 19:02
ჰემოლოზი ხომ არ იყო? გვითხარი რა ნუ გვაწვალებ...

Posted by: Thandrus 12 Sep 2009, 19:04
თუ ჰემოლიზი არის, მაშინ ერთი იდეა - ვთქვათ, გლუკოზა-6-ფოსფატ დეჰიდროგენაზის ნაკლებობა ხომ არ აქვს? ამას შეუძლია ინტრავასკულარული ჰემოლიზის გამოწვევა და თირკმლის მწვავე უკმარისობის მოცემა. თან, როგორც ვიცი, ეს ჰემოლიზური კრიზები დიაბეტური კეტოაციდოზის დროსაა ხშირი.

Posted by: Professor Xachikian 12 Sep 2009, 19:04
Dru
Thandrus
QUOTE
რომ ჰემოლიზი ხდება...

QUOTE
ჰემოლოზი ხომ არ იყო? გვითხარი რა ნუ გვაწვალებ...

მიმართულება სწორია. შარდის ამალიზი იყო მკვეთდარ ჰემ-დადებითი.
პაციენტს ჰემოლიზი არ ქონია - ჰემოგლობინი და ჰემატოკრიტი ნორმაში იყო შენარჩუნებული, და სისხლის მიკროსკოპული ანალიზიც ნორმაში იყო. ასევე ნორმაში იყო MCV (ერითროციტების მოცულობა)

Posted by: Dru 12 Sep 2009, 19:06
შარდის წითელი ფერი არის კიდევ პორფირიის დროს...
* * *
რატომღაც დაზიანდა ინტერსტიცია და წარმოიშვა დიდი რაოდენობით პორფირინები?!....

Posted by: Thandrus 12 Sep 2009, 19:10
Professor Xachikian

მიოგლობინი თუ გაზომეთ?

შეიძლება რაბდომიოლიზის გამო განვითარდა ეს ყველაფერი. რაბდომიოლიზის გამოწვევა კი ბევრ მეტაბოლურ დარღვევას შეუძლია.

Posted by: Professor Xachikian 12 Sep 2009, 19:10
ვახ... ჭედავს ჩემთან არ მაპოსტინებს....

ეს რაც ადრე დავწერე:

Thandrus:
იდეა კარგია. თანაც, ეს დაავადება ხშირია გარკვეულ პოპულაციაში. მაგრამ რადგან ჰემოლოზი არ იყო, G-6-PD დონე არ განსაზღვრულა

Dru:
ჰაპტოგლობინის ზუსტი დონე არ მაქვს აქ, მაგრამ ამ ანალიზმა ჰემოლიზზე ეჭვი არ დაადასტურა

Posted by: Thandrus 12 Sep 2009, 19:14
Professor Xachikian

ანუ, რაბდომიოლიზიც გამოვრიცხეთ? user.gif

Posted by: Professor Xachikian 12 Sep 2009, 19:14
Thandrus
up.gif up.gif up.gif
რაბდომიოლოზი. სავარაუდოდ, გამოწვეული ჰიპოკალემიის გამო, და გავითარებული თურკმლის ტუბულარული პიგმენტოპათია.
CPK (კრეატინ-ფოსფოკინაზა) იყო ~25,000 (ნორმა 170-მდე)
პაციენტს დაეწყო ჰემოდიალიზი, და აგრესიული სითხეების გადასხმა ბიკარბონატით, შარდის PH-ის მომატების მიზნით.
მდგომარეობა რამდებიმე კვირაში გამოსწორდა.

Posted by: Thandrus 12 Sep 2009, 19:30
Professor Xachikian

bis.gif

მიდი, კიდევ რამე დადე რა smile.gif

Posted by: Thandrus 9 Oct 2009, 20:52
აუჰ... თითქმის ერთი თვეა არავის შემოუხედავს ამ თემაში... აბა, მოდით, ერთად გამოვაცოცხლოთ smile.gif

აბა, თუ მეტყვით ამ კაცს რა ჭირს smile.gif

Posted by: Blind_Torture_Kill 9 Oct 2009, 21:52
მარაზმია მარა რა ფერია
ესეთი რაღაცა პირველად ვნახე

Posted by: Thandrus 9 Oct 2009, 22:05
მარაზმი გასაგებია, მაგრამ ეს ფერი რამ გამოიწვია?

Posted by: Romina 10 Oct 2009, 21:17
გამარჯობა ყველას smile.gif ძალიან მომენატრა აქ ყოფნა.

ერთ მარტივ ქეისს დავდებ, ოღონდ ნამდვილია :-))))))) თან ისეთი, სულ რომ დაგამახსოვრდება, იმიტომ რომ ამ დიაგნოზზე ეჭვი არ მივიტანე და შეცდომა იყო, უპატიებელი :-)

კლინიკაში მოვიდა 56 წლის ქალი. ანამნეზში 8 თვიანი ცხელება. წონაში დაიკლო ამ ხნის განმავლობაში დაახლოებით 30 კგ. 7 თვის წინ დაესვა ფილტვის ტუბერკულოზის დიაგნოზი, რომელსაც უმკურნალეს. ამ ეტაპზე ამ მხრივ ჩივილები აღარ აღენიშნება. სიმპტომებიდან კიდევ ყურადღებას იქცევდა ძლიერი საერთო სისუსტე. სისხლის საერთო ანალიზში ედს-40. დანარჩენი მაჩვენებლები ნორმა. შარდის საერთო ანალიზი - ნორმა. ყველა რევმატული სინჯი ნორმა იყო. ანუ სისტემური დაავადებები იმ ეტაპზე გამოირიცხა. ჩვენთან მოსვლამდე ჩატარებული ჰქონდა ძვლის ტვინის პუნქცია და მუცლის და გულმკერდის ღრუს კტ. რაიმე პათოლოგია არ გამოვლინდა. პაციენტმა აღნიშნა რომ გაურკვეველი დაავადებით გარდაეცვალა მეუღლე 1 წლის წინ, საქართველოს ერთ-ერთი რაიონის საავადმყოფოში. სავარაუდო დიაგნოზი იყო მენინგიტი.

მოკლედ ჩვენ დიაგნოზი ვერ დავსვით და რეკომენდაცია მივეცით რომ ისევ დაბრუნებულიყო ფტიზიატრთან. იმიტომ რომ გულმკერდის რენტგენი ჩვენც გავიმეორეთ და ეჭვი მივიტანეთ ტუბერკულოზის რეციდივზე.

2 თვის შემდეგ სულ სხვა კლინიკაში შემთხვევით ვნახე ეს პაცინტი და გავიგე დიაგნოზი. თქვენ რას იფიქრებდით?

Posted by: Thandrus 11 Oct 2009, 01:51
Romina

სიმსივნე ხომ არა? user.gif

Posted by: Masked 11 Oct 2009, 03:41
QUOTE (Thandrus @ 9 Oct 2009, 20:52 )
აუჰ... თითქმის ერთი თვეა არავის შემოუხედავს ამ თემაში... აბა, მოდით, ერთად გამოვაცოცხლოთ smile.gif

აბა, თუ მეტყვით ამ კაცს რა ჭირს smile.gif

ცYანოზე, არა დავადება, არამედ არის სიმპტომი ფილტვების , სასუნტქი ორგანოების ასევე გულის დავადებეის დროს.
ამ შემთხვევაში პაციენტი ვარაუდიტ არის დაბადებული ან ადრე შეცენილი გულისდავადებით, ძვლების შტ. განვიტარება მაძლევს ვიპიქრო ეს.

Posted by: Thandrus 11 Oct 2009, 04:47
Masked

არა, ციანოზი არაა smile.gif

Posted by: Romina 11 Oct 2009, 08:38
Thandrus

სიმსივნე არ ჰქონდა. მართალია გამოკვლევების მიუხედავად დიდი შანსია მაინც სადმე გამოგეპაროს სიმსივნე, მაგრამ ამ კონკრეტულ შემთხვევაში არ იყო. :-)
აბა კიდე დაფიქრდი.

Posted by: Thandrus 11 Oct 2009, 17:40
Romina

ჰმ... თირეოტოქსიკოზი მაშინ...

Posted by: donvaso 11 Oct 2009, 17:45
Romina
AIDS?????????????

Posted by: Thandrus 11 Oct 2009, 17:49
donvaso

QUOTE
AIDS?????????????


ამ შემთხვევაში არამგონია ფილტვში ლოკალიზებულ ტუბერკულოზთან გვქონოდა საქმე...

Posted by: donvaso 11 Oct 2009, 17:53
Thandrus
შიდსის სიმპტომები
არამოტივირებული ცხელება;
ძლიერი საერთო სისუსტე;
წონაში კლება სხეულის მასის 10%-ით და მეტად;
ოფლიანობა;
ლიმფური კვანძების გადიდება;
ფაღარათი;
სხვადასხვა სახის გამონაყარი კანსა და ლორწოვან გარსებზე;
თეთრი ფერის ნადები ენაზე;
გახანგრძლივებული ფილტვების ანთება, რომელიც ცუდად ექვემდებარება ჩვეულებრივ ანტიბაქტერიულ თერაპიას;
ნერვული სისტემის სხვადასხვა სახის დაზიანება (დემენცია, მენინგიტი, ენცეფალიტი, მიელოპათია, პერიფერიული ნეიროპათია) და სხვა.

პირველი რაც მომივიდა თავში ეგ იყო.... აი ბევრი რამე ემთხვევა თან.... smile.gif

Posted by: Romina 11 Oct 2009, 18:35
donvaso
Thandrus

კი შიდსი იყო smile.gif ყოჩაღ. რამდენიმე თვის შემდეგ შემთხვევით ვნახე ინფექციურში. ძალიან დამწყდა გული, რომ არ ვიფიქრე ამ დაავადებაზე. რისკ ჯგუფებში არ შედიოდა, საქართველოს ერთ-ერთი რაიონის სოფელში ცხოვრობდა და იმდენად ვერ წარმოვიდგენდი, რომ არც კი მიფიქრია, რაც ნამდვილად არასწორი მიდგომაა. ტუბერკულოზის არსებობა, სხვა თანმხლები დაავადების გარეშე და ასეთი საეჭვო ანამნეზი ქმრის შესახებ საკმარისი უნდა ყოფილიყო დასაეჭვებლად :-(. ქმარს ალბათ პირდაპირ ნეირო-შიდსი გამოუვლინდა და მალევე მოკვდა ზუსტი დიაგნოზის დასმამდე.

ჩვენ რომ გავგზავნეთ პულმონოლოგთან, იმან ნახა რომ ტუბერკულოზის რეციდივი იყო მკურნალობის შემდეგ და ის დაეჭვდა, რომ შეიძლება შიდსი ყოფილიყო. გაგზავნა ინფექციურში და იქ დადასტურდა. კატასტროფულად დაბალი ჰქონდა T- ჰელპერების რიცხვი. ეხლა უკვე კარგად არის.

Posted by: Thandrus 11 Oct 2009, 18:54
Romina

საინტერესო იყო smile.gif

წინა გვერდზე დადებულ სურათს გადმოვიტან:

რა სჭირს ამ კაცს?


Posted by: donvaso 11 Oct 2009, 19:12
Romina
პირველი რაც იყო ეგ მომაფიქრდა რაც დაწერე იქიდან გამომდინარე....
smile.gif


Thandrus
რაღაც ქრონიკული ინტოქსიკაცია ხო არ არის, მაგ; ტყვიით, ან რკინით...
რიავი...
ისე, მაგარ ფერზე როა ფაქტია... biggrin.gif biggrin.gif

Posted by: Romina 11 Oct 2009, 19:23
Thandrus

QUOTE
საინტერესო იყო

მადლობა 2kiss.gif
donvaso
QUOTE
რაღაც ქრონიკული ინტოქსიკაცია ხო არ არის, მაგ; ტყვიით, ან რკინით...


მართლა გავს რაღაც მეტალის დაგროვებას მოწამვლის ან ნივთიერებათა ცვლის მოშლის გამო. (რკინის მაგალითად). მაგრამ ასე მთლიანად კანის შეფერვა არასდროს მინახია
smile.gif
* * *
Thandrus

გვითხარი რა რა პასუხი, დავიტანჯე ამ "გალურჯებული" კაცის ყურებით :-)

Posted by: Thandrus 11 Oct 2009, 22:28
donvaso
Romina

QUOTE
რაღაც ქრონიკული ინტოქსიკაცია ხო არ არის, მაგ; ტყვიით, ან რკინით...


QUOTE
მართლა გავს რაღაც მეტალის დაგროვებას მოწამვლის


სწორ გზას ადგახართ smile.gif

კიდევ ცოტა ხანიც იფიქრეთ, მერე კი გეტყვით smile.gif

Posted by: Romina 11 Oct 2009, 22:42
Thandrus
QUOTE
სწორ გზას ადგახართ


მე ეხლა გამოშტერების გზას ვადგავარ და ხვალ ვიფიქრებ კიდე, თუ მანამდე არ გამოიცნეს smile.gif

Posted by: PHOBOSS 11 Oct 2009, 23:22
QUOTE (Thandrus @ 11 Oct 2009, 22:28 )
donvaso
Romina

QUOTE
რაღაც ქრონიკული ინტოქსიკაცია ხო არ არის, მაგ; ტყვიით, ან რკინით...


QUOTE
მართლა გავს რაღაც მეტალის დაგროვებას მოწამვლის


სწორ გზას ადგახართ smile.gif

კიდევ ცოტა ხანიც იფიქრეთ, მერე კი გეტყვით smile.gif

რარაც შხამს გავს რაც მეტჰემოგლობინს წარმოქმნის, შეიძლება ბენზოლის ან ნიტრიტების წარმოებულია რამე?

Posted by: Thandrus 11 Oct 2009, 23:35
PHOBOSS

არა, ეგეთი შხამი არაა. თან ეს მდგომარეობა, კანისა და თვალების ასეთი ფერის მიუხედავად, სიცოცხლეს საფრთხეს არ უქმნის.

Posted by: PHOBOSS 12 Oct 2009, 01:04
QUOTE (Thandrus @ 11 Oct 2009, 23:35 )
PHOBOSS

არა, ეგეთი შხამი არაა. თან ეს მდგომარეობა, კანისა და თვალების ასეთი ფერის მიუხედავად, სიცოცხლეს საფრთხეს არ უქმნის.

მაშინ არგირია ჭირს მაგ სიმპატიურ კაცს gigi.gif ვერცხლით "მოწამლულა" gigi.gif

Posted by: Thandrus 12 Oct 2009, 02:07
PHOBOSS

QUOTE
მაშინ არგირია ჭირს მაგ სიმპატიურ კაცსვერცხლით "მოწამლულა"


აბსოლუტურად მართალია! up.gif up.gif up.gif

არგირიას ვერცხლის შემცველი პრეპარატების ხანგრძლივი დროის განმავლობაში მიღება იწვევს. რადგან ამ კაცს თვალებიც ჩალურჯებული აქვს, ამას შეიძლება არგიროზიც ეწოდოს. სურათი ვიკიპედიიდან იყო აღებული, მგონი თქვენთვისაც საინტერესო იქნება ამ პატარა სტატიის წაკითხვა (იმის გათვალისწინებით, რომ წიგნებშიც არ წერია ამაზე ბევრი)

http://en.wikipedia.org/wiki/Argyria

აი ამ ლინკზეც რამდენიმე ლურჯი ადამიანის სურათი biggrin.gif

http://images.google.com/images?hl=en&safe=off&client=opera&rls=en&um=1&sa=1&q=argyria&aq=0&oq=Argyr&aqi=g7&start=0

Posted by: PHOBOSS 12 Oct 2009, 10:48
ერთ პატარა ქეისს მეც დავდებ smile.gif
პაციენტი 62 წლის ქალი შემოვიდა კლინიკაში ეპიგასტრიუმში ძლიერი ტკივილით, გულისრევით ღებინებით. ტკივილი დაეწყო მეზობელთან კამათის შემდეგ. თანმხლები დაავადებები: არტერიული ჰიპერტენზია, შაქრიანი დიაბეტი ტიპი II. კლინიკაში შემოსვლისას არტ. წნევა 140/90მმ, პულსი - 80. კანი მკრთალი, ლორწოვანები ვარდისფერი. ფილტვების აუსკულტაციით რაიმე პათოლოგია არ დაფიქსირდა. პაციენტს ტკივილი მოეხსნა 10 მგ მორფინის ი.ვ. შეყვანის შემდეგ.
ე.კ.გ.:http://radikal.ru/F/s19.radikal.ru/i192/0910/c7/d8b36165245a.jpg.html
ექოკარდიოგრაფია: http://radikal.ru/F/i059.radikal.ru/0910/5c/3c1f95eedf75.jpg.html
კორონარული ანგიოგრაფია და ვენტრიკულოგრაფია:http://radikal.ru/F/s47.radikal.ru/i118/0910/5a/f57fe16d4758.jpg.html
კარდიომარკერები: ტკივილის დასაწყისიდან12 საათში troponin I – 0,046mkg/l; კრეატინფოსფოკინაზა MB ფრაქცია – 31 ერთ/ლ. ტკივილის დასაწყისიდან 24საათის შემდეგ კრეატინფოსფოკინაზა MB ფრაქცია – 27 ერთ/ლ.
თქვენი დიაგნოზი?

P.S. შემთხვევიდან 2 თვის შემდეგ პაციენტს ჩაუტარდა განმეორებითი ექოკარდიოგრაფია, რაზეც რაიმე პათოლოგიური ცვლილება არ დაფიქსირდა.

Posted by: Thandrus 12 Oct 2009, 15:19
PHOBOSS

კარდიოგრამიდან გამომდინარე. წინა კედლის იშემიის ეპიზოდი მგონია. კლინიკა და ანალიზებიც, ჩემი აზრით, ამაზე მიუთითებს.

Posted by: PHOBOSS 12 Oct 2009, 16:24
QUOTE (Thandrus @ 12 Oct 2009, 15:19 )
PHOBOSS

კარდიოგრამიდან გამომდინარე. წინა კედლის იშემიის ეპიზოდი მგონია. კლინიკა და ანალიზებიც, ჩემი აზრით, ამაზე მიუთითებს.

კარდიომარკერების მაჩვენებლები ძალიან დაბალია მიოკარდიუმის ნეკროზისთვის, 2 ჯერ მაინც უნდა იყოს კრეატინფოსფოკინაზა მომატებული (ნორმა 24ერთ/ლ) თუ მართლა ინფარქტია. თან კორნაროგრაფიას დააკვირდი, წინა კედელს რა არტერიაც კვებავს აბსოლუტურად ჯანმრთელია. ერთი სიტყვით ინფარქტი არაა wink.gif

Posted by: Thandrus 12 Oct 2009, 17:18
PHOBOSS

ინფარქტი არ მითქვამს, იშემია ვიგულისხმე, ინფარქტის გარეშე. მაშინ მარკერები არ იქნებოდა მომატებული. მაგრამ, რადგან ამბობ, რომ წინა კედლის მკვებავი არტრია ჯანმრთელია, ესეიგი ეს არ არის...

ექოზე, მგონი გული ყველა სურათში ზომით თითქმის ერთნაირია... კონსტრიქციული პერიკარდიტი ხომ არაა? gigi.gif

Posted by: PHOBOSS 12 Oct 2009, 17:36
QUOTE (Thandrus @ 12 Oct 2009, 17:18 )
PHOBOSS

ინფარქტი არ მითქვამს, იშემია ვიგულისხმე, ინფარქტის გარეშე. მაშინ მარკერები არ იქნებოდა მომატებული. მაგრამ, რადგან ამბობ, რომ წინა კედლის მკვებავი არტრია ჯანმრთელია, ესეიგი ეს არ არის...

ექოზე, მგონი გული ყველა სურათში ზომით თითქმის ერთნაირია... კონსტრიქციული პერიკარდიტი ხომ არაა? gigi.gif

არა, პერიკარდიტი არაა, პერიკარდიუმს თითქმის ვერც კი შენიშნავ ამ სურათებზე, ანუ ნორმაში აქვს. კარგად დააკვირდი ექოსკოპიის სურათს და ვენტრიკულოგრაფიას, მარცხენა პარკუჭი იკუმშება მაგრამ მთლად ნორმალურად არა. აი ზუსტად მანდ უნდა ეძებო პასუხი smile.gif
პატარა მინიშნებას მოგცემ: ეს არაა რაიმე გავრცელებული დაავადება, ძალიან იშათი ფენომენია და უცნაური სახელი ქვია. smile.gif

Posted by: mtvareuli 12 Oct 2009, 18:38
PHOBOSS
QUOTE
უცნაური სახელი ქვია

ტაკოცუბოს კარდიომიოპათია? biggrin.gif

Posted by: PHOBOSS 12 Oct 2009, 19:39
QUOTE (mtvareuli @ 12 Oct 2009, 18:38 )
PHOBOSS
QUOTE
უცნაური სახელი ქვია

ტაკოცუბოს კარდიომიოპათია? biggrin.gif

სწორია!!!! ხო მაგარი სახელი აქვს? gigi.gif ტაკოცუბოს კარდიომიოპათია ანუ სტრესული კარდიომიოპათია, ანუ აპიკალური ბალონური სინდრომი. ინტაქტური კორონარების ფონზე მწვავე კორონარული სინდრომის სურათი და გარდამავალი აპიკალური აკინეზია ახასიათებს smile.gif

Posted by: Romina 12 Oct 2009, 20:04
PHOBOSS

მეც კი ვიცოდი ეგ
boli.gif smile.gif smile.gif

Posted by: PHOBOSS 12 Oct 2009, 22:46
QUOTE (Romina @ 12 Oct 2009, 20:04 )
PHOBOSS

მეც კი ვიცოდი ეგ
boli.gif  smile.gif  smile.gif

gigi.gif ვიცი რომ იცოდი, ვამაყობ ასეთი ღირსეული მოსწავლე რომ მყავდა ე.კ.გ.-ს სფეროში jump.gif ამას ისეთი სახელი აქვს რომ ერთხელ თუ მოისმინე არასდროს დაგავიწყდება.
http://radikal.ru/F/s57.radikal.ru/i155/0910/f0/6f28509c5a31.jpg.html

Posted by: Romina 12 Oct 2009, 22:48
PHOBOSS

QUOTE
ღირსეული მოსწავლე

ოროსანი მოსწავლე
biggrin.gif biggrin.gif

Posted by: PHOBOSS 12 Oct 2009, 22:55
QUOTE (Romina @ 12 Oct 2009, 22:48 )
PHOBOSS

QUOTE
ღირსეული მოსწავლე

ოროსანი მოსწავლე
biggrin.gif biggrin.gif

lol.gif lol.gif საეტაპო გამოცდა როგორ ვერ მოვაწყვეთ ბლოკადების ბოლოს cry.gif cry.gif ვიცი როგორ გიყვარს ჩათვლები და გამოცდები gigi.gif

Posted by: Romina 12 Oct 2009, 22:59
PHOBOSS

biggrin.gif biggrin.gif ხო, 1 კურსზე დავრჩი biggrin.gif თემიდან გადახვევას აქვს ადგილი და დავივორნებით smile.gif

Posted by: PHOBOSS 12 Oct 2009, 23:17
QUOTE (Romina @ 12 Oct 2009, 22:59 )
PHOBOSS

biggrin.gif  biggrin.gif ხო, 1 კურსზე დავრჩი  biggrin.gif თემიდან გადახვევას აქვს ადგილი და დავივორნებით  smile.gif

მე მაპატიეთ, ჩემი ბრალია vik.gif
* * *
აბა მოვიდა კიდევ ერთი მარტივი შემთხვევა smile.gif პაციენტი 69 წლის ქალი, ძირითადი ჩივილი ხშირი სისხლდენა ცხვირიდან, რომელიც კონსერვატული თერაპიით რამდენიმე წუთში იხსნება. ოჯახში დედას და დას ჰქონდათ ხშირი სისხლდენა ცხვირიდან და იშვიათად მელენა.
გულმკერდის რენტგენოგრამა: http://www.radikal.ru
http://radikal.ru/F/s05.radikal.ru/i178/0910/63/ba99f36b03c0.jpg.html
კონტრასტული კ.ტ.: http://www.radikal.ru
http://radikal.ru/F/s56.radikal.ru/i151/0910/db/0d3015fd31e0.jpg.html


თქვენი დიაგნოზი?

Posted by: Romina 13 Oct 2009, 16:40
PHOBOSS
არ ვიცი, პირველი რაც მაფიქრდება ვეგენერის გრანულომატოზია, მაგრამ დედა და და მაეჭვებს და მელენა smile.gif smile.gif

Posted by: PHOBOSS 13 Oct 2009, 21:25
QUOTE (Romina @ 13 Oct 2009, 16:40 )
PHOBOSS
არ ვიცი, პირველი რაც მაფიქრდება ვეგენერის გრანულომატოზია, მაგრამ დედა და და მაეჭვებს და მელენა smile.gif smile.gif

no.gif no.gif no.gif არა, ეგ არაა gigi.gif ფილტვის რენტგენოგრამაზე რას ხედავ ბარემ გვითხარი wink.gif

Posted by: Mrs_Zum 13 Oct 2009, 23:44
გუდპასჩერი? :| ..........................................

Posted by: PHOBOSS 14 Oct 2009, 00:17
QUOTE (Mrs_Zum @ 13 Oct 2009, 23:44 )
გუდპასჩერი? :| ..........................................

არრრა no.gif გუდპასჩერს ცხვირიდან სისხლდენა არ ახასიათებს. კლინიკუად ვლინდება ძირითადად ალვეოლური ჰემორაგია, ჰემოპტოე, ჰილუსებთან ახლოს ინფილტრატები გლომერულოპათია. კიდე თუ სწორად მახსოვს (გაიხაროს ნათელა ლექტორმა love.gif ) მემკვიდრული არაა და არც მელენას იწვევს smile.gif

Posted by: Mrs_Zum 14 Oct 2009, 01:28
მაინც დავწერე რა იცი რა ხდება biggrin.gif

Posted by: PHOBOSS 14 Oct 2009, 01:35
QUOTE (Mrs_Zum @ 14 Oct 2009, 01:28 )
მაინც დავწერე რა იცი რა ხდება biggrin.gif

კარგია yes.gif ხო იცი კამათში იბადება ჭეშნარიტებაო. ექიმმა ყველაფერზე უნდა იფიქროს უამრავი რამ ივარაუდოს, მერე იქნებ გაუმართლოს და დასვას სწორი დიაგნოზი gigi.gif მიდი აბა შემდეგი ვარაუდი დაწერე gigi.gif

Posted by: Romina 14 Oct 2009, 21:15
PHOBOSS

ხო ხედავ შენი ქეისი არავის არ მოეწონა! tongue.gif

Posted by: PHOBOSS 14 Oct 2009, 21:28
ხო მასეა sad.gif არადა საკმაოდ მარტივი ქეისია. მემკვიდრული ჰემორაგიული ტელეანგიექტაზიაა დიაგნოზი, რანდიუ-ოსლერის დაავადება. წავედი მაშინ მეც cry.gif

Posted by: tergauli 14 Oct 2009, 21:45
S.O.S
იქნებ ვინმემ იცოდეს სად აკეთებენ ყველაზე უტყუარ სისხლის ანალიზს თბილისში ან საქართველოში

Posted by: Romina 14 Oct 2009, 21:58
PHOBOSS
QUOTE
წავედი მაშინ მეც 



smile.gif smile.gif არ იტირო, მცხეთობას ზეიმობენ ალბათ და იმიტომ არ შემოიჭყიტეს. ფილტვში მრგვალი ჩრდილი დაბლა რა არის ვერ გავიგე მაინც.

tergauli
რა ანალიზს, საერთოს თუ გულისხმობთ ბევრგან კეთდება კარგად. მაგალითად ლაბორატორია "მრჩეველში" - მის. ალ. ყაზბეგის 9.

Posted by: PHOBOSS 14 Oct 2009, 22:03
QUOTE (Romina @ 14 Oct 2009, 21:58 )
PHOBOSS
QUOTE
წავედი მაშინ მეც 



smile.gif smile.gif არ იტირო, მცხეთობას ზეიმობენ ალბათ და იმიტომ არ შემოიჭყიტეს. ფილტვში მრგვალი ჩრდილი დაბლა რა არის ვერ გავიგე მაინც.


არტერიო-ვენოზური მალფორმაციაა, არტერიაც და ვენაც კარგად ჩანს რენტგენზე, კ.ტ.-ზეც მალფორმაციის კარგი კონტრასტირებაა.

P.S. მცხეთობას წესით მცხეთელებიც უნდა აღნიშნავდნენ wink.gif

Posted by: tergauli 14 Oct 2009, 22:03
Romina

საერთოსაც და სისხლის ბაქტეოროლოგიურ ანალიზს,დათესვის ჩათვლით (ოღონდ სეფსისის ცენტრი არა mad.gif )

Posted by: Romina 14 Oct 2009, 22:13
tergauli
"ციტოში" აკეთებენ ბაქტერიოლოგიურ ანალიზს კარგად, მაშინ საერთოც მანდ გაიკეთეთ. ფალიაშვილზე გადავიდა მგონი ეგ ლაბორატორია. ზუსტად არ ვიცი.
* * *
PHOBOSS
AVM როა გავიგე. მაგრამ დაბლა ანევრიზმაა?

Posted by: Rain Dog 15 Oct 2009, 20:54
QUOTE
ხო ხედავ შენი ქეისი არავის არ მოეწონა!


რატომ, მე მომეწონა ძალიან jump.gif ეჰ, რა დროს გამებრიდა კომპი sad.gif ტაკოცუბოს კარდიომიოპათია კი გასამეორებელი მაქვს... აღარ მახსოვდა როგორ გამოიყურებოდა ექოზე biggrin.gif

P. S.

Thandrus-ი ვარ smile.gif

Posted by: Romina 15 Oct 2009, 20:58
Rain Dog

გილოცავ ახალ ამპლუას
smile.gif smile.gif

Posted by: Rain Dog 15 Oct 2009, 20:59
Romina

ამპლუა არა, სხვისი ნიკია gigi.gif

Posted by: PHOBOSS 16 Oct 2009, 11:06
QUOTE (Romina @ 14 Oct 2009, 22:13 )
PHOBOSS
AVM როა გავიგე. მაგრამ დაბლა ანევრიზმაა?

ეგ სფერული ჩრდილია ზუსტად მალფორმაცია. ორი ზოლი რომ მიდის კარისკენ ეგაა არტერია და ვენა. თუ დააკვირდები რენტგენზე მთლად ჰოჰომგენური არაა სფერო, ამით შეიძლება განასხვავო ანევრიზმისგან.

Posted by: mtvareuli 16 Oct 2009, 13:47
როგორც ვხედავ, ახალი ქეისი არ არის, ამიტომ მე დავდებ ერთ სურათს.


რას ხედავთ, რა ჰქვია და რომელ დაავადებას ახასიათებს



Posted by: mtvareuli 16 Oct 2009, 13:49
მეორესაც დავდებ ყოველი შემთხვევისთვის..........



Posted by: agnia 16 Oct 2009, 18:18
Wilson's disease


რა არის სურათზე, რომელი დაავადებაა?

http://www.radikal.ru

Posted by: Blind_Torture_Kill 19 Oct 2009, 21:15
mtvareuli
კაიზერ ფლეშერის რგოლი

agnia
ბაბეზიოზი

Posted by: agnia 19 Oct 2009, 23:42
Blind_Torture_Kill
QUOTE
ბაბეზიოზი



no.gif

ცოტა რთულად მისახვედრი სურათი იყო, ახლა უფრო ადვილს დავდებ smile.gif

დაავადება და გამომწვევი
http://www.radikal.ru


Posted by: Blind_Torture_Kill 19 Oct 2009, 23:57
agnia

მალარია მაშინ თუ ესეც არაა მაშინ კლინიკური სურათიც მიაშველე

Posted by: agnia 20 Oct 2009, 07:46
Blind_Torture_Kill

კი, მალარიაა, ტროპიკული მალარია. სურათზე ჩანს რომ falciparum-ია smile.gif

Posted by: Blind_Torture_Kill 20 Oct 2009, 09:12
................................................
აბა ექიმებო აქ რა გვაქვს ?

Posted by: Thandrus 22 Oct 2009, 15:11
Blind_Torture_Kill

Dawson's Fingers? user.gif

Posted by: Blind_Torture_Kill 22 Oct 2009, 20:44
Thandrus
QUOTE
Dawson's Fingers?



no.gif

Posted by: vano_t 23 Oct 2009, 00:12
60 წლის მამაკაცი მოდის კლინიკაში ჩივილების გარეშე. წელიწადში 2-ჯერ ეს ავადმყოფი იკეთებს ლაბორატორიებს. თავს კარგად გრძნობს. ფიზიკური გამოკვლევაც ნორმალურია.

ალერგიები არ აქვს

იღებს: ლიზინოპრილს, მეტოპროლოლს, ჰიდროქლორთიაზიდს, სიმვასტატინს.

ლაბორატორიები აჩვენებს შემდეგს: გაკეთდა che16, რომელშიც შედის თითქმის ყველა ძირითადი გამოკვლევები. აქედან K იყო 5.6 (ნორმა 5-ის დაბლა); Ca იყო 10.5 (ნორმა 10-ის დაბლა), კრეატინინი და გამოთვლილი GFR (გლომელურების ფილტრაციის სიჩქარე) ნორმაშია. CBC-იც (სისხლის უჯრედების დათვლა) ნორმაშია.

რა არის დიაგნოზი? რა გამოკვლევებს გაუკეთებთ და როგორ უმკურნალებთ ავადმყოფის?

Posted by: LUKA-BRAZI 24 Oct 2009, 10:32
vano_t
ვანო! როგორ ხარ? smile.gif რამდენი ხანი არ შევხვედრილავრთ ფორუმის მასშტაბით smile.gif) რა ამბებია შენსკენ? smile.gif

Blind_Torture_Kill
დათო, ბოლო-ბოლო რას ვშვებით იმ ამბავზე? user.gif რას აპირებ? აპირებ რამეს საერთოდ?

Posted by: vano_t 26 Oct 2009, 09:22
LUKA-BRAZI
QUOTE
vano_t
ვანო! როგორ ხარ? smile.gif რამდენი ხანი არ შევხვედრილავრთ ფორუმის მასშტაბით smile.gif) რა ამბებია შენსკენ? smile.gif

კარგად ვარ ლუკა. იცოცხლე მოკითხვისათვის (და ისედაც შე კაცო wink.gif). თავად როგორ გიკითხო ფორუმის მასშტაბებით? smile.gif

Posted by: LUKA-BRAZI 26 Oct 2009, 14:21
vano_t
QUOTE
კარგად ვარ ლუკა. იცოცხლე მოკითხვისათვის (და ისედაც შე კაცო wink.gif). თავად როგორ გიკითხო ფორუმის მასშტაბებით? smile.gif

მეც არამიშავს ვანო. მერე რაღაც მინდა გკითხო.... smile.gif

Posted by: Thandrus 26 Oct 2009, 14:40
vano_t

ჩემი აზრით, საინტერესო პაციენტია. აქ მოყვანილი ლაბორატორიული მონაცემებით ვივარაუდებდი, რომ ჰიპერკალცემიის მიზეზი ჰიდროქლორთიაზიდია, ჰიპერკალემიის - თიაზიდებით გამოწვეული ჰიპოკალემიისათვის თავის არიდების მიზნით დანიშნული ელანაპრილი.
* * *
LUKA-BRAZI

და შენ თვითონ სად ხარ დაკარგული? biggrin.gif წინა გვერდებზე არ ჩანხარ... smile.gif

Posted by: vano_t 26 Oct 2009, 19:48
Thandrus
QUOTE
vano_t

ჩემი აზრით, საინტერესო პაციენტია. აქ მოყვანილი ლაბორატორიული მონაცემებით ვივარაუდებდი, რომ ჰიპერკალცემიის მიზეზი ჰიდროქლორთიაზიდია, ჰიპერკალემიის - თიაზიდებით გამოწვეული ჰიპოკალემიისათვის თავის არიდების მიზნით დანიშნული ელანაპრილი.
* * *

არა, ამ შემთხვევაში არ არის ასე. ზოგადად კი შეუძლია დიურეტიკს გამოიწვიოს ჰიპერკალცემია. ენალაპრილის ჩვენება არ არის მაგი, რომ ჰიპოკალემიის თავის არიდების მიზნით დაინიშნოს. მაგ დროს კალიუმი შეგიძლია დაუმატო უბრალოდ.

როგორ მიუდგებოდი ასეთი ავადმყოფს? ცოტა "ნამიოკს" მოგცემ. ადრე ეს პრობლემა არასდროს არ ქონია ამ ავადმყოფს (არც ჰიპერკალცემია და არც ჰიპერკალემია).

Posted by: LUKA-BRAZI 26 Oct 2009, 20:18
vano_t
ფარისებრახლო ჯირკვლის პრობლემები აქვს ვითომ? user.gif

Thandrus
სანდროს გაუმარჯოს, როგორ ხარ? smile.gif)

Posted by: Thandrus 26 Oct 2009, 20:39
LUKA-BRAZI

რავიცი, არა მიშავს რა... დაიწყე მომზადება?

vano_t

ლუკასნაირად, მეც, ჯერ-ჯერობით, PTH-ს გავზომავდი

Posted by: vano_t 26 Oct 2009, 20:54
LUKA-BRAZI
QUOTE
vano_t
ფარისებრახლო ჯირკვლის პრობლემები აქვს ვითომ? user.gif

არა

მოკლედ, დაწერეთ რას გაუკეთებდით ამ პაციენტს-რა ტესტებს, რა გამოკითხვებს, რა გამოკვლევებს და როგორ უმკურნალებდით.

Posted by: LUKA-BRAZI 26 Oct 2009, 21:24
Thandrus
ალბათ ნოემბრიდან შევუდგები...... smile.gif

Posted by: Thandrus 26 Oct 2009, 21:50
კარგი, მაშინ ნელ-ნელა მივყვები: ისტორიაში რამე მნიშვნელოვანი ხომ არ ჰქონია პაციენტს (ძვლის დავადებები, სიმსივნე... ვიტამინებს ხომ არ იღებდა)? სისხლში ალბუმინის კონცენტრაცია რამდენია (მისმა შემცირებას შეუძლია გამოიწვიოს სისხლში თავისუფალი კალიუმისა და კალციუმის დონის მომატება)?

Posted by: vano_t 26 Oct 2009, 22:29
QUOTE (Thandrus @ 26 Oct 2009, 21:50 )
კარგი, მაშინ ნელ-ნელა მივყვები: ისტორიაში რამე მნიშვნელოვანი ხომ არ ჰქონია პაციენტს (ძვლის დავადებები, სიმსივნე... ვიტამინებს ხომ არ იღებდა)? სისხლში ალბუმინის კონცენტრაცია რამდენია (მისმა შემცირებას შეუძლია გამოიწვიოს სისხლში თავისუფალი კალიუმისა და კალციუმის დონის მომატება)?

არა, ჯანმრთელი ავადმყოფია. გარდა ჰიპერტენზიისა პრობლემა არ აქვს. ალბუმინიც ნორმაში იყო.

Posted by: LUKA-BRAZI 26 Oct 2009, 23:44
vano_t
აბა ბევრი ბანანი, კარტოფილი და რძე მიიღო? biggrin.gif მოკლედ ვანო ძაღლის თავი მგონი მის წამლებშია. საერთოდ რატომ იღებს მაგ წამლებს? ჰიპერტენზიის სამკურნალოდ ლიზინოპრილი გასაგებია. ბეტა-ბლოკერი რატომ აქვს დანიშნული? სიმვასტატინი, ალბათ ჰიპერლიპიდემიის გამო, მაგრამ შარდმდენი? მაგ პაციენტს გულის მხრივ ჩივილები არ აქვს? იქნებ აქვს შეგუბებითი მოვლენებიც?

მეორე ვარიანტი: ჰიპერკალცემია გამოწვეულია D ვიტამინის სიჭარბით (?), შესაბამისად მეტი კალციუმის ათვისება ხდება. ან თიაზიდების ბრალია. ასევე დეჰიდრატაციის. ამ შემთხვევაში სისხლში წყლის დონის დაქვეითება ნიშნავს ელექტროლიტების დონის აწევას.... მოკლედ.....

Posted by: Thandrus 26 Oct 2009, 23:55
LUKA-BRAZI

პარალელურად ვფიქრობთ რა lol.gif

დაახლოებით ეგრე ვფიქრობ, მაგრამ რამდენიმე მომენტია, ჩემი აზრით: ბანანი კარტოფილი, რძე და ვიტამინი D მხოლოდ ჰიპერკალცემიას ახსნის. დეჰიდრატაციის შემთხვევაში კი, კალიუმისა და კალციუმის გარდა სხვა ელექტროლიტების კონცენტრაციაც მომატებული იქნებოდა...

თუმცა მაინც, შემდეგ ტესტად, ვიტამინ D-ს კონცენტრაცია რომ გავზომოთ, კარგი იქნება smile.gif
* * *
QUOTE
მოკლედ ვანო ძაღლის თავი მგონი მის წამლებშია


სულ პირველად მაგაზე დავფიქრდი - თიაზიდებს ჰიპერკალცემიის გამოწვევა შეუძლიათ, ელანაპრილს და მეტოპროლოლს კი ჰიპერკალემიის, მაგრამ ვანომ გვანიშნა, სხვა მიმართულებით წადითო და რავიცი gigi.gif

Posted by: LUKA-BRAZI 27 Oct 2009, 00:10
Thandrus
ნუ მაშინ აღარ ვიცი....
თან ვანომ თქვა:
QUOTE
ჯანმრთელი ავადმყოფია. გარდა ჰიპერტენზიისა პრობლემა არ აქვს.

და
QUOTE
ადრე ეს პრობლემა არასდროს არ ქონია ამ ავადმყოფს (არც ჰიპერკალცემია და არც ჰიპერკალემია).

თან თირკმელიც ნორმაშია.... მოკლედ ვანო, ტექნიკურ პერსონალს მიმართეთ, მგონი ეგ აპარატი გაგიფუჭდათ givi.gif

ეს ხუმრობით და რაღაზე ვიფიქროთ კიდე სანდრო? smile.gif

Posted by: Thandrus 27 Oct 2009, 01:04
LUKA-BRAZI

რავიცი, აღარც მე მაქვს მეტი იდეა (ისეთი, რომ ორივე ელექტროლიტის მომატება ახსნას) biggrin.gif

ჩავუტაროთ ტოქსიკოლოგიური ანალიზი წამლების კონცენტრაციაზე, თუ ეს რამეს მოგვცემს givi.gif

თუ არა და, ვანომ მიგვანიშნოს კიდევ რამე biggrin.gif

Posted by: vano_t 27 Oct 2009, 01:47
კაი მაშინ, კიდევ მივანიშნებ სხვა რაიმეს. როცა კლინიკაში მოდის ავადმყოფი, სწრაფი ანამნეზით და ფიზიკური გამოკვლევით ჯანმრთელია და ლაბორატორიული ცვლილების საფუძველი ერთი შეხედვით არ არსებობს, მაშინ დაავადებების გარდა კიდევ უნდა იფიქრო ლაბორატორიული მონაცმების ცვლილების მიზეზე. ჰიპერკალემიასაც და ჰიპერკალცემიასაც შეიძლება ქონდეს ასეთი მიზეზი smile.gif ეს წმინდად პრაქტიკული საკითხია, რომელიც ძალიან ხშირად დაგჭირდებათ პრაქტიკაში.

ახლა თქვენზეა კიდევ სხვა მიზეზების დასახელება და შემდეგი არასაჭირო ჩარევიდან ავადმყოფის გადარჩენა wink.gif

Posted by: Thandrus 27 Oct 2009, 02:00
vano_t

ანალიზის დროს სისხლი ვენიდან აუღეს ხო? ტრავმული ვენოპუნქციის დროს არის ხოლმე ჰიპერკალემია (მკვდარი უჯრედებიდან გამოსული კალიუმის ხარჯზე). ფალს-პოზიტიური ჰიპერკალცემიაც, ჰარისონის თქმით, ყველაზე ხშირად "უნებლიე ჰემოკონცენტრაციისას" გვხვდება.

ანუ, ეს მონაცემები ანალიზების არტეფაქტებია?

Posted by: semiteli 27 Oct 2009, 02:22
ლიზინოპრილს პერედოზირებაა ან დიურეტიკს არ ღებულობდა რაღაც პერიოდი მასთან ერთად (გიდროქლორტიაზიდს)
უნდა გაკეთდეს ჰემატიკრიტი და თუ დადასტურდა – დაენიშნოს მეტი სითხის მიღება(ან გადასხმა ერთი გაუკეტდეს ფიზიოლოგიური ხსნარის) , შეუმცირდეს ლიზინოპრილის დოზა

Posted by: vano_t 27 Oct 2009, 05:02
Thandrus
QUOTE
ანალიზის დროს სისხლი ვენიდან აუღეს ხო? ტრავმული ვენოპუნქციის დროს არის ხოლმე ჰიპერკალემია (მკვდარი უჯრედებიდან გამოსული კალიუმის ხარჯზე). ფალს-პოზიტიური ჰიპერკალცემიაც, ჰარისონის თქმით, ყველაზე ხშირად "უნებლიე ჰემოკონცენტრაციისას" გვხვდება.

ანუ, ეს მონაცემები ანალიზების არტეფაქტებია?

ჰო ეგ არის, ჰემოლიზის გამოა ჰიპერკალემია. ყველა შემთხვევაში, პრაქტიკაში (ტესტირების დროს როცა გაძლევენ ლაბორატორიებს, მაშინ სხვა საქმეა) უნდა გადაამოწმო ლაბორატორიული მონაცემები, სანამ რაიმეს დიაგნოზს და მკურნალობას დაიწყებდე. ლაბორატორიული შეცდომები ბევრი ხდება. ამ შემთხვევაში ამ ავადმყოფს ლაბორატორია ჩაუტარდა ერთ განსაკუთრებულ დღეს. კერძოდ, ამ დღეს სავადმყოფო იძლევა ლაბორატორიებზე დიდ ფასდაკლებებს და უამრავი ავადმყოფი მოდის ერთი დღის განმავლობაში. აღებული სისხლი ხშირად "უქმად" დევს სანამ ანალიზატორში გაუშვებენ. ხდება სისხლის ჰემოლიზი და კალიუმის მომატება. ეს შეიძლება კიდევ გქონდეს მაშინ, როცა სისხლის აღებისას ტორნიკეტი დიდი ხნის განმავლობაშია დადებული აღების ადგილის პროქსიმალურად. კალციუმის მომატება არ გამიგია ჰემოლიზის გამო, თუმცა გამორიცხული არაფერია. კალციუმის მცირე მომატება რომ გაქვს, მაინც უნდა გადაამოწმო კალციუმი სანამ PTH-ს შეუკვეთავ. ყველაზე ხშირი მიზეზი მაინც დეჰიდრატაცია და კალციუმის ან ვიტამინ D-ს ჭარბი მიღებაა. თავიდან შეგიძლია უბრალოდ მარტივი პრობლემები გადაჭრა (კალციუმის და ვიტამინ D-ს შემცველი პროპარატების შემცირება ან მოხსნა, რძის და რძის პროდუქტების მოხმარების შემცირება, კარგი ჰიდრატაცია და მერე გადაუსინჯო ლაბორატორია ისევ. ამ ავადმყოფის შემთხვევაში ლაბორატორიები ვიზიტის დღეს გადამოწმდა და კალციუმი მოვიდა 9.4 და კალიუმი 4.5.

Posted by: LUKA-BRAZI 27 Oct 2009, 21:27
semiteli
QUOTE
ლიზინოპრილს პერედოზირებაა ან დიურეტიკს არ ღებულობდა რაღაც პერიოდი მასთან ერთად (გიდროქლორტიაზიდს) უნდა გაკეთდეს ჰემატიკრიტი და თუ დადასტურდა – დაენიშნოს მეტი სითხის მიღება(ან გადასხმა ერთი გაუკეტდეს ფიზიოლოგიური ხსნარის) , შეუმცირდეს ლიზინოპრილის დოზა

შენ სადღაც ახსენე ექიმი არ ვარო და ამდენი საიდან იცი? smile.gif

Posted by: Romina 27 Oct 2009, 22:08
დიდად გამოსაცნობი ქეისი არ არის (თან ამის მსგავსი ადრეც დავდე), მაგრამ მაინც დავდებ. უბრალოდ იმის დასადასტურებლად, რომ ჩვენთან ბევრ ექიმს უჭირს საკუთარი სფეროს იქით გახედვა და შეცდომას უშვებს sad.gif

სასწრაფო დახმარების მანქანამ კლინიკაში მოიყვანა 32 წლის ქალი. ძირითადი სიმპტომები იყო მაღალი ცხელება და ჰიპოტენზია, ძლიერი საერთო სისუსტე. აღნიშნულმა პაციენტმა 4 თვის წინ მომართა კლინიკას ჩივილებით: წონაში კლება, სუბფებრილიტეტი, თმის ცვენა, ვასკულიტის ტიპის გამონაყარი ხელებზე, ძლიერი საერთო სისუსტე. ჩატარდა ლაბორატორიული კვლევა. LE-უჯრედები არ აღმოჩნდა, ანტი-ds.DNA ნორმის ფარგლებში, მხოლოდ ANA იყო ნორმის ზედა ზღვარზე. პაციენტს მაინც დაესვა სავარაუდო სისტემური წითელი მგლურას დიაგნოზი და ექიმმა დაიწყო მეთილპრედნიზოლონით მკურნალობა (4 მგ-იანი, 8 აბი დღეში).

მოკლედ როცა სასწრაფომ მოიყვანა ეს დიაგნოზი 4 თვის დასმული ჰქონდა და ისევ 8 აბზე იყო. სასწრაფოს ექიმმაც და მოგვიანებით მისმა მკურნალმა ექიმმაც ეს ეპიზოდი მგლურას გამწვავებად ჩათვალა. და მეორე დღეს პაციენტს დაეწყო პრედნიზოლონის პულს-თერაპიით მკურნალობა. მისი მდგომარეობა კიდევ უფრო დამძიმდა, გადაყვანილ იქნა ღუდუშაურის კლინიკაში და იქ გარდაიცვალა. სწორი დიაგნოზი იქ დაესვა, მაგრამ გვიან იყო sad.gif

რა ექნებოდა თქვენი აზრით?

Posted by: vano_t 28 Oct 2009, 12:36
QUOTE (Romina @ 27 Oct 2009, 22:08 )
დიდად გამოსაცნობი ქეისი არ არის (თან ამის მსგავსი ადრეც დავდე), მაგრამ მაინც დავდებ. უბრალოდ იმის დასადასტურებლად, რომ ჩვენთან ბევრ ექიმს უჭირს საკუთარი სფეროს იქით გახედვა და შეცდომას უშვებს sad.gif

სასწრაფო დახმარების მანქანამ კლინიკაში მოიყვანა 32 წლის ქალი. ძირითადი სიმპტომები იყო მაღალი ცხელება და ჰიპოტენზია, ძლიერი საერთო სისუსტე. აღნიშნულმა პაციენტმა 4 თვის წინ მომართა კლინიკას ჩივილებით: წონაში კლება, სუბფებრილიტეტი, თმის ცვენა, ვასკულიტის ტიპის გამონაყარი ხელებზე, ძლიერი საერთო სისუსტე. ჩატარდა ლაბორატორიული კვლევა. LE-უჯრედები არ აღმოჩნდა, ანტი-ds.DNA ნორმის ფარგლებში, მხოლოდ ANA იყო ნორმის ზედა ზღვარზე. პაციენტს მაინც დაესვა სავარაუდო სისტემური წითელი მგლურას დიაგნოზი და ექიმმა დაიწყო მეთილპრედნიზოლონით მკურნალობა (4 მგ-იანი, 8 აბი დღეში).

მოკლედ როცა სასწრაფომ მოიყვანა ეს დიაგნოზი 4 თვის დასმული ჰქონდა და ისევ 8 აბზე იყო. სასწრაფოს ექიმმაც და მოგვიანებით მისმა მკურნალმა ექიმმაც ეს ეპიზოდი მგლურას გამწვავებად ჩათვალა. და მეორე დღეს პაციენტს დაეწყო პრედნიზოლონის პულს-თერაპიით მკურნალობა. მისი მდგომარეობა კიდევ უფრო დამძიმდა, გადაყვანილ იქნა ღუდუშაურის კლინიკაში და იქ გარდაიცვალა. სწორი დიაგნოზი იქ დაესვა, მაგრამ გვიან იყო sad.gif

რა ექნებოდა თქვენი აზრით?

ბევრი რამ შეიძლება ყოფილიყო. რაზე ჩაუტარდა გამოკვლევები? ჰიპერთირეოიდიზმი (thyroid storm, როგორც გართულება), ქრონიკული ინფექციები (აბსცესი სადმე, ოსტეომიელიტი, ტუბერკულოზი, HIV), ლიმფომები, ლეიკემიები და ა.შ. ყველაფერი უნდა იყოს დიფ დიაგნოზში და შესაბამისი კვლევები ჩაუტარდეს. თუ გაესინჯა TSH, გულმკერდის რენტგენი, CBC თავისი პათოლოგიური უჯრედების გამოკვლევით, მუცლის და გულმკერდის კომპიუტერული ტომოგრაფია, კანის გამონაყარის ბიოფსია?

მოკლედ რა ტესტები ჩაუტარდა და რა აჩვენა ტესტებმა?

Posted by: Blind_Torture_Kill 28 Oct 2009, 12:39
QUOTE
მაღალი ცხელება და ჰიპოტენზია, ძლიერი საერთო სისუსტე


სეფტიური შოკი

Posted by: Romina 28 Oct 2009, 12:49
vano_t
Blind_Torture_Kill

საქმე სწორედ ის არის, რომ ექიმმა ბევრი აღარ იფიქრა და დიაგნოზი დასვა, თავისი სფეროს ფარგლებში. მაგ გამოკვლევები რომ ჩაეტერებინა, ნახევარი მაინც არ დაუშვებდა შეცდომას. რაც დავწერე იმ ტესტების გარდა გაკეთდა შარდის საერთო - ნორმა და სისხლის საერთო - ედს -30. მერე აღარ გამეორებულა ანალიზები. მოკლედ შიდსი იყო. მერე გაირკვა, რომ ქმარი ჰყავდა ინტრავენური ნარკომანი. იმუნოდეფიციტს + კიდე იმუნოსუპრესია 4 თვის მანძილზე და ბოლოს პულს-თერაპია sad.gif

Posted by: vano_t 28 Oct 2009, 12:59
QUOTE (Blind_Torture_Kill @ 28 Oct 2009, 12:39 )
QUOTE
მაღალი ცხელება და ჰიპოტენზია, ძლიერი საერთო სისუსტე


სეფტიური შოკი

სეპტიური შოკი მწვავე დაავადებაა. ავადმყოფი ან კვდება საათებში/დღეებში, ან გადარჩება და აღარ ექნება დაავადება. ამ ავადმყოფს ქრონიკული პრობლემა ქონდა.

Posted by: Blind_Torture_Kill 28 Oct 2009, 13:21
QUOTE
სეპტიური შოკი მწვავე დაავადებაა. ავადმყოფი ან კვდება საათებში/დღეებში, ან გადარჩება და აღარ ექნება დაავადება. ამ ავადმყოფს ქრონიკული პრობლემა ქონდა.


სიკვდილის მიზეზი ვიგულისხმე

Posted by: Thandrus 28 Oct 2009, 15:19
Romina

QUOTE
მოკლედ შიდსი იყო. მერე გაირკვა, რომ ქმარი ჰყავდა ინტრავენური ნარკომანი. იმუნოდეფიციტს + კიდე იმუნოსუპრესია 4 თვის მანძილზე და ბოლოს პულს-თერაპია


საშინელებაა sad.gif

Blind_Torture_Kill

წინა გვერდზე რომ სურათი დადე, რა არის იმაზე?

Posted by: Blind_Torture_Kill 28 Oct 2009, 17:39
Thandrus

მიდი გამოიცანი smile.gif

Posted by: vano_t 28 Oct 2009, 21:33
QUOTE (Blind_Torture_Kill @ 28 Oct 2009, 13:21 )
QUOTE
სეპტიური შოკი მწვავე დაავადებაა. ავადმყოფი ან კვდება საათებში/დღეებში, ან გადარჩება და აღარ ექნება დაავადება. ამ ავადმყოფს ქრონიკული პრობლემა ქონდა.


სიკვდილის მიზეზი ვიგულისხმე

სიკვდილის უშუალო მიზეზი კი შეიძლება ეგ იყოს და ხშირია კიდეც.

Posted by: vano_t 5 Nov 2009, 08:02
პრაქტიკულ შემთხვევას დავდებ.

24 წლის გოგო მოდის ER-ში სუნთქვის უკმარისობის ჩივილით. პრობლემა დაეწყო ER-ში მოსვლამდე 45 წუთით ადრე. პაციენტის მიხედვით, მას აქვს ალერგია ხახვზე (კანზე აყრის). მიუხედავად ამისა, მაინც ჭამა ხახვი და მალე დაეწყო სუნთქვის უკმარისობა. სახლში რამოდენიმეჯერ იხმარა ალბუტეროლის ინჰალატორი, მაგრამ ამან სიმპტომები არ მოუხსნა. ასევე აქვს ხველა.

ანამნეზში აღსანიშნავია ხშირი შაკიკის შეტევები და ზოგადი შფოთვა (generalized anxiety disorder).

წამლებიდან იღებს რემერონს შფოთვისათვის და ალბუტეროლის ინჰალატორს.

ER-ში, სანამ ავადმყოფს ნახავდა ექიმი, პაციენტს დაენიშნა ალბუტეროლის ნებულიზებური ფორმა (nebulized albuterol) და უკეთესად იგრძნო თავი.

გასინჯვისას: წნევა 135/70; ტემპერატურა 36,5; გულისცემის სიხშირე 122 დარტყმა წუთში; პულსური ოქსიმეტრია 97 %;

ყურების ოტოსკოპიური დათვალიერებით აღინიშნება მხოლოდ წარსული ოტიტების ნიშნები (თეთრი ფიბროზული "ნადები" დაფის აპკზე); პირის ღრუ ნორმალური ლორწოვანი გარსით და ნორმალური ნუშისებური ჯირკვლებით/ხახით. კისერზე მოსმენისას არ აქვს სტრიდორი. პაციენტს არ აღენიშნება გადიდებული ლიმფური კვანძები. ფილლტვები სუფთაა ყოველგვარი wheezing-ის (მე მგონი მსტვენავი ხიხინი ქვია თუ არ ვცდები) და crackles-ის (მე მგონი სველი ხიხინი ქვია თუ არ ვცდები) გარეშე.

აბა რა დამატებით კითხვებს დასვამთ (რატომ?), გამოკვლევებს ჩაატარებთ (რატომ?), რა არის შესაძლო დიფერენციალური დიაგნოზები და როგორ უმკურნალებთ.

P.S. თუ ასეთი ფორმატი არ მოგწონთ, არ მოგერიდოთ, მითხარით smile.gif მე თვითონ მგონია, რომ ასეთი ფორმატით უფრო ისწავლის ადამიანი (მათ შორის მეც და ყველაც ინფორმაციის გაცვლის საფუძველზე) და ამიტომ ვდებ ასეთი ფორმატით შემთხვევებს.

Posted by: Blind_Torture_Kill 5 Nov 2009, 10:23
QUOTE
აბა რა დამატებით კითხვებს დასვამთ (რატომ?), გამოკვლევებს ჩაატარებთ (რატომ?), რა არის შესაძლო დიფერენციალური დიაგნოზები და როგორ უმკურნალებთ.


CBC ით დავიწყოთ

Posted by: vano_t 5 Nov 2009, 10:45
QUOTE (Blind_Torture_Kill @ 5 Nov 2009, 10:23 )
QUOTE
აბა რა დამატებით კითხვებს დასვამთ (რატომ?), გამოკვლევებს ჩაატარებთ (რატომ?), რა არის შესაძლო დიფერენციალური დიაგნოზები და როგორ უმკურნალებთ.


CBC ით დავიწყოთ

14 500 (ნორმის ზედა ზღვარი 10 000); აქედან 80 % ნეიტროფილები და 3 % bands (მცირედ მარცხნივ გადახრა მოკლედ)

Posted by: MAIN KAMPF 5 Nov 2009, 12:59
სუნთქვის სიხშირე რამდენია. ფილტვების მოსმენისას თუ ტარდება სუნთქვა შესუსტებული ხომ არ არის?
QUOTE
პაციენტს დაენიშნა ალბუტეროლის ნებულიზებური ფორმა (nebulized albuterol) და უკეთესად იგრძნო თავი.

უკეთესად იგრძნო რას ნიშნავს? რა სიმპტომები დარჩა კიდევ ან თუ დარჩა საერთოდ? მაინც მგონია რომ ეს დაკავშირებულია ალერგიულ რეაქციასთან, მაგრამ მაინც გავაკეთებდი გულმკერდის რენტგენოგრაფიას. რატომ? ასეთივე სიმპტომები შეიძლება ჰქონდეს სპონტანურ პნევმოთორაქს.

Posted by: lgogokhia 5 Nov 2009, 23:35
MAIN KAMPF
QUOTE
სპონტანურ პნევმოთორაქს.

QUOTE
ფილლტვები სუფთაა ყოველგვარი wheezing-ის (მე მგონი მსტვენავი ხიხინი ქვია თუ არ ვცდები) და crackles-ის (მე მგონი სველი ხიხინი ქვია თუ არ ვცდები) გარეშე.

ანუ ალბათ ნიშნავს რომ breath sound-ები იყო და ნორმალური, არა?

panic attack ხო არ ქონდა? მაგრამ ლეიკოციტოზი მაინც რაღაცაზე მიუთითებს..
რა ჩანს გულმკერდის რენტგენოგრაფიაზე?

Posted by: basa-ttt 5 Nov 2009, 23:36
QUOTE
გულისცემის სიხშირე 122 დარტყმა წუთში

ბევრია
გული გამორიცხეთ?
მაგ პაროქსიზმი - WPW

QUOTE
ასევე აქვს ხველა

როგორი ხველა?


QUOTE
24 წლის გოგო მოდის ER-ში სუნთქვის უკმარისობის ჩივილით. პრობლემა დაეწყო ER-ში მოსვლამდე 45 წუთით ადრე. პაციენტის მიხედვით, მას აქვს ალერგია ხახვზე (კანზე აყრის). მიუხედავად ამისა, მაინც ჭამა ხახვი და მალე დაეწყო სუნთქვის უკმარისობა

სავარაუდოდ ლორწოვანის გარსის შეშუპებაა..

Posted by: MAIN KAMPF 6 Nov 2009, 00:18
QUOTE
სავარაუდოდ ლორწოვანის გარსის შეშუპებაა..
no.gif

QUOTE
კისერზე მოსმენისას არ აქვს სტრიდორი


Posted by: niniaa 6 Nov 2009, 00:40
ჰგავს ასთმურ შეტევას,მაგრამ...საინტერესოა ,აქამდე ასეთი შეტევა თუ ჰქონია?

Posted by: MAIN KAMPF 6 Nov 2009, 01:51
QUOTE
ჰგავს ასთმურ შეტევას,


no.gif
QUOTE
ფილლტვები სუფთაა ყოველგვარი wheezing-ის (მე მგონი მსტვენავი ხიხინი ქვია თუ არ ვცდები) და crackles-ის (მე მგონი სველი ხიხინი ქვია თუ არ ვცდები) გარეშე


QUOTE
panic attack ხო არ ქონდა

საინტერესო აზრია
smile.gif

Posted by: basa-ttt 6 Nov 2009, 01:55
QUOTE
ფილლტვები სუფთაა ყოველგვარი wheezing-ის (მე მგონი მსტვენავი ხიხინი ქვია თუ არ ვცდები) და crackles-ის (მე მგონი სველი ხიხინი ქვია თუ არ ვცდები) გარეშე

წამლის მიღების შემდეგ შეიძლება ხიხინი აღარ მოისმინებოდეს.


QUOTE
panic attack ხო არ ქონდა

ლეიკოციტები?

Posted by: MAIN KAMPF 6 Nov 2009, 02:22
[quote]წამლის მიღების შემდეგ შეიძლება ხიხინი აღარ მოისმინებოდეს.[/quote]



yes.gif
მაგრამ[quote]უკეთესად იგრძნო თავი.[/quote] არ ნიშნავს სიმპტომების გაქრობას

ლეიკოციტები? [/quote]
თუნდაც ალბუტამოლის მუდმივი ხმარების შემდეგ




smile.gif

Posted by: vano_t 6 Nov 2009, 02:50
MAIN KAMPF
QUOTE
სუნთქვის სიხშირე რამდენია. ფილტვების მოსმენისას თუ ტარდება სუნთქვა შესუსტებული ხომ არ არის?
QUOTE
პაციენტს დაენიშნა ალბუტეროლის ნებულიზებური ფორმა (nebulized albuterol) და უკეთესად იგრძნო თავი.

უკეთესად იგრძნო რას ნიშნავს? რა სიმპტომები დარჩა კიდევ ან თუ დარჩა საერთოდ? მაინც მგონია რომ ეს დაკავშირებულია ალერგიულ რეაქციასთან, მაგრამ მაინც გავაკეთებდი გულმკერდის რენტგენოგრაფიას. რატომ? ასეთივე სიმპტომები შეიძლება ჰქონდეს სპონტანურ პნევმოთორაქს.

სუნთქვის სიხშირე იყო 34/წუთში.

"უკეთესად იგრძნო" ვგულისმობ უკეთესად იგრძნო სუბიექტურად თავი-ანუ სუნთქვის უკმარისობის შეგრძნება და "მოჭიმვის შეგრძნება" გაუმჯობესდა.

გულმკერდის რენტგენი (ექიმის წაკითხვით, ოფიციალური რადიოლოგიური დასკვნა არ არის ჯერ) არ მოიცავს რაიმე ინფილტრატს, გულის ზომები ნორმალურია, პლევრის რღუში სითხე არ არის, ფილტვების ველები სიმეტრიულია ორივე მხარეს.

lgogokhia
QUOTE
panic attack ხო არ ქონდა? მაგრამ ლეიკოციტოზი მაინც რაღაცაზე მიუთითებს..
რა ჩანს გულმკერდის რენტგენოგრაფიაზე?
შესაძლებელია, და ალბათ ყველაზე სავარაუდო დიაგნოზი ეგ არის. ლეიკოციტოზი თავისთავად არაფერზე მიუთითებს და ნებისმიერი სტრესის დროს შეიძლება ავიდეს მაღლა. თუმცა, როცა ლეიკოციტოზი გაქვს, სხვა სერიოზული რამეები არ უნდა გამოგრჩეს. მოკლედ პანიკის შეტევა ერთი სავარაუდო დიაგნოზია. პნეუმოთორაქსიც არის შესაძლებელი, მაგრამ არსებული მონაცემების საფუძველზე (ტკივილის არარსებობა, ნორმალური აუსკულტაცია, ნორმალური პულსური ოქსიმეტრია, ნორმალური რენტგენი რაც მთავარია) ნაკლებ ალბათურია.

basa-ttt
QUOTE
ბევრია
გული გამორიცხეთ?
მაგ პაროქსიზმი - WPW
ბევრია ნამდვილად. და გულის პრობლემების (მაგალითად არითმიების) გამორიცხვაც აუცილებელია, გეთანხმები. ეკგ აჩვენებს სინუსურ ტაქიკარდიას, P-ს ამპლიტუდა 1.5 მმ, ხანგრძლივობა 0.06, QRS ღერძი +98 გრადუსი, PR ინტერვალი 0.16; ნორმალური QRS კომპლექსები და ST-T.

QUOTE
როგორი ხველა?
მშრალი და ხველა ძირითადად გაძლიერდა ამ შეტევის შემდეგ.

დანარჩენზე ჩემი აზრი ასეთია: ასთმის შეტევაც არის შესაძლებელი. იშვიათად, (ყოველ შემთხვევაში მყავდა ავადმოყოფი, რომელსაც მეიოში ასთმის დიაგნოზი დაუსვის და საერთოდ არასდროს ქონია მსტვენავი ხიხინი) შეიძლებაო ასეთი ასთმის არსებობაო.

ლეიკოციტები რომ აქვს, ამით რაიმეზე უდა (ან არ უნდა) გავამახვილოთ ყურადღება?

კიდევ რაზე იფიქრებთ ახალგაზრდა (რეპრპოდუქციული ასაკის) ქალში, რომელიც მოდის ტაქიკარდიით და ტაქიპნეით?

Posted by: MAIN KAMPF 6 Nov 2009, 03:39
თირეოტოქსიკოზი ხომ არ აქვს rolleyes.gif
დიაფრაგმის ნერვის გაღიზიანებამაც იცის მაგ:ორსულობის დროს. boli.gif

Posted by: Thandrus 6 Nov 2009, 03:57
vano_t

რაღაცეები მაინტერესებს: პაციენტს ისტორიაში რამე სხვა საინტერესო არ ჰქონია? ალბუტეროლს რისთვის იღებს (ასთმის გამო?) ? წინა ალერგიული შეტევების დროს (ხახვზე) ასეთივე სიმპტომები ჰქონდა?

საავადმყოფოში მოყვანის და პირველადი მკურნალობის შემდეგ რა სიმპტომები დარჩა?

Posted by: basa-ttt 6 Nov 2009, 04:36
QUOTE
კიდევ რაზე იფიქრებთ ახალგაზრდა (რეპრპოდუქციული ასაკის) ქალში, რომელიც მოდის ტაქიკარდიით და ტაქიპნეით?

ნუუ იმედია ექოსკოპია გაკეთდა შინაგან ორგანოებზე და მათ შორის მცირე მენჯზე.



QUOTE
მშრალი და ხველა ძირითადად გაძლიერდა ამ შეტევის შემდეგ.

მაინც ალერგიული ფონი გაქვს...
ასთმა ან ობსტრუქცია რჩება მაინც...


QUOTE
გულისცემის სიხშირე 122 დარტყმა წუთში

QUOTE
PR ინტერვალი 0.16

ტაქიკარდიის ფონზე ბევრი ხომ არაა?
ნუ ნორმები სადღაც მიწერია პულსის მიხედვით.
აი ახლა ვნახე - 122 მაჯაზე უნდა იყოს 0,12
და არა 0,16
ანუ ბლოკადა გამოდის?

Posted by: vano_t 6 Nov 2009, 07:04
მოდი, ისე რო არ გამოვიდეს "გამოიცანი მე რას ვფიქრობ", რაღაც პრინციპებზე მივუთითებ. როცა ER-ში მოდის ავადმყოფი, პირველ რიგში უნდა იფიქრო ყველანაირ სერიოზულ, მწვავე და კატასტროფულ პრობლემებზე. ჩემი მიდგომით, როცა ავადმყოფი მოდის სასწრაფოში, მისი დიაგნოზი "ყველაფერია", რათა არ გამომრჩეს სერიოზული პრობლემა (თუმცა ამას უმეტესად ვერ ვახერხებ და ბევრჯერ მრჩება მაინც).

ჩვენს შემთხვევევაში რამოდენიმე ასეთ პრობლემას მივაქციეთ ყურადღება: ანაფილაქსიური რეაქცია/ასთმის მწვავე შეტევა და პნეუმოთორაქსი.

თირეოტოქსიური კრიზი (ან თირეოტოქსიკოზი) ალბათ შესაძლებელია, მაგრამ ჩემი გამოცდილებით (თუმცა ამ ამბავში დიდი გამოცდილება არ მაქვს) ავადმყოფებს წინ უძღვის რაღაც სიმპტომები, რომელიც ამას მიუთითებს. შეიძლება კიდეც უნდა შემეკვეთა TSH, მაგრამ არ შემიკვეთია-არც განმიხილია ეგ დიაგნოზი, სიმართლე რომ გითხრათ. ავადმყოფი კლინიკაში უნდა მოვიდეს follow up-ზე და თუ კიდევ ტაქიკარდიულია (სხვა რამესაც გამოვკითხავ), მაშინ აუცილებლად გავსინჯავ TSH-ს.

ახლა შევაჯამოთ: მოდის ავადმყოფი სუნთქვის უკმარისობით (სუბიექტურად, მაგრამ ობიექტურად არ არის ჰიპოქსიური), სინუსური ტაქიკარდიით, ტაქიპნეით და სავარაუდო ეს ყველაფერი დაიწყო ხავხვის ჭამის შემდეგ. აქ მივუთითებ იმას, რომ ამ ავადმყოფის ყველა ER ვიზიტის ვიტალური ნიშნები გადავათვალიერე და ამ ავადმყოფს არასდროს არ ქონია გულისცემის სიხშირე 95-ზე მეტი და სუნთქვის სიხშირე 22-ზე მეტი; ამასთანავე, ბოლო EKG-ც 6 თვის წინ ზუსტად ისეთივე იყო, როგორც ახლა.

ახლა ყველა შესაძლო კარდიოვასკულარული კატასტროფაზე და მწვავე ინფექციურ პრობლემებზე უნდა ვიფიქრო. ეს ჩემი მიდგომაა ნაწილობრივ. მარტივად, ანამნეზში და ფიზიკური გამოკვლევით უნდა შევკრიბო ყველაფერი, რომელიც შეიძლება მიუთითებდეს სეფსისს და ინფექციის რაიმე კერას; რომელიც შეიძლება მიუთითებდეს კარდიოვასკულარულ პრობლემებს. ბასამ ახსენა WPW, მარა ეკგ-ზე მაგის ნიშნები არ არის.

Thandrus
QUOTE
რაღაცეები მაინტერესებს: პაციენტს ისტორიაში რამე სხვა საინტერესო არ ჰქონია? ალბუტეროლს რისთვის იღებს (ასთმის გამო?) ? წინა ალერგიული შეტევების დროს (ხახვზე) ასეთივე სიმპტომები ჰქონდა?

საავადმყოფოში მოყვანის და პირველადი მკურნალობის შემდეგ რა სიმპტომები დარჩა?
ისტორიაში კი ქონდა რაღაც მნიშვნელოვანი, მაგრამ პაციენტს არ მოუცია ეს ყველაფერი. ამიტომ, შენ უნდა შეკრიბო მნიშვნელოვანი, რაც პაციენტმა არ გადმოგცა. შენ კონკრეტულად რა გაინტერესებს? (შეიძლება მე არც დამისვია ეგ კითხვა, მაგრამ შენ რომ დასვამ, ვისწავლით ორივე)

ალბუტეროლს იღებდა ასთმის გამო მოთხოვნილებისამებრ (PRN). ასთმის სერიოზული შეტევები არ ქონდა არასდროს და გაციების დროს ხმარობდა ძირითადად ალბუტეროლს. ასეთი შეგრძნება სუნთქვის უკმარისობის ადრე არ ქონია.

MAIN KAMPF
QUOTE
დიაფრაგმის ნერვის გაღიზიანებამაც იცის მაგ:ორსულობის დროს.

თირეოტოქსიკოზზე ადრეც დავწერე, მაგრამ ორსულობა ყველა ვარიანტში უნდა გამორიცხო ახალგაზრდა გოგოს შემთხვევაში სულ მცირე 2 მიზეზის გამო: 1) ორსულობის გართულებები რომ გამორიცხო (სავსებით შესაძლებელია, რომ ავადმყოფს ქონდეს საშვილოსნოს გარე ორსულობა მაგალითად და შინაგანი სისხლდენა, რამაც შეიძლება მოგცეს ტაქიკარდიაც და ტაქიპნეაც); 2) წამლები და გამოკვლევები ისე შეარჩიო, რომ ნაყოფისათვის საზიანო არ იყოს. ისე ეს ავადმყოფი არ იყო ორსულად.

basa-ttt
QUOTE
ნუუ იმედია ექოსკოპია გაკეთდა შინაგან ორგანოებზე და მათ შორის მცირე მენჯზე.
არა, ექსოკოპია არ გაკეთებულა. მუცლის და მენჯის ფიზიკური დათვალიერებით არაფერი ტკივილი არ აღენიშნებოდა; ნაწლავის ხმები ნორმალურია; და ანამნეზშიც არ არის აბდომინალური პრობლემების ნიშნები. მნიშვნელოვანი აქ კიდევ ის არის, რომ ამ ასაკის ქალები იღებენ გარკვეულ წამლებს, რომელთაც შეუძლიათ სერიოზული პრობლემა გამოიწვიონ და ამაზეც უნდა ვიფიქროთ.

QUOTE
მაინც ალერგიული ფონი გაქვს...
ასთმა ან ობსტრუქცია რჩება მაინც...
შეიძლება, გამორიცხულ არ არის. მე ვერ ვამბობ დარწმუნებით. შეიძლება პანიკური შეტევაც იყოს. შეიძლება ორივე ერთად იყოს. შეიძლება სხვა რამეც არის.

QUOTE
ტაქიკარდიის ფონზე ბევრი ხომ არაა?
ნუ ნორმები სადღაც მიწერია პულსის მიხედვით.
აი ახლა ვნახე - 122 მაჯაზე უნდა იყოს 0,12
და არა 0,16
ანუ ბლოკადა გამოდის?
არა, 0.12-დან 0.2-მდე ნორმაა. შემოკლებულის დროს pre-excitation-ზე ფიქრობ ზოგადად (თუ დელტა ტალღები ახლავს, WPW-ზე, მაგალითად). 0.2-ის შემდეგ პირველი ხარისხის ბლოკია.

Posted by: lgogokhia 6 Nov 2009, 11:46
vano_t
QUOTE
მნიშვნელოვანი აქ კიდევ ის არის, რომ ამ ასაკის ქალები იღებენ გარკვეულ წამლებს, რომელთაც შეუძლიათ სერიოზული პრობლემა გამოიწვიონ და ამაზეც უნდა ვიფიქროთ.

მწეველი თუ იყო და თუ გაისინჯა PT, aPTT, ფიბრინოგენი, თრომბოციტები და D-დიმერი. ან რამე ანგიოლოგიური გამოკვლევა ხო არ ჩაუტარდა დამატებით?

P.S. birth control pill-ები და პატარა ემბოლი? maybe???

Posted by: niniaa 6 Nov 2009, 12:14
vano_t
QUOTE
კიდევ რაზე იფიქრებთ ......, რომელიც მოდის ტაქიკარდიით და ტაქიპნეით?

QUOTE
დანარჩენზე ჩემი აზრი ასეთია: ასთმის შეტევაც არის შესაძლებელი. იშვიათად, (ყოველ შემთხვევაში მყავდა ავადმოყოფი, რომელსაც მეიოში ასთმის დიაგნოზი დაუსვის და საერთოდ არასდროს ქონია მსტვენავი ხიხინი) შეიძლებაო ასეთი ასთმის არსებობაო.

ტაქიკარდია და ტაქიპნოე ჯდება ასთმის შეტევის კლინიკაში, გამოდის ჯერ ასთმაც ვერ გამოვრიცხეთ,ხომ?
ავადმყოფი კონტრაცეპტივებს ხომ არ იღებს?

Posted by: lgogokhia 6 Nov 2009, 12:44
QUOTE
ავადმყოფი კონტრაცეპტივებს ხომ არ იღებს?

მე ასე დავსვამდი ამ შეკითხვას ავადმყოფი კონტრაცეპტივებს ხომ იღებს? იშვიათად თუ შეგხვდება გოგო მაგ ასაკში რომელიც არ იღებს კონტრაცეპტივებს, ლაპარაკი მაქვს პაციენტების იმ პოპულაციაზე რაზეცაა ქეისში ლაპარაკი.. რატომღაც my gut tells me რომ ალერგიული შეიძლება იყოს ავადმყოფი, მაგრამ მწვავე ალერგიული ან კარდიოლოგიური პრობლემა ნაკლებად სავარაუდოა.. მაგრამ მაინც რო გამოირიცხოს ასთმა, სპირომეტრია თუ გაკეთდა? მაგრამ სპირომეტრიამდე მაინც კოაგულაციის მაჩვენებლებს გამოვიკვლევდი.. მე მაინც უფრო ემბოლიისკენ ვიხრები.. ვნახოთ რა მოხდება..

Posted by: niniaa 6 Nov 2009, 13:08
vano_t
QUOTE
მას აქვს ალერგია ხახვზე (კანზე აყრის).

კანზე გამონაყარი ამ შემთხვევაში არ განუვითარდა?
QUOTE
სანამ ავადმყოფს ნახავდა ექიმი, პაციენტს დაენიშნა ალბუტეროლის ნებულიზებური ფორმა (nebulized albuterol) და უკეთესად იგრძნო თავი.

ჩვენ არ გვაქვს ინფორმაცია ექიმის მოსვლამდე ანუ სანამ უკეთ იგრძნობდა თავს, რა აუსკულტაციური მონაცემები იყო ? თუ არ გვაქვს,მაშინ ვერც იმის მტკიცებას დავიწყებთ, რომ ფილტვებში მანამდე არაფერი იყო,ხომ?
QUOTE
იღებს რემერონს შფოთვისათვის

შფოთვით რა ფსიქიკური პათოლოგიაა ,,შეფუთული "?

Posted by: Thandrus 6 Nov 2009, 17:05
vano_t

ჰო... მაშინ ასე დავსვამ კითხვას, ხახვის ჭამის შემდეგ, ზედმეტი ალბუტეროლი ხომ არ მიიღო? რაღაც საეჭვოდ ჰგავს ამ წამლის გვერდითი მოვლენები (ტაქიკარდია, ნერვიულობა, მშრალი პირი, ზოგჯერ პარდოქსული ბრონქოსპაზმი) და ამ პაციენტის სიმპტომები ერთმანეთს...

Posted by: niniaa 6 Nov 2009, 17:26
Thandrus
QUOTE
ჰო... მაშინ ასე დავსვამ კითხვას, ხახვის ჭამის შემდეგ, ზედმეტი ალბუტეროლი ხომ არ მიიღო? რაღაც საეჭვოდ ჰგავს ამ წამლის გვერდითი მოვლენები (ტაქიკარდია, ნერვიულობა, მშრალი პირი, ზოგჯერ პარდოქსული ბრონქოსპაზმი) და ამ პაციენტის სიმპტომები ერთმანეთს...

მაშინ (nebulized albuterol)-ს მდგომარეობა არ უნდა გაეუარესებინა?

Posted by: Thandrus 6 Nov 2009, 18:19
niniaa

ხოდა, ასეც შეიძლება მომხდარიყო. ხახვზე მცირედი ალერგიული რეაქცია მისცა, სახლში მიღებულმა ალბუტეროლმა კი, როგორც წერია, "ვერ უშველა", ანუ შეიძლება კიდევ უფრო დაამძიმა სიტუაცია და ასეთი სიმპტომები მოგვცა.

Posted by: niniaa 6 Nov 2009, 18:27
Thandrus
QUOTE
ხახვზე მცირედი ალერგიული რეაქცია მისცა,

თუ ხახვზე ალერგიული რეაქცია კანზე გამონაყრის სახით გამოიხატება და ამ შემთხვევაში ხახვის ჭამის მერე არ იყო, მაშინ ეს ხახვზე ალერგიის არსებობასაც ეჭვქვეშ ხომ არ დააყენებს?

Posted by: MAIN KAMPF 6 Nov 2009, 19:25
მოკლედ ეხლა რამოდენიმე ციტატას გავუკეთებ კომენტარს და შემდეგ გავაკეთებ დასკვნას რას გავაკეთებდი და სავარაუდო დიაგნოზი.
QUOTE
: 1) ორსულობის გართულებები რომ გამორიცხო (სავსებით შესაძლებელია, რომ ავადმყოფს ქონდეს საშვილოსნოს გარე ორსულობა მაგალითად და შინაგანი სისხლდენა, რამაც შეიძლება მოგცეს ტაქიკარდიაც და ტაქიპნეაც

საშვილოსნოსგარე ორსულობა აქ არ მიგულისხმია, ნაყოფო აწვება დიაფრაგმას და დიაფრაგმის ნერვს მას შეუძლია გამოიწვიოს შესაბამისი მოვლენები, ნუ ეს უფრო საინფორმაციო პასაჟი იყო.
QUOTE
თირეოტოქსიური კრიზი (ან თირეოტოქსიკოზი) ალბათ შესაძლებელია

ზუსტად ჯდება ლეიკოციტოზსაც ამართლებს, თუმცა ამასაც გამოვრიცხავ თუ ვიხელმძღვანელებ ჩვენს ხელთ არსებული მონაცენებით ვგულისხმობ ავადმყოფი იტყოდა რომ აქვს ჩიყვი მათთვის ეს ცნობილია ყოველთვის რაც გაადვილებდა დიაგნოზის დასმას. ხაზი მინდა გავუსვა რომ როგორც პირველი ასევე მეორე შემთხვევა ჩემს პრაქტიკაში ყოფილა, მეორე საკმაოდ ხშირად და სიცოცხლისათვის განსაკუთრებულად საშიშია.

QUOTE
მე მაინც უფრო ემბოლიისკენ ვიხრები

რა ვიცი ასაკით რომ ახალგაზრდა არ იყოს შეიძლება გამეფიქრა მაგრამ მე მგონია არა.
QUOTE
მაშინ (nebulized albuterol)-ს მდგომარეობა არ უნდა გაეუარესებინა?

სწორი შენიშვნაა, ჩემი აზრით მანამდე მიღებულმა წამლებმა nebulized albuterol)-მიღების დროს მიაღწიეს პიკს და ეს პრობლემაც შესაბამისად დარეგულირდა.

QUOTE
ჩვენ არ გვაქვს ინფორმაცია ექიმის მოსვლამდე ანუ სანამ უკეთ იგრძნობდა თავს, რა აუსკულტაციური მონაცემები იყო ? თუ არ გვაქვს,მაშინ ვერც იმის მტკიცებას დავიწყებთ, რომ ფილტვებში მანამდე არაფერი იყო,ხომ?

აქ ბევრი რამე არ გვაქვს თუნდაც გულმკერდის პერკუსია , შაქარი დანარჩენებზე წინაში დავსვი კითხვა
მოკლედ ჩემი აზრით რახანც ეს ყველაფერი დაიწყო იმის შემდეგ რაც შეიქმნა ალერგიული ფონი და შესაბამისად ავადმყოფს ჰქონდა ასთმის შეტევა მაგრამ აქ მთავარია ის რომ საავადმყოფოში უკვე მდგომარეობა საგრძნობლად გაუმჯობესდა მიღებული პრეპარატი ეფექტური აღმოჩნდა შეიძლება დამშვიდებით გამოკვლევების ჩატარება და ზუსტი დიაგნოზის დასმა.
ჰო კიდევ სტაციონარში დავამატებდი ანტიჰისტამინურ პრეპარატს ასევე სედუქსენი ან სიბაზონი რაც ხელთ იქნებოდა და კორტიკოსტეროიდებს ჩავიგდებდი ხალათის ჯიბეში.
აქ ალბათ იქნებიან თერაპევტები რეანიმატოლოგები რომელთაც უფრო მეტი გამოცდილება აქვთ და უფრო გაგვანათლებენ საინტერესო იქნება
smile.gif biggrin.gif biggrin.gif

Posted by: niniaa 6 Nov 2009, 19:35
MAIN KAMPF
QUOTE
მოკლედ ჩემი აზრით რახანც ეს ყველაფერი დაიწყო იმის შემდეგ რაც შეიქმნა ალერგიული ფონი

არ დაგვავიწყდეს ამ ავადმყოფის ფსიქური მდგომარეობა....ამიტომ:
ვიცით კი რომ შეიქმნა ალერგიული ფონი?! ჩვენთვის ხომ ჯერ არ გაუციათ პასუხი კითხვაზე :ჰქონდა ალერგიული გამონაყრი კანზე ხახვის მიღების შემდეგ?

Posted by: MAIN KAMPF 6 Nov 2009, 21:29
QUOTE
არ დაგვავიწყდეს ამ ავადმყოფის ფსიქური მდგომარეობა


yes.gif

Posted by: lgogokhia 7 Nov 2009, 00:32
vano_t
მოდი ეხლა და გვითხარი რამე რა biggrin.gif

Posted by: anarxisti 7 Nov 2009, 00:53
QUOTE (niniaa @ 6 Nov 2009, 19:35 )
MAIN KAMPF
QUOTE
მოკლედ ჩემი აზრით რახანც ეს ყველაფერი დაიწყო იმის შემდეგ რაც შეიქმნა ალერგიული ფონი

არ დაგვავიწყდეს ამ ავადმყოფის ფსიქური მდგომარეობა....ამიტომ:
ვიცით კი რომ შეიქმნა ალერგიული ფონი?! ჩვენთვის ხომ ჯერ არ გაუციათ პასუხი კითხვაზე :ჰქონდა ალერგიული გამონაყრი კანზე ხახვის მიღების შემდეგ?

არავითარი "ალერგია" არ უნდა იყოს...

ყველაზე უფრო სავარაუდოა ფილტვის ემბოლია, თუ ესტროპროგესტერონულ პრეპარატებს ღებულობს, და ამასთან ერთად ოჯახში იყო თრომბოემბოლიის შემთხვევები და კიდევ თუ მწეველია...
თუმც მარტო ამ პრეპარატების მიღებაც საკმარისია თრომბოემბოლიისათვის..

Posted by: lgogokhia 7 Nov 2009, 00:56
anarxisti
QUOTE
არავითარი "ალერგია" არ უნდა იყოს...
ყველაზე უფრო სავარაუდოა ფილტვის ემბოლია, თუ ესტროპროგესტერონულ პრეპარატებს ღებულობს, და ამასთან ერთად ოჯახში იყო თრომბოემბოლიის შემთხვევები და კიდევ თუ მწეველია...
თუმც მარტო ამ პრეპარატების მიღებაც საკმარისია თრომბოემბოლიისათვის..

yes.gif yes.gif 2kiss.gif ალერგია შეიძლება დიფ დიაგნოზისთვის, ძალიან ნაკლებად სავარაუდო, მაგრამ როგორც ზემოთ ავღნიშნე პირველ რიგში ემბოლოას გამოვრციხავდი..

Posted by: vano_t 7 Nov 2009, 04:06
lgogokhia
QUOTE
მოდი ეხლა და გვითხარი რამე რა biggrin.gif

ხო, ყველაფერი შესაძლებელია smile.gif

ფლიტვის ემბოლიაზე უნდა იფიქრო ყველა ვარიანტში (ახალგაზრდობას არ აქვს მნიშვნელობა, როცა ავადმყოფი იღებს ჩასახვის საწინააღმდეგო პრეპარატებს). D-დაიმერზე ამბობენ კარგი სენსიტიური ტესტია, რაც იმას ნიშნავს, რომ შეგიძლია ეფექტურად გამორიცხო დაავადება (ისე ამაზეც კამათია). თუ მომატებულია, ვეღარაფერს ვერ იტყვი-ბევრი მდგომარეობისათვის არის დამახასიათებელი მომატება ალერგიის და ასთმის ჩათვლით. კლინიკურად, უნდა ეძებო მარჯვენა გულის გადიდების ნიშნები: ქვემო კიდურების შესიება (ამან შეიძლება DVT-საც, ანუ ღრმა ვენების თრობოზსაც მიუთითოს), რაც ავადმყოფს არ ქოდნა. ამ ავადმყოფს არც ქვემო კიდურების ტკივილი ქონდა და ისტორიაში მნიშვნელოვანი რისკ ფაქტორები DVT-ს არ ქონდა (დიდი ხანი მოგზაურობა, ოჯახური ისტორია, ქვემო კიდურებზე ჩარევები). ეკგ-ზეც არ იყო P-პულმონალე; QRS იყო მარჯვნივ გადახრილი, მაგრამ ძველ ეკგ-ზეც ქონდა იგივე მარჯვნივ გადახრა; რენტგენზე მარჯვენა კონტური გულისა ნორმალური იყო. სპირალური კტ არ შემიკვეთია და არც v/q (ვენტილაცია/პერფუზიის სკანირება). მიზეზი? სოფლის კლინიკაა და ამისათვის ავადმყოფი 120 კმ-ზე უნდა გაუშვა მინიმუმ. არასტაბილური რომ ყოფილიყო, ყველა ვარიანტში გავუშვებდი.

გარდა ამისა, სეფსისიც ნაკლებ სავარაუდო იყო. სიცხე არ ქონდა, ტოქსიური არ იყო, გამოკითხვით და მთლიანი გასინჯვით ინფექციის სავარაუდო კერა არ აღინიშნებოდა.

მოკლედ, ავადმყოფმა მიიღო ნებულიზებული ალბუტეროლი და სოლუმედროლი, პლიუს ზანაქსი (xanax). გაუმჯობესდა მკვეთრად. ტაქიკარდია არ შეცვლილა. სახლში იქნა გაწერილი და ვუთხარით რა სიმპტომებისათვის მიექცია ყურადღება. კლინიკაში უნდა მოვიდეს ორშაბათს.

Posted by: lgogokhia 7 Nov 2009, 04:28
vano_t
tongue.gif tongue.gif თუ რამე საინტერესო მოხდა დაწერე მერე.. მე რატომღაც ემბოლიაზე როდესაცაა საუბარი ყოველთვის უფრო კატასტროფულ სიტუაციებზე გამირბის გული, ამიტომ შენი მინიშნება
QUOTE
მნიშვნელოვანი აქ კიდევ ის არის, რომ ამ ასაკის ქალები იღებენ გარკვეულ წამლებს, რომელთაც შეუძლიათ სერიოზული პრობლემა გამოიწვიონ და ამაზეც უნდა ვიფიქროთ.
უკვე გადამწყვეტი იყო smile.gif

Posted by: MAIN KAMPF 7 Nov 2009, 12:40
მოკლედ ყველაფრიდან გამომდინარე ასევე მიღებული პრეპარატების ეფექტურობა, საშუალებას გვაძლევს ვიფიქროთ რომ ეს იყო ასთმური შეტევა. ერთი რამე მინდა ავღნიშნო ცოტა არ იყოს გამიკვირდა რატომ გამახვილდა ასეთი ყურადღება ემბოლიაზე
QUOTE
(ახალგაზრდობას არ აქვს მნიშვნელობა, როცა ავადმყოფი იღებს ჩასახვის საწინააღმდეგო პრეპარატებს

მერე? უმრავლესობა იღებს. თუ რომელიმეს პრაქტიკაში გქონიათ ამის გამო ემბოლიის მიზეზი. და როდის ყოფილა ალბუტეროლით მოხსნილიყო. თვითონ ეს წამალი უკვე ტესტია.
QUOTE
რენტგენზე მარჯვენა კონტური გულისა ნორმალური იყო. სპირალური კტ არ შემიკვეთია და არც v/q (ვენტილაცია/პერფუზიის სკანირება). მიზეზი? სოფლის კლინიკაა და ამისათვის ავადმყოფი 120 კმ-ზე უნდა გაუშვა მინიმუმ. არასტაბილური რომ ყოფილიყო, ყველა ვარიანტში გავუშვებდი.

სად მუშაობ? ძალიან გავს სამხრეთ აფრიკის ე.წ. rural hospital-ის სიტუაციას
biggrin.gif
რამე ახალი გამოუშვი კიდევ თუ გაქვს baby.gif

Posted by: niniaa 7 Nov 2009, 13:09
v[b]vano_t
[quote]ტაქიკარდია არ შეცვლილა. ს[/quoteano_t[/b]
ანუ რჩება 122?
ჩვეულებრივ როგორი პულსის სიხშირე აქვს ?ჩაუტარდა 2 საათიანი თერმომეტრიზაცია?როგორია ედსი , ერითროციტები , ჰემოგლობინი,ფორმულა და კიდევ არ გიპასუხიათ კითხვაზე :გამონაყარი თუ ქონდა კანზე.

Posted by: lgogokhia 7 Nov 2009, 21:35
მე მგონი ეს ქეისი მორჩა, დავსვით დიაგნოზიც და ვუმკურნალეც და გავგზავნეთ დაკვირვებაზე. აგერ მე დავდებ ახალს ეხლა:

28 წლის ჯანმრთელი, სექსობრივად აქიუტრი მამაკაცი მიდის ექიმთან დილას კლინიკაში წლიურ გამოკვლევებზე. პაციენტი უჩივის ძლიერ დაღლილობას, უძილობას, იძინებს ნაშუადღევს და დილას ანუ ძილის რითმი არის ორფაზიანი დღე-ღამის განმავლობაში. ავადმყოფს ჩაუტარდა საკმაოდ დაწვრილებითი გამოკვლევა, რამაც გამოავლინა შემდეგი ცვლილებები: პროლაქტინი 11.4 (ნორმა 11.0), საერთო ტესტოსტერონი 359(ნორმა 370), თავისუფალი ტესტოსტერონი -ნორმა, SHBG - ნორმა, AST მომატებული. ავადმყოფი ჰიპერპროლაქტინემიის წინასწარი დიაგნოზით გაიგზავნა MRI-ზე სადაც რადიოლოგი ორაზროვნად მიუთითებს 2 მმ ზომის "შესაძლო" განსხვავებული სიმკვრივის უბანს. ავადმყოფი გაიგზავნა ენდოკრინოლოგთან შემდეგი მკურნალობისთვის, დაესვა პროლაქტინომის დიაგნოზი და დაენიშნა კაბერგოლინი (დოპამინის აგონისტი). 2 თვის მკურნალობის შედეგად ავადმყოფს არ აღენიშნება გაუმჯობესება დაღლილობის მხრივ, ზემოთაღნიშნული მაჩვენებლები იგივეა გარდა პროლაქტინის რომლის დონე აღმოჩნდა განუსაზღვრადი ანუ 0. ავადმყოფს მოეხსნა კაბერგოლინი და შესტავაზეს ტესტოსტერონის ჩანაცვლებითი მკურნალობა, რაზეც მან განაცხადა უარი და მოითხოვა მეორე და მესამე ენდოკრინოლოგის კონსულტაცია. ორივე ენდოკრინოლოგმა გაიმეორა ანალიზები და პლუს IGF-I რომელიც ასევე აღმოჩნდა ნორმის ფარგლებში და დაადასტურა რომ პროლაქტინომის დიაგნოზი იყო მცდარი და კაბერგოლინის მიღების საჭიროება დაისვა ეჭვქვეშ. აღმოჩნდა რომ აღნიშნული მცირე/უმნიშვნელო ცვლილებები ჰორმონულ პროფილში გამოწვეული იყო მეორადად, მოეხსნა ენდოკრინოლოგიური მეთვალყურეობა და გაგზავნილ იქნა სპეციალისტთან. პაციენტის გამოკითხვით გაირკვა რომ ის ჩივილების დაწყებას უკავშირებს ერთდროულად სამ ფაქტორს - ავტოავარია, მელატონინის მიღება საკვები დანამატის სახით და განსხვავებული დროის ზონებში ყოფნას გარკვეული პერიოდის განმავლობაში. დასვა დიაგნოზი, და უმნიშვნელო მკურნალობის და ცხოვრების სტილის ცვლილებით ავადმყოფის მდგომარეობა საგრძნობლად გაუმჯობესდა და აღნიშნული ჰორმონული მარკერები შეიცვალა ნორმამდე (თუმცა საკითხავია იყო შეცვლილი თუ არა საერთოდ). ასევე დამატებითი სპეციფიკური გამოკვლევით გამოვლინდა სხვა დიაგნიზი რომელიც გარკვეულ როლს თამაშობდა პაციენტის ჩივილებში.

მე მგონი ყველაფერი ვთქვი და უკვე ძალიან მარტივია დიაგნოზის გამოცნობა ეხლა, მაგრამ მაინც აბა გამოთქვით აზრი რა იყო ძირითადი დიაგნოზი, რატომ აირია ენდოკრინოლოგიურ პრობლემასთან და რა მკურნალობა დაენიშნა. მე რომ მივუთითო სპეციალისტი რომელთანაც იქნა გაგზავნილი პასუხი იქნება ძალიან მარტივი, ამიტომ შევჩერდები ენდოკრინოლოგზე და ახსნით მაგის შემდეგ განვიტარებული მოვლენების მიმრთულება. ისიც რო არ დავწერო რაც დავწერე, მაშინ საერთოდ ძნელი იქნება რამე ლოგიკის პოვნა, ამიტომ ყველაფერი ძალიან გავამარტივე.. უბრალოდ ერთი მომენტია სასწავლი - რატომ აირია პრობლემა ენდოკრინოლოგიურთან..

vano_t
სულ მინდოდა შენთვის ამ ამბავის მოყოლა, მაგრამ მავიწყდებოდა.. biggrin.gif biggrin.gif და ნამდვილად დავრწმუნდი რომ ხანდახან ზედმეტი სამედიცინო შესაძლებლობები უტვინო ექიმის ხელში პირიქით ავნებს პაციენტს. ამიტომ იმ "სოფლის კლინიკის" შესაძლებლობები უფრო სასარგებლოა ადამაინისთვის თუ მას ჭკვიანი ექიმი მარტავს..

Posted by: Blind_Torture_Kill 7 Nov 2009, 22:52
QUOTE
გასინჯვისას: წნევა 135/70; ტემპერატურა 36,5; გულისცემის სიხშირე 122 დარტყმა წუთში; პულსური ოქსიმეტრია 97 %;


ესეთი მაჩვენებელი შესაძლებელია ემბოლიის დროს ?

Posted by: MAIN KAMPF 8 Nov 2009, 12:18
QUOTE
რატომ აირია პრობლემა ენდოკრინოლოგიურთან..

მელატონინის ერთ ერთი ფუნქციაა ძილის რეგულაცია, ასევე სხვა ჰორმონების სეკრეციაზე მოქმედებს, მაგ. აფერხებს გონადოტროპინის სეკრეციას. რატომ არის პროლაქტინი მომატებული თუნდაც უმნიშვნელოდ თუ იღებდა მელატონინს?
rolleyes.gif

Posted by: niniaa 8 Nov 2009, 13:09
lgogokhia
QUOTE
მე რომ მივუთითო სპეციალისტი რომელთანაც იქნა გაგზავნილი პასუხი იქნება ძალიან მარტივი, ა

სექსუალური ფუნქცია ექნებოდა დარღვეული, მაგრამ რატომ.....ვენეროლოგი?
* * *
QUOTE
lgogokhia

სად დაიკარგე,გაგვეცი პასუხი.

Posted by: lgogokhia 8 Nov 2009, 14:16
MAIN KAMPF
QUOTE
მელატონინის ერთ ერთი ფუნქციაა ძილის რეგულაცია, ასევე სხვა ჰორმონების სეკრეციაზე მოქმედებს, მაგ. აფერხებს გონადოტროპინის სეკრეციას. რატომ არის პროლაქტინი მომატებული თუნდაც უმნიშვნელოდ თუ იღებდა მელატონინს?

მელატონინის როგორც ჰორმონის მოქმედება ნამდვილად ძილის ციკლს უკავშირდება და სხვა ბევრსაც, მაგრამ მე სწორედ იმიტომ ავღნიშნე საკვები დანამატის სახით, რომ ეგზოგენურ მელატონინს საკმაოდ მოკლე ნახევარდაშლის პერიოდი აქვს რის გამოც პრეპარატის სახით მისი მიღება მნიშვნელოვან ზეგავლენას არ ახდენს ჰორმონულ მარკერებზე, რასაც თან ემატება ის ბარიერები რაც პრეპარატს ხვდება პერ-ოს მისი მიღების შედეგად. ამიტომ მელატონინის შემცველი საკვები დანამატები უფრო ზოგადმასტიმულირებელ და უმნიშვნელო ზეგავლენას ახდენს რეალურად ძილის ფაზებზე და როგორც საკვები დანამატი ის არ წარმოადგენს FDA რეგულირებად პრეპარატს. ეს ყველაფერი შეეხება ეგზოგენურად მიღებულ მელატონინს საკვები დანამატის სახით. რაც შეეხება მელატონინის დონეს სისხლში არ გამოკვლეულა და არც მგონია საერთოდ მას დიაგნოსტიკური ღირებულება ქონდეს. ისე მართალი ხარ, სხვათაშორის გამოითქვა აზრი/ჰიპოთეზა რომ ტესტოსტერონის ვარიაციაში და მეორადად პროლაქტინის მომატებაში შეიძლება ამ ეგზოგენურ მელატონინსაც გარკვეული როლი ეთამაშა, თუმცა ეს რათქმაუნდა რეალურ საფუძველს მოკლებული იყო, უბრალოდ მხოლოდ რამდენიმე სტატისტიკურად არასარწმუნო კვლევებშია აღწერილი მსგავსი ეფექტი. ზოგი კვლევა კი პირიქით ძილის ფაზაზე საპირისპირო ეფექტებს აღწერს. რეალური საფუძველი დიაგნოსტიკის იყო განსხვავებული, მელატონინი აქ უბრალოდ იმიტომ ვახსენე რომ მიმართულება უფრო ადვილი ყოფილიყო.

niniaa
QUOTE
ვენეროლოგი? სექსუალური ფუნქცია ექნებოდა დარღვეული,

სულ სხვა გზით მიდიხარ, ავადმყოფი მსგავს სიმპტომატიკას საერთოდ არ აღნიშნავდა და ფიზიკური გამოკვლევითაც არანაირ სექსობრივი გზით გადამდებ დაავადებებზე არ ყოფილა ეჭვი. ავადმყოფი არის HIV, HCV, HBV უარყოფითი. სექსობრივი ცხოვრება აქტიური, ჩივილების გარეშე. სპერმის გამოკვლევისას ინფექციური, მორფოლოგიური, რაოდენობირივი ან სხვა სახის პათოლოგიური ცვლილებები არ გამოვლინდა..

დასვით შეკითხვები გამოკვლევებზე და პაციენტის კონკრეტულ ჩივილებზე, ძალიან დეტალური და ვირტუალურად ყველანაირი გამოკვლევა ჩატარდა. დიაგნოზი არის საკმაოდ მარტივი და ის ყველაფერი რაზეც ვილაპარაკე მეორადი. უბრალოდ უფრო კლინიკურ დედუქციას საჭიროებს..

Posted by: Cousteau 8 Nov 2009, 14:41
QUOTE
და გაგზავნილ იქნა სპეციალისტთან

ფსიქიატრთან?

Reference Levels Prolactin Standard

Women 2.8-29.2 ng/mL
Men 2.1-17.7 ng/mL

ნორმა გამოსდის მაგას? spy.gif

The normal range for total testosterone is incredibly broad -- 250 to 1,200 nanograms (ng) per deciliter (dl).If total level are in the "low normal" range, less than 400 ng/dl levels should be monitored, If levels are below normal, less than 250 ng/dl, your physician may recommend replacement therapy.


Posted by: lgogokhia 8 Nov 2009, 15:00
Cousteau
QUOTE
Reference Levels Prolactin Standard

Women 2.8-29.2 ng/mL Men 2.1-17.7 ng/mL

ნორმა გამოსდის მაგას? spy.gif

The normal range for total testosterone is incredibly broad -- 250 to 1,200 nanograms (ng) per deciliter (dl).If total level are in the "low normal" range, less than 400 ng/dl levels should be monitored, If levels are below normal, less than 250 ng/dl, your physician may recommend replacement therapy.

პროლაქტინიც და ტესტოსტერონიც იყო გამოთვლილი იყო არა-რეფერენს სტანდარტით, ლაბორატორიის ნორმები შედეგებში იყო მითითებული, რაც მეც ავღნიშნე. ისე სხვადასხვა ლაბორატორიისგან მიღებული ვარიაციაც საკმაოდ იყო (3 სხვადასხვა ლაბ-დან), თუმცა ჩემი point არის აქ ის რომ ვარიაცია ნორმისგან იყო მცირე და როგორ აღმოჩნდა დასაშვები კონკრეტულ სიტუაციაში, ვინაიდან ბოლო გამოკვლევაზე პაციენტის აღნიშნული მაჩვენებლები ლაბორატორიის რეფერენს-სტანდარტებით იყო ნორმალური.. rolleyes.gif

QUOTE
ფსიქიატრთან?
QUOTE
ვენეროლოგი?
ცივი და ცხელის თამაში რო არ დავიწყოთ ახსენით რატომ თვლი ფსიქიატრი ან ვენეროლოგი მკურნალობდა.


Posted by: MAIN KAMPF 8 Nov 2009, 15:06
[quote]ფსიქიატრთან?[/q
ფსიქოლოგ ანდროლოგი
boli.gif

Posted by: lgogokhia 8 Nov 2009, 15:09
Cousteau
QUOTE
Reference Levels Prolactin Standard

ისე საკმაოდ დამაბნია მაგან, მაგრამ არ ვიცოდი რომ ბევრ ლაბორატორიას მოწყობილობაზე(ალბათ) დამოკიდებულებით საკუთარი რეფერენს ნორმები აქვთ.. good point!!

MAIN KAMPF
QUOTE
ფსიქოლოგ ანდროლოგი

რატომ? ისე არც ვიცი ასეთი სპეციალისტი თუ არსებობს.. ფსიქოლოგია არასამედიცინო სპეციალობაა, ანუ MD-ები არ არიან ანდროლოგებისგან განსხვავებით..

Posted by: MAIN KAMPF 8 Nov 2009, 16:31
მინდოდა მეთქვა ფსიქოლოგი. ანდროლოგი.

smile.gif

Posted by: niniaa 8 Nov 2009, 17:11
lgogokhia
რა ტრავმები ჰქონდა ავტოავარიის დროს?
მელატონინის დამატება საკვებში... რატომ?
რატომ უხდება სხვადასხვა სარტყელში ყოფნა და რა სიხშირით?

Posted by: lgogokhia 8 Nov 2009, 22:01
niniaa
QUOTE
რა ტრავმები ჰქონდა ავტოავარიის დროს?

მნიშვნელოვანი ტრავმები თვითონ არ ქონდა, მაგრამ აღენიშნა პოსტტრავმული სტრესი გამოხატული უძილობით და აგზნებით (anxiety), ეწეოდა ბევრს.

QUOTE
მელატონინის დამატება საკვებში... რატომ?

თვითონ იღებდა როგორც დამხმარე საშუალებას ძილისთვის, ვინაიდან ის ხელმისაწვდომია შეზღუდვის გარეშე.. მაგრამ როგორც ადრე ავღნიშნე მელატონინი ნაკლებად არის მნიშვნელოვანი ამ შემთხვევაში.

QUOTE
რატომ უხდება სხვადასხვა სარტყელში ყოფნა და რა სიხშირით?

წელიწადში 1-2ჯერ.

სიმარტივისთვის დავაკონკრეტებ რომ ავადმყოფს ქონდა ორფაზიანი ძილი 24 საათის პერიოდში. იძინებდა შუადღის 6-7 საათიდან ღამის 10-11 საათამდე და დილის 4-5 საათიდან 9 საათამდე. სისხლი ჰორმონებისთვის ყოველთვის იყო აღებული დილის 7 საათზე, როდესაც პაციენტს უზდებოდა იმაზე ადრე ადგომა კლინიკაში კონსულტაციისთვის, ვიდრე ის ყოველდღიურად იღვიძებდა სამსახურში წასასვლელად.

Posted by: MAIN KAMPF 8 Nov 2009, 22:35
მოკლედ ეხლა დროა გვითხრა რა არის მიზეზი და დიაგნოზი jump.gif

Posted by: lgogokhia 8 Nov 2009, 23:54
MAIN KAMPF
პაციენტი გაგზავნილ იქნა ძილის ნევროლოგთან შემდგომ გამოკვლევაზე. დეტალური ანამნეზით ძილის ელექტროფიზიოლოგიური და რესპირატორული (მე მგონი ასე შეიძლება ითქვას) აღმოჩნდა რომ მას ქონდა ცირკადიანული რითმის დარღვევა და ობსტრუქციული აპნეა. როგორც ცნობილია ზოგიერთი ჰორმონი, განსაკუთრებით პროლაქტინს ახასიათებს ცირკადიანული ვარიაციები და ჩაითვალა რომ შესაძლოა მცირე ვარიაციები განპირობებული იყო პაციენტისთვის არასწორ დროში სისხლის გამოკვკევასთან, ვინაიდან მოგვიანებით დღის განმავლობაში აღებული სისხლში აღნიშნული ჰორმონული მარკერები იყო უფრო ახლოს ნორმასთან. გარდა ამისა აღსანიშნავია პაციენტის მედიკამენტური ანამნეზი, ის მოკლე პერიოდის განმავლობაში სხვადასხვა დროს იღებდა ზიბანს (სიგარეტის მოწევა ავღნიშნე) და ზოლოფტს (PTSD ავღნიშნე). პაციენტს აეკრძალა ყველა ზემოთჩამოთვლილი წამლები, დაენიშნა ნუვიჯილი და მკვეთრი შუქით მკურნალობა და მიეცა კონკრეტული მითითებები და დარიგებები. ნუვიჯილის ეფექტურობა არ იყო განსაკუთრებით გადამწყვეტი ალბათ, თუმცა მკვეთრი შუქით მკურნალობამ, ყოველდღიური დღის აქტოვობების შეცვლამ და სიგარეტის თავის დანებების შედეგად აღნიშნული ძილის პრობლემები მოგვარდა და 3 თვის შემდგომ ჰორმონული გამოკვლევების შედეგად მაჩვენებლები უკვე იყო რეფერენს სტანდარტებში. ასევე საგრძნობლად გაუმჯობესდა პაციენტის ზოგადი განწყობა, ენერგიულობა დღის განმავლობაში და ყველა მსგავსი ჩივილი მოეხსნა. მართალია შეიძლება ცოტა გაურკვეველია ენდოკრინოლოგიური კავშირი ამ პრობლემასთან, მაგრამ მკურნალობის შედეგების გათვალისწინებით ზემოთაღნიშნული ჩაითვალა ყველაზე რაციონალურ ახსნად. საკმაოდ დიდი გამოცდილების მქონე ენდოკრინოლოგმა თავიდანვე გამოთქვა მსგავსი აზრი, ვინაიდან ვარიაცია არ იყო იმდენად მნიშვნელოვანი რომ ჰიპოგონადიზმზე ყოფილიყო ეჭვი.

Posted by: basa-ttt 10 Nov 2009, 20:16
QUOTE
ძილის ნევროლოგთან

ეს ახალი პროფესიაა?
"ძილის მინისტრის" ვარიანტია?

Posted by: lgogokhia 12 Nov 2009, 08:44
basa-ttt
QUOTE
ეს ახალი პროფესიაა? "ძილის მინისტრის" ვარიანტია?

არა, სხვაშორის ძველი.. უზარმაზარი ძილის კლინიკით და მთელი თავისი ატრიბუტებით. smile.gif

Posted by: basa-ttt 12 Nov 2009, 09:07
ცოტა ხანში ალბათ მარჯვენა და მარცხენა ყურის ექიმი ცალ ცალკე სპეციალობები გახდება..

gigi.gif

Posted by: vano_t 14 Nov 2009, 08:49
Blind_Torture_Kill
QUOTE
QUOTE
გასინჯვისას: წნევა 135/70; ტემპერატურა 36,5; გულისცემის სიხშირე 122 დარტყმა წუთში; პულსური ოქსიმეტრია 97 %;


ესეთი მაჩვენებელი შესაძლებელია ემბოლიის დროს ?
შეიძლება. ისიც არის შესაძლებელი, რომ ავადმყოფს ქონდეს ქრონიკული უსიმპტომო ფილტვების ემბოლიები და წლების მერე მოვიდეს ფილტვის წრეში განვითარებული ჰიპერტენზიით ამის ფონზე.

lgogokhia
QUOTE
სულ მინდოდა შენთვის ამ ამბავის მოყოლა, მაგრამ მავიწყდებოდა..
მთავრია ყველაფერი კარგად არის smile.gif

ფორუმს smile.gif
ახალ შემთხვევას დავდებ. დაახლოებით 3 კვირის წინ მოიყვანეს ავადმყოფი საავადმყოფოში. 24 წლის ბიჭი რომელსაც სავარაუდოდ ქონდა გულყრა. ავადმყოფი მუშაობდა oil rig-ზე (მოკლედ, ნავთობის საქაჩის დასამონტაჟებელი აგრეგატია) და გარეთ (გარეთ ტემპერატურა იყო დაახლოებით 0 ცელსიუსით.) იპოვა თანამშრომელმა უგონოდ. ავადმყოფი მოყვანილი იქნა სამუშაოს ზედამხედველის მიერ. ზედამხედველს აქვს მხოლოდ ეს ინფორმაცია. ის ამბობს, რომ უნახია გულყრები და იცის როგორია გულყრა. იმის მიხედვით რაც მას გადმოსცა მეორე მუშამ (ვინც ავადმყოფი გარეთ ნახა უგონოთ), ზედამხედველი ფიქრობს რომ ავადმყოფს გულყრა ქონდა. მეორე მუშა არ ახლდა თან და ვერ შევკრიბე მეტი ინფორმაცია. თვითონ ავადმყოფს (A-ს) ახსოვს ეს: ოთახში იჯდნენ ბიჭები და ისვენებდნენ. A-მ შეუძლოდ იგრძნო თავი. ასე ეგონა ცხელი იყო. შემდეგ გარეთ გავიდა და ახსოვს, რომ გონება დაკარგა. იმის მერე ბევრი არაფერი ახსოვს, სანამ მეორე მუშამ (B-მ) არ იპოვა. წარსულის ისტორია უმნიშვნელო.

ქირურგიები არ ქონია.

წამლებს არ იღებს.

ალერგიები არ აქვს.

ოჯახური/გენეტიკური ისტორია უმნიშვნელო.


დეტალური ანამნეზით, ავადმყოფს 2 დღე ქონდა სუბიექტური ცხელება, მშრალი ხველა, კუნთების ტკივილი, ყელის ტკივილი.

პულსი 90, წნევა 140/80-ზე, ტემპერატურა 36.8, პულსოქსიმეტრია 98 %, სუნთქვა 12.

ფიზიკური გამოკვლევა ზოგადად ნორმალურია.

CBC/BUN/Creatinine/Cl/HCO3/Na/Ka ნორმაშია.

როგორია ამ ავადმყოფთან მიდგომა?

გამოკვლევის დეტალებს და გამოკითხვის დეტალებს არ ვწერ. ბევრი რამ თქვენ უნდა შემეკითხოთ. თუ გამოკვლევისას და გამოკითხვისას არ გამომრჩა ეგ ნაწილი, მაშინ გიპასუხებთ.

Posted by: lgogokhia 14 Nov 2009, 09:44
vano_t
ტრავმა ხომ არ აქვს ანამნეზში? გლუკოზის დონე? was he exposed to any kind of gas/substance, რომელსაც შეეძლო ჰიპოქსია გამოეწვია? ეკგ თუ გაკეთდა და რა რამე რითმის ან სხვა დარღვევა ხომ არ იყო? ეეგ თუ იყო ხელმისაწვდომი?

Posted by: vano_t 14 Nov 2009, 11:35
lgogokhia
QUOTE
ტრავმა ხომ არ აქვს ანამნეზში?
ტრამვას გულისწასვლამდე უარყოფს.

QUOTE
გლუკოზის დონე?
გლუკოზა ნორმალური იყო

QUOTE
was he exposed to any kind of gas/substance, რომელსაც შეეძლო ჰიპოქსია გამოეწვია?
სამუშაო ადგილი safe იყო და ზედამხედველი უარყოფს ამას.

QUOTE
ეკგ თუ გაკეთდა და რა რამე რითმის ან სხვა დარღვევა ხომ არ იყო?
ეკგ ნორმალური იყო.

QUOTE
ეეგ თუ იყო ხელმისაწვდომი?
არა, სოფლის პირობებში მაგი კომფორტია biggrin.gif რისთვის გინდა ეეგ? მანამდე არ შეგიძლია სხვა რამეებით დაადგინო რასაც ეძებ?

Posted by: lgogokhia 14 Nov 2009, 13:01
vano_t
QUOTE
არა, სოფლის პირობებში მაგი კომფორტია

J1 prison-ზე ხო არ ხარ შენ შემთხვევით? biggrin.gif if u know what I mean.. lol

QUOTE
რისთვის გინდა ეეგ? მანამდე არ შეგიძლია სხვა რამეებით დაადგინო რასაც ეძებ?

და რას ვეძებ რო? ან სინკოპე დაკავშირებული ფიზიკურ გადაძაბვასთან/sleep deprivation-თან და ა.შ. ან ეპილეფსია? მაგრამ შენ არ დადებდი ასეთ მარტივ ქეისს.. აბა დაველოდები ჩემზე ჭკვიანებს თუ შენ hint-ს ვერ იმეტებ..

Posted by: niniaa 14 Nov 2009, 13:04
vano_t
QUOTE
ტრამვას გულისწასვლამდე უარყოფს.

ეს იყო გულისწასვლა თუ გულყრა?
თუ გულყრა იყო, მაშინ ენცეფალოგრამა თუ გაკეთდა?

* * *
QUOTE
ავადმყოფს 2 დღე ქონდა სუბიექტური ცხელება,

სუბიექტური ცხელება,რას ნიშნავს, ავადმყოფს აცხელებდა და თერმომეტრი სიცხეს არ აჩვენებდა?
* * *
QUOTE
მშრალი ხველა, კუნთების ტკივილი, ყელის ტკივილი.

ამას რას უკავშირებს?

Posted by: lgogokhia 14 Nov 2009, 13:14
vano_t
შენი დაწერილის მიხედვით ჩავთვალე, რომ ვერ გითხრა ვერავინ თუ როგორ დაკარგა გონი, მართალი ვარ? თუ ზედამხედველმა "კლასიკური" გულყრა აღგიწერა (სწორია ეგ სიტყვა?)?

Posted by: Thandrus 14 Nov 2009, 13:54
vano_t

MRI ან რამე სხვა სახის თავის ტვინის ხომ არ გაკეთებულა? აქ დაწერილიდან, პირველი შთაბეჭდილებით, შეიძლება ჰიპოთალამუსის ტემპერატურის მაკონტროლებელ ცენტრში ყოფილიყო არანორმალური ნერვული აქტივობა (რომელიც შეიძლება რიგ მდგომარეობებს გამოეწვია, მაგალითად სიმსივნეს), რომელიც შემდეგ სხვაგანაც გავრცელდა (ნუ, ეს ისე, ჰიპოთეზის დონეზე gigi.gif)

Posted by: vano_t 14 Nov 2009, 14:13
lgogokhia
QUOTE
QUOTE
არა, სოფლის პირობებში მაგი კომფორტია

J1 prison-ზე ხო არ ხარ შენ შემთხვევით? biggrin.gif if u know what I mean.. lol

ვიყავი, მაგრამ 2 წლის წინ მოვიხადეს სროკი biggrin.gif

QUOTE
QUOTE
რისთვის გინდა ეეგ? მანამდე არ შეგიძლია სხვა რამეებით დაადგინო რასაც ეძებ?

და რას ვეძებ რო? ან სინკოპე დაკავშირებული ფიზიკურ გადაძაბვასთან/sleep deprivation-თან და ა.შ. ან ეპილეფსია? მაგრამ შენ არ დადებდი ასეთ მარტივ ქეისს.. აბა დაველოდები ჩემზე ჭკვიანებს თუ შენ hint-ს ვერ იმეტებ..
სწორად მიდიხარ: სინკოპე (საერთოდ და არა მაინცდამაინც სპეციფიური ფორმა) უნდა გაარჩიო ეპილეფსიისაგან. ამისათვის ანამნეზიც და ფიზიკური გამოკვლევაც (ლაბორატორიების ჩათვლით) იმ მიმართულებით უნდა წაიყვანო, რომ ამათი დიფერენცირება მოახდინო. თან ავადმყოფსაც უნდა უმკურნალო ამასობაში.

ანამნეზი არასრულყოფილია მაგალითად და გაცილებით მეტის გაგება შეიძლება.

QUOTE
შენი დაწერილის მიხედვით ჩავთვალე, რომ ვერ გითხრა ვერავინ თუ როგორ დაკარგა გონი, მართალი ვარ? თუ ზედამხედველმა "კლასიკური" გულყრა აღგიწერა (სწორია ეგ სიტყვა?)?
ზედამხედველმა ვერ მითხრა, იმიტომ რომ გადმოცემით იცოდა ყველაფერი. მასაც სხვამ უთხრა ეს ყველაფერი და იმ სხვამ უშუალოდ ნახაო. ზედამხედველის აღწერილს ვერ დაეყრდნობი. იმის გარდა, რომ ამ ავადმყოფს მთელი სხეულის ტრემორი ქონდა, ამ ზედამხედველმა არაფერი იცის სხვა.

niniaa
QUOTE
ეს იყო გულისწასვლა თუ გულყრა?
თუ გულყრა იყო, მაშინ ენცეფალოგრამა თუ გაკეთდა?
ეგ შენ უნდა გაარკვიო. ავადმყოფმა იცის რომ გონება დაკარგა და მანამდე ქონდა შეგრძნება, რომ გონებას კარგავდა.

QUOTE
სუბიექტური ცხელება,რას ნიშნავს, ავადმყოფს აცხელებდა და თერმომეტრი სიცხეს არ აჩვენებდა?
მასეც შეიძლება ჩათვალო, მაგრამ ჩვენს შემთხვევაში აცხელებდა, ოღონდ ტემპერატურა არ გაუსინჯია.

QUOTE
ამას რას უკავშირებს?
რას გულისხმობ?

Thandrus
QUOTE
MRI ან რამე სხვა სახის თავის ტვინის ხომ არ გაკეთებულა? აქ დაწერილიდან, პირველი შთაბეჭდილებით, შეიძლება ჰიპოთალამუსის ტემპერატურის მაკონტროლებელ ცენტრში ყოფილიყო არანორმალური ნერვული აქტივობა (რომელიც შეიძლება რიგ მდგომარეობებს გამოეწვია, მაგალითად სიმსივნეს), რომელიც შემდეგ სხვაგანაც გავრცელდა (ნუ, ეს ისე, ჰიპოთეზის დონეზე gigi.gif)
MRI არ გაკეთებულა, პატარა ჰოსპიტალია (და არ არის ამის საშუალება), მაგრამ CT სკანირებას აუცილებლად გავუკეთებდი ამის საშუალება რომ ყოფილიყო. CT-ზე ამ შემთხვევაში არ იყო გადაუდებელი მოთხოვნილება (ავადმყოფი ნევროლოგიურად და კარდიოვასკულარულ ამბავში სტაბილური იყო) და რომ ყოფილიყო აუცილებლად გავუშვებდი დიდ საავადმყოფოში. ისე გაწერის მერე იმიჯინგი თავის ნამდვილად უნდა.

ჰო კიდევ: common diseases are more common then uncommon diseases smile.gif ხშირ შემთხვევებზე იფიქრე. უფრო პრაქტიკულს შემთხვევებს ვდებ, რაც ხშირია. მგონია ასე უფრო გავუზიარებთ ერთმანეთს გამოცდილებას. ხანდახან მასეთსაც შევაპარებ ისე.


ახლა მე წავედი და ხვალ გავცემ დანარჩენ კითხვებს პასუხს. აბა დროებით.

Posted by: niniaa 14 Nov 2009, 17:51
vano_t
QUOTE
ავადმყოფმა იცის რომ გონება დაკარგა და მანამდე ქონდა შეგრძნება, რომ გონებას კარგავდა.

გულის წასვლის დროს გრძნობენ ხოლმე გონებას რომ კარგავენ, მაგრამ ვინ თქვა, რომ ეპილეფსიანები არ გრძნობენ?მე მყავდა ექთანი ეპილეფსიით დაავადებული, რომელიც წინასწარ გრძნობდა და ცდილობდა, შეტევის დროს ცუდად არ დაცემულიყო.
საეჭვო შემთხვევაში აუცილებელია სასწრაფოდ გაკეთდეს ენცეფალოგრამა .ჩავატარებდი ნევროპათოლოგის კონსულტაციას.
QUOTE
ზედამხედველი ფიქრობს რომ ავადმყოფს გულყრა ქონდა.

გულყრაში რა იგულისხმება: ტონური და კლონური კრუნჩხვები?დუჟი ჰქონდა, ენის დაზიანება, უნებლიე შარდვა?
QUOTE
ავადმყოფს 2 დღე ქონდა სუბიექტური ცხელება, მშრალი ხველა, კუნთების ტკივილი, ყელის ტკივილი.

ავადმყოფი ამას უკავშირებს გაციებას? რაიმე ინფექციას? მოყვანის დროს ეს ჩივილები აღარ ჰქონდა? როგორია ედსი, ლეიკოციტები და ფორმულა, ჰემოგლობინი?140/80 წნევა მისთვის მაღალი არ არის? ტაქიკარდია შემდეგ დღეებშიც რჩებოდა?

Posted by: Thandrus 14 Nov 2009, 20:54
vano_t

QUOTE
ჰო კიდევ: common diseases are more common then uncommon diseases


გავითვალისწინებ smile.gif

ნუ, მაშინ ვკითხოთ თვითონ ამ პაციენტს, თუ ჰქონდა post-ictal state, ანუ გონებაზე რომ მოვიდა როგორ გრძნობდა თავს.

კიდევ, ესეთი რამე პირველად დაემართა?

თუ ვთქვათ სინკოპეზე წავალთ, მაშინ თავიდან D-Dimer-ი გავზომოთ...

Posted by: LUKA-BRAZI 15 Nov 2009, 00:25
basa-ttt
QUOTE
ცოტა ხანში ალბათ მარჯვენა და მარცხენა ყურის ექიმი ცალ ცალკე სპეციალობები გახდება..

თუ ყურის ანატომია, ფიზიოლოგია და პათოლოგია ძალიან დაიხვეწა, ალბათ ეგრეც მოხდება biggrin.gif

vano_t
Thandrus
lgogokhia
ახალი ქეისი რო დაიდება ჩემი ნიკიც დაფლუდეთ რო შემოვიხედო ხოლმე რა smile.gif ეხლა ამ ქეისს ვეღარ მივყვები....

Posted by: lgogokhia 15 Nov 2009, 00:55
vano_t
QUOTE
სწორად მიდიხარ: სინკოპე (საერთოდ და არა მაინცდამაინც სპეციფიური ფორმა) უნდა გაარჩიო ეპილეფსიისაგან. ამისათვის ანამნეზიც და ფიზიკური გამოკვლევაც (ლაბორატორიების ჩათვლით) იმ მიმართულებით უნდა წაიყვანო, რომ ამათი დიფერენცირება მოახდინო. თან ავადმყოფსაც უნდა უმკურნალო ამასობაში.
ანამნეზი არასრულყოფილია მაგალითად და გაცილებით მეტის გაგება შეიძლება.

ანემია არ ქონდა - არა. ელექტროლიტები იყო ნორმაში - იყო. გლუკოზა ნორმა. არ აღწერს მსგავს გულყრას ადრე? არა. არ ახსოვს არაფერი? არა. კრუნჩხვა ქონდა გულყრის დროს? რიავი მარა მგონი არა. მედიკამენტებს ლეგალურს თუ არალეგალურს არ იღებდა? არა. კტ-ზე გაუშვებ მალე - გაუშვებ. დღეს კარგადაა - კარგადაა..

how about ავტონომიური სინკოპე ფიზიკური სტრესისგან გამოწვეული (აღნიშნავ ტრემორს), თუნდაც ტემპერატურის ცვალებადობა, ანუ შიგნით იყვნენ თბილად და 0 გრადუსზე გამოვიდნენ და პირიქით. საძილე არტერიას თუ უქენი რამე და რა მოხდა მერე? (რამე პრაქტიკული/კლინიკური/ღირებული გამოყენება თუ აქვს დღეს მაგას?) აბა რა ვქნა, თუ რამეს არ მალავ, დავტოვებდი პაციენტს რამდენიმე ხნით დაკვირვებაზე და თუ ყველაფერი კარგადაა გავუშვებდი სახში. smile.gif

Posted by: basa-ttt 15 Nov 2009, 00:56
QUOTE
how about ავტონომიური სინკოპე ფიზიკური სტრესისგან გამოწვეული (აღნიშნავ ტრემორს), თუნდაც ტემპერატურის ცვალებადობა, ანუ შიგნით იყვნენ თბილად და 0 გრადუსზე გამოვიდნენ და პირიქით. საძილე არტერიას თუ უქენი რამე და რა მოხდა მერე? (რამე პრაქტიკული/კლინიკური/ღირებული გამოყენება თუ აქვს დღეს მაგას?) აბა რა ვქნა, თუ რამეს არ მალავ, დავტოვებდი პაციენტს რამდენიმე ხნით დაკვირვებაზე და თუ ყველაფერი კარგადაა გავუშვებდი სახში

ეეგ სჭირდება აუცილებლად.
თვალის ფსკერის გამოკვლევა
და თუ კარგი ცხოვრებაა კტ.

Posted by: vano_t 15 Nov 2009, 01:00
niniaa
QUOTE
გულის წასვლის დროს გრძნობენ ხოლმე გონებას რომ კარგავენ, მაგრამ ვინ თქვა, რომ  ეპილეფსიანები არ გრძნობენ?მე მყავდა ექთანი ეპილეფსიით დაავადებული, რომელიც წინასწარ გრძნობდა და  ცდილობდა, შეტევის დროს ცუდად არ დაცემულიყო.
საეჭვო შემთხვევაში აუცილებელია სასწრაფოდ გაკეთდეს ენცეფალოგრამა .ჩავატარებდი ნევროპათოლოგის კონსულტაციას.
მე არ მითქვამს მასეთი რამ, რომ ეპილეფსიანი არ გრძნობს. შენ იკითხე გულყრა იყო თუ ეპილეფსიაო. მე გიპასუხე, ავადმყოფმა იცის, რომ გონება დაკარგა. ავადმყოფი ვერ გეტყვის იმას, რომ გულყრა ქონდა თუ სინკოპე (ცნობილიც რომ იყოს მისთვის გულყრის ისტორია). ენცეფალოგრამის სასწრაფო აუცილებლობა არ არსებობს. თუ ეჭვი გაქვს გულყრაზე, მაშინ დაიწყებ ანტიეპილეპტიკს ინტრავენურად. ნევროლოგის კონსულტაცია კარგია, მაგრამ არ გყავს სპეციალისტი და შენით უნდა გადაწყვიტო.

QUOTE
QUOTE
ზედამხედველი ფიქრობს რომ ავადმყოფს გულყრა ქონდა.

გულყრაში რა იგულისხმება: ტონური და კლონური კრუნჩხვები?დუჟი ჰქონდა, ენის დაზიანება, უნებლიე შარდვა?
კარგი კითხვებია. გამოკვლევით, ენა ნორმალური იყო და არ ქონდა ენის დაზიანების ნიშნები. შარვალი და ტრუსიკიც მშრალი იყო. ზედამხედველმა იცი მხოლოდ ის, რაც ვთქვი (რომ ავადმყოფს, გადმოცემით ქონდა სხეულის "კანკალი"). კიდევ ის, რომ ავადმყოფს არ აქვს გულყრის ისტორია ანამნეზში.

QUOTE
ავადმყოფი ამას უკავშირებს გაციებას? რაიმე ინფექციას?  მოყვანის დროს  ეს ჩივილები აღარ ჰქონდა? როგორია ედსი, ლეიკოციტები და ფორმულა,  ჰემოგლობინი?140/80 წნევა მისთვის მაღალი არ არის? ტაქიკარდია შემდეგ დღეებშიც რჩებოდა?
ტაქიკარდია არ ქონია ავადმყოფს. ეს წნევაც სტრესის შემდეგ ნორმალურია. ავადმყოფმა როგორ უნდა დაუკავშიროს თავისი სიმპტომები ინფექციას? ეს ექიმის გასარკვევი არ არის? სიმპტომებით შესაძლებელია ქონოდა გრიპი (გრიპის სეზონია თან). influenza A და influenza B ჩატარდა და ტესტები უარყოფითი მოვიდა. ასევე გაიგზავნა ტესტი H1N1-ზე (მოგვიანებით ეს ტესტიც უარყოფითი იყო). CBC (რაც ნორმალური იყო) მოიცავს ლეიკოციტებს და ფორმულას და ჰემოგლობინს-ეს ჩემი ბრალია, დამეზარა CBC-ს განმარტება smile.gif ედს-ს მე პირადად იშვიათად ვუკვეთავ თუ გარკვეულ დიაგნოზზე არ ვფიქრობ. ამ შემთხვევაში არ შემიკვეთია (შეიძლება იყოს საფუძველი და მე ვერ ვხედავ).

გულყრა დიფერენციალურ დიაგნოზში ნამდვილად უნდა შედიოდეს სანამ არ გამორიცხავ უფრო მეტი ანამნეზით და გამოკვლევლით. თუ გულყრაზე ფიქრობ, მაშინ მის ყველა მიზეზზეც უნდა იფიქრო. ამ შემთხვევაში, თუ ადამიანს აქვს სიცხე და თავის ტკვილი, მხედველობაში უნდა გქონდეს მენინგიტიც. გამოკვლევით, ლაბორატორიებით და ანამნეზით ნაკლებ სავარაუდოა თუმცა.

Thandrus
QUOTE
ნუ, მაშინ ვკითხოთ თვითონ ამ პაციენტს, თუ ჰქონდა post-ictal state, ანუ გონებაზე რომ მოვიდა როგორ გრძნობდა თავს.

კიდევ, ესეთი რამე პირველად დაემართა?

თუ ვთქვათ სინკოპეზე წავალთ, მაშინ თავიდან D-Dimer-ი გავზომოთ...
პირველად დაემართა ასეთი რამ. ავადმყოფი ვერ გეტყვის საერთოდ post-ictal state ქონდა თუ არა, იმიტომ რომ მაგას გარეთა დამკვირვებელი ადგენს. D-dimer-ით სინკოპეს როგორ დაადგენ, ან რა ინფომრაციას მოგცემს სინკოპეზე?

tip: ყველასთან (და ყველაზე მნიშვნელოვან წყაროსთან მათ შორის) არ გვისაუბრია ვისგანაც ინფორმაცია შეიძლება შევკრიბოთ smile.gif

Posted by: lgogokhia 15 Nov 2009, 01:02
LUKA-BRAZI
QUOTE
ჩემი ნიკიც დაფლუდეთ

რომ ვიცოდე ეგ რას ნიშნავს ვიზამდი უეჭველი biggrin.gif

QUOTE
דברים טובים מגיעים למי לחכות

ჰიბრუ იცი? ისრაელში ვარ ნამყოფი და ვისწავლე ლო, სლიხა და კიდევ დამავიწყდა რაღაცეები biggrin.gif

Posted by: basa-ttt 15 Nov 2009, 01:02
QUOTE
ტრუსიკიც

ნიფხავი

Posted by: Thandrus 15 Nov 2009, 01:39
vano_t

QUOTE
ავადმყოფი ვერ გეტყვის საერთოდ post-ictal state ქონდა თუ არა, იმიტომ რომ მაგას გარეთა დამკვირვებელი ადგენს.


არ შეუძლია ავადმყოფს, რომ გითხრას, გონზე მოსვლის შემდეგ გაბრუებული იყო თუ არა? user.gif

QUOTE
D-dimer-ით სინკოპეს როგორ დაადგენ, ან რა ინფომრაციას მოგცემს სინკოპეზე?


სხვა რამე ვიგულისხმე, მეთქი თუ გულყრას გამოვრიცხავთ და სინკოპეს გზით წავალთ, შეიძლება ეს ფილტვის ემბოლიით იყოს გამოწვეული (რომელმაც პირველი შეხედვით დიდი პათოლოგიური ცვლილებები შეიძლება ანალიზებში არ მოგვცეს) ჰოდა, D-dimer-ის ანალიზიც ამისათვის მინდოდა.

QUOTE
tip: ყველასთან (და ყველაზე მნიშვნელოვან წყაროსთან მათ შორის) არ გვისაუბრია ვისგანაც ინფორმაცია შეიძლება შევკრიბოთ


ვის გულისხმობ, თვითონ პაციენტს, თუ ვინც ის პაციენტი ასეთ მდგომარეობაში ნახა? biggrin.gif

კიდევ რამდენიმე სავარაუდო ვარინტი, რაც მომაფიქრდა: ჰიპოთერმიას, იდეაში, ხომ შეუძლია ასეთი სიმპტომების გამოწვევა? (თუმცა მაშინ ალბათ გულის რითმის ან სხვა პრობლემებიც იქნებოდა + დაკლებული სხეულის ტემპერატურა). ანდაც, ჰიპოგლიკემიური კომა... შეტევამდე პაციენტი მშიერი ხომ არ იყო და დიდი დატვირთვა ხომ არ მიუღია? ალკოჰოლს ხომ არ ეტანება?

Posted by: LUKA-BRAZI 15 Nov 2009, 01:53
lgogokhia
QUOTE
რომ ვიცოდე ეგ რას ნიშნავს ვიზამდი უეჭველი biggrin.gif

ნიკზე დააწკაპუნე ხოლმე smile.gif


Posted by: Mirandaa 15 Nov 2009, 02:16
ეს ნახეთ ყოველ დღე სუ ახალ ახალი http://www.ogarko.net . ეხლაც მაგარი ტექსტი და ინტრო რეკლამა დაუმატებიათ smile.gif

Posted by: Thandrus 15 Nov 2009, 02:20
Mirandaa

????????????

Posted by: LUKA-BRAZI 15 Nov 2009, 02:33
Thandrus
QUOTE
????????????

!!!!!!!!!!!!!!!!!!


biggrin.gif

Posted by: Thandrus 15 Nov 2009, 02:40
LUKA-BRAZI

gigi.gif

უბრალოდ რაღაც სპამის შთაბეჭდილება დატოვა smile.gif

Posted by: LUKA-BRAZI 15 Nov 2009, 03:11
არა ეგ იყო ტრიგერული ნიშანი მარსელი პატარა მწვანე კაცუნების დედამიწაზე შემოსევის smile.gif


Posted by: lgogokhia 15 Nov 2009, 04:39
Mirandaa
wtf? რატომ ყველა თემაში გიწერია ეგ რაღაცა.. არ შევედი სპეციალურად.. აი ეგრე biggrin.gif biggrin.gif

LUKA-BRAZI
ჰიბრუ? და კიდევ ეგ ხინკალები მიდოდა ამ მშიერ კაცს, რომელიც დღეს საღამოს 6 საათზე ადგა.. biggrin.gif biggrin.gif

Posted by: vano_t 15 Nov 2009, 10:19
lgogokhia
QUOTE
ანემია არ ქონდა - არა. ელექტროლიტები იყო ნორმაში - იყო. გლუკოზა ნორმა. არ აღწერს მსგავს გულყრას ადრე? არა. არ ახსოვს არაფერი? არა. კრუნჩხვა ქონდა გულყრის დროს? რიავი მარა მგონი არა. მედიკამენტებს ლეგალურს თუ არალეგალურს არ იღებდა? არა. კტ-ზე გაუშვებ მალე - გაუშვებ. დღეს კარგადაა - კარგადაა..

how about ავტონომიური  სინკოპე ფიზიკური სტრესისგან გამოწვეული (აღნიშნავ ტრემორს), თუნდაც ტემპერატურის ცვალებადობა, ანუ შიგნით იყვნენ თბილად და 0 გრადუსზე გამოვიდნენ და პირიქით. საძილე არტერიას თუ უქენი რამე და რა მოხდა მერე? (რამე პრაქტიკული/კლინიკური/ღირებული გამოყენება თუ აქვს დღეს მაგას?) აბა რა ვქნა, თუ რამეს არ მალავ, დავტოვებდი პაციენტს რამდენიმე ხნით დაკვირვებაზე და თუ ყველაფერი კარგადაა გავუშვებდი სახში. smile.gif
კაი, მაშინ, სხვა რამესაც დავამატებ. ვინც თვითმხილველი იყო, იმას ვესაუბრე ტელეფონით (21-ე საუკუნეში ტელეკომუნიკაცია დიდი სისწრაფით მიდის smile.gif). თვითმხილველისგან ეს გამოირკვა: ოთახში იყვნენ ბიჭები. პაციენტმა უთხრა იმათ, რომ ცხლად გრძნობდა თავს (თუ როგორც ითქმება I felt hot). მერე გარეთ გავიდნენ და ერთმანეთს დაცილდნენ. დაახლოებით 15 წუთში თვითმხილველმა გადაწყვიტა მოენახულებინა პაციენტი და იმ ადგილას რომ მივიდა სადაც პაციენტი მუშუაობდა, პაციენტი ნახა ძირს დაგდებული უგონოდ. პაციენტს არავითარი კრუნჩხვები არ აღენიშნებოდა. შემდეგ თვითმხილველმა გამოაფხიზლა პაციენტი და გონზე რომ მოვიდა ავადმყოფი, მაშინ დაეწყო კანკალი. ტრემორის დროს პაციენტი უკვე აზრზე იყო.

პაციენტის მოყოლით, გარეთ რომ გავიდნენ ეგ და მისი თანამშრომელი და შემდეგ ერთმანეთს დაცილდნენ, დაახლოებით 15 წუთში იგრძნო რომ გულისწასვლის შეგრძნება ქონდა და მერე არაფერი ახსოვს. ახსოვს უკვე როცა მისმა თანამშრომელმა გამოაფხიზლა. გულის წასვლის წინ არ ქონია არავითარი ტკვილი გულის არეში, არ ქონია გულის ფრიალის/სიხშირის შეგრძნება. არ ქონია რაიმე აურასავით. არ ქონია სუნთქვის უკმარისობა.

მოკლედ, როგორც ჩანს, სინკოპე იყო სავარაუდოდ. და სავარაუდოდ რამოდენიმე ფაქტორით გამოწვეული-სიცხე, მერე სიცივეში მუშაობა, შეიძლება ცოტათი დეჰიდრირებულიც იყო და ხეზ. გული ნაკლებ სავარაუდოა, მარა მე ყველა ვარიანტში გამოვრიცხავდი გაწერის მერე (შეიძლება ჰოლტერი დაენიშნოს, ექო გაუკეთდეს). როგორც ჩანს დიდი ხანიც არ ქონდა წასული გონება ავადმყოფის და თვითმხილველის მონაყოლების შეჯერებით.

კიდევ გაუკეთდა CPK და ნორმაში იყო (ეპილეფსიის დროს ხშირად ადის CPK მაღლა). ავადმყოფი არალეგალური წამლების მოხმარებას უარყოფს. თუმცა, მაინც გაიგზავნა urine drug screen და უარყოფითი მოვიდა.

კიდევ მნიშვნელოვანი რა არის, როცა ავადმყოფი იქცევა და გონებას კარგავს (ანუ, წაქცევის მერე რაზე უნდა გამახვილდეს კიდევ ყურადღება)? და კიდევ საავადმყოფოში რო დააკვირდები, რეებს მოითხოვ, რას დააკვირდები?

Thandrus
QUOTE
კიდევ რამდენიმე სავარაუდო ვარინტი, რაც მომაფიქრდა: ჰიპოთერმიას, იდეაში, ხომ შეუძლია ასეთი სიმპტომების გამოწვევა? (თუმცა მაშინ ალბათ გულის რითმის ან სხვა პრობლემებიც იქნებოდა + დაკლებული სხეულის ტემპერატურა). ანდაც, ჰიპოგლიკემიური კომა... შეტევამდე პაციენტი მშიერი ხომ არ იყო და დიდი დატვირთვა ხომ არ მიუღია? ალკოჰოლს ხომ არ ეტანება?
კი, ეს ყველაფერი მნიშვნელოვანია ძალიან. მაგრამ, ჰიპოთერმიას ნორმალური ტემპერატურით გამორიცხავ. ალკოჰოლს არ ეტანებოდა და არც სუნი ჰქონდა. გლუკოზა ლაბარატორიით ნორმალური იყო (90 მგ/დლ-მმოლებში არ ვიცი რამდენია, მარა სადღაც 4-მდეა ალბათ). შიმშლიმა ჰიპოგლიკემიის გარეშე შეიძლება მოგცეს ეგ და შეიძლება ამ შემთხვევაშიც ქონდა რაიმე კავშირი (ეს ბიჭები დიდი ხანი მუშაობენ ხშირად შესვენების გარეშე).

QUOTE
სხვა რამე ვიგულისხმე, მეთქი თუ გულყრას გამოვრიცხავთ და სინკოპეს გზით წავალთ, შეიძლება ეს ფილტვის ემბოლიით იყოს გამოწვეული (რომელმაც პირველი შეხედვით დიდი პათოლოგიური ცვლილებები შეიძლება ანალიზებში არ მოგვცეს) ჰოდა, D-dimer-ის ანალიზიც ამისათვის მინდოდა.
კი, PE უნდა გქონდეს მხედველობაში ყოველლთვის. მარა თუ PE-ზე ფიქრობ, რომელმაც სინკოპე გამოიწვია, მაშინ დიდი ემბოლი უნდა იყოს გაჭედილი, რომელიც პრედატვირთვას მნიშვნელოვნად შეამცირებს და ესენი ძალიან სერიოზული ავადმყოფები არიან. პრინციპში, შესაძლებელია პატარამაც მოგცეს. ამ ავადმყოფს რისკ ფაქტორები არ ქონდა, ჰიპოქსია არ ქონდა, ტაქიკარდია არ ქონდა. ჩემი აზრით ნაკლებ სავარაუდოა.

Posted by: lgogokhia 15 Nov 2009, 11:21
vano_t
QUOTE
კიდევ მნიშვნელოვანი რა არის, როცა ავადმყოფი იქცევა და გონებას კარგავს (ანუ, წაქცევის მერე რაზე უნდა გამახვილდეს კიდევ ყურადღება)? და კიდევ საავადმყოფოში რო დააკვირდები, რეებს მოითხოვ, რას დააკვირდები?

ხომ არ იყოს რამე ტრანზიტორული იშემიური მოვლენა? დავაკვირდები CVS მაჩვენებლებს.. ანუ როგორც ვთქვი ადრე, ჩავთვლი სინკოპე-დ. დავაკვირდები პაციენტს, გავაგზავნი კტ-ზე, ჰოლტერს გავუკეთებ კიდევ (გეთანხმები). ჩავუტარებ რამე დატვირთვის ცდებს.. თუ ყველაფერი კარგადაა წავიდეს სახლში.. რავი აბა მეტი რა უნდა ვქნა ვერ ვხვდები..

Posted by: LUKA-BRAZI 15 Nov 2009, 12:42
lgogokhia
ჰიბრუ ვიცი ელემენტარულზე ელემენტარული..... შენ კი არა მეც კარგა ხანს ვუყურებდი მაგ სურათს smile.gif

ამ ქეისმა დამაინტერესა და თავიდან წავიკითხე smile.gif ჩავერთვები მეც...
vano_t
lgogokhia
QUOTE
ხომ არ იყოს რამე ტრანზიტორული იშემიური მოვლენა?

სად? ტვინში თუ გულში? თუ გულში, მაშინ არამგონია ტკივილის ეპიზოდის გარეშე ჩაევლო მაგ ამბავს.

მოკლედ ასეთი სცენარი: თუ გარეთ ყინვაა, მაშინ მოსასვენებელ ოთახში სითბო ექნებოდათ + თბილადაც ეცვათ + თუ შესვენების გაეშე იმუშავეს და მშივრებიც იყვნენ (მაგრამ გლუკოზა ნორმაა სისხლში ჰო? spy.gif) -> პერიფერიული ვაზოდილატაცია, სისხლის გადანაწილება პერიფერიაზე, შესაბამისად პაციენტმა I felt hot-ო და გავიდა გარეთ, გარეთ გასვლისას შეიძლება ორთოსტატიური ჰიპოტენზიაც ქონდა, გარეთ კი 0 გრადსუზე რეფლექსური სპაზმი და სისხლის მიმოქცევის კიდევ უფრო გაუარესება, შესაბამისად სინკოპე..... მაგრამ ეს ძალიან არამგონია smile.gif

თვაბრუსხვევა ხომ არ ქონდა? მენიერის დაავადებას ვგულისხმობ......

და კიდევ, წინა დღეებში ქონდა მსგავსი სიცხის შეგრძნება. ეს რატომ მიგავიწყდათ?? სარქველებზე ვეგეტაციები ხომ არ აქვს, საიდანაც პატარა ემბოლია გაიჭედა სადმე თავში? ვანო ხო გაქვთ CT მანდ? გაკეთდა? ან გულზე ექო?

Posted by: Cousteau 15 Nov 2009, 13:27
vano_t

შეიძლება ეწერა სადმე და ვერ ვნახე

ადრე ჰქონია თუ არა ეგეთი რამე?
''გულის წასვლის'' გარდა სხვა სიმპტომები საერთოდ არ ქონია?
რამე წამალს ხო არ სვავს ან რამე ლეგალური წამლის abuse ხო არ ყოფილა?

ნეზნაუ... ფეოქრომოციტომას შეიძლება ქონდეს ეგეთი presentation, მარა თავს არ დავდებ მაგაზე smile.gif

Posted by: LUKA-BRAZI 15 Nov 2009, 13:37
Cousteau
QUOTE
ადრე ჰქონია თუ არა ეგეთი რამე?

არაო...
QUOTE
''გულის წასვლის'' გარდა სხვა სიმპტომები საერთოდ არ ქონია?

მგონი არც ეგო...
QUOTE
რამე წამალს ხო არ სვავს ან რამე ლეგალური წამლის abuse ხო არ ყოფილა?

ნწუ smile.gif
QUOTE
ნეზნაუ... ფეოქრომოციტომას შეიძლება ქონდეს ეგეთი presentation, მარა თავს არ დავდებ მაგაზე

კარგი ვერსიაა yes.gif

Posted by: niniaa 15 Nov 2009, 14:52
vano_t
QUOTE
ენცეფალოგრამის სასწრაფო აუცილებლობა არ არსებობს

ე.ი. თავიდანვე გამოირიცხა გულყრა, თორემ გულყრის შემთხვევაში ელექტროფიზიოლოგიური ცვლილებები უფრო კარგად გამოჩნდება, რაც უფრო სწრაფად ჩატარდება ენცეფალოგრაფია, ხომ?
გულის ეხოსკოპია თუ ჩატარდა?
რადგან ავადმყოფი დაეცა, ყურადღება უნდა მივაქციოთ ხომ არ მოხდა ტრავმირება , განსაკუთრებით თავის.
ფაქტია, რომ ავადმყოფი 2 დღით ადრე უკვე შეუძლოდ გრძნობდა თავს,ჩაეკითხეთ, გაცივდა თუ გარშემო ვინმეს ჰქონდა ანალოგიური სიმპტომები?
QUOTE
ტაქიკარდია არ ქონია ავადმყოფს. ეს წნევაც სტრესის შემდეგ ნორმალურია.

90 პულსსდა 140/80 წნევას ნორმად ვერ ჩავთვლით, თუ გულისწასვლის მეორე დღესაც ასევე რჩება/მე ეს მქონდა მხედველობაში/,ხომ?

Posted by: Ni-L 15 Nov 2009, 17:01
მუცლის ღრუს გამოვიკვლევდი(ელენთას და სხვ.)

Posted by: Thandrus 15 Nov 2009, 17:58
Cousteau

QUOTE
ფეოქრომოციტომას შეიძლება ქონდეს ეგეთი presentation


შეიძლება, მაგრამ ამ დროს ჩემი აზრით, გაცილებით უფრო მეტი სიპტომი ექნებოდა... მაგალითად ჰიპერგლიკემია.

vano_t

ისე, ოთახში იყვნენო და გარეთ სიცივე იყოო, ამბობ... ოთახში რითი თბებოდნენ? ნახშირორჟანგი ანდა სულაც CO ხომ არ იყო იქ მომატებული? CO-თი მოწამვლას შეუძლია ასეთი სიმპტომების მოცემა.

Posted by: LUKA-BRAZI 15 Nov 2009, 19:00
Thandrus
QUOTE
ისე, ოთახში იყვნენო და გარეთ სიცივე იყოო, ამბობ... ოთახში რითი თბებოდნენ? ნახშირორჟანგი ანდა სულაც CO ხომ არ იყო იქ მომატებული? CO-თი მოწამვლას შეუძლია ასეთი სიმპტომების მოცემა.

ჰო მაგრამ სხვები რატომ არ მოიწამლნენ?

Posted by: Thandrus 15 Nov 2009, 21:04
LUKA-BRAZI

QUOTE
ჰო მაგრამ სხვები რატომ არ მოიწამლნენ?


სხვადასხვა ადამიანისათვის სხვადასხვა დოზა შეიძლება იყოს მომწამვლელი...

Posted by: lgogokhia 15 Nov 2009, 22:12
QUOTE
სად? ტვინში თუ გულში?

ტვინზე მაქვს ლაპარაკი..

QUOTE
ფეოქრომოციტომას შეიძლება ქონდეს ეგეთი presentation, მარა თავს არ დავდებ მაგაზე

კარგი აზრია..

QUOTE
შეიძლება, მაგრამ ამ დროს ჩემი აზრით, გაცილებით უფრო მეტი სიპტომი ექნებოდა... მაგალითად ჰიპერგლიკემია.

მე მგონი არ არის აუცილებელი ჰიპერგლიკემია რო ქონდეს.

QUOTE
ნახშირორჟანგი ანდა სულაც CO ხომ არ იყო იქ მომატებული

არავოო smile.gif წაიკითხეთ ზემოთ რა წერია..

Posted by: Thandrus 15 Nov 2009, 22:36
lgogokhia

QUOTE
მე მგონი არ არის აუცილებელი ჰიპერგლიკემია რო ქონდეს.


ჰიპერგლიკემია მაგალითისათვის მოვიყვანე. ფეოქრომოციტომას ძალიან მრავალმხრივი პრეზენტაცია შეიძლება ჰქონდეს (Great Masquerader-ს მაგიტომაც ეძახიან) მაგრამ კაცის ერთადერთი სიმპტომი სინკოპე იყოს, უცნაური არაა?

თუმცა ძნელი არ უნდა იყოს, ამდენი განსჯის მაგივრად, კატექოლამინების და მეტანეფრინის დონე გავზომოთ სისხლში და გამოვრიცხოთ ან დავადგინოთ ფეოქრომოციტომაა თუ არა.

QUOTE
არავოო წაიკითხეთ ზემოთ რა წერია..


ჰო, ამომივარდა მხედველობიდან, ოქსიმეტრიაც კარგი იყო gigi.gif

Posted by: lgogokhia 16 Nov 2009, 03:40
Thandrus
QUOTE
კატექოლამინების და მეტანეფრინის დონე გავზომოთ სისხლში

yes.gif yes.gif მაგრამ არის მაგის საშუალება?

Posted by: basa-ttt 16 Nov 2009, 09:22
QUOTE
გონზე რომ მოვიდა ავადმყოფი, მაშინ დაეწყო კანკალი.

შესცივდა...
gigi.gif

მარტივადაა საქმე =
გრიპის მერე დასუსტებულზე ვეგეტაც. სისტემის სპაზმი იყო
სავარაუდოდ ჰიპოტონიის გამო წაუვიდა გული.
რაის ფეოქრომოციტომა.
gigi.gif

მეც წამსვლია გული გრიპის მერე ნაავადმყოფარზე.
წინასწარ რაღაცნაირი შეგრძება მქონდა
ცუდად ვარ - თითქოს უჰაერობას ვგრძნობდი და დაწოლა მინდოდა...

მართალია ვანო -
დაკვირვება უნდა
და თუ განმეორდა ასეთი მოვლენა
მერე უკვე საფუძვლიანი გამოკვლევა -
დაწყებული სიმსივნის ძიებით ტვინში,
დამთავრებული გულის პათოლოგიით

Posted by: niniaa 16 Nov 2009, 15:19
basa-ttt
QUOTE
მარტივადაა საქმე =გრიპის მერე დასუსტებულზე სავარაუდოდ ჰიპოტონიის გამო წაუვიდა გული.

გეთანხმები. თუნდაც არასაკმარისი რაოდენობით სითხის მიღება, გამოიწვევდა ჰოპოვოლემიას+პატარა ოთახში თუნდაც მცირე უჰაერობა იქნებოდა მაპროვოცირებელი.

Posted by: LUKA-BRAZI 16 Nov 2009, 15:24
niniaa
QUOTE
გეთანხმები. თუნდაც არასაკმარისი რაოდენობით სითხის მიღება, გამოიწვევდა ჰოპოვოლემიას+პატარა ოთახში თუნდაც მცირე უჰაერობა იქნებოდა მაპროვოცირებელი.

ჰო, მუშა ვერსიაა, მაგრამ ხომ იცი, გონება ყოველთვის უფრო რთულისკენ გარბის smile.gif)

Posted by: Cousteau 16 Nov 2009, 15:41
QUOTE (niniaa @ 16 Nov 2009, 15:19 )
basa-ttt
QUOTE
მარტივადაა საქმე =გრიპის მერე დასუსტებულზე ვეგეტაც. სისტემის სპაზმი იყო სავარაუდოდ ჰიპოტონიის გამო წაუვიდა გული.

გეთანხმები. თუნდაც არასაკმარისი რაოდენობით სითხის მიღება, გამოიწვევდა ჰოპოვოლემიას+პატარა ოთახში თუნდაც მცირე უჰაერობა იქნებოდა მაპროვოცირებელი.

რამდენი ''არასაკმარისი სითხე'' უნდა დაგაკლდეს რო გული წაგივიდეს?

QUOTE
მარტივადაა საქმე =
გრიპის მერე დასუსტებულზე ვეგეტაც. სისტემის სპაზმი იყო
სავარაუდოდ ჰიპოტონიის გამო წაუვიდა გული.
რაის ფეოქრომოციტომა.


რისი სპაზმი უნდა ქონდეს პაციენტს რომ ჰიპოტონია დაემართოს? spy.gif






თუ ვაზოვაგალური სინკოპეაქ მაშინ უნდა გამოვიცნოთ ეს რამ ''ჩართო''

Posted by: niniaa 16 Nov 2009, 18:37
Cousteau
QUOTE
რამდენი ''არასაკმარისი სითხე'' უნდა დაგაკლდეს რო გული წაგივიდეს?

გრიპის დროს მაქსიმალურად უნდა იყოს ორგანიზმი დატვირთული სითხით.ჯანმრთელმა ადამიანმა დაახლოებით ორ ლიტრამდე უნდა მიიღოს სითხე დღე-ღამეში,ჰოდა , თვითონ მიხვდი, ვირუსული ინფექციის დროს რამდნი იქნება საჭირო.

Posted by: LUKA-BRAZI 16 Nov 2009, 21:48
ვანო, ქამ ბექ ნიგგა, გივ ას სამ ჰინთს biggrin.gif გული შეუმოწმეთ?

Posted by: lgogokhia 16 Nov 2009, 22:16
QUOTE
სავარაუდოდ ჰიპოტონიის გამო წაუვიდა გული.
რაის ფეოქრომოციტომა.

სინკოპე რომ იყო ამაზე უკვე შევთანხმდით კაი ხნის წინ. ფეოქრომოციტომა აქ დიფ.დიაგნოზისთვის არის და რა გაქვს მაგის საწინააღმდეგო კატეგორიულად? უბრალოდ იმ პრინციპით რო ვიხელმძღვანელოთ რომ ყველა შესაძლო ვარიანტი გამოირიცხოს.

QUOTE
მართალია ვანო -
დაკვირვება უნდა
და თუ განმეორდა ასეთი მოვლენა
მერე უკვე საფუძვლიანი გამოკვლევა -
დაწყებული სიმსივნის ძიებით ტვინში,
დამთავრებული გულის პათოლოგიით

რამე ახალი გვითხარი ბარემ wink.gif

აგერ მოვა უფროსი და გვეტყვის რამეს..

Posted by: Cousteau 16 Nov 2009, 22:25
QUOTE (niniaa @ 16 Nov 2009, 18:37 )
Cousteau
QUOTE
რამდენი ''არასაკმარისი სითხე'' უნდა დაგაკლდეს რო გული წაგივიდეს?


გრიპის დროს მაქსიმალურად უნდა იყოს ორგანიზმი დატვირთული სითხით.ჯანმრთელმა ადამიანმა დაახლოებით ორ ლიტრამდე უნდა მიიღოს სითხე დღე-ღამეში,ჰოდა , თვითონ მიხვდი, ვირუსული ინფექციის დროს რამდნი იქნება საჭირო.

გრიპის ვირუსი წყალზე მოთხოვნას ზრდის ორგანიზმში?

Posted by: LUKA-BRAZI 16 Nov 2009, 22:37
QUOTE
გრიპის ვირუსი წყალზე მოთხოვნას ზრდის ორგანიზმში?


ალბათ იგულისხმა რომ თუ სიცხე აქვს ადამიანს და თან ფიზიკურად შრომობს (პაციენტი მუშაა ჰო?) ბერ სითხეს დაკარგავს ოფლის სახით ან ფილტვებიდან ორთქლის სახით და იქნებ ფაღარათითაც smile.gif)

Posted by: niniaa 17 Nov 2009, 16:14
Cousteau
QUOTE
გრიპის ვირუსი წყალზე მოთხოვნას ზრდის ორგანიზმში?

რა პრიმიტიული კითხვები გაქვს, პირველკურსელი ხარ?
საერთოდ, ბევრი სითხე არა მარტო ჰიპოვოლემიის თავიდან ასაცილებლადაა საჭირო გრიპის დროს, არამედ ინტოქსიკაციის შესამცირებლადაც.

Posted by: Cousteau 17 Nov 2009, 18:08
QUOTE (niniaa @ 17 Nov 2009, 16:14 )
Cousteau
QUOTE
გრიპის ვირუსი წყალზე მოთხოვნას ზრდის ორგანიზმში?

რა პრიმიტიული კითხვები გაქვს, პირველკურსელი ხარ?
საერთოდ, ბევრი სითხე არა მარტო ჰიპოვოლემიის თავიდან ასაცილებლადაა საჭირო გრიპის დროს, არამედ ინტოქსიკაციის შესამცირებლადაც.


yes.gif, მეორეკურსელივარ user.gif შენ რომელკურსელიხარ?

როგორ იწვევს გრიპის ვირუსი ჰიპოვოლემიას,როგორ ამცირებს ბევრი წყლის დალევა ''ინტოქსიკაციას'' და რამდენი სითხე უნდა მიიღოს პაციენტმა რომ ''ინტოქსიკაცია'' შეუმცირდეს, რო გავნათლდე ცოტა user.gif

Posted by: niniaa 17 Nov 2009, 18:31
Cousteau
QUOTE
როგორ იწვევს გრიპის ვირუსი ჰიპოვოლემიას,როგორ ამცირებს ბევრი წყლის დალევა ''ინტოქსიკაციას'' და რამდენი სითხე უნდა მიიღოს პაციენტმა რომ ''ინტოქსიკაცია'' შეუმცირდეს, რო გავნათლდე ცოტა

რადგან მეორე კურსელი ხარ, გეპატიება, მაგრამ მოდი და დაფიქრდი და იქნებ შენ თვითონ მიხვდე რატომ? ამით საექიმო აზროვნება გაგივითარდება.
QUOTE
შენ რომელკურსელიხარ?

როდის ვიყავი სტუდენტი აღარც მახსოვს!

Posted by: Cousteau 17 Nov 2009, 18:38
QUOTE (niniaa @ 17 Nov 2009, 18:31 )
Cousteau
QUOTE
როგორ იწვევს გრიპის ვირუსი ჰიპოვოლემიას,როგორ ამცირებს ბევრი წყლის დალევა ''ინტოქსიკაციას'' და რამდენი სითხე უნდა მიიღოს პაციენტმა რომ ''ინტოქსიკაცია'' შეუმცირდეს, რო გავნათლდე ცოტა

რადგან მეორე კურსელი ხარ, გეპატიება, მაგრამ მოდი და დაფიქრდი და იქნებ შენ თვითონ მიხვდე რატომ? ამით საექიმო აზროვნება გაგივითარდება.

QUOTE
შენ რომელკურსელიხარ?

როდის ვიყავი სტუდენტი აღარც მახსოვს!

დამეხმარე user.gif

Posted by: LUKA-BRAZI 17 Nov 2009, 18:47
user posted image

smile.gif

Posted by: Cousteau 17 Nov 2009, 18:48
QUOTE (LUKA-BRAZI @ 17 Nov 2009, 18:47 )
user posted image

smile.gif

yes.gif

ინფლუენზათი გამოწვეული ჰიპოვოლემიით მოკვდა yes.gif

Posted by: lgogokhia 17 Nov 2009, 22:27
იცით რაზე ვკაიფობ? ღიმილისმომგვრელ ამპარტავნობას აბსოლუტურად არანაირი "ტრაკის" გარეშე, საერთოდ. ეს კომუნისტური გადმონაშთია, რომელიც ბევრის აზროვნებაში ჯერ კიდევ შემორჩა და ვხედავ ხოლმე ხანდახან biggrin.gif biggrin.gif ეს ისე ლირიული გადახვევის პონტში wink.gif

Cousteau
კუსტო ინფლუენზით მოკვდა? დედა სირცხვილო.. smile.gif მართლა არ ვიცოდი..


Posted by: lgogokhia 19 Nov 2009, 10:45
მიუხედავად იმისა რომ ბოლო ქეისი არ არის ამოხსნილი ახალი უნდა დავდო, რომელიც ჩემი აზრით საინტერესოა და გთხოვთ წინა ქეისი არ ჩააგდოთ, ორივეზე ვიმსჯელოთ:

52 წლის მარტოხელა კაცი შემოდის ემერჯენსიში კრუნჩხვითი სტატუსით, უგონოდ, ტემპერატურა 39ცელსიუსი. იმდენი რაღაცა ჭირს რომ საიდან დავიწყო არ ვიცი. ავადმყოფი იღუპება რამდენიმე წუთში შემოსვლიდან, თუმცა მოხერხდა გარკვეული ინფორმაციის შეგროვება. ავადმყოფი ბოლო რამდენიმე წლების განმავლობაში მკურნალობდა ბიპოლარული დაავადებას და რამდენიმე თვის წინ დაუდგინდა ენის ვირუსული ლეიკოპლაკია. ბოლო პერიოდში უჩიოდა პირის სიმშრალეს, ენის შესიებას, კანის ქავილს, კანჭის კუნთების ტკივილს, გაღიზიანებადობას/აგიტაციას, საღამოობით დეპრესიის გაძლიერებას, მოუსვენრობას, ტრემორს, არარეგულარულ თავის ტკივილს, ეპიზოდურ ჰიპერტენზიას, ბრუქსიზმს, ცივ ოფლდენას, უნებლიე კუნთების შეკუმშვას. ღებულობდა არც მეტი არც ნაკლები შემდეგ დანიშნულებას:

დილას: ასპირინი, სინთროიდი (ლევოთიროქსინი), ციტომელი (ლიოთირონინი), პროპრანოლოლი, დეზოქსინი (მეტამფეტამინი), დექსედრინი (დექსტროამფეტამინი), ბენიკარი (ოლმეზარტანი), მულტივიტამინი, მაგნიუმი, ვიტამინი D3, ლოვაზა (ომეგა 3 ეთილ ეთერი), კალციუმი

შუადღეს: დექსედრინი და პროპრანოლოლი

საღამოს: ზეტია (ეზეტიმიბი), ალპრაზოლამი, ციტალოპრამი, ლესქოლი (ფლუვასტატინი), ტრაზოდონი, ზიპრექსა (ოლანზაპინი)

ასევე ენაზე ხმარობდა: მიცელექსი (კლოტრიმაზოლი), ფლუცინონიციდის გელი (გლუკოკორტიკოიდი).

მოკლედ, ამის მეტი ინფორმაცია მე არ გამაჩნია. იმსჯელეთ პაციენტის სიკვდილის მიზეზზე და პრეპარატებზე რომლითაც მკურნალობდა და რატომ, რა ასე შემდეგ.. რამდენიმე მნიშვნელოვანი მომენტია განსასჯელი, როგორიცაა წამლის გვერდითი ეფექტები, ურთიერთქმედება და ა.შ. რა შთაბეჭდილება გრჩებათ ანამნეზის წაკითხვის შემდეგ..
* * *
P.S. წარმოდგენა არ მაქვს როგორ მოახერხა ამდენი წამლის დანიშნულების შოვნა.. ვანო, შესაძლებელია თუ არა სხვადასხვა ექიმისგან მიიღოს სხვადასხვა დანიშნულება პაციენტმა ისე რომ არ გაიგოს მეორე ექიმმა წინა ექიმის დანიშნულების შესახებ?

Posted by: Cousteau 19 Nov 2009, 16:56
QUOTE
რა შთაბეჭდილება გრჩებათ ანამნეზის წაკითხვის შემდეგ..

ის შთაბეჭდილება მრჩება რომ სამედიცინო დაწესებულებების abuse-ი ქონდა ამ კაცს : )

სეროტონინის სინდრომია?

საკაიფო ქეისია : D yes.gif

შეიძლება ვცდები და რამე სხვა drug interaction-ისგან მოკვდა user.gif
მერე ჩავუჯდები ყველა წამლის განხილვას ცალცალკე user.gif

Posted by: LUKA-BRAZI 19 Nov 2009, 17:05
lgogokhia
აუფ LG ეს რა დადე? smile.gif)

რაღაცეებს დავაზუსტებ:
QUOTE
რამდენიმე თვის წინ დაუდგინდა ენის ვირუსული ლეიკოპლაკია

AIDS ქონდა ? სხვა რომელი დაავადება ვლინდება ენის ლეიკოპლაკიით? მე ვიცი შიდსი, ქრონიკული ირიტაცია ლოკალური რომელიც შეიძლება იყოს კიბოსწინარე დაავადებაც....

მოკლედ ბიპოლარული დაავადება იგივე მანიაკალურ-დეპრესიული სინდრომია ჰო? ჰოდა ალბათ ფსიქიატრებმა მისცეს მაგხელა დანიშნულებაც.... + ლეიკოპლაკიაზე სხვა ექიმმა. მერე ალბათ ღვიძლი გამოვიდა მწყობრიდან -> drug free radicals induced hepatitis... და მაგის ტვინს დაემატა ღვიძლის დეზინტოქსიკაციური ფუნქციის დარღვევის შედეგებიც.... 39 გრადუსიც ცენტრალური წარმოშობის მგონია ვიდრე რაიმე ზედდართული ინფექციური აგენტის....

Posted by: Blind_Torture_Kill 19 Nov 2009, 23:23
lgogokhia

აუტოფსიის შედეგები დადე smile.gif
* * *
QUOTE
იმსჯელეთ პაციენტის სიკვდილის მიზეზზე


QUOTE
დილას: ასპირინი, სინთროიდი (ლევოთიროქსინი), ციტომელი (ლიოთირონინი), პროპრანოლოლი, დეზოქსინი (მეტამფეტამინი), დექსედრინი (დექსტროამფეტამინი), ბენიკარი (ოლმეზარტანი), მულტივიტამინი, მაგნიუმი, ვიტამინი D3, ლოვაზა (ომეგა 3 ეთილ ეთერი), კალციუმი

შუადღეს: დექსედრინი და პროპრანოლოლი

საღამოს: ზეტია (ეზეტიმიბი), ალპრაზოლამი, ციტალოპრამი, ლესქოლი (ფლუვასტატინი), ტრაზოდონი, ზიპრექსა (ოლანზაპინი)


აი შესაძლო ვარიანტები

Posted by: Thandrus 20 Nov 2009, 00:55
lgogokhia

QUOTE
დილას: ასპირინი, სინთროიდი (ლევოთიროქსინი), ციტომელი (ლიოთირონინი), პროპრანოლოლი, დეზოქსინი (მეტამფეტამინი), დექსედრინი (დექსტროამფეტამინი), ბენიკარი (ოლმეზარტანი), მულტივიტამინი, მაგნიუმი, ვიტამინი D3, ლოვაზა (ომეგა 3 ეთილ ეთერი), კალციუმი

შუადღეს: დექსედრინი და პროპრანოლოლი

საღამოს: ზეტია (ეზეტიმიბი), ალპრაზოლამი, ციტალოპრამი, ლესქოლი (ფლუვასტატინი), ტრაზოდონი, ზიპრექსა (ოლანზაპინი)

ასევე ენაზე ხმარობდა: მიცელექსი (კლოტრიმაზოლი), ფლუცინონიციდის გელი (გლუკოკორტიკოიდი).


wow.gif

ჰო... ამდენი წამალი ერთად არ მინახავს მგონი... ეს კაცი მარტო ბიპოლარულს და ლეიკოპლაკიას მკურნალობდა? სხვა რამეც ექნებოდა ალბათ....

კლინიკური სურათით, ეს უფრო Neuroleptic Maglignant Syndrome-ს ჰგავს. სეროტონერგული სინდრომისაგან განსხვავებით, ის შედარებით ხანგრძლივი დროის განმავლობაში (1-3 დღე vs. საათები) ვითარდება, შედარებით ნაკლებად მაღალი ტემპერატურა ახასიათებს და კუნთების ტკივილი აქვს გამოხატული. თან აქ მოცემული წამლებიდან, ოლანზაპინს შეეძლო ეს გამოწევია.

თუმცა, აქ იმდენ წამალს იღებს, რომ რაღაც სხვა მგომარეობის ატიპიური სურათიც შეიძლება რომ იყოს.

სჯობს, ამ პაციენტის სისხლის ანალიზი გავაკეთოთ კრეატინ კინაზაზე, LDH-ზე და კიდევ რამდენიმე ფერმენტზე. NMS-ის შემთხევაში ესენი მომატებული უნდა იყოს (რაბდომიოლიზის გამო).

Posted by: LUKA-BRAZI 20 Nov 2009, 01:13
მე თურმანაულის სამტომეულს გადავშლი და იქნებ რამე წავიკითხო მაგ წამლებზე. ლოლ. sad.gif
* * *
A mnemonic used to remember the features of NMS is FEVER

* F - Fever
* E - Encephalopathy
* V - Vitals unstable
* E - Elevated enzymes (elevated CPK)
* R - Rigidity of muscles

biggrin.gif
* * *
მარა მაგაზე მაღალი სიცხე სეროტონინის სინდრომს ახასიათებს როგორც წავიკითხე. ასევე კუნთების უნებლიე კუმშვაც უფრო სეროტონინის სინდრომზე მიუთითებს, ვიდრე NMS-ზე. ამას უფრო კუნთების რიგიდობა ახასიათებსო...
* * *
QUOTE
3 წევრი: LUKA-BRAZI, lgogokhia, Thandrus

ნეტა რას ვაკეთებთ აქ 02:10 am-ზე სანდრო? LG გასაგებია, მაგრამ შენ და მე? smile.gif

Posted by: Thandrus 20 Nov 2009, 02:16
LUKA-BRAZI

QUOTE
მარა მაგაზე მაღალი სიცხე სეროტონინის სინდრომს ახასიათებს როგორც წავიკითხე


როგორც ვიცი, სეროტონინის სინდრომის დროს ტემპერატურა ხშირად 40' - 41 'C გრადუსია, NMS-ს დროს სადღაც 38 - 39.

QUOTE
კუნთების უნებლიე კუმშვაც უფრო სეროტონინის სინდრომზე მიუთითებს, ვიდრე NMS-ზე. ამას უფრო კუნთების რიგიდობა ახასიათებსო...


ამაში მართალი ხარ, მაგრამ მდგომარეობის სხვა ნიუანსები, ჩემი აზრით, უფრო NMS-ზე მიუთითებს. ამიტომაც, რადგან ეს კაცი როგორც სეროტონერგულ, ისე ნეიროლეპტიკურ საშუალებებს იღებდა, რაღაც ატიპიური, ამ ორ სინდრომს შორის რაღაც საშუალო მდგომარეობა შეიძლება ყოფილიყო.

იდეალურ ვარიანტში, სისხლში ყველა "ეჭვმიტანილი" წამლის კონცენტრაციის ანალიზი უნდა გაკეთდეს, რომ ზუსტად შედეგი დადგინდეს...

* * *
QUOTE
ნეტა რას ვაკეთებთ აქ 02:10 am-ზე სანდრო?


მე პირადად, ქეისმა დამაინტერესა... მგონი შენც smile.gif ისე, ახლა მეცადინეობის დაწყებას ვაპირებდი, მაგრამ ახლა არ ვიცი, რა ვქნა gigi.gif

Posted by: lgogokhia 20 Nov 2009, 12:04
Cousteau
QUOTE
სეროტონინის სინდრომია?

yes.gif yes.gif მიუხედავად იმისა რომ მეტამფეტამინები არ იწვევენ განსაკუთრებულად სეროტონინის გამოყოფას, მათი გამოყენება SSRI პრეპარატებთან მაინც სიფრთხილით უნდა იყოს, ვინაიდან ეს უფრო დოზირებაზეა დამოკიდებული. ამ შემთხვევაში იყო კლასიკურად გამოხატული drug abuse, ავადმყოფი ადრე მკურნალობდა შიზოფრენიის დიაგნოზით და უამრავჯერ drug abuse-ზე.

LUKA-BRAZI
QUOTE
AIDS ქონდა ? სხვა რომელი დაავადება ვლინდება ენის ლეიკოპლაკიით? მე ვიცი შიდსი, ქრონიკული ირიტაცია ლოკალური რომელიც შეიძლება იყოს კიბოსწინარე დაავადებაც....

კარგი შენიშვნაა, თუმცა ავადმყოფი იყო HIV -, თუმცა ძალიან ბევრი ფაქტორი შეიძლება აქ ლეიკოპლაკიის მიზეზი ყოფილიყო, იმუნოდეფიციტი, ქრონიკული ირიტაცია და ასე შემდეგ.

QUOTE
მოკლედ ბიპოლარული დაავადება იგივე მანიაკალურ-დეპრესიული სინდრომია ჰო? ჰოდა ალბათ ფსიქიატრებმა მისცეს მაგხელა დანიშნულებაც.... + ლეიკოპლაკიაზე სხვა ექიმმა. მერე ალბათ ღვიძლი გამოვიდა მწყობრიდან -> drug free radicals induced hepatitis... და მაგის ტვინს დაემატა ღვიძლის დეზინტოქსიკაციური ფუნქციის დარღვევის შედეგებიც.... 39 გრადუსიც ცენტრალური წარმოშობის მგონია ვიდრე რაიმე ზედდართული ინფექციური აგენტის...
ლოგიკური მსჯელობაა თუმცა ძალიან განზოგადოებული. ტემპერატურა კი ნამდვილად ცენტრალური იყო.
Blind_Torture_Kill
QUOTE
აუტოფსიის შედეგები დადე smile.gif

არ მაქვს.. ერთერთ გრანდ რაუნდზე მოვისმინე და რაც დავწერე მაგის იქეთ არაფერი არ ვიცი smile.gif

Thandrus
QUOTE
კლინიკური სურათით, ეს უფრო Neuroleptic Maglignant Syndrome-ს ჰგავს. სეროტონერგული სინდრომისაგან განსხვავებით, ის შედარებით ხანგრძლივი დროის განმავლობაში (1-3 დღე vs. საათები) ვითარდება, შედარებით ნაკლებად მაღალი ტემპერატურა ახასიათებს და კუნთების ტკივილი აქვს გამოხატული. თან აქ მოცემული წამლებიდან, ოლანზაპინს შეეძლო ეს გამოწევია.

აბსოლუტურად გეთანხმები, იყო ამის განხილვა კიდევაც როგორც შესაძლო ვარიანტი, თუმცა კლინიკურად მაინც სეროტონინული სინდრომის სიმპტომატიკა იყო, ვიდრე NMS რომელზეც ქეისში ვილაპარაკე. ... მიუხედავად იმისა რომ შესაძლოა NMშ ყოფილიყო, მაინც საბოლოოდ სეროტონინზე იყო ბოლო ლაპარაკი.
QUOTE
როგორც ვიცი, სეროტონინის სინდრომის დროს ტემპერატურა ხშირად 40' - 41 'C გრადუსია, NMS-ს დროს სადღაც 38 - 39.
ეს ალბათ ყველაზე ფარდობითია. სპეციფიკური კრიტერიუმების მიხედვით ამ დროს ტემპ არის 38-ზე მეტი..

ბოლო შეკითხვას დავსვამ კიდევ ამ ქეისზე: რატომ ღებულობდა ავადმყოფი სინთროიდს და ციტომელს?

Posted by: LUKA-BRAZI 20 Nov 2009, 17:48
lgogokhia
smile.gif

კარგი ქეისი იყო. კიდე დადე LG smile.gif

Posted by: Thandrus 21 Nov 2009, 17:28
lgogokhia

QUOTE
ბოლო შეკითხვას დავსვამ კიდევ ამ ქეისზე: რატომ ღებულობდა ავადმყოფი სინთროიდს და ციტომელს?


რავიცი... ათასი რამე შეიძლება ყოფილიყო. არ ვიცი, შეიძლება ზოგიერთი მიღებული წამალი ჰიპოთალამუს-ჰიპოფიზ-ფარისებრი ჯირკვლის ღერძში რაღაც დარღვევას იწვევს, ანდა პაციენტს თვითონ ჰქონდა რაღაც ამასთან დაკავშირებული ანომალია.

შეიძლება, უბრალოდაც Drug Abuse იყო.

მგონი სჯობს შენ თვითონ გვითხრა, რატომ იღებდა smile.gif

Posted by: LUKA-BRAZI 21 Nov 2009, 21:35
მარტივი ქვიზი დროის გასაყვანად smile.gif

აბა რა ხდება ამ სურათზე? რა პროცესია აღწერილი, რა ქვია ბოლო ქრომოსომას და რომელი პათოლოგიის დროს გვხვდება იგი...

გამოცნობა დასწრებაზეა givi.gif


Posted by: lgogokhia 21 Nov 2009, 23:25
Thandrus
QUOTE
რავიცი... ათასი რამე შეიძლება ყოფილიყო. არ ვიცი, შეიძლება ზოგიერთი მიღებული წამალი ჰიპოთალამუს-ჰიპოფიზ-ფარისებრი ჯირკვლის ღერძში რაღაც დარღვევას იწვევს, ანდა პაციენტს თვითონ ჰქონდა რაღაც ამასთან დაკავშირებული ანომალია.

შეგახსენებ რომ პაციენტი წლების განმავლობაში მკურნალობდა ბიპოლარულ დაავადებას smile.gif

LUKA-BRAZI
ფილადელფიის ტრანსლოკაცია და ქრონიკული მიელოგენური ლეიკემია? ეგ მახსენდება 9 და 22 ქრომოსომაზე...

Posted by: LUKA-BRAZI 21 Nov 2009, 23:52
lgogokhia
QUOTE
ფილადელფიის ტრანსლოკაცია და ქრონიკული მიელოგენური ლეიკემია? ეგ მახსენდება 9 და 22 ქრომოსომაზე...

up.gif

ჰო smile.gif

Posted by: Thandrus 22 Nov 2009, 00:38
lgogokhia

როგორც ვიცი, ლითიუმით მკურნალობას შეიძლება ჰიპოთიროიდიზმი ახასიათებდეს. მაგრამ ამ წამლებში ლითიუმი მითითებული არაა. კიდევ, შეიძლება, რომ ეს კაცი ბიპოლარულის დეპრესიის ფაზაში ყოფილიყო. ამ დროს შეიძლება ჰიპოთიროიდიზმი იყოს (TSH-ისადმი ფარისებრი ჯირკვლის მგრძნობიარობის შემცირების გამო) ან vice versa. ალბათ ამ დეპრესიის გამო აძლევდნენ.

Posted by: lgogokhia 22 Nov 2009, 23:40
Thandrus
QUOTE
როგორც ვიცი, ლითიუმით მკურნალობას შეიძლება ჰიპოთიროიდიზმი ახასიათებდეს. მაგრამ ამ წამლებში ლითიუმი მითითებული არაა.

yes.gif yes.gif ეხლა არ ღებულობს მაგრამ ღებულობდა 15 წელი.. ავღნიშნე რომ წლების განმავლობაში მკურნალობდა ბიპოლარულ დაავადებას..

Posted by: Thandrus 23 Nov 2009, 00:29
lgogokhia

ანუ, მაგის გამოა ჰიპოთიროიდიზმი?

Posted by: lgogokhia 23 Nov 2009, 11:11
Thandrus
QUOTE
ანუ, მაგის გამოა ჰიპოთიროიდიზმი?

ეგრეა.

Posted by: lgogokhia 24 Nov 2009, 11:22
vano_t
QUOTE
პაციენტს არავითარი კრუნჩხვები არ აღენიშნებოდა. შემდეგ თვითმხილველმა გამოაფხიზლა პაციენტი და გონზე რომ მოვიდა ავადმყოფი, მაშინ დაეწყო კანკალი. ტრემორის დროს პაციენტი უკვე აზრზე იყო.


როდის უნდა დაგვააფდეითო ამ პაციენტზე? boli.gif

Posted by: LUKA-BRAZI 26 Nov 2009, 17:59
lgogokhia
LG აფდეითებს მაგ პაციენტზე ალბათ დააგვიანდება ჯერ კიდევ smile.gif

მიდი რა, დადე რამე კარგი ტვინის ასამუშავებელი ქეისი და განვიხილოთ smile.gif თუ არადა მე მოვიფიქრებ რამეს smile.gif

Posted by: Thandrus 2 Dec 2009, 19:50
აი ერთი პატარა ქეისი smile.gif

71 წლის დიაბეტიანმა კაცმა ქუჩაში გონება დაკარგა. ის სასწრაფო დახმარების მანქანამ Emergency Room-ში გადაიყვანა. იქ მიყვანისას პაციენტი ფხიზელი, მაგრამ ლეთარგიული იყო, ჰქონდა მარჯვენამხრივი gaze preference, არ შეეძლო მარცხენა მხარეს თვალების მოძრაობა. ამასთან ერთად, მისი მარჯვენა ხელისა და ფეხის კუნთების ძალა იყო 2 / 5, მარჯვენამხრივი ბაბინსკის ნიშნით.

1. რომელი ლოკალიზაციის დაზიანებამ შეიძლება გამოიწვიოს ასეთი სიმპტომების ერთობლიობა?
2. პაციენტის ასაკის ისტორიისა და პრეზენტაციის გათვალისწინებით, რა არის ყველაზე სავარაუდო დიაგნოზი?

Posted by: E-36 3 Dec 2009, 10:48
ინტრაცერებრალური სისხლჩაქცევა ვითოომ ? offtopic.gif



ბაბინსკი რააა ანუ პირამიდული ტრაქტი ჩაგვიხშო თაან ? boli.gif

რაღაცეები მოვიგონე მგონი cry.gif

წავედი ტრავმატოლოგიის თემაში sad.gif
იქ არ ვიგონებ ჭეშმარიტებას ვღაღადებ sad.gif

Posted by: Thandrus 3 Dec 2009, 11:34
E-36

QUOTE
ინტრაცერებრალური სისხლჩაქცევა ვითოომ ?


ეს ერთ-ერთი მიზეზი შეიძლება რომ იყოს smile.gif

QUOTE
ბაბინსკი რააა ანუ პირამიდული ტრაქტი ჩაგვიხშო თაან ?


კი, დააზიანა.

ახლა ზუსტი ლოკალიზაცია მისათითებელი (Hint: თვალებთან დაკავშირებულ სიმპტომატიკას დაუკავშირეთ მარჯვენამხრივი სისუსტე).

Posted by: LUKA-BRAZI 3 Dec 2009, 16:56
E-36
მასე ვწვალობთ და ვჯახირობთ ყველანი აქ კაცო smile.gif იყავი არ წახვიდე smile.gif ბევრ კარგ რამეს ისწავლი და გაიგებ ამ თემაში yes.gif ვეცადოთ ეს თემა არ მოკვდეს smile.gif

Thandrus
გამოდის რომ დაზანება არის მარცხენა მხარეს?? რახან მარჯვნივ აქვს სისუსტე (პირამიდული გზები იკვეთება), თვალებს კი მარცხენა მხარეს ვერ ამოძრავებს....

პ.ს. პრიონ, პრიონ... biggrin.gif

Posted by: E-36 3 Dec 2009, 21:26
LUKA-BRAZI
ამ თემას დიდი ხანია ვკითხულობ უბრალოდ თავს ვიკავებ ხოლმე დაპოსტვისგან smile.gif უფრო მირჩევნია ტრავმატოლოგიისკენ ვპოსტო ვიწრო სპეციალობის ამბავში smile.gif

Thandrus
მე მაინც ინტრაცერებრალური სისხლჩაქცევისკენ მივდივარ...
ლოკალიზაციის მხრივ დაზიანებას შეიძლება ადგილი ქონდეს კეფის მარცხენა წილში. მიკროანგიოპათიაა ალბათ,თან შეიძლება დიაბეტურიც.შეიძლება ასევე ჰიპოფიზის ადენომაც არის არ არის გამორიცხული,თურქული კეხის წინ არ არის მხედველობის ჯვარედინი ?

სხვა ვერაფრით დაგვეხმარები ? smile.gif

Thandrus
1. რომელი ლოკალიზაციის დაზიანებამ შეიძლება გამოიწვიოს ასეთი სიმპტომების ერთობლიობა?

ჩემი პასუხია ამ ეტაპზე: კეფის მარცხენა წილის ქერქის დიაბეტური მიკროანგიოპათია
ან ჰიპოფიზის ადენომა punch.gif

Posted by: Thandrus 3 Dec 2009, 22:18
E-36

QUOTE
მარცხენა წილის ქერქის დიაბეტური მიკროანგიოპათიაან ჰიპოფიზის ადენომა


კეფის მარცხენა წილის დაზიანება მოგცემდა ორივე visul field-ის მარჯვენამხრივ სენსორულ დეფიციტს. მე კი არ მითქვამს, რომ მას აღქმასთან რამე პრობლემა ჰქონდა. მხედველობის ჯვარედინის დაზიანება კი გამოიწვევდა ბიტემპორალურ ჰემიანოპიას, რის დროსაც ორივე თვალის visual field-ის პერიფერიული ნახევრის სენსორული დეფიციტია.

ამ პაციენტს კიდევ თვალთან დაკავშირებული მოტორული დეფიციტი აქვს (არ შეუძლია მარცხენა მხარეს თვალების მოძრაობა).

LUKA-BRAZI

QUOTE
გამოდის რომ დაზანება არის მარცხენა მხარეს?? რახან მარჯვნივ აქვს სისუსტე (პირამიდული გზები იკვეთება), თვალებს კი მარცხენა მხარეს ვერ ამოძრავებს....


მართალი ხარ, დაზიანება მარცხენა მხარესაა, და კორტიკოსპინალური (ანუ იგივე პირამიდული) ტრაქტია ჩათრეული. ახლა უნდა გაიხსენო ის, რაც პასუხს აგებს თვალის მოძრაობაზე; და კიდევ თავის ტვინის რომელ ლოკუსში არის კორტიკოსპინალური ტრაქტი და ეს "რაღაც" ახლოს ერთმანეთთან და პასუხიც ხელში გექნება smile.gif

კიდევ ერთი მინიშნება: როდესაც თვალები სისუსტის მხარეს "იყურებიან" (ანუ იმ მხარეს, სადაც ხელისა და ფეხის სისუსტეა) ამას wrong-way eyes-ს უწოდებენ. ეს ნიშანი შეიძლება თავის ტვინის რამდენიმე ადგილის დაზიანებამ გამოიწვიოს.

ჰოდა ახლა თქვენი ჯერია smile.gif

QUOTE
პ.ს. პრიონ, პრიონ...


gigi.gif ეჰ, მომიწევს ალბათ ამეებზე რეფერატის გაკეთება sad.gif

Posted by: LUKA-BRAZI 3 Dec 2009, 23:00
Thandrus
QUOTE
მართალი ხარ, დაზიანება მარცხენა მხარესაა, და კორტიკოსპინალური (ანუ იგივე პირამიდული) ტრაქტია ჩათრეული. ახლა უნდა გაიხსენო ის, რაც პასუხს აგებს თვალის მოძრაობაზე; და კიდევ თავის ტვინის რომელ ლოკუსში არის კორტიკოსპინალური ტრაქტი და ეს "რაღაც" ახლოს ერთმანეთთან და პასუხიც ხელში გექნება smile.gif

კიდევ ერთი მინიშნება: როდესაც თვალები სისუსტის მხარეს "იყურებიან" (ანუ იმ მხარეს, სადაც ხელისა და ფეხის სისუსტეა) ამას wrong-way eyes-ს უწოდებენ. ეს ნიშანი შეიძლება თავის ტვინის რამდენიმე ადგილის დაზიანებამ გამოიწვიოს.

აუუუ თითქოს მახსოვს და თან არც მახსოვს ეგ პონტები biggrin.gif თსსუ-ს კურაციულ სისტემას რა ვუთხარი!
მოკლედ მაგ პაციენტს ნერვუს ოფთალმიკუსი, ქიაზმა და ტრაქტი ინტაქტური აქვს, რახან აღქმასთან პრობლემა არ აქვს. არ დავუკვირდი, პირველივე პოსტიდანვე ჩანდა რომ ოკულომოტორიუსის პრობლემებია მანდ. კორტიკოსპნალური გზა, წინა ცენტრალური ხვეულიდან იწყება და თუ არ ვცდები სადღაც წიტელ ბირთვთან ახლოს უნდა იყვნენ ერთმანეთთან ოკულომოტორიუსი და პირამიდული გზები??????????? აუ არადა კურაციის დროს ამაზე ნაღდად მეცოდინებოდა პასუხი biggrin.gif

მეორე ვარიანტი: დაზიანება არის ოკულომოტორიუსის ბირთვებში (ერთერთში) და არა თვითონ ნერვში.... ოჰ..... biggrin.gif

QUOTE
gigi.gif ეჰ, მომიწევს ალბათ ამეებზე რეფერატის გაკეთება sad.gif

მე შარშან დავწერე და კათედრის გამგეს მაგრად გაუკვირდა, მილიონში 1-ს ემართება და საიდან გამოჩხრიკეო smile.gif რო დაწერ, წამაკითხე, მაინტერესებს, თუ გინდა ჩემსას გაჩვენებ smile.gif
* * *
E-36
QUOTE
ამ თემას დიდი ხანია ვკითხულობ უბრალოდ თავს ვიკავებ ხოლმე დაპოსტვისგან smile.gif უფრო მირჩევნია ტრავმატოლოგიისკენ ვპოსტო ვიწრო სპეციალობის ამბავში

იქაც და აქაც biggrin.gif

Posted by: Thandrus 4 Dec 2009, 00:16
LUKA-BRAZI

თანთათან სწორ პასუხს უახლობდები, მაგრამ გეზი ცოტათი უნდა შეცვალო smile.gif

ოკულომოტორიუსის პარალიზი სხვანაირ სიმპტომატიკას მოგცემს: ეს არის 1. down-and-out თვალები (რომესაც გუგა მოთავსსებულია გვერდით და ქვემოთ) 2. სრული პტოზი, ანუ თვალი დახუჭულია (ოკულომოტორიუსი ლევატორ პალპებრესაც აინერვირებს) და 3. გაფართოებული გუგა, რომელიც სინათლეს ვერ პასუხობს შეკუმშვით (რადგან ოკულომოტორიუსის პარასიმპათიკური ნაწილია დაზიანებული).

ამიტომ... სხვა ნერვებზე გადადი biggrin.gif



Posted by: LUKA-BRAZI 4 Dec 2009, 00:41
Thandrus
QUOTE
ოკულომოტორიუსის პარალიზი სხვანაირ სიმპტომატიკას მოგცემს: ეს არის 1. down-and-out თვალები (რომესაც გუგა მოთავსსებულია გვერდით და ქვემოთ) 2. სრული პტოზი, ანუ თვალი დახუჭულია (ოკულომოტორიუსი ლევატორ პალპებრესაც აინერვირებს) და 3. გაფართოებული გუგა, რომელიც სინათლეს ვერ პასუხობს შეკუმშვით (რადგან ოკულომოტორიუსის პარასიმპათიკური ნაწილია დაზიანებული).

მართალი ხარ, მართალი yes.gif

QUOTE
ამიტომ... სხვა ნერვებზე გადადი

ნერვუს აბდუცეეეეეეეეეეეეეეეენს ! bis.gif
რა, არა? spy.gif
biggrin.gif
ან Trochlear nerve, nut less likely smile.gif

პ.ს. ნეიროანატომიაზე ხართ დ.ღ.-სთან? biggrin.gif

Posted by: E-36 4 Dec 2009, 18:32
აუF მე ვაფშე საით წავედი sad.gif

თურმე რა ხდება cry.gif

Thandrus
დაწერე რა სწორი პასუხი user.gif

Posted by: Thandrus 4 Dec 2009, 19:07
LUKA-BRAZI
E-36

კარგით, მაშინ დავწერ პასუხს.

QUOTE
ნერვუს აბდუცეეეეეეეეეეეეეეეენს !


დაზიანებულია პონსის (ხიდის) მარცხენა ნაწილი. პონსში მოთავსებულია აბდუცენსის ბირთვი. გარდა იმისა, რომ აბდუცენსის ნუკლეუსიდან აბდუცენს ნერვი მიდის იფსილატერალური თვალის ლატერალურ რექტუსში. ეს ბირთვი არგრეთვე დაკავშირებულია ოკულომოტორულ, ვესტიბულარულ, აბდუცენს (მეორე მხარის) და ოკულომოტორიუსის ბირთვებთან სტრუქტურით, რომელსაც Medial Longitudinal Fasciculus (MLF) ჰქვია. ამ სტრუქტურის სწალობით, აბდუცენსის ბირთვი უფრო მეტს აკეთებს, ვიდრე უბრალოდ იფსილატერალური თვალის აბდუქციაა, ის ორივე თვალის ჰორიზონტალურ მოძრაობას აკონტროლებს.

მაშ ასე, აბდუცენსის ბირთვის დაზიანებისას ვერცერთი თვალი ვერ იხედება დაზიანებული მხარისაკენ. გარდა ამისა, აბდუცენსის ბირთვთან ახლოს მოთავსებულია ე. წ. Paramedian Pontine Reticular Formation (PPRF) რომელსაც აბდუცენსის ბირთვში სიგნალები მოაქცს ქერქიდან და რიგი სხვა სტრუქტურებიდან. PPRF-ს დაზიანებამაც აბდუცენსის ბირთვის დაზიანებისნაირ თვალისმიერ სიმპტომებს მოგვცემს.

ახლა, ლუკა, რაც შეეხება შენს ვარაუდს (ნერვუს აბდუცენს): აბდუცენსის ნერვის დაზიანების დროს თვითონ ბირთვი თუ ნორმაშია, მაშინ horizontal gaze დაზიანებული არ იქნება. ანუ, სისუსტე იქნება მხოლოდ იფსილატერალური თვალის აბდუქციაში, მეორე თვალი კი წესრიგში იქნება.

ახლა რაც ზოგადად wrong-way eyes-ის ნიშანს შეეხება: ის შეიძლება რამდენიმე მიზეზით იყოს გამოწვეული: 1. კრუნჩხვითი აქტიურობა კორტექსში (კორტექსში მოთვსებულია ე. წ. frontal eye field, რომელიც კონტრალატერალურ მხარეს თვალის სწაფ მოძრაობას, საკადებს, უზრუნველჰყოფს - ასე რომ, ამ დროს თვალები კრუნჩხვის საპირისპირო მხარეს იქნება მიმართული; ამასთან ერთად თუ მოტორული კორტექსიცაა ჩათრეული, მაშინ კონტრალატერალურ მხარეს სისუსტე შეიძლება მოგვცეს) 2. დიდი სისხლჩაქცევა თალამუსში (სისხლჩაქცევამ თალამუსის ახლოს გამავალი კორტიკოსპინალური ტრაქტი შეიძლება დააზიანოს და თვალები სუსტი მხარისაკენ მიმართოს; ამ დროს პაციენტი ხშირად ღრმა კომაში იმყოფება.) 3. პონსის დაზიანება (ამაზე უკვე დავწერე)

მოკლედ, MRI-მ აჩვენა, რომ ამ პაციენტს პონსის ინფარქტი ჰქონდა. მიზეზი - პაროქსიზმული წინაგულოვანი ფიბრილაციის შედეგად გულში წამოქმნილი თრომბი, რომელიც ბაზილარულ არტერიაში მოხვდა და საბოლოოდ ბაზილარული არტერიის პატარა ტოტი, პონსში გამომავალი სისხლძარღვი დაახშო.

იმედია ქეისი მოგეწონათ smile.gif

Posted by: E-36 4 Dec 2009, 19:13
QUOTE (Thandrus @ 4 Dec 2009, 19:07 )
იმედია ქეისი მოგეწონათ smile.gif

ძალიან up.gif

სერიოზული ქეისი იყო მართლა up.gif up.gif




next pleasee fig.gif

Posted by: LUKA-BRAZI 4 Dec 2009, 20:56
Thandrus
QUOTE
იმედია ქეისი მოგეწონათ

კი კარგი იყო up.gif

smile.gif

Posted by: Thandrus 21 Dec 2009, 18:54
ამოვწიოთ თენა...

vano_t, წინა გვერდებზე რომ პაციენტზე გვიამბობდი, რა ბედი ეწია?

Posted by: E-36 21 Dec 2009, 19:36
QUOTE (Thandrus @ 21 Dec 2009, 18:54 )
ამოვწიოთ...

+1 smile.gif

დადეთ რამე ქეისი smile.gif

Posted by: MAIN KAMPF 21 Dec 2009, 19:43
დავდებ მაგრამ არ ვიცი როგორ დავდო აქ სურათი მასწავლეთ smile.gif

Posted by: Thandrus 21 Dec 2009, 19:51
MAIN KAMPF

ადვილია. თუ 1) სურათი 100კბ-ზე ნაკლებია, მაშინ "preview post"-ს დააჭერ და იქ ~file attachment"-ის ოპცია გამოჩნდება და იქ მიამაგრებ სურათს. 2) 100კბ-ზე დიდია, მაშინ

http://images.beef.ge/

ამ საიტზე ატვირთე სურათი და მერე იქიდან Direct Image Link (BB Code):-ს ლინკი შემს პოსტში დააკოპირე smile.gif

Posted by: E-36 21 Dec 2009, 19:56
QUOTE (MAIN KAMPF @ 21 Dec 2009, 19:43 )
დავდებ მაგრამ არ ვიცი როგორ დავდო აქ სურათი მასწავლეთ smile.gif

100კ-მდე სურათს თვითონ ფორუმი გვაძლევს უფლებას დავდოთ...
თუ მეტია ზომაში
http://www.radikal.ru/
შევედით აქ...
შემდგომ ავირჩიეთ სურათი BROWSE
შემდგომ ზაგრუზიტ...
შემდგომ მეორე გრაფაში მოცემული კარტინკა ვ ტეკსტე-ში ლინკი პირდაპირ დააკოპირე აქ და სურათიც გამოჩნდება smile.gif


Thandrus
უი აგიხსნია უკვე smile.gif

Posted by: MAIN KAMPF 21 Dec 2009, 20:04
http://www.radikal.ru
სურათი დავაგდე ვნახოთ თუ გამოჩნდა.
პაციენტი მოყვანილია ავტოსაგზაო შემთხვევიდან. იყო უგონო მდგომარეობაში სასწრაფოს მიერ დაინტუბირებული იქნა. საავადმყოფოში შემოსვლისთანავე გაუკეთდა რ-გრაფია. რა ჩანს სურათზე, აზრები chups.gif

Posted by: LUKA-BRAZI 21 Dec 2009, 20:48
რა საინტერესო დაჩრდილვაა? biggrin.gif თან ნეკნის კიდის გასწვრივ, თითქოს სახაზავით გაუხაზავს ვინმეს....

ჰაერია მუცლის ღრუში? baby.gif
* * *
P.S. MAIN KAMPF კარგი ქენი რომ ეს თემა ამოწიე up.gif


Posted by: lgogokhia 21 Dec 2009, 20:53
MAIN KAMPF
დიაფრაგმის რუპტურა? baby.gif user.gif

Posted by: vano_t 21 Dec 2009, 22:00
QUOTE (MAIN KAMPF @ 21 Dec 2009, 20:04 )
სურათი დავაგდე ვნახოთ თუ გამოჩნდა.
პაციენტი მოყვანილია ავტოსაგზაო შემთხვევიდან. იყო უგონო მდგომარეობაში სასწრაფოს მიერ დაინტუბირებული იქნა. საავადმყოფოში შემოსვლისთანავე გაუკეთდა რ-გრაფია. რა ჩანს სურათზე, აზრები chups.gif

პნევმოპერიტონეუმი და პნევმოთორაქსი ერთად ჩანს. დაიფრაგმა არ ჩანს (LG-ს ვეთანხმები). შეიძლება ავადმფოფს აქვს ტრავმული პნეუმოთორაქსი დიაფრაგმის გახეთქვით (რაპტურე), რაც მოგცემს პნევმოპერიტონეუმსაც. ან მუცლის ღრუს რომელიმე ორგანოს პერფორაცია (ან დიდი ჭრილობა, საიდანაც ჰაერი შეიძლება მოხდეს მუცლის ღრუში) და დიაფრაგმის გახეთქვა, რაც მოგცემს პნევმოთორაქს; ან ორივე ერთად.

Posted by: MAIN KAMPF 21 Dec 2009, 22:32
biggrin.gif ყურადღებით დააკვირდით
QUOTE
თითქოს სახაზავით გაუხაზავს ვინმეს....
საინტერესო მიგნებაა
yes.gif ერთი მიხვედრება გითხარით
QUOTE
სასწრაფოს მიერ დაინტუბირებული იქნა.

biggrin.gif აბა სცადეთ რაღაცეებით უახლოვდებით, როცა მეტყვით მაშინ ავხსნი მანამდე თუ არავინ გამოიცნო. შემდეგ ახალი ქეისი წამოვა smile.gif

Posted by: anarxisti 21 Dec 2009, 22:43
QUOTE (MAIN KAMPF @ 21 Dec 2009, 22:32 )
biggrin.gif ყურადღებით დააკვირდით
QUOTE
თითქოს სახაზავით გაუხაზავს ვინმეს....
საინტერესო მიგნებაა
yes.gif ერთი მიხვედრება გითხარით
QUOTE
სასწრაფოს მიერ დაინტუბირებული იქნა.

biggrin.gif აბა სცადეთ რაღაცეებით უახლოვდებით, როცა მეტყვით მაშინ ავხსნი მანამდე თუ არავინ გამოიცნო. შემდეგ ახალი ქეისი წამოვა smile.gif

უცხო სხეულია "ჩარჭობილი" ??
ღვიძლი ხომ არაა მკერდის ღრუში "გადასული"?

Posted by: LUKA-BRAZI 21 Dec 2009, 22:47
anarxisti
QUOTE
ღვიძლი ხომ არაა მკერდის ღრუში "გადასული"?

ვაა, საინტერესოა! თითქოს მარცხენა წილის კიდე უნდა იყოს ის სწორი ხაზი baby.gif

და ინტუბაცია რა შუაში უნდა იყოს?

MAIN KAMPF
დაგვაინტრიგე biggrin.gif

Posted by: Cousteau 21 Dec 2009, 22:49
საყლაპავი დაუინტუბირეს ?
კუჭი ანუ

Posted by: anarxisti 21 Dec 2009, 22:52
QUOTE (Cousteau @ 21 Dec 2009, 22:49 )
საყლაპავი დაუინტუბირეს                                            ?

ინტუბაციის პრობლემაა, კი, ოღონდ არა საყლაპავის...

კანქვეშა ემფიზემაა, ტრაქეის გასკდომა დაზიანების გამო!

Posted by: LUKA-BRAZI 21 Dec 2009, 22:53
შესწორება შემაქვს ჩემს პოსტში: ღვიძლი მასეთ დაჩრდილვას არ იძლევა მგონი.....

QUOTE
საყლაპავი დაუინტუბირეს ?

და სადმე ხვრელიდან გადავიდა მუცლის ღრუში? კარგი იდეა უნდა იყოს smile.gif
* * *
QUOTE
კანქვეშა ემფიზემაა, ტრაქეის გასკდომა დაზიანების გამო!

და ის სწორი ხაზი ანარხისტ? რამეს მართლა ნისნავს თუ დაგვაბნია ქეისის ავტორმა? smile.gif

Posted by: MAIN KAMPF 21 Dec 2009, 22:54
[quote]quote]turboman]
biggrin.gif შუაშია ძალიანაც biggrin.gif კითხვები:
1.ჰაერი არის თუ არა მუცლის ღრუში?
2.გამომდინარე პირველი კითხვიდან თუ არის ჰაერი სად მიმანიშნეთ მოხაზეთ და კიდევ[quote]და ინტუბაცია რა შუაში უნდა იყოს? [/quote]
biggrin.gif

Posted by: LUKA-BRAZI 21 Dec 2009, 22:57
MAIN KAMPF
კაცო ინტუბაცია რა შუაში უნდა იყოს მეთქი მე ვთქვი ეგ ანარხისტის პოსტზე, რომ თუ რვიძლია მანდ საქმეში ინტუბაცია რაღა შუაშია თქო smile.gif


Posted by: MAIN KAMPF 21 Dec 2009, 22:58
QUOTE
საყლაპავი დაუინტუბირეს ?
კუჭი ანუ

შეწყდა მსჯელობა ეგ არის
bis.gif დააკვირდით კუჭის კონტურს როგორ იმეორებს. alk.gif ათი ოქრო ჩემგან biggrin.gif

Posted by: Cousteau 21 Dec 2009, 22:58
: ))))))))))))))))))

QUOTE
ათი ოქრო ჩემგან

2kiss.gif
5ს მე დავიტოვებ და 5იც ხალხს დავურიგოთ : D

Posted by: lgogokhia 21 Dec 2009, 22:59
წინასწარ გეუბნებით რომ სუიციდს ჩავიტარებ თუ ეს საყლაპავის ინტუბაცია ან დაკავშირებული გართულებებია.. biggrin.gif biggrin.gif

Posted by: LUKA-BRAZI 21 Dec 2009, 22:59
კუსტომ გვაჯობა ყველას! smile.gif

წავიდა შემდეგი ქეისი smile.gif
* * *
შემდეგი ქეისი თორემ დაგვორნავთ ყველას! gigi.gif

Cousteau
დღგ უნდა გადაიხადო ფორუმის ადმინისტრაციასთან yes.gif და მოგების გადასახადიც გეკისრება givi.gif

Posted by: MAIN KAMPF 21 Dec 2009, 23:06
ცოტა ხანში წამოვა ახალი ქეისი. ეს ფოტო ესპანელი ექიმების გამოგზავნილია. ქალბატონია გამოსახული მოიფიქრეთ რა დაავადება შეიძლება სჭირდეს. ერთადერთი რაც შემიძლია ვთქვა რამოდენიმე წელია ავადმყოფობდა, წინა ქეისისაგან განხვავებით რთულია smile.gif

Posted by: lgogokhia 21 Dec 2009, 23:07
vis.gif vis.gif vis.gif vis.gif წავედი მე სადმე ჩემთვის მოვკვდები ჩუმად!!! ეხლა ეგ სურატი უნდა დავუგზავნო ყველას ვინც 03-სჰი მუშაობს.. ვიცნობ რამდენიმეს.. MAIN KAMPF რახან აქ დადე, ჩავთვლი რომ არ არის სურათი კონფიდენციალური..

MAIN KAMPF
აბა ფოტო?

Posted by: LUKA-BRAZI 21 Dec 2009, 23:09
MAIN KAMPF
სურათი არ ჩანს smile.gif

Posted by: MAIN KAMPF 21 Dec 2009, 23:11
QUOTE
ეხლა ეგ სურატი უნდა დავუგზავნო ყველას ვინც 03-სჰი მუშაობს..

არ გინდა, ჩვენ აქ ვიმსჯელოთ ჩვენთვის,
yes.gif
* * *
http://www.radikal.ru

Posted by: lgogokhia 21 Dec 2009, 23:14
MAIN KAMPF
ოკ.. no problem.. რა მაინტერესებს - პაციენტს რა უთხარით, რა გჭირსო? თუ ვერ მოასწარით?

Posted by: Thandrus 21 Dec 2009, 23:17
MAIN KAMPF

ტერატომა ან ომფალოცელე...

ისე, რადიოლოგია ჩემი სუსტი წერტილია და შეგიძლიათ ზუსტად ამიხსნათ, წინა სურათზე სად რა და როგორ იყო? gigi.gif ვერ აღვიქვი.

Posted by: anarxisti 21 Dec 2009, 23:18
QUOTE (MAIN KAMPF @ 21 Dec 2009, 23:11 )
QUOTE
ეხლა ეგ სურატი უნდა დავუგზავნო ყველას ვინც 03-სჰი მუშაობს..

არ გინდა, ჩვენ აქ ვიმსჯელოთ ჩვენთვის,
yes.gif
* * *
http://www.radikal.ru

საშვილოსნოს გიგანტური ფიბრომა?
ან საკვერცხის კეთილთვისებიანი სიმსივნე (ფიბროტეკომა)??

დიდხანს ავდმყობობდაო... ე ი ავთვისებიანი რამ არ უნდა იყოს
კანის ცვლილებები მეორადია, უფრო

Posted by: LUKA-BRAZI 21 Dec 2009, 23:18
ტერატომა???

ეგ დისემბრიოგენეზული პათოლოგია არ არის? ტერატომა შეიძლება მაგხელა იყოს? eek.gif
* * *
ანარხისტს უფრო ვემხრობი. სიმეტრიულად არის რაღაც წარმონაქმნი და უფრო გიგანტური ფიბრომა შეიძლება იყოს...... მეორე ვარიანტი არ ვიცი წესიერად...... ისე, რომელიმე ჰიპოგასტრიუმის ორგანოს პრობლემა უნდა იყოს დანამდვილებით (IMHO biggrin.gif)

Posted by: MAIN KAMPF 21 Dec 2009, 23:24
QUOTE
საშვილოსნოს გიგანტური ფიბრომა?

გასაგებია შენი სპეციალობაა
biggrin.gif ისე ჩვენმა გინეკოლოგებმაც კი ნახეს მაგრამ დღემდე ჰგონიათ რომ დიდი ზომის თიაქარია lol.gif lol.gif ყოჩაღ bis.gif მეგონა დიდი სჯა ბაასი წავიდოდა

Posted by: LUKA-BRAZI 21 Dec 2009, 23:25
მივულოცოთ anarxist-ს smile.gif

bis.gif

Posted by: MAIN KAMPF 21 Dec 2009, 23:27
quote]წინა სურათზე სად რა და როგორ იყო? ვერ აღვიქვი[/quote]
კონტური მეორდება კუჭისას თუ დაუკვირდები ყურადღებით, აქ მონიშვნა სურათზე არ ვიცი როგორ კეთდება თორემ გაჩვენებდი

Posted by: Thandrus 21 Dec 2009, 23:27
LUKA-BRAZI

QUOTE
ტერატომა შეიძლება მაგხელა იყოს?


შეიძლება იყოს, მაგრამ გეთანხმები, სისულელე ვთქვი lol.gif სად ეს ქალი და სად ტერატომა biggrin.gif

QUOTE
საშვილოსნოს გიგანტური ფიბრომა?


ჰო... ცოტათი დაფიქრება იყო საჭირო tongue.gif

MAIN KAMPF

მარა, ისე მაინტერესებს, აქამდე ეს ქალი რას ელოდებოდა? სიმსივნის ზომის მსოფლიო რეკორდის დამყარება უნდოდა? biggrin.gif

Posted by: anarxisti 21 Dec 2009, 23:28
QUOTE (MAIN KAMPF @ 21 Dec 2009, 23:24 )
QUOTE
საშვილოსნოს გიგანტური ფიბრომა?

გასაგებია შენი სპეციალობაა
biggrin.gif ისე ჩვენმა გინეკოლოგებმაც კი ნახეს მაგრამ დღემდე ჰგონიათ რომ დიდი ზომის თიაქარია lol.gif lol.gif ყოჩაღ bis.gif მეგონა დიდი სჯა ბაასი წავიდოდა

მყოლია ეგეთი პაციენტი... მთლად ამხელა არ იყო, მართალია..
რაც მთავარია.. არაფერი არ აწუხებდა gigi.gif

აგერ მსგავსი კეისიც, მაგრამ მაინც რა არის, ზუსტად... 68 წლის ბებია იყო gigi.gif ,სახლში მარტო ცხოვრობდა, დავარდა, ფეხი მოიტეხა და ამის გამო მოხვდა საავადმყოფოში

Posted by: LUKA-BRAZI 21 Dec 2009, 23:29
ერთი ქეისიც და გავდივართ givi.gif

Posted by: MAIN KAMPF 21 Dec 2009, 23:31
quote]მარა, ისე მაინტერესებს, აქამდე ეს ქალი რას ელოდებოდა? სიმსივნის ზომის მსოფლიო რეკორდის დამყარება უნდოდა?
ჰო ეგ შეკითხვა მეც გამიჩნდა და ასეთი პასუხი იყო რომ ეს ქალი სადღაც მიყრუებულ სოფელში მთაში ცხოვრობდა,არ ვიცი შესაბამისად არც ინტელექტი უჭრიდა ალბათ. ოპერაცია გაუკეთდა

Posted by: Ni-L 21 Dec 2009, 23:37
anarxisti
მაგ ბებოს გულის პრობლემები რომ აქვს ეგ არაფერია....
ხოდა, მუცელში სიმსივნეა>>>?

Posted by: LUKA-BRAZI 21 Dec 2009, 23:39
Ni-L
QUOTE
მაგ ბებოს გულის პრობლემები რომ აქვს ეგ არაფერია....

და რა, დილატაცია აქვს ვითომ? spy.gif მგონი გულის მდებარეობა აქვს მასეთი + გაზრდილი წნევა მუცლის ღრუში და დიაფრაგმის მაღლა დგომა + კონსტიტუციური აგებულება....... რავიცი აბა smile.gif

Posted by: MAIN KAMPF 21 Dec 2009, 23:39
QUOTE
ხოდა, მუცელში სიმსივნეა>>>?

ზემოთ რა არის საყლაპავია კონტრასტირებული თუ არტე ფაქტია


Posted by: Ni-L 21 Dec 2009, 23:45
სასულე დეფორმირებულია, ხოდა მანდ მგონი მიმდებარე ლიმფური კვანძები გადიდებულია, გული ხო მინი გიგანტია თავისი აორტით, თუმცა ამხელა გულს მასეთი ერ ეკადრება... biggrin.gif

Posted by: anarxisti 21 Dec 2009, 23:47
QUOTE (Ni-L @ 21 Dec 2009, 23:37 )
anarxisti
მაგ ბებოს გულის პრობლემები რომ აქვს ეგ არაფერია....
ხოდა, მუცელში სიმსივნეა>>>?

გულის პრობლემები მერე დაეწყო...ოპერაციის მერე ჰემოდინამიკის დარღვევის გამო..

კი, სიმსივნეა, კეთილთვისებიანი, საკვერცხის გიგანტური სეროზული ცისტადენომა.. 18 ლიტრი იყო შიგთავსი... ესეც მრავალი წელი ზრდიდა gigi.gif მარტოხელა იყო და არც "აწუხებდა".
ოპერაციის მერე სიარულიც კი უჭირდა, სიმძიმის ცენტრი დაერღვა..

აგერ ოპერაციის წინ


Posted by: Ni-L 21 Dec 2009, 23:51
anarxisti
ეგ არაფერი, ერთხელ შემთხვევით მოხვდა 8 თვის ორსული საავადმყოფოში, ხოდა მაშინ გაიგო რომ ორსულად იყო, არც მუცლის ზომები აწუხებდა და არც სიმძიმის ცენტრი გადატანა...

Posted by: anarxisti 21 Dec 2009, 23:54
QUOTE (Ni-L @ 21 Dec 2009, 23:51 )
anarxisti
ეგ არაფერი, ერთხელ შემთხვევით მოხვდა 8 თვის ორსული საავადმყოფოში, ხოდა მაშინ გაიგო რომ ორსულად იყო, არც მუცლის ზომები აწუხებდა და არც სიმძიმის ცენტრი გადატანა...

ეგ ჩვეულებრივი ამბავია... ორსულობის ნეგაცია ჰქვია..
მე მყავდა ერთი ახალგაზრდა პაციენტი ეგეთი, 8 თვის ორსული, დედამ მოიყვანა, შარვალი არ ეტევაო!!! gigi.gif

Posted by: Ni-L 21 Dec 2009, 23:55
რენტგენზე დადეთ რა რამე კიდევ, საინტერესოა.... smile.gif

Posted by: LUKA-BRAZI 22 Dec 2009, 00:00
ძალიან კარგია კლინიკური შემთხვევების განხილვა smile.gif სასარგებლო და საინტერესო. გავაგრძელოთ რა რეგულარულად smile.gif

Posted by: MAIN KAMPF 23 Dec 2009, 13:34
ყურადღებით შეხედეთ ამ სურათს ხარისხი არ უვარგა, რა პათოლოგია შეიძლება დაინახოთ?
http://www.radikal.ru

Posted by: donvaso 23 Dec 2009, 21:19
MAIN KAMPF
მარჯვენა დიაფრაგმის რელაქსაცია(მაღლა დგომა)(ღვიძლის გადიდების ხარჯზე)?

Posted by: MAIN KAMPF 23 Dec 2009, 21:40
QUOTE
მარჯვენა დიაფრაგმის რელაქსაცია(მაღლა დგომა)(ღვიძლის გადიდების ხარჯზე)?

no.gif თუმცა მაღლა დგომა არის, სხვა მიზეზი

Posted by: donvaso 23 Dec 2009, 22:19
MAIN KAMPF
სხვა ვერაფერს ვხედავ.... sad.gif

Posted by: MAIN KAMPF 23 Dec 2009, 22:24
ამ სურათზე მეც გამიჭირდა დანახვა ერთმა ჩემმა კოლეგამ მომაწოდა ჰოდა ეს სურათი გადაუღეს პაციენტს რომელსაც საყლაპავის ტრავმული გასკდომა ჰქონდა, მარჯვნივ თუ დაუკვირდები არის კონტრასტის გამოსვლა smile.gif

Posted by: donvaso 23 Dec 2009, 22:25
MAIN KAMPF
ეგ სურათის დეფექტი მეგონა..... sad.gif

Posted by: MAIN KAMPF 23 Dec 2009, 22:33
მეც რა დაგიმალო ეგრე მეგონა ის კი არა გამიკვირდა მთლიან კონტრასტირებას ვერ ვხედავ, მაგრამ ფაქტია ავადმყოფს ავტო ავარიის შემდგომ დაემართა smile.gif
* * *
ეხლა დავდებ ძალინ საინტერესო სურათებს, ეს დაავადება უმეტესად ცენტრალურ აფრიკაშია გავრცელებული და იშვიათია. თუ ვინმემ იცის მიპასუხოს biggrin.gif
http://www.radikal.ru
http://www.radikal.ru
http://www.radikal.ru
ასევე სჭირდება თუ არა ამ დაავადებას ოპერაცია. პ.ს. სურათები ცუდი სანახავია puke.gif

Posted by: Thandrus 23 Dec 2009, 23:26
MAIN KAMPF

ეს მგონი ბურკიტის სიმსივნე უნდა იყოს, არა?

QUOTE
ასევე სჭირდება თუ არა ამ დაავადებას ოპერაცია


მკურნალობა უმთავრესად ქემოთერაპიულია, თუმცა ზოგჯერ მგონი ოპერაციასაც აკეთებენ...

Posted by: LUKA-BRAZI 24 Dec 2009, 00:18
QUOTE
მგონი ბურკიტის სიმსივნე უნდა იყოს

ვეთანხმები yes.gif

Posted by: vano_t 24 Dec 2009, 06:10
ბურკიტის ლიმფომა ყველაზე სავარაუდო დიაგნოზია, ყველა გეთანხმებით. თუმცა, ყველა სიმსივნის დროს სხვა დიფერენციალური დიაგნოზებიც არის შესაძლებელი. 3 საინტერესო სურათს დავდებ:
user posted image
ამ ბაშვს, მაგალითად, აღმოაჩნდა შვანომა.

user posted image
ამ ქალს მარჯვენა ყბაყურა ჯირვლის სიმსივნე აქვს.

user posted image
ამ ბაშვს კიდევ აღმოაჩნდა რაღაც სიმსივნე, რომელსაც ქვიებია გიგანტური ცემენტომა (Gigantiform Cementoma)

******************************

68 წლის ქალი მოდის კლინიკაში (ერთ-ერთი established პაციენტია). დაახლოებით 4 კვირის წინ ვნახეთ კლინიკაში ტერფის ორმხრივი ძლიერი ტკვილებით. ავადმყოფი გაშვებული იქნა ორთოპედთან და დაესვა Gout-ის (ქართულად პოდაგრა, თუ არ ვცდები) დიაგნოზი. შარდმჟავა 11-მდე ქონდა ამავე დროს. ორთოპედმა ავადმყოფი დასვა ალოპურინოლზე და მელოქსიკამზე (არასტერიოდიული ანთების საწინააღმდეგო პრეპარატია). კლინიკაში მოვიდა იმისათვის, რომ follow-up გაგვეკეთებინა. ამ ავადმყოფს აქვს ასევე თირკმლების ქრონიკული უკმარისობა თირკმლის არტერიების სტენოზის გამო. ამიტომ კრეატინინიც და შარდმჟავაც შევამოწმეთ ელექტროლიტებთან ერთად. ავადმყოფის საშუალო კრეატინინი მერყეობდა 1.5 და 1.8-ს შორის (დაახლოებით შეესაბამება კრეტინინის კლირენსს 45-50). ბოლო შარდოვანა დაახლოებით 30 იყო. ამ ვიზიტზე კრეატინინი აწეული იყო 2.3-მდე და შარდოვანაც 56-მდე. ასევე მაღალი ქონდა კალიუმი. სავარაუდოდ თირკმლების ფუნქციის მწვავე დაქვეითება გამოწვეული იყო მელოქსიკამით და ავადმყოფს შეუწყდა მელოქსიკამი. დაენიშნა კეიექსალატი (kayexalate, რომელიც კალიუმის შეწოვის დასაქვეითებლად გამოიყენება ჰიპერკალემიის დროს). ასევ დროებით შეუწყდა რამიპრილი (აგფ ინჰიბიტორია) ჰიპერკმალემიის და თირკმლის მწვავე უკმარისობის გამო.

ავადმყოფი დაბარებულ იქნა 1 კვირაში ლაბორატორიების გადასამწომებლად. დაახლოებით მეოთხე დღეს ავადმყოფი მოვიდა ემერჯენსიში შემდეგი ჩივილებით: წინა ღამით გამოაყარა. აღწერით ეს გამონაყარი მცირე ზომის იყო, შემაღლებული და ქავილი ქონდა ძალიან. გამონაყარი ქონდა ზედა კიდურებზე, მკერდზე, მუცელზე და ცოტა ქვედა კიდურებზეც. იმავე ღამეს გამონაყარმა თითქმის გაუარა და მეორე დილით ტუჩები გაუსივდა.

ავადმყოფს თირკმლის ქრონიკული უკმარისობის გარდა აქვს ჰიპერტენზია.

წამლებიდან იღებდა რამიპრილს, რომელიც 4 დღის წინ შევუწყვიტეთ, ალოპურინოლს; რომელიც 4 კვირის წინ დაეწყო; კეიექსალატს, რომელიც 4 დღის წინ დაეწყო, რომელიღაც კალციუმის არხის ბლოკატორს (ზუსტად აღარ მახსოვს რომელს) ჰიპერტენზიისათვის მრავალი წლის განმავლობაში.

1) რაზე უნდა იქნეს ყურადღება მიქცეული ფიზიკური გამოკველევის დროს აუცილებლად?

2) შესაძლო დიფერენციალური დიაგნოზები.

3) იმერჯენსიში ავადმყოფის მკურნალობა და იმერჯენსიდან გაშვებისას ავადმყოფი მკურნალობა რა არის?

თუ დამატებით კითხვები გაქვთ, არ მოგერიდოთ smile.gif

Posted by: lgogokhia 24 Dec 2009, 14:02
vano_t
გამონაყრიდან რამდენ ხანში მოხვდა ემერჯენსიში? რამიპრილი როდის შეუწყვიტეთ, ამ ეპიზოდამდე თუ მის შემდეგ (ანგიოედემა????) ? ალოპურინოლის ჰიპერსენსიტიურობბგჰკჯჰ;კ;'ლკ - hypersensitivity syndrome? CBC-ში რა ხდებოდა? და თირკმლის ფუნქციაშიც? მე ვგულისხმობ ეპიზოდის შემდეგ..
* * *
მე შეკითხვა მაქვს, რომელიც სხვა თემაში არ მინდა გავიტანო, რადგან ამ თემის პასუხის გამცემი ხალხი მხოლოდ აქ იკრიბება: 60 წლის ქალს აქვს შესაძლო ინფლუენზა და არის რისკის ჯგუფში (რადიოთერაპიის შემდგომი პერიოდი). 2 დღე ქონდა სუბფებრილური ტემპერატურა, რინიტი, ხველა/ყელის ტკივილი and all that jazz. დაახლოებით 10 წლის წინ ქონდა TB, რომლის შემდეგ რეაქტივაცია არ მომხდარა მიუხედავად ბევრი ხელსაყრელი მომენტისა. დღეს პოლიკლინიკის ექიმმა დაუნიშნა ცეფტრიაქსონი (პრეპარატი პეო - რა ხილია არ ვიცი ოღონდ), კურანტილი (ისე გულისთვის კაიაო biggrin.gif მოვუხსენი), მილდრონატი (უხარია და გაიკეთოს), კესტინი (ებასტინი, შველის მის ალერგიულ რინიტს კარგად) და რაღაც გაურკვეველი წარმოშობის საკვები დანამატი, რომელიც მოვუხსენი, რადგან სხვა, უკეთესი რეპუტაციის დანამატებს ღებულობს. რამდენად მიზანშეწონილად მიგაჩნიათ ცეფტრიაქსონის ჩანაცვლება ლევოფლოქსაცინით (ლევოქსიმედი) ანამნეზიდან გამომდინარე.

მაპატიეთ გაოფება, მაგრამ მჭირდება თქვენი აზრი..

Posted by: MAIN KAMPF 24 Dec 2009, 17:02
QUOTE
ეს მგონი ბურკიტის სიმსივნე უნდა იყოს, არა?

yes.gif
QUOTE
ინფლუენზა
მასზე როგორც ცეფტრიაქსონი(პეო)უშვებს ქართული წარმოება "ჯმპ" ასევე ლევოფლოქსაცინი (ფტორქინოლონი) მოქმედებს მე აზრს ვერ ვხედავ შეცვლისა მითუმეტეს დღეს დანიშნული პრეპარატი პეო ავადმყოფს არ მიუღია და შესაბამისად ეფექტს ვერც ნახავდი, კურანტილი
biggrin.gif გასაგებია.
QUOTE
ანამნეზიდან გამომდინარე.
რა მაინც? კონკრეტულად რამ გადაგაწყვეტინა ანტ. შეცვლა

Posted by: lgogokhia 24 Dec 2009, 17:12
MAIN KAMPF
ლევოფლოქსაცინი კარგად ფარავს community თუ health care associated პნევმონიებს და პლუს ტბ-ს რეაქტივაციის პროფილაქტიკისთვის, ამიტომ ვთქვი ანამნეზიდან გამომდინარე. ცეფტრიაქსონს არა მგონია ეფექტი ქონდეს მიკობაქტერიაზე. მხოლოდ ამ მიზნით აქვს თუ არა შეცვლას აზრი და არის თუ არა მნიშვნელოვანი მიკობაქტერიების coverage იქნას გათვალისწინებული ამ შემთხვევაში ანამნეზიდან გამომდინარე?

Posted by: MAIN KAMPF 24 Dec 2009, 17:48
QUOTE
ტერფის ორმხრივი ძლიერი ტკვილებით. ავადმყოფი გაშვებული იქნა ორთოპედთან და დაესვა Gout-ის (ქართულად პოდაგრა, თუ არ ვცდები)
გარდა ტერფისა თუ ჰქონდა ჩივილები სხვა სახსრებზე იდაყვი , მაჯა და ა.შ. ფეხის დიდი თითები იყო თუ არა შესიებული დეფორმირებული ამას იმიტომ ვამბობ რომ მე რაც მინახია პოდაგრის შეტევა დაწყებულა ამ თითებიშ ძლიერი ტკივილებით და შესიებით. სხვა გამოკვლევები სისხლის საერთო ანალიზი შემოსვლის დროს რაც გაუკეთდებოდა ალბათ რა ხდებოდა? გაუკეთდა თუ არა თირკმელების ექოსკოპია ამ ავადმყოფებს ახასიათებთ ლითიაზი (ეს არაფერს შეცვლიდა მაგრამ ურიგო არ იქნებოდა)

QUOTE
მელოქსიკამი
ამას შეეძლო კალიუმის მომატება გამოეწვია.
QUOTE
ალოპურინოლს; რომელიც 4 კვირის წინ დაეწყო;
თირკმელების უკმარისობის დროს თავის შეკავება სჯობდა ეს უფრო გააღრმავებდა პროცესს, გამონაყარიც იცის,დაეწყო "კოლხიცინით". დამატებითი გამოკვლევები: ფარისებრი ჯირკვლის ჰორმონებზე+ კალციუმი.
QUOTE
შესაძლო დიფერენციალური დიაგნოზები.

თქვენ ეჭვი გეპარებათ რომ არ იყო პოდაგრა თუ? მაშინ დიფ დიაგნოზი ფსევდო პოდაგრასთან(Pseudogout)
smile.gif
QUOTE
) იმერჯენსიში ავადმყოფის მკურნალობა და იმერჯენსიდან გაშვებისას ავადმყოფი მკურნალობა რა არის?
ვერ გავიგე
smile.gif
* * *
QUOTE
ლევოფლოქსაცინი კარგად ფარავს community თუ health care associated პნევმონიებს და პლუს ტბ-ს რეაქტივაციის

ვერ დაგეთანხმები ეს პრეპარატი უფრო ალტერნატიული პრეპარატია , თუ შეცვლაა პირველ რიგში მაინც ჯობია გამოყენებული იქნეს კომბინირებული ბეტა ლაქტამაზის ინჰიბიტორი ანტიბიოტიკი მაგ ამოქსიკლავი, ანდა თუნდაც უფრო იაფი მეორე თაობის ცეფალოსპორინი ცეფუროქსიმი(ზინაცეფი)

smile.gif
* * *
QUOTE
QUOTE
ლევოფლოქსაცინი კარგად ფარავს community თუ health care associated პნევმონიებს და პლუს ტბ-ს რეაქტივაციის


ვერ დაგეთანხმები ეს პრეპარატი უფრო ალტერნატიული პრეპარატია , თუ შეცვლაა პირველ რიგში მაინც ჯობია გამოყენებული იქნეს კომბინირებული ბეტა ლაქტამაზის ინჰიბიტორი ანტიბიოტიკი მაგ ამოქსიკლავი, ანდა თუნდაც უფრო იაფი მეორე თაობის ცეფალოსპორინი ცეფუროქსიმი(ზინაცეფი)

წინა პოსტში დამიწერა შემთხვევით
biggrin.gif

Posted by: vano_t 24 Dec 2009, 21:09
lgogokhia
QUOTE
გამონაყრიდან რამდენ ხანში მოხვდა ემერჯენსიში? რამიპრილი როდის შეუწყვიტეთ, ამ ეპიზოდამდე თუ მის შემდეგ (ანგიოედემა????) ? ალოპურინოლის ჰიპერსენსიტიურობბგჰკჯჰ;კ;'ლკ - hypersensitivity syndrome? CBC-ში რა ხდებოდა? და თირკმლის ფუნქციაშიც? მე ვგულისხმობ ეპიზოდის შემდეგ..
კარგი კითხვებია. რამიპრილი შეუწყდა 3 დღით ადრე, სანამ ტუჩები გაუსივდებოდა და გამონაყარი განუვითარდებოდა. მაგრამ, მე შევეცადე მენახა case report-ები აგფ ინჰიბიტორების შეწყვეტის შემდეგ თუ შესაძლოა ანგიოედემის განვითარება, რომელიც სტატისტიკურად იძლევა კორელაციას. emedicine-ზე ვნახე რაღაც ინფო. ახლა არ მაქვს ლინკი. მერე, თუ დაგაინტერესებს, დავლინკავ. ალოპურინოლზე ალერგიაც შესაძლებელია, რა თქმა უნდა. kayexalate-ზე არ შეიძლება ალერგია? CBC ნორმალური იყო. თირკმლის ფუნქცია ER-ში არ შემიმოწმებია და შემოწმდა გუშინ (1 კვირის თავზე წინა ვიზიტიდან). კალიუმი ჩამოვიდა 4.4-ზე და კრეატინინი 1.7-ზე. მარა BUN აიწია კიდევ 70-მდე. ავადმყოფი არ ჩანს დეჰიდრატირებული გამოკვლევით. დიდი რაოდენობით სითხეებსაც იღებს დღის განმავლობაში. ერთადერთი რაზეც ვფიქრობ BUN-ის აწევის გახანგრძლივებას არის ის, რომ ER-ში ავადმყოფი დავიწყეთ სტეროიდებზე 5 დღით.

QUOTE
მე შეკითხვა მაქვს, რომელიც სხვა თემაში არ მინდა გავიტანო, რადგან ამ თემის პასუხის გამცემი ხალხი მხოლოდ აქ იკრიბება: 60 წლის ქალს აქვს შესაძლო ინფლუენზა და არის რისკის ჯგუფში (რადიოთერაპიის შემდგომი პერიოდი). 2 დღე ქონდა სუბფებრილური ტემპერატურა, რინიტი, ხველა/ყელის ტკივილი and all that jazz. დაახლოებით 10 წლის წინ ქონდა TB, რომლის შემდეგ რეაქტივაცია არ მომხდარა მიუხედავად ბევრი ხელსაყრელი მომენტისა. დღეს პოლიკლინიკის ექიმმა დაუნიშნა ცეფტრიაქსონი (პრეპარატი პეო - რა ხილია არ ვიცი ოღონდ), კურანტილი (ისე გულისთვის კაიაო biggrin.gif მოვუხსენი), მილდრონატი (უხარია და გაიკეთოს), კესტინი (ებასტინი, შველის მის ალერგიულ რინიტს კარგად) და რაღაც გაურკვეველი წარმოშობის საკვები დანამატი, რომელიც მოვუხსენი, რადგან სხვა, უკეთესი რეპუტაციის დანამატებს ღებულობს. რამდენად მიზანშეწონილად მიგაჩნიათ ცეფტრიაქსონის ჩანაცვლება ლევოფლოქსაცინით (ლევოქსიმედი) ანამნეზიდან გამომდინარე.

მაპატიეთ გაოფება, მაგრამ მჭირდება თქვენი აზრი..
თუ გადავწყვიტავ ანტიბიოტიკი მივცე, მაშინ ლევაქვინი ჯობია როცეფინს. როცეფინს არ აქვს coverage ატიპიური პნევმონიისათვის. მარა, შეიძლება არც დაუნიშნო ავადმყოფს ანტიბიოტიკი საერთოდ-გააჩნია თუ აქვს ქრონიკული რესპირატორული ან გულის სერიოზული პრობლემები. რადიოთერაპია ნამდვილად არის მაჩვნებელი იმისა, რომ ავადმყოფი შეიძლება იმუნოსუპრესირებული იყოს. მარა, რამდენი ხანი გავიდა რადიოთერაპიის შემდეგ და სხეულის რა ნაწილში ქონდა თერაპირა. საბოლოო ჯამში, ისე, ანტიბიოტიკის დანიშვნაც არ დანიშვნა ძნელი საკითხია და ბევრი ექიმი ნიშნავს თითქმის ყოველთვის. საბოლოოდ, ექიმს თუ უნდა გამართლება მოუძებნოს, ალბათ ნახავს კიდეც. მე მინიმალურად ვცდილობ ავადმყოფისათვის წამლების მიცემას და უფრო ხშირად ვარიდებ თავს თუ შესაძლებელია.

MAIN KAMPF
ტერფის გარდა სხვა ჩივილები არ ქონდა. თუმცა, ამის დიფერენციალური დიაგნოზი არ მქონია მხედველობაში. სახსრების ამბავში ავადმყოფი კონსულტანტთან იქნა გაგზავნილი, რომელმაც პრობლემას მოუარა (სიმპტომები აღარ აქვს ავადმყოფს). მე უბრალოდ მაინტერესებდა გამონაყართან და ტუჩების გასიებასთან დაკავშირებული დიაგნოზები.

კოლხიცინზე არ ყოფილა ავადმყოფი, მარტო ალოპურინოლზე იყო. კოლხიცინი (ან ინდომეტაცინი) ალბათ იმიტომაც არ დაუნიშნეს, რომ ავადმყოფს თირკმლის უკმარისობა ქონდა. თუმცა, ალბათ მელოქსიკამის არიდებაც კარგი იქნებოდა.

QUOTE
QUOTE
იმერჯენსიში ავადმყოფის მკურნალობა და იმერჯენსიდან გაშვებისას ავადმყოფი მკურნალობა რა არის?
ვერ გავიგე
სასწრაფოში რა მკურნალობა უნდა ჩაუტარდეს და რაზე უნდა იქნას ფიზიკური გამოკვლევა ორიენტირებული? და სახლში გაშვებისას რა მკურნალობა უნდა ჩაუტარდეს?


Posted by: lgogokhia 25 Dec 2009, 10:37
vano_t
ანუ გავეცი პასუხი თუ არა ვერ ვხვდები? იყო რამე სხვა რომელზეც იფიქრე დიფ დიაგნოზის თუ საბოლოო დიაგნოზის დროს? რაც შეეხება მკურნალობას, ალბათ სტეროიდები, როგორც აღნიშნე, ემერჯენსიში კი ჩვეულებრივი ალერგიული ემერჯენსების მკურნალობა?? ან რამე დამატებითი hint მოგვეცი smile.gif

QUOTE
თუ გადავწყვიტავ ანტიბიოტიკი მივცე, მაშინ ლევაქვინი ჯობია როცეფინს. როცეფინს არ აქვს coverage ატიპიური პნევმონიისათვის. მარა, შეიძლება არც დაუნიშნო ავადმყოფს ანტიბიოტიკი საერთოდ-გააჩნია თუ აქვს ქრონიკული რესპირატორული ან გულის სერიოზული პრობლემები. რადიოთერაპია ნამდვილად არის მაჩვნებელი იმისა, რომ ავადმყოფი შეიძლება იმუნოსუპრესირებული იყოს. მარა, რამდენი ხანი გავიდა რადიოთერაპიის შემდეგ და სხეულის რა ნაწილში ქონდა თერაპირა. საბოლოო ჯამში, ისე, ანტიბიოტიკის დანიშვნაც არ დანიშვნა ძნელი საკითხია და ბევრი ექიმი ნიშნავს თითქმის ყოველთვის. საბოლოოდ, ექიმს თუ უნდა გამართლება მოუძებნოს, ალბათ ნახავს კიდეც. მე მინიმალურად ვცდილობ ავადმყოფისათვის წამლების მიცემას და უფრო ხშირად ვარიდებ თავს თუ შესაძლებელია.

რესპირატორული ან გულის პრობლემები არ აქვს. რადიოთერაპია ჩაუტარდა 3 თვის წინ მცირე მენჯის არეში (?) საშვილოსნოს ტოტალური რეზექციის შემდეგ. დღეს გავიგე რომ არავითარი ტემპერატურა და სიმპტომები აღარ აქვს უკვე, ამიტომ ან გადაიტანა ან საერთოდ არაფერი არ ჰქონდა. ალერგიულია და რინიტი ხშირად ემართება, ამიტომ ანტიბიოტიკოთერაპიის საკითხს გადაწყვეტენ მდგომარეობის გაუარესების შემთხვევაში.. thnx!!

MAIN KAMPF
QUOTE
ვერ დაგეთანხმები ეს პრეპარატი უფრო ალტერნატიული პრეპარატია , თუ შეცვლაა პირველ რიგში მაინც ჯობია გამოყენებული იქნეს კომბინირებული ბეტა ლაქტამაზის ინჰიბიტორი ანტიბიოტიკი მაგ ამოქსიკლავი, ანდა თუნდაც უფრო იაფი მეორე თაობის ცეფალოსპორინი ცეფუროქსიმი(ზინაცეფი)

მე ლევოფლოქსაცინზე მიკობაქტერიების გამო გავამახვილე ყურადღება, რომელიც შენ არ გიხსენებია და არ ვიცი მიაქციე თუ არა ყურადღება. thnx smile.gif

Posted by: vano_t 25 Dec 2009, 11:38
lgogokhia
QUOTE
ანუ გავეცი პასუხი თუ არა ვერ ვხვდები? იყო რამე სხვა რომელზეც იფიქრე დიფ დიაგნოზის თუ საბოლოო დიაგნოზის დროს? რაც შეეხება მკურნალობას, ალბათ სტეროიდები, როგორც აღნიშნე, ემერჯენსიში კი ჩვეულებრივი ალერგიული ემერჯენსების მკურნალობა?? ან რამე დამატებითი hint მოგვეცი smile.gif

კი გაეცი პასუხი. ალერგიული რეაქციაა ალბათ (შეიძლება ანგიოედემაც იყოს). kayexalate ერთი ვარიანტია; მეორე ვარიანტია ალოპურინოლი; მესამე ვარიანტია სხვა რაიმე (მაგალითად, რაიმე ახალი პარფიუმერია, საკვები, საპონი, შამპუნი და დღიური მოხმარების ათასი წვრილმანი ახალი ტანსაცმლის ჩათვლით), რაც ანამნეზში არ აღენიშნება. ფიზიკური გამოკვლევისას მხედველობაში მქონდა ენის დათავლიერება (ენა ხომ არ არის შეშუპებული), ტრაქეის მოსმენა (სტრიდორი ხომ არ არის) და ფილტვების მოსმენა wheezing-ზე. აქედან დადებითი რომელიმე დადებითი ნიშანი განსაზღვრავს იმას თუ რამდენად close control არის საჭირო და საჭიროა თუ არა, რომ ავადმყოფი ინტენსიურში მოთავსდეს.

ალოპურინოლი უნდა შეუწყდეს ალბათ. თუმცა, სხვადასხვა ექიმი სხვადასხვანაირად მიუდგება ამ საკითხს.

ჰაუსის (დოქტორ ჰაუსს ვგულისხმობ smile.gif) კეისი არ არის. იმისათვის დავდე, რომ ხშირად მარტივი კეისებზე პასუხის გაცემა ფაქტიურად შეუძლებელია, როცა რამოდენიმე სავარაუდო მიზეზი გაქვს დაავადების განვითარების.

დანარჩენებს სხვები დაამატებენ ალბათ.

Posted by: lgogokhia 25 Dec 2009, 12:12
vano_t
QUOTE
kayexalate

ამ პრეპარატს ბევრი ინფორმაცია ვერ ვნახე, მაგრამ რაც ვნახე ყველაზე სერიოზული გვერდითი ეფექტად აღწერილია ნაწლავის ნეკროზის შემთხვევები, ალერგიაზე სტანდარტული ფრაზების მეტი არაფერია, ამიტომ უფრო late onset angioedema givi.gif ან უფრო ალოპურინოლის ალერგიულ რეაქციაზე უფრო ვიფიქრებდი და მოვუხსნიდი.. რეალურად რა მოხდა, მოეხსნა ალოპურინოლი?

P.S. Merry Christmas!! smile.gif

Posted by: vano_t 25 Dec 2009, 13:07
lgogokhia
QUOTE
QUOTE
kayexalate

ამ პრეპარატს ბევრი ინფორმაცია ვერ ვნახე, მაგრამ რაც ვნახე ყველაზე სერიოზული გვერდითი ეფექტად აღწერილია ნაწლავის ნეკროზის შემთხვევები, ალერგიაზე სტანდარტული ფრაზების მეტი არაფერია, ამიტომ უფრო late onset angioedema givi.gif ან უფრო ალოპურინოლის ალერგიულ რეაქციაზე უფრო ვიფიქრებდი და მოვუხსნიდი.. რეალურად რა მოხდა, მოეხსნა ალოპურინოლი?
ალერგია ყველა წამალმა შეიძლება მოგცეს (შეიძლება არც მოგცეს სინამდვილეში, მაგრამ safe assumption არის). ალოპურინოლი მოეხსნა. კეიექსალატი ისედაც ბოლო დოზა იყო-თუმცა შესაძლო ალერგია უნდა ჩაიწეროს ისტორიაში.

QUOTE
P.S. Merry Christmas!! smile.gif
დიდი მადლობა და შენც გილოცავ.

Posted by: LUKA-BRAZI 25 Dec 2009, 13:09
აუ როგორ დამაგვიანდა ამ ქეისზე! mad.gif

user.gif

Posted by: MAIN KAMPF 26 Dec 2009, 23:34
შეკითხვა ადვილია, ხანდახან გახსენება კარგია, ეს სიტუაცია შეიძლება ამბულატორიულად გასწორდეს.
რა არის სურათზე გამოსახული და როგორ მოიქცეოდით
http://www.radikal.ru

Posted by: irakli222 27 Dec 2009, 00:00
სწორი ნაწლავის გამოვარდნა?
ამბულატორულ პირობებში რა გავუკეთო არ ვიცი (სტაცინარში პროქტოლოგებმა იფიქრონ ამაზე)

Posted by: LUKA-BRAZI 27 Dec 2009, 00:15
QUOTE
სწორი ნაწლავის გამოვარდნა?

ვეთანხმები.....
QUOTE
ამბულატორულ პირობებში რა გავუკეთო არ ვიცი (სტაცინარში პროქტოლოგებმა იფიქრონ ამაზე)

ამასაც ვეთანხმები givi.gif

თუმცა ალბათ კუნთების რელაქსაცია და მასაჟი (ადგილობრივი)...... ინტერნეტში ძებნა მეზარება smile.gif)

Posted by: lgogokhia 27 Dec 2009, 00:32
MAIN KAMPF
ვეთანხმები ზემოთ გამოთქმულ აზრებს, სწორი ნაწლავის პროლაფსი.. სწორ ნაწლავს შევაბრუნებდი შესაბამის პირობებში და შემდეგში ვუმკურნალებდი ძირითად დაავადებას. ოპერაციის დასახელება მე მგონი არაა საჭირო smile.gif თუ სხვა რამეა არ ვიცი user.gif

Posted by: Thandrus 27 Dec 2009, 01:35
QUOTE
სწორი ნაწლავის პროლაფსი


კი მაგრამ, ამის ამბულატორიულად გასწორება შესაძლებელია? user.gif

ბუასილიც ხომ შეიძლება იყოს ეს? (ამბობენ, ზოგჯერ ჰგავს ბუასილი full-thickness რექტალურ პროლაფსს)

Posted by: E-36 27 Dec 2009, 05:22
QUOTE
კი მაგრამ, ამის ამბულატორიულად გასწორება შესაძლებელია? user.gif

+1
საინტერესოა
......................................

Posted by: lgogokhia 27 Dec 2009, 05:23
Thandrus
QUOTE
კი მაგრამ, ამის ამბულატორიულად გასწორება შესაძლებელია?

მაგის არა, და საერთოდ პროლაფსის ნაკლებად. ანუ იმას ვგულისხმობ რომ ალბათ შევეცდებოდი შებრუნებას თავიდან რო იქნას აცილებული შესაძლო ნეკროზი intussusception-ის გამო (თან ეტყობა მოხუცია) და მოვამზადებდი საბოლოო მკურნალობისთვის, i.e. ოპერაციისათვის თუ მდგომარეობა იძლება ამის საშუალებას. საერთოდ ამბულატორიულად და სტაციონარულად არ არის ზუსტი ტერმინი.

QUOTE
ბუასილიც ხომ შეიძლება იყოს ეს? (ამბობენ, ზოგჯერ ჰგავს ბუასილი full-thickness რექტალურ პროლაფსს)

კი, მაგრამ ეს ძალიან დიდია და რავი უფრო მგონია პროლაფსია. user.gif

მიკვირს ხოლმე ასეთ ექსტრემალურ სურათებს რო ვხედავ თუ რატომ არ მიდიან ექიმთან ადრე ეს ოჯახაყვავებულები...

Posted by: vano_t 27 Dec 2009, 09:21
Thandrus
QUOTE
QUOTE
სწორი ნაწლავის პროლაფსი


კი მაგრამ, ამის ამბულატორიულად გასწორება შესაძლებელია? user.gif

ბუასილიც ხომ შეიძლება იყოს ეს? (ამბობენ, ზოგჯერ ჰგავს ბუასილი full-thickness რექტალურ პროლაფსს)

მაგის გასწორება სახლშიც შეიძლება. სწორი ნაწლავის პროლაფსი უფრო ბავშვების პრობლემაა ვიდრე მოზარდების (თუმცა ორსული ან მშობიარე ქალის პრობლემაც შეიძლება იყოს). ბავშვებში როცა რაღაც მიზეზის გამო პროლაფსი ხშირია, მშობელსაც ასწავლიან გასწორებას. მარტივია ძალიან ამის რედუქცია. უბრალოდ ერთხელ უნდა გააკეთო და მეორედ აღარ გექნება პრობლემა ძირითადად.

ეს ბუასილი არ შეიძლება იყოს. პრინციპში ყველაფერი შესაძლებელია, მაგრამ ძალიან დიდია ბუასილისათვის, უსაშველოდ დიდი. ბუასილს კიდევ (როცა ანთება ხდება მისი) მოლურჯო ფერი დაკვრავს.

თუმცა, დიდი ზომა ასევე შეიძლება მოგცეს საშვილოსნოს პროლაფსმა. ამათი გარჩევა ადვილია. საშვილოსნოს პროლაფსი საშოდან გამოდის და სწორი ნაწლავისა-ანუსიდან. ასევე მარტივია საშვილოსნოს პროლაფსის ჩასწორება.

ორივე შემთხვევაში ხელთათმანს გაიკეთებ და რაიმე ლუბრიკანტს მოუსმევ პროლაბირებულ ორგანოს. მერე მიაწვები ფრთხილად ზევითკენ (შესაბამის ღრუსაკენ).

lgogokhia
QUOTE
მიკვირს ხოლმე ასეთ ექსტრემალურ სურათებს რო ვხედავ თუ რატომ არ მიდიან ექიმთან ადრე ეს ოჯახაყვავებულები...
ხშირად ეგეთი პრობლემა ემართებათ რაიმე ნევროლოგიური ტიპის ავადმყოფებს, რომელთაც ასევე გონებრივი პრობლემებიც აქვთ. გარდა ამისა, ზოგ შემთხვევაში, რამდენადაც გასაკვირი არ უნდა იყოს, ავადმყოფი ჯამრთელიც რომ იყოს, შეიძლება ეგრევე ვერ შეამჩნიოს პრობლემა. პროლაფსი კიდევ უცებ შეიძლება დიდი გახდეს. დაახლოებით 2 კვირის წინ ER-ში მოვიდა ავადმყოფი შარდის შეკავების ჩივილებით. ავადმყოფს ვიცნობ, კლინიკაში ხშირად ვნახულობ. ალცაიმერის დასაწყისი აქვს. ფოლის კათეტერი უნდა ჩაედგა ექთანს და უშველებელი პროლაფსი ქონდა საშვილოსნოსი, ისე რომ საერთოდ არ აწუხებდა გარდა შარდის შეკავების შეგრძნებისა. არადა, რამოდენიმე საათის უნდა ყოფილიყო ეს პროლაფსი, სულ მცირე.

Posted by: MAIN KAMPF 27 Dec 2009, 10:34
QUOTE
მაგის გასწორება სახლშიც შეიძლება. სწორი ნაწლავის პროლაფსი უფრო ბავშვების პრობლემაა ვიდრე მოზარდების (თუმცა ორსული ან მშობიარე ქალის პრობლემაც შეიძლება იყოს). ბავშვებში როცა რაღაც მიზეზის გამო პროლაფსი ხშირია, მშობელსაც ასწავლიან გასწორებას. მარტივია ძალიან ამის რედუქცია. უბრალოდ ერთხელ უნდა გააკეთო და მეორედ აღარ გექნება პრობლემა ძირითადად.
მართალი ხარ
yes.gif yes.gif
QUOTE
ორივე შემთხვევაში ხელთათმანს გაიკეთებ და რაიმე ლუბრიკანტს მოუსმევ პროლაბირებულ ორგანოს. მერე მიაწვები ფრთხილად ზევითკენ (შესაბამის ღრუსაკენ).

ამაშიც მართალი ხარ
yes.gif yes.gif
მაგრამ ამ ქეისის დადების მიზანი არა მარტო ის რომ შეგვეხსენებინა ერთმანეთისათვის ეს პათოლოგია არამედ მისი ლიკვიდაციის ხერხებიც. საქმე იმაშია, რომ ვისაც ჩაუსწორებია პროლაპსი არის სიძნელე, განსაკუთრებით თუ დიდი ზომისაა სიძნელე კიდევ იმაშია რომ ლუბრიკატის წასმის შემდეგ გამოვარდნილი ნაწილი იწყებს სრიალს და მანიპულაცია შესაძლებელია გაგრძელდეს რაც არასასიამოვნოა როგორც პაციენტისათვის ასევე ექიმისათვის(დამღლელია). ერთერთმა ჩემმა კოლეგამ მასწავლა პატარა ხერხი და შესაბამისად დავდებ სურათებს. ეს მანიპულაცია მე გამოვიყენე პაციენტზე და ძალიან კმაყოფილი დავრჩი. რაშია საქმე ამ გამოვარდნილ ნაწლავს გარედან მოვაყრით შაქრის ფხვნილს biggrin.gif გასაგებია რატომაც რომ არ ისრიალოს შემდეგ ფრთხილი მოძრაობებით ვაბრუნებთ უკან ეფექტურია gigi.gif http://www.radikal.ruhttp://www.radikal.ru

Posted by: vano_t 27 Dec 2009, 12:30
QUOTE (MAIN KAMPF @ 27 Dec 2009, 10:34 )
ეს მანიპულაცია მე გამოვიყენე პაციენტზე და ძალიან კმაყოფილი დავრჩი. რაშია საქმე ამ გამოვარდნილ ნაწლავს გარედან მოვაყრით შაქრის ფხვნილს biggrin.gif გასაგებია რატომაც რომ არ ისრიალოს შემდეგ ფრთხილი მოძრაობებით ვაბრუნებთ უკან ეფექტურია

yes.gif თანაც ყველანაირი შაქარი არ წავა, მხოლოდ გრანულირებული (ძალიან წვრილად გაცრილი რომ არის, არ ივარგებს). შენ რაც თქვი, იმასთან ერთად გრანულირებული შაქარი სითხეს იწოვს და ზომებს ამცირებს შეშუპებული ნაწლავის.

Posted by: MAIN KAMPF 27 Dec 2009, 12:42
ახალი ქეისი. ნახეთ სურათები და აზრები smile.gif
URL=http://www.radikal.ru]user posted image[/URL]
http://www.radikal.ru
http://www.radikal.ru

Posted by: Blind_Torture_Kill 27 Dec 2009, 13:00
MAIN KAMPF

ცოტა ისტორიაც დაამატე ხოლმე
ანასარკას გავს
ეს ექიმოზები რატომ აქვს ინფოს მერე გავარკვევთ

Posted by: MAIN KAMPF 27 Dec 2009, 13:03
ავადმყოფი შიდსიანია, მისი ერთ ერთი გართულებაა baby.gif

Posted by: Blind_Torture_Kill 27 Dec 2009, 13:05
MAIN KAMPF

კაპოში სარკომა

Posted by: lgogokhia 27 Dec 2009, 13:05
MAIN KAMPF
კაპოშის სარკომა?

Posted by: MAIN KAMPF 27 Dec 2009, 13:10
QUOTE
კაპოშის სარკომა?

რა თქმა უნდა
bis.gif biggrin.gif

Posted by: utilizatori 30 Dec 2009, 00:57
აქ ვიქტორინა გაქვთ ? smile.gif
------

Posted by: Thandrus 30 Dec 2009, 04:43
utilizatori

ჩვენგან ერთ-ერთი კლინიკურ შემთხვევას გვთავაზობს და მერე ვხნით / ვიხილავთ smile.gif

Posted by: E-36 31 Dec 2009, 17:50
Коллеги, с Новым годом! Радости, успехов, здоровья, мира! Будьте счастливы!!!

Posted by: MAIN KAMPF 31 Dec 2009, 18:34
გილოცავთ ახალ წელს გისურვებთ ბედნიერებას, სიხარულს იმედებიბს გამართლებას.http://elar.ge/

Posted by: Thandrus 7 Jan 2010, 04:25
გილოცავთ შობას!

აბა, ამ ქეისზე რას იტყვით:

22 წლის მამაკაცი, უმნიშვნელო სამედიცინო ისტორიით, აეროპორტიდან სასწრაფოდ გადაყვანილია Emergency Department-ში ნახევრადფხიზელ მდგომარეობაში. პაციენტი მალარიულ ზონაში სამკვირიანი მოგზაურობის შემდეგ უკან ბრუნდებოდა. მისი მეგობრის თქმით, თვითმფრინავში ასვლისას მას ნორმალური მენტალური სტატუსი ჰქონდა. მგზავრობისას, მან ისადილა და წაუძინა. თვითმფრინავი რომ დაჯდა, მეგობარმა პაციენტი ვეღარ გააღვიძა. მეგობარმა დახმარება ითხოვა და პაციენტი ED-ში გააქანეს. თვითმფრინავის იმ რეისზე მყოფ სხვა მგზავრებს არანაირი სიმპტომი არ აქვთ.

ფიზიკური შემოწმების შედეგები - ათლეტური აღნაგობის ახალგაზრდა კაცი, რომელიც გონებადაბინდული და მინიმალურად გამოფხიზლებადია (arousable). სასიცოცხლო ნიშნები: ტემპერატურა - 98.7ºF (37.0ºC), პულსი - 85 bpm, წნევა - 110/70, სუნთქვის სიხშირე - 7 br/min, ჟანგბადის სატურაცია - 98% ოთახის ჰაერზე. შეინიშნება კანის დიფუზური flushing-ი, მაგრამ სხვა რაიმე დაზიანების გარეშე. გულის აუსკულტაცია ნორმალურია. ფილტვები სუფთა ორივე მხარეს. თავისა და კისრის შემოწმების შედეგი უმნიშვნელოა. მუცლის გამოკვლევით მოისმინება ნორმალური ნაწლავური ხმები. გადაჭიმვა, პალპაციისადმი მგრძნობიარობა ან ორგანომეგალია არ აღინიშნება. რექტალური გამოკვლევა აჩვენებს ნორმალურ სპინქტერულ ტონუსს, ჰემ-ნეგატიურ განავალს და მასების არარსებობას.

ლაბორატორიული ანალიზები - CBC - ნორმალური WBC რაოდენობა მარცხნივ გადახრის გარეშე. დანარჩენი გამოკვლევებიც ნომაშია: კრეატინინი 1.2 mg/dL (106.1 μmol/L), გლუკოზ 90 mg/dL (5.0 mmol/L), PT - 12.1 წამი, PTT - 28.5 წამი. შარდის specific gravity არის 1.010 და ბაქტერიებისადმი ნეგატიურია. პაციენტმა 4 თვის წინ HIV-ს ტესტი გაიკეთა, რომელიც ნეგატიური აღმოჩნდა.

ამის გარდა გადაიღეს რენტგენი (სურათი მოცემულია):

რა დიაგნოზზე იფიქრებთ?

Posted by: mtvareuli 7 Jan 2010, 04:33
Thandrus

ეს ოვალური რაღაცეები რა არის spy.gif

Posted by: Thandrus 7 Jan 2010, 04:40
mtvareuli

QUOTE
ეს ოვალური რაღაცეები რა არის


ოოო, ეგაა მთავარი gigi.gif


OFFTOPIC:
შენც ეკლესიაში იყავი თუ ინსომნია? user.gif


Posted by: irakli222 7 Jan 2010, 11:20
ნარკოტიკები გადმოჰქონდა მაგ ახალგაზრდას.
არსებობს ასეთი მეთოდი.
ერთერთი პაკეტი გაიხსნა და ნარკოტიკებით მწვავე მოწამვლის სურათი განვითარდა.

Posted by: LUKA-BRAZI 7 Jan 2010, 13:15
QUOTE
ნარკოტიკები გადმოჰქონდა მაგ ახალგაზრდას. არსებობს ასეთი მეთოდი.

ვეთანხმები, თან ლოკალიზაციის მიცხედვით როგორც ჩანს ორ ულუფად აქვს გადაყლაპული პაკეტები biggrin.gif

Posted by: Thandrus 7 Jan 2010, 15:50
irakli222
LUKA-BRAZI

საღოლ! up.gif

ახლა, რომელი ნარკოტიკია, ისიც თქვით ბარემ... (სიმპტომებიდან გამომდინარე)

Posted by: Cousteau 7 Jan 2010, 17:35
QUOTE (Thandrus @ 7 Jan 2010, 15:50 )

ახლა, რომელი ნარკოტიკია, ისიც თქვით ბარემ... (სიმპტომებიდან გამომდინარე)

ოპიოიდია რომელიმე? spy.gif

user posted image

user posted image

Posted by: mtvareuli 7 Jan 2010, 17:37
Thandrus
QUOTE
ახლა, რომელი ნარკოტიკია, ისიც თქვით ბარემ...

ოპიოიდების ჯგუფის

Posted by: LUKA-BRAZI 7 Jan 2010, 18:53
Cousteau
რეტინობლასტომა? user.gif

Posted by: MAIN KAMPF 7 Jan 2010, 22:41
მაგას კონტეინერები ჰქვია რაც კუჭში ჩანს. რამოდენიმე წლის წინ ვმუშაობდი მოსკოვში, ახლოს იყო შერემეტევოს აეროპორტი და ხშირად მოყავდათ მანდედან ხალხი განსაკუთრებით ნიგერიელები, უკეთდებოდათ რენტგენი. ერთხელ ოპერაციაც გავუკეთეთ და ამოვიღეთ 10 ცალი კინდერის კვერცხი რომ არის სათამაშო დევს პლასმასის ბურთია საკმაოდ დიდია
biggrin.gif როგორ ყლაპავენ მიკვირს. ყოველთვის იყო ოპიუმი, სხვა ნარკოტიკი მაშინ არ გვინახია. სიმპტომებიდან რაც დასხელებულია
flushing ახასიათებს. ისე მე როგორც ვიცი ოპიოიდებს გუგების შევიწროებითაც არჩევენ, კოკაინს პირიქით გაფართოება ახსიათებთ.

კარგი ქეისია
bis.gif

Posted by: Thandrus 7 Jan 2010, 23:26
Cousteau
mtvareuli
MAIN KAMPF

მართლები ხართ სამივენი, მას ჰეროინი გადაჰქონდა. მკურნალობა დაიწყო ნალოქსონის ინფუზიით (დიდი ხნის განმავლობაში) და შემდეგ ოპერაციით, რომლის შედეგადაც ეს პაკეტები ამოიღეს.

Posted by: Cousteau 8 Jan 2010, 00:18
QUOTE (Thandrus @ 7 Jan 2010, 23:26 )
რომლის შედეგადაც ეს პაკეტები ამოიღეს.

ამოიღეს და სხვანაირ, უფრო პატარა პაკეტებში გადააფასოეს : D

LUKA-BRAZI
დიახ, დიახ, რეტინობლასტომაა, აბა ის თქვი უფრო faimlial არის თუ sporadic?

Posted by: LUKA-BRAZI 8 Jan 2010, 00:21
Cousteau
ეგ აღარ ვიცი smile.gif თუ ვერავინ გამოიცნო მაშინ მოკლედ აგვიხსენი რა რას ნიშნავს smile.gif

Posted by: Thandrus 8 Jan 2010, 00:49
Cousteau

ჰმ... მხოლოდ ამ სურათით შეიძლება გამოვლენა თუ რომელი ტიპი აქვს? user.gif

დაახლოებით ნახევარი ნახევარზეა, როგორც ვიცი, faimlial vs. sporadic...

მოკლედ, ოჯახში თუ ვინმეს ჰქონდა...


Posted by: LUKA-BRAZI 8 Jan 2010, 00:53
Cousteau
Approximately 6% of newly diagnosed retinoblastoma cases are familial and 94% are sporadic.

http://www.medscape.com/viewarticle/491384_2

Posted by: Cousteau 8 Jan 2010, 00:59
QUOTE
ჰმ... მხოლოდ ამ სურათით შეიძლება გამოვლენა თუ რომელი ტიპი აქვს?
მგონი არა : D


QUOTE (LUKA-BRAZI @ 8 Jan 2010, 00:53 )
Cousteau
Approximately 6% of newly diagnosed retinoblastoma cases are familial and 94% are sporadic.

http://www.medscape.com/viewarticle/491384_2

jump.gif jump.gif jump.gif


Posted by: Thandrus 8 Jan 2010, 01:28
LUKA-BRAZI

QUOTE
Approximately 6% of newly diagnosed retinoblastoma cases are familial and 94% are sporadic.


ხო... არასწორად მახსოვდა ესეიგი biggrin.gif

ნექსთ, ფლიზ!

Posted by: LUKA-BRAZI 8 Jan 2010, 01:51
Thandrus
It's your turn nigga gigi.gif

Posted by: anarxisti 9 Jan 2010, 22:45
ტელევიზორში შევესწარი გადაცემას და ამან მიბიძგა ეს პათოლოგია დამედო..
მოკლედ საქმე ეხებოდა თურქეთს.. ცნობილი ფირმები იქ ახდენენ ჯინსების დამუშავებას იმისათვის რომ ეს ჯინსები უფრო მოდური იყოს;
ხოდა ის ხალხი ვინც ამაზე მუშაობდა ძალიან მძიმედ ხდება ავად..
იმდენად მძიმედ, რომ ეს მეთოდი ჯინსების დამუშავების, აიკრძალა, ახლახან.
დაავადება ეხება ფილტვებს.
რა პათოლოგიასთან გვაქვს საქმე?

Posted by: donvaso 9 Jan 2010, 22:52
კირს/ცემენტს ხმარობდნენ ჯინსების წარმოებაში? biggrin.gif biggrin.gif

Posted by: E-36 9 Jan 2010, 23:17
anarxisti
გახანგრძლივებული ფილტვების ანთება ?

ჯინსების დამუშავება ხდებოდა საღებავებით ?
და საღებავი შეიცავდა ტყვიას ?

კლინიკა არაფერი გვაქვს ? punch.gif

ტყვიით ქრონიკული ინტოქსიკაცია cry.gif

Posted by: Thandrus 9 Jan 2010, 23:24
anarxisti

სილიკოზთან smile.gif

Posted by: E-36 9 Jan 2010, 23:31
QUOTE (Thandrus @ 9 Jan 2010, 23:24 )
anarxisti

სილიკოზთან smile.gif

და მტვრის შესუნთქვა საიდან ხდებოდა? spy.gif
ქვებზე ხეხავდნენ ჯინსებს რომ უფრო მოდური ყოფილიყო?

Posted by: Thandrus 10 Jan 2010, 00:03
E-36

ალბათ Sandblatsing-ის მეთოდით ამუშავებდნენ ჯინსებს. ამ მეთოდში კიდევ სილიკა გამოიყენება უხშირესად. სილიკოზიც იმიტომ ვივარაუდე biggrin.gif

http://en.wikipedia.org/wiki/Sandblasting

Posted by: E-36 10 Jan 2010, 00:13
QUOTE (Thandrus @ 10 Jan 2010, 00:03 )
E-36

ალბათ Sandblatsing-ის მეთოდით ამუშავებდნენ ჯინსებს. ამ მეთოდში კიდევ სილიკა გამოიყენება უხშირესად. სილიკოზიც იმიტომ ვივარაუდე biggrin.gif

http://en.wikipedia.org/wiki/Sandblasting

აჰააა გასაგებიაა up.gif alk.gif


მარა მაინც საინტერესოა რა ხდებოდა დაწესებულებაში სადაც ცნობილი ფირმები ჯინსებს ამუშავებდნენ punch.gif

Posted by: donvaso 10 Jan 2010, 00:13
სილიკოზი ეგეთ გაკირვას იძლევა?????????

Posted by: anarxisti 10 Jan 2010, 00:16
QUOTE (Thandrus @ 9 Jan 2010, 23:24 )
anarxisti

სილიკოზთან smile.gif

საღოლ up.gif(თურქულია -ო)
ეგაა.
ისე ცოტა მოგეცადა შეი კაი კაცო! biggrin.gif
ადრე, ევროპაში მეშახტეებში ყოფილა გავრცელებული..

მაღალი წნევით ხდება ჯინსებზე სილის(ქვიშის) "მისხურება" (ვერ ვპოულობ ტერმინს, სორრი).."გასახეხად", ხოდა ის ასხლეტილი ნაწილაკები ფილტვებში ილექება...
კლინიკაა სუნთქვის უკმარისობა.. ქრონიკული.
მკურნალობა არ არსებობსო! (არ ვიცი)
ფილტვის გადანერგვაო, მხოლოდ.

ჯინსის ჩაცმა აღარ მომინდა როცა ვნახე ის საწ###ბი.

Posted by: E-36 10 Jan 2010, 00:21
anarxisti
საინტერესო ქეისი იყო up.gif

Thandrus up.gif up.gif up.gif




NEXT fig.gif

Posted by: mtvareuli 10 Jan 2010, 15:40
QUOTE
რა  არის  ეს?                            

Posted by: LUKA-BRAZI 10 Jan 2010, 18:38
mtvareuli
მარიკა, სანამ სურათს ატვირთავ ფორუმზე, მანამდე სახელი შეუცვალე თორემ "save image as"-ს რომ დააკლიკებ, სახელს გიჩვენებს ავტომატურად smile.gif

ეგ არის ჰერპეს სიმპლექსით გამოწვეული კერატიტი biggrin.gif

Posted by: mtvareuli 10 Jan 2010, 18:53
LUKA-BRAZI

ეუფ biggrin.gif


კარგი, მაშინ მითხარით რატომ არის ჰერპესვირუსული კერატიტი, სურათის სახელის გარდა რითი მიხვდებოდით?


Posted by: Cousteau 10 Jan 2010, 20:58
QUOTE (mtvareuli @ 10 Jan 2010, 18:53 )
LUKA-BRAZI

ეუფ biggrin.gif


კარგი, მაშინ მითხარით რატომ არის ჰერპესვირუსული კერატიტი, სურათის სახელის გარდა რითი მიხვდებოდით?

დენდრიტია პატარა (მგონი)

ესე ჯობია user posted image

Posted by: mtvareuli 11 Jan 2010, 00:58
Cousteau
QUOTE
დენდრიტია პატარა (მგონი)

მგონი არა, ნამდვილად დენდრიტია yes.gif მაგრამ არ ისე პატარა


Posted by: Cousteau 11 Jan 2010, 01:01
QUOTE (mtvareuli @ 11 Jan 2010, 00:58 )
Cousteau
QUOTE
დენდრიტია პატარა (მგონი)

მგონი არა, ნამდვილად დენდრიტია yes.gif მაგრამ არ ისე პატარა

thats what i call არც ისე პატარა დენდრიტი

user posted image

: )

Posted by: Cousteau 19 Jan 2010, 20:21
46 წლის, ალბერტ ფორუმიანი, მოდის თქვენთან შემდეგი ჩივილებით:
ხელი ''მწიწკნის'', მარჯვენა ხელი მტკივა, მარჯვენა ხელი მაქ დაბუჟებული, ავდილად ვიღლები, ბოლო 3 თვის განმავლობაში წონა დავიკელი (არადა როგორ უნდა დამეკლო, სულ ვჭამ და ვჭამ), და სუსტადვარ ძალიან, ექიმომიშველერამე

ობიექტურად: მარჯვენა პტოზი, მიოზი, მარჯვენა ფილტვის აპიკალურ ნაწილში სუნთქვა ცუდად ისმინება, იმის გამო რო პაციენტი მწეველია (და ეწევა 16 წლიდან) შეგეშინდათ ''მაინც სიმსივნე არ იყოსო'', გადაუღეთ რენტგენი, რენტგენზე არის ეს

user posted image

რა ჭირს ბატონ ალბერტს? spy.gif

Posted by: LUKA-BRAZI 19 Jan 2010, 21:16
Cousteau
ტუბერკულოზი? მაგრამ ტუბერკულოზი ადვილი პასუხია, შენ მასეთ ქეისს არ დადებდი, "ზნაჩიტ" არ არის ტუბერკულოზი smile.gif

პ.ს.
ბიჯო ძალიან dense არის ეს წიგნი........

დარწმუნებული არ ვარ რომ Nerst Equation-ის სწავლა ჩვენს მიზანს ძალიან მიგვაახლოვებს user.gif

smile.gif
* * *
პ.ს.ს

ეგ რაღაც დაჩრდილვა მარჯვენა სუბკლავიას და არტერიებს აწვება (ხელის "წიწკვნა", დაბუჟება) + აწვება ნერვებს (ცდომილი?) -> მიოზი, პტოზი...... ჰა? spy.gif
* * *
მგონი სისულელე დავწერე ჰო?... biggrin.gif

Posted by: donvaso 19 Jan 2010, 21:36
გვერდიითი სურათი არ გვაქვს ან ჭრილი??
ხოდა, მარჯვენა ზემო წილის სიმსივნეა ალბათ....

Posted by: Thandrus 19 Jan 2010, 21:51
Cousteau

Pancoast-ის სიმსივნეა...

Posted by: Cousteau 19 Jan 2010, 22:26
QUOTE (Thandrus @ 19 Jan 2010, 21:51 )
Cousteau

Pancoast-ის სიმსივნეა...

უმმმმ up.gif
იეს მაი ფელოვ ჯორjიანს დას ისტ Pancoast's Tumor

and the name of the syndrome is...

სინდრომს რა ქვია?


OFFTOPIC:
Horny Pamela



PS
QUOTE
Nerst Equation


Fuck Ners Equation
თუმცა მე ვიცი ერთი ჩემზე ჭკვიანი და მაგას ვკითხოთ ხო არ მოდის მსგავსი რაღაცეები


Posted by: Thandrus 19 Jan 2010, 22:54
Cousteau

QUOTE
and the name of the syndrome is...

სინდრომს რა ქვია?


ცოტათი ვერ მივხვდი, რას გულისხმობ.

პანკოსტის სიმსივნით გამოწვეულ სინდრომს პანკოსტის სინდრომი ეწოდება.

QUOTE
მარჯვენა პტოზი, მიოზი


ეს მარჯვენამხრივი ჰორნერის სინდრომია biggrin.gif (+ ალბათ ანჰიდროზი)

ისე, პანკოსტის სიმსივნე ბრახიალური პლექსუსის ქვედა trunk-ს აზიანებს, რასაც Klumpke's Palsy ჰქვია...

ხოდა რავიცი, კიდევ რამეა სხვა? user.gif

Posted by: Cousteau 19 Jan 2010, 23:09
QUOTE (Thandrus @ 19 Jan 2010, 22:54 )

ეს მარჯვენამხრივი ჰორნერის სინდრომია biggrin.gif (+ ალბათ ანჰიდროზი)

yes.gif yes.gif yes.gif
მაგას ვკითხულობდი : )

Posted by: LUKA-BRAZI 20 Jan 2010, 00:01
Cousteau
QUOTE
თუმცა მე ვიცი ერთი ჩემზე ჭკვიანი და მაგას ვკითხოთ ხო არ მოდის მსგავსი რაღაცეები

იასნად, არ მოდის biggrin.gif it's not a chemistry exam.... only clinically oriented dude, only clinically... wink.gif as far as I'm concerned..... no? spy.gif
* * *
Thandrus
როგორც ყოველთვის მოწოდების სიმაღლეზე ხარ biggrin.gif

Posted by: Thandrus 20 Jan 2010, 00:07
LUKA-BRAZI

QUOTE
იასნად, არ მოდისit's not a chemistry exam.... only clinically oriented dude, only clinically...as far as I'm concerned..... no?


USMLE-ზე თუ ამბობ, მე რაც შთაბეჭდილება დამრჩა, ვერ თითქმის ვერ საერთოდ ვერ იტყვი რა მოგივა. მაგალითად, ვისიც ვიცი, ზოგი ამბობს, ბიოქიმია თითქმის არ მოდისო, ზოგი კი - ძალიან ბევრი კითხვაა ბიოქიმიიდანო... ასე რომ, გამართლებაზეა რა gigi.gif ამიტომ, ყველაფრისათვის უნდა იყო მზად biggrin.gif

ახლა, ნერსტის განტოლება სუფთა სახით შეიძლება არ მოგივიდეს, მაგრამ ბიოქიმიის და ფიზიოლოგიის შესწავლაში ძალიან გამოგადგება, ასე რომ სჯობს ზედმეტი ნახევარი საათი დახარჯო და მერე გაგიადვილდეს smile.gif

QUOTE
როგორც ყოველთვის მოწოდების სიმაღლეზე ხარ


tongue.gif

Posted by: LUKA-BRAZI 20 Jan 2010, 00:15
Thandrus
შენ ჩააბარე სტეპ 1?! როდის, როგორ, სად, რამდენ ქულაზე?! ბიჯო არ უნდა გვითხრა? ერთი ოჯახივით ვართ smile.gif)

QUOTE
მაგალითად, ვისიც ვიცი, ზოგი ამბობს, ბიოქიმია თითქმის არ მოდისო, ზოგი კი - ძალიან ბევრი კითხვაა ბიოქიმიიდანო...

აი ჰო yes.gif ბევრი კითხვა იგივე პათოლოგიიდან ძალიან გავს ბიქიმიურს და პირიქით, მაგიტომაც ძნელია თქვა რა საიდან მოდის. მაგრამ ნერსტის ფორმულა რომ შემხვდეს გამოცდაზე, სკანდალს მოვაწყობ biggrin.gif საკუთარ თავთან რა თქმა უნდა biggrin.gif ლოგარითმები და etc. მე მარტო ის დავიმახსოვრე რომ მაგ ფორმულას იონების კონცენტრაცია გადაყავს მილივოლტებში.

Posted by: Thandrus 20 Jan 2010, 00:46
LUKA-BRAZI

QUOTE
შენ ჩააბარე სტეპ 1?! როდის, როგორ, სად, რამდენ ქულაზე?! ბიჯო არ უნდა გვითხრა? ერთი ოჯახივით ვართ )


არა კაცო, ჯერ რა ჩავაბარე... ხანგრძლივი მზადების პროცესში ვარ ჯერ biggrin.gif ნელ-ნელა მივიწევ მაგ მიზნისაკენ smile.gif

Posted by: LUKA-BRAZI 20 Jan 2010, 01:19
Thandrus
QUOTE
ნელ-ნელა მივიწევ მაგ მიზნისაკენ

To infinity and beyond! smile.gif)

Posted by: lgogokhia 20 Jan 2010, 10:59
QUOTE
ხელი ''მწიწკნის'', მარჯვენა ხელი მტკივა, მარჯვენა ხელი მაქ დაბუჟებული, ავდილად ვიღლები, ბოლო 3 თვის განმავლობაში წონა დავიკელი (არადა როგორ უნდა დამეკლო, სულ ვჭამ და ვჭამ), და სუსტადვარ ძალიან, ექიმომიშველერამე. ობიექტურად: მარჯვენა პტოზი, მიოზი, მარჯვენა ფილტვის აპიკალურ ნაწილში სუნთქვა ცუდად ისმინება, იმის გამო რო პაციენტი მწეველია (და ეწევა 16 წლიდან) შეგეშინდათ ''მაინც სიმსივნე არ იყოსო'', გადაუღეთ რენტგენი, რენტგენზე არის ეს


რათქმაუნდა smile.gif
QUOTE
Pancoast-ის სიმსივნეა...

QUOTE
ჰორნერის სინდრომია


რატომ უფრო ხშირად არ დებთ რამე სადისკუსიო შეკითხვებს? აუცილებელი არ არის რომ რამე ქეისი იყოს.. ყველა მეცადინეობთ UშMLE-სთვის და დარწმუნებული ვარ სადღაც რამე გექნებათ (პლუს რისფექთ ყველას და მე გაძლევთ გარანტიას რომ UშMLE-ს ჩააბარებთ )... best of luck ყველას!!!

Posted by: LUKA-BRAZI 20 Jan 2010, 14:32
lgogokhia
რამდენ მოლეკულა ATP-ს ხარჯავს Na/K ATPase ერთ ციკლზე? biggrin.gif

Posted by: Thandrus 20 Jan 2010, 16:04
LUKA-BRAZI

QUOTE
რამდენ მოლეკულა ATP-ს ხარჯავს Na/K ATPase ერთ ციკლზე?


ეუF... biggrin.gif

ერთ ცალს მგონი. tongue.gif

Posted by: LUKA-BRAZI 20 Jan 2010, 16:25
Thandrus
QUOTE
ერთ ცალს მგონი. tongue.gif

არა! ძლივს რაღაც არ "გამაზე" კაცო? lol.gif სულ გამოცნობა შეიძლება? biggrin.gif biggrin.gif

Posted by: Thandrus 20 Jan 2010, 17:02
LUKA-BRAZI

QUOTE
არა! ძლივს რაღაც არ "გამაზე" კაცო?სულ გამოცნობა შეიძლება?


ჰმ... მაშ რამდენი? biggrin.gif

ციტატა დევლინიდან: "Movement of Na and K is an electrogenic antiport process, with three Na ions moving out and two K ions into tthe sell for each ATP molecule hydrolyzed.

თუ სხვანაირად გავიგე, ვერ მივხვდი, რომელ ციკლზე მელაპარაკები? user.gif

Posted by: LUKA-BRAZI 20 Jan 2010, 17:14
Thandrus
ციტატა კონსტანცოდან:

The transport cycle is illustrated in Figure 1-6. The cycle begins with the enzyme in the E1 state, bound to ATP. In the E1 state, the ion-binding sites face the intracellular fluid, and the enzyme has a high affinity for Na+; three Na+ ions bind, ATP is hydrolyzed, and the terminal phosphate of ATP is transferred to the enzyme, producing a high-energy state, E1∼P. Now, a major conformational change occurs, and the enzyme switches from E1∼P to E2∼P. In the E2 state, the ion-binding sites face the extracellular fluid, the affinity for Na+ is low, and the affinity for K+ is high. The three Na+ ions are released from the enzyme to extracellular fluid, two K+ ions are bound, and inorganic phosphate is released from E2. The enzyme now binds intracellular ATP, and another major conformational change occurs that returns the enzyme to the E1 state; the two K+ ions are released to intracellular fluid, and the enzyme is ready for another cycle.

ATP-ები მე გავამუქე. პირდაპირ არ წერია, მაგრამ ტექსტიდან მასე გამოდის რომ E1-E2-ზე იხარჯება ერთი ატფ, ხოლო E2-E1-ზე იხარჯება კიდევ ერთი ატპ, მაგრამ სურათზე მასე არ არის. მგონი მართლა 1 ატფ იხარჯება და მეორე ატფ-ში P ფოსფატური ჯგუფი იგულისხმება? მაგრამ ტექსტიდან სულ სხვა აზრი გამოდის და......

ესეც Figure 1-6:

Posted by: Thandrus 20 Jan 2010, 17:44
LUKA-BRAZI

QUOTE
ATP-ები მე გავამუქე. პირდაპირ არ წერია, მაგრამ ტექსტიდან მასე გამოდის რომ E1-E2-ზე იხარჯება ერთი ატფ, ხოლო E2-E1-ზე იხარჯება კიდევ ერთი ატპ, მაგრამ სურათზე მასე არ არის. მგონი მართლა 1 ატფ იხარჯება და მეორე ატფ-ში P ფოსფატური ჯგუფი იგულისხმება? მაგრამ ტექსტიდან სულ სხვა აზრი გამოდის და......


მგონი არ გამოიტანე სწორი დასკვნა. ციკლი თავიდან რომ დავიწყოთ: არის E1-ATP-ს კომპლექსი. ეს კომპლექსი რომ Na-ს სამ იონს მიიერთებს (და Mg-ს იონებსაც, რომელიც აუცილებელი კოფაქტორია ამ ტუმბოსი) მაშინ ამ ATP-ს ჰიდროლიზი მოხდება ADP-დ და ფოსფატად. და ეს ფოსფატი მიუერთდება ამ ტუმბოს ალფა subunit-ს. ნატრიუმის იონების უჯრედს გარეთ გამოდევნის პროცესში ტუმბოს კომფორმაცია E1-დან E2-დან იცვლება, რის დროსაც კალიუმის იონებისადმი აფინურობა იმატებს და E2-ფოსფატის კომპლექსი იერთებს კალიუმის ორ იონს. ეს რომ მოხდება, E2-დან ფოსფატის ჰიდროლიზი ხდება. შემდეგ, კალიუმის იონები უჯრედში შევლენ (ჯერ კიდევ ენზიმზე ბმულები), ტუმბოს ATP მიუმაგდება, კალიუმის იონები ტუმბოს ჩამოცილდება და ისევ საწყის ATP-E1-ის კომფორმაციას ვიღებთ,

ჯამში, მოხდა 1 ATP-ს ჰიდროლიზი ADP-დ და ფოსფატად, სამი Na იონი გამოვიდა გარეთ და ორი Ka იონი შევიდა უჯრედში smile.gif

Posted by: LUKA-BRAZI 20 Jan 2010, 21:29
Thandrus
ეგ მექანიზმის სავსებით გასაგებია, მაგრამ მაშინ კონსტანცოს რატომ უწერია ასე?
QUOTE
The three Na+ ions  are released from the enzyme to extracellular fluid, two K+ ions are bound, and inorganic phosphate is released from E2. The enzyme now binds intracellular ATP, and another major conformational change occurs

თუ 1 ატფ ჰიდროლიზდება (რაც ალბათ ასეცაა), მაშინ რატომ უწერია ასე გამოკვეთილად რომ E2->E1 ფაზის დროს უკავშირდება კიდევ ატფ-ს-ო? თან ეგ უწერია ფოსფატური ჯგუფის ჩამოცილების შემდეგ. მეც ვფიქრობ რომ 1 ატფ იხარჯება, მაგრამ რატომღაც 100%-ით დარწმუნებული არ ვარ, სწორედ მასეთი ორაზროვნად ნათქვამის გამო......

Posted by: ab-ba 20 Jan 2010, 21:41
ეს რა ქეისებია? თქვენს ცოდნას ამზეურებთ ბიოქიმიაში.
რა, საინტერესო შემთხვევები შემოგაკლდათ?

Posted by: LUKA-BRAZI 20 Jan 2010, 21:44
ab-ba
და თუ რაღაც გაურკვეველ საკითხს განვიხილავთ ერთმანეთში, რა, პრობლემაა?

QUOTE
თქვენს ცოდნას ამზეურებთ ბიოქიმიაში.

?


Posted by: Thandrus 20 Jan 2010, 22:25
LUKA-BRAZI

QUOTE
მაშინ რატომ უწერია ასე გამოკვეთილად რომ E2->E1 ფაზის დროს უკავშირდება კიდევ ატფ-ს-ო? თან ეგ უწერია ფოსფატური ჯგუფის ჩამოცილების შემდეგ.


ხო, მერე, რა გინდა, კალიუმის შესვლასთან ერთად ფოსფატი მოსძვრება, ATP დაუკავშირდება და E2 გადავა E1-ში. აქ მთავრდება ციკლი. შემდეგ ახალი ციკლი იწყება ნატრიუმის იონები, ATP-ს ჰიდროლიზი და ა.შ.

Posted by: LUKA-BRAZI 20 Jan 2010, 23:01
Thandrus
ჰო სანდრო ეგ კი, ეჭვიც აღარ მეპარება რომ 1 ატფ მაგრამ მაინც რატომ წერია ასე? biggrin.gif
QUOTE
The enzyme now binds intracellular ATP, and another major conformational change occurs that returns the enzyme to the E1 state; the two K+ ions are released to intracellular fluid, and the enzyme is ready for another cycle.

აი ეს რომ ვთარგმნოთ სიტყვასიტყვით გამოვა რომ: ენზიმი უკავშირდება ინტრაცელულურ ატფ-ს, ხდება კონფორმაციული გარდაქმნა და ენზიმი ბრუნდება E1 პოზიციაში, გამოათავისუფლებს K-ის იონებს და მზად არის მომდევნო ცილკისთვის.

ე.ი. E1-ში დასაბრუნებლად ატფ უნდაო ასე არ გამოდის? არადა ერთი ატფ უკვე დაიხარჯა E1-დან E2-ში გადასვლის დროს.

მგონი ვერ ვარ სრულ ჭკუაზე biggrin.gif კონსტანცოს ეგ ციტატა რომ წაიკითხე, არ დაგრჩა ისეთი შთაბეჭდილება თითქოს 2-ს ხარჯავდეს ტუმბო? smile.gif

Posted by: Thandrus 20 Jan 2010, 23:12
LUKA-BRAZI

QUOTE
ე.ი. E1-ში დასაბრუნებლად ატფ უნდაო ასე არ გამოდის? არადა ერთი ატფ უკვე დაიხარჯა E1-დან E2-ში გადასვლის დროს.


არა, აქ უნდა ATP-ს დაკავშირება და არა ჰიდროლიზი. შემდეგ, მეორე ციკლი რომ დაიწყება, ამ მიბმული ATP-ის ჰიდროლიზი მოხდება. შენ binding და hydrolysis გეშლება ერთმანეთში და იმიტომ დაიბენი smile.gif

Posted by: LUKA-BRAZI 20 Jan 2010, 23:45
Thandrus
ჰმმ, ჰო ალბათ ეგაა smile.gif ვერ წარმომედგინა binding ჰიდროლიზის გარეშე რატომღაც smile.gif ეხლა ეკონომიკური კრიზისია თორემ ადრე 5 ატფ-ზე მუშაობდნენ ნატრი-კალის ტუმბოები biggrin.gif კაპიკები ღირდა ატფ. რას გვიშვება ეს მთავრობა! biggrin.gif

Posted by: Thandrus 21 Jan 2010, 01:17
LUKA-BRAZI

ხო... ადრე NAD-ი 3 ATP-ს გვაძლევდა და FAD-ი 2-ს. ახლა კიდევ NAD-ი 2,5-ს გვაძლევს, FAD-ი კიდევ 1,5-ს... ეჰ, საით მივექანებით... ბატონო დააავით, გვეშველება?

Posted by: lgogokhia 21 Jan 2010, 01:50
ab-ba
რა შუაშია გამომზეურება? აქ ისედაც ჩანს ვინ რას ამზეურებს biggrin.gif ეს ისე ფიზიოლოგიაა..

LUKA-BRAZI
შენ ნუ სვამ პროვოკაციულ შეკითხვებს. smile.gif ეს ვებ-საიტი თუ იცით? http://library.med.utah.edu/WebPath/EXAM/QUIZ/QUIZIDX.html ერთ-ერთი must resource-ა U of Utah-ს სამედიცინო სტუდენტებისთვის..

აბა ასე ექსპრომტად დასმული შეკითხვა: ავადმყოფი 23 წლის გოგონა თავის ტკივილის, კრუნჩხვების, ღებინების და მოძრაობის კოორდინაციის დარღვევით. ისტორიიდან აღსანიშნავია სარკომის ანამნეზი პირველი ხარისხის ნათესავში.. აბა ჩქარა დიაგნოზი biggrin.gif ნუ მომთხოვთ სურათებს, ლაბ-ებს და ა.შ. პასუხი ან არ იცით ან იცით smile.gif

Posted by: LUKA-BRAZI 21 Jan 2010, 12:59
lgogokhia
სარკომა მეტასტაზებით თავის ტვინში? ს
სტატისტიკურად რბილი ქსოვილების სარკომა იძლევა ყველაზე ხშირად მეტასტაზებს თავის ტვინში, მსგავსი ნევროლოგიური სიმპტომატიკითო.... http://jjco.oxfordjournals.org/cgi/content/full/29/5/245#tbl1_1

QUOTE
ეს ვებ-საიტი თუ იცით? http://library.med.utah.edu/WebPath/EXAM/QUIZ/QUIZIDX.html ერთ-ერთი must resource-ა U of Utah-ს სამედიცინო სტუდენტებისთვის..

სუპერ რესურსია! კიდევ მინდა მასეთები! smile.gif მაგით უნდა შევიმოწმოთ თავი...... მეტა: Cousteau, Blind_Torture_Kill

Posted by: Thandrus 21 Jan 2010, 13:23
lgogokhia

ჰმ... ამ კლინიკური პრეზენტაციით არ ვარ დარწმუნებული. აბა მაინც ვცდი, ლი-ფრაუმენის სინდრომი ხომ არაა შემთხვევით? პაციენტი ახალგაზრდაა, მის ახლო ნათესავს კი ამ სინდრომისათვის ხშირი სიმსივნე (სარკომა) აქვს... ყველა კრიტერიუმს ვერ აკმაყოფილებს, მაგრამ ახლოსაა.

Posted by: LUKA-BRAZI 21 Jan 2010, 13:43
Thandrus
ჰო, გადავიკითხე და მართლა გავს. ლინკი ჩემნაირი უცოდინრებისათვის smile.gif http://emedicine.medscape.com/article/987356-overview

Posted by: lgogokhia 21 Jan 2010, 15:07
and the Oscar goes to Thandrus smile.gif რათქმაუნდა
QUOTE
ლი-ფრაუმენის სინდრომი
UშMLE-ში მსგავსი vague პრეზენტაციები ხშირია, ამიტომ crucial არის რომ მსგავსი ჰინტები დაიმახსოვრო smile.gif

აბა კიდევ შეკითხვა ექსპრომტად: 4 წლის ბიჭი ისევ კრუნჩხვებით და გონებრივი ჩამორჩენილობით. 1 წლის ასაკში დაესვა გულის რაბდომიომის დიაგნოზი. ფუნდოსკოპიით რეტინაზე ჩანს დაზიანება.. დიაგნოზი????

პ.ს. რაბდომიომას დავამატე "გულის"ან გულში თუ როგორაც არის..

Posted by: mtvareuli 21 Jan 2010, 15:12
lgogokhia

ტუბერული სკლეროზი


თუ როგორაა ქართულად spy.gif


Posted by: lgogokhia 21 Jan 2010, 15:28
mtvareuli
კეე wink.gif

37 წლის უშვილო მამაკაცი მოიწამლა პროპანით... დიაგნოზი? LOL თუ დიაგნოზს გამოიცნობთ მიხვდებით რატომ დავწერე ეგრე უაზროდ.. მაგაზე უკეტთესად არ მომდის ტავში ..

Posted by: Cousteau 21 Jan 2010, 15:33
QUOTE (lgogokhia @ 21 Jan 2010, 15:28 )
mtvareuli
კეე wink.gif

37 წლის უშვილო მამაკაცი მოიწამლა პროპანით... დიაგნოზი? LOL თუ დიაგნოზს გამოიცნობთ მიხვდებით რატომ დავწერე ეგრე უაზროდ.. მაგაზე უკეტთესად არ მომდის ტავში ..

Klinefelter's syndrome

QUOTE
ეს იმ პონტშია? რა დაავადებას იწვევს მალარიის გამომწვევიო...

ალბათ იმ პონტში დაწერა რომ რა უნდა ჭირდეს უშვილო მამაკაცს რომელიც პროპანის სუნს ვერ იგებს (ანუ პროპანის ოდორანტის სუნდს)

Posted by: mtvareuli 21 Jan 2010, 15:33
lgogokhia
QUOTE
მამაკაცი მოიწამლა პროპანით... დიაგნოზი?

ეს იმ პონტშია? რა დაავადებას იწვევს მალარიის გამომწვევიო.... biggrin.gif
* * *
Cousteau

აჰა, გასაგებია smile.gif

პ.ს. დაბადებისდღეს გილოცავ wink.gif

Posted by: Cousteau 21 Jan 2010, 15:36
Thandrus
შენ ინტერნალ მედიცინის გარდა რამე რო აირჩიო თავს ჩამოვიხრჩობ : D
very very respect up.gif

QUOTE
აჰა, გასაგებია

შეიძლება ვცდები : D ეს ისე ვიფიქრე user.gif
შეიძლება კარტაგენერი იყოს... ნეზნაუ

QUOTE
პ.ს. დაბადებისდღეს გილოცავ

thanxu 2kiss.gif

Posted by: mtvareuli 21 Jan 2010, 15:42
Cousteau
QUOTE
შეიძლება ვცდები : D ეს ისე ვიფიქრე
შეიძლება კარტაგენერი იყოს... ნეზნაუ

კალმანის სინდრომიც შეიძლება იყოს, ანოსმია და ჰიპოგონადიზმი ახასიათებს

Posted by: lgogokhia 21 Jan 2010, 15:56
mtvareuli
QUOTE
კალმანის სინდრომიც შეიძლება იყოს, ანოსმია და ჰიპოგონადიზმი ახასიათებს

yes.gif 2kiss.gif

Cousteau
QUOTE
ალბათ იმ პონტში დაწერა რომ რა უნდა ჭირდეს უშვილო მამაკაცს რომელიც პროპანის სუნს ვერ იგებს (ანუ პროპანის ოდორანტის სუნდს)

yes.gif
QUOTE
Klinefelter's syndrome

მიუხედავად იმისა რომ კლაინფელტერის სინდრომის დროს შესაძლებელია იზოლირებული ანოსმია იყოს (ვნახე რამდენიმე სტატია, თორემ სტანდარტულად მე მგონი რივიუ ბუქებში არ არის), ამ კონკრეტული ორი სიმპტომის (ჰიპოგონადიზმი და ანოსმია) დროს პირველ რიგში კალმანის სინდრომი მეცემა მე პირადად გონებაში..

გილოცავ მეც დაბადების დღეს smile.gif

very very big respect ნამდვილად ჩემგან ყველას!!! honestly..

Posted by: Cousteau 21 Jan 2010, 17:31
QUOTE (lgogokhia @ 21 Jan 2010, 15:56 )

Cousteau
QUOTE
ალბათ იმ პონტში დაწერა რომ რა უნდა ჭირდეს უშვილო მამაკაცს რომელიც პროპანის სუნს ვერ იგებს (ანუ პროპანის ოდორანტის სუნდს)

yes.gif
QUOTE
Klinefelter's syndrome

მიუხედავად იმისა რომ კლაინფელტერის სინდრომის დროს შესაძლებელია იზოლირებული ანოსმია იყოს (ვნახე რამდენიმე სტატია, თორემ სტანდარტულად მე მგონი რივიუ ბუქებში არ არის), ამ კონკრეტული ორი სიმპტომის (ჰიპოგონადიზმი და ანოსმია) დროს პირველ რიგში კალმანის სინდრომი მეცემა მე პირადად გონებაში..

გილოცავ მეც დაბადების დღეს smile.gif

very very big respect ნამდვილად ჩემგან ყველას!!! honestly..

ამმ, კლაიფელტერს ანოსმია რო ქონდა ეგ არ ვიცოდი, თავიდან ვიფიქრე კართაგენერი, წიპა, ცილიარული დისკინეზიის გამო, ვითომ ყნოსვის რეცეპტორებზეც რამე ექნებოდა, მერე მივგუგლე და კალმანი და კლაინფელტერი ვნახე, ვინაიდან გარტყმაში არ ვარ კალმანის სინდრომი რაარი დავწერე კლაინფელტერი : )))

Posted by: Blind_Torture_Kill 21 Jan 2010, 20:13
Cousteau
QUOTE
ამმ, კლაიფელტერს ანოსმია რო ქონდა ეგ არ ვიცოდი, თავიდან ვიფიქრე კართაგენერი, წიპა, ცილიარული დისკინეზიის გამო, ვითომ ყნოსვის რეცეპტორებზეც რამე ექნებოდა, მერე მივგუგლე და კალმანი და კლაინფელტერი ვნახე, ვინაიდან გარტყმაში არ ვარ კალმანის სინდრომი რაარი დავწერე კლაინფელტერი : )))

biggrin.gif

მეც გილოცავ დაბ. დღეს მალე მენახე უსაში
sa.gif

Posted by: Thandrus 21 Jan 2010, 21:08
mtvareuli

საღოლ, ნთვარე up.gif

Cousteau

QUOTE
შენ ინტერნალ მედიცინის გარდა რამე რო აირჩიო თავს ჩამოვიხრჩობ : D


არ ვიცი ჯერ რა... ენდოკრინოლოგია ან ინფექციონისტობა მინდა.

QUOTE
ვინაიდან გარტყმაში არ ვარ კალმანის სინდრომი რაარი დავწერე კლაინფელტერი : )))


ემბრიოგენეზის დროს, გონადოტროპული ჰორმონის გამომამუშავებელი ნეირონების მიგრაცია უნდა მოხდეს olfactory placode-დან ჰიპოთალამუსში. როდესაც ეს არ ხდება (უხშირესად ცილა ანოსმინის მუტაციის გამო), მაშინ უფუნქციო ნეირონები პლაკოდში რჩებიან და olfactory bulb-ის განვითარებას უშლიან ხელს. ამიტომაც ვიღებთ ორ მთავარ სიმპტომს, ჰიპოგონადიზმსა და ანოსმიას.

კლაინფელტერთან არ ვიცოდი ანოსმიაც თუ იყო ზოგჯერ ასოცირებული.

Posted by: Thandrus 21 Jan 2010, 21:23
აბა ერთი ეს მითხარით, რა არის smile.gif

Posted by: Blind_Torture_Kill 21 Jan 2010, 21:37
QUOTE
აბა ერთი ეს მითხარით, რა არის


პაპილომებს გავს smile.gif

Posted by: mtvareuli 21 Jan 2010, 21:47
Thandrus
QUOTE
ემბრიოგენეზის დროს, გონადოტროპული ჰორმონის გამომამუშავებელი ნეირონების მიგრაცია უნდა მოხდეს olfactory placode-დან ჰიპოთალამუსში

კავშირი სასქესო და საყნოსავ სისტემებს შორის :ფერომონები: tan.gif


Posted by: Thandrus 21 Jan 2010, 22:14
Blind_Torture_Kill

QUOTE
პაპილომებს გავს


არა... ეს უფრო საშინელი სანახავია (ჩემი აზრით) biggrin.gif

mtvareuli

QUOTE
კავშირი სასქესო და საყნოსავ სისტემებს შორის :ფერომონები:




chups.gif

Posted by: LUKA-BRAZI 21 Jan 2010, 22:24
საღოლ ხალხო, საღოლ! მაგრები ხართ ყველა! yes.gif სანდრო, გიგო, მარიკა და დათო. Doc. G-ც მაგარია!! yes.gif

მე კიდევ უვიცი, გაუნათლებელი და ბნელი. წავეთრევი წიგნს გადავშლი.

mtvareuli
შენ სულ მაშინ უნდა შემოხვიდე ხოლმე როცა რამდენიმე თემის და-Merge-ვა მავიწყდება ხოლმე რა! lol.gif
* * *
თემატური კატალოგი გვინდა მართლაც.... თორემ ანბანზე ხშირად სხვანაირად არის თუმცა ესეც კარგია. მეტა: ტანკე biggrin.gif

Posted by: lgogokhia 22 Jan 2010, 04:33
Thandrus
QUOTE
არა... ეს უფრო საშინელი სანახავია (ჩემი აზრით)

ვერ ვხვდები, მაგრამ ინფექციურ/პარაზიტული მგონია რატომღაც..

LUKA-BRAZI
QUOTE
ე კიდევ უვიცი, გაუნათლებელი და ბნელი. წავეთრევი წიგნს გადავშლი.

შენ კი მორჩი თვითგვემას smile.gif))) რეებს წერ თუ იცი..

Posted by: Thandrus 22 Jan 2010, 21:26
lgogokhia

QUOTE
ვერ ვხვდები, მაგრამ ინფექციურ/პარაზიტული მგონია რატომღაც..


მართალი ხარ, ინფექციურია, მაგრამ არა პარაზიტული (ნუ, პროტოზოა / ჰელმინტების გაგებით)

Posted by: Thandrus 22 Jan 2010, 23:15
აქეთაც გადმოვიტან......................

Posted by: LUKA-BRAZI 24 Jan 2010, 16:59
Thandrus
სადრო რატომ გვაწვალებ? smile.gif) ერთი Guardian იყო, სურათის მეტს რომ არაფერს დებდა და რომ ვწვალობდით გამოცნობაზე (Guardian, ვხუმრობ biggrin.gif); ახლა შენც დაიწყე? biggrin.gif

Posted by: Thandrus 24 Jan 2010, 18:15
LUKA-BRAZI

ნუ რავიცი... კლინიკურს ისეთს ვერაფერს დავამატებდი, რაც სურათზე არ ჩანს.

კარგი, გეტყვით მაშინ - ეს ქრომობლასტომიკოზია. მახსოვს ადრე კუსტომ დადო ქრომოს სურათი (ხელზე იყო მეჭეჭისებრი წარმონაქმნი). ეს სურათი კი იმიტომ დავდე, რომ მეც გამაკვირვა, როდესაც ვნახე... ხის სოკოებს ჰგავს ფაქტიურად biggrin.gif

Posted by: LUKA-BRAZI 24 Jan 2010, 21:20
მოკლედ რა ხდება:

ეს თემა, უდავოდ ძალიან კარგი თემაა თავისი არსით და დანიშნულებით. ადრე უფრო საინტერესო იყო, განსაკუთრებით მისი II ნაწილი smile.gif მაგრამ "რა არის ეს?" თემის მიმართულება არის კლინიკური და შეკითხვებიც შესაბამისი თემატიკიდან იდება.

მაგრამ ამ თემაში ვინც ვპოსტავთ, ასე თუ ისე ვიცით ვინც ვართ და რას წარმოვადგენთ. უმეტესობა სტუდენტები ან რეზიდენტები ვართ. ექიმები ცოტანი თუ შემოდიან აქ, შესაბამისად ჩვენთვის ამ თემას სავარჯიშო ფუნქცია აქვს კლინიკურ ცოდნაში, რაც იგივეა რაც S2 CK. ეს ყველაფერი კარგია, მაგრამ რატომ არ ვივარჯიშოთ S1-შიც? თანაც თუ დავუკვირდებით ყველანი S1-თან უფრო ახლოს ვართ... ურიგო არ იქნება თუ ერთმანეთს ამ მხრივაც წავავარჯიშებთ... S1-ის კითხვები ძალიან ბევრია და ტესტებიც უამრავი, სახლშიც ვაკეთებთ, მაგრამ ფორუმზე განხილვა უფრო სახალისო და უფრო პროდუქტიულიც კი შეიძლება იყოს გარკვეულწილად.

ჰოდა თქვენი აზრი მაინტერესებს ამ საკითხზე:
1. გინდათ თუ არა, რომ შეკითხვები დაიდოს S1-ის თემატიკიდან? საბაზისო საგნები ანუ....
2. გაკეთდეს ახალი თემა მაგისთვის, თუ ამ თემას გადავარქვა სახელი, რაიმე შუალედური დვარქვა და ამავე თემაში გავაჩაღოთ დისკუსიები?
3. კითხვები იყოს როგორც ქართულად ისე ინგლისურად. პასუხებიც შესაბამისად. სადაც explanation იქნება დასაწერი, დავწეროთ ინგლისურად/ქართულად და ა.შ. who's for and against? biggrin.gif

ესეც თემის ფლუდი. ვისაც სურს დააფიქსიროს თავისი მოსაზრება.
Thandrus 121
LUKA-BRAZI 93
Professor Xachikian 57
Blind_Torture_Kill 54
lgogokhia 47
MAIN KAMPF 36
Cousteau 35
vano_t 33
basa-ttt 33
Romina 29
PHOBOSS 22
mika9 22
niniaa 16
mtvareuli 13
E-36 12
anarxisti 9
donvaso 9
Mrs_Zum 5
Dru 5
Ni-L 5
irakli222 4
Solveig 4
Tornike Alashvili 3
agnia 3
Rain Dog 2
tergauli 2
TIALATA 1
semiteli 1
utilizatori 1
Mirandaa 1
svani67 1
Masked 1
ab-ba 1
BadbadGirl 1
infinity-infinity 1

c'mon niggas biggrin.gif this won't put too much effort on you smile.gif)

თუ ერთ თემაში გავაკეთებთ ამ ყველაფერს, მაშინ კლინიკური და "არაკლინიკური" კითხვები ერთმანეთს შეენაცვლება. ვთვქათ Cousteau დადებს კითხვას რომელიმე საბაზისო საგნიდან, რომელიც შედის S1-ში, ხოლო ვთქვათ MAIN KAMPF დადებს კითხვას კლინიკიდან და ა.შ.

თუ ცალ-ცალკე თემას გავაკეთებთ, მაშინ ამ თემაში იქნება კლინიკური ქეისები, ახალ თემაში კი იქნება კითხვები ე.წ. საბაზისო საგნებიდან.

მოკლედ ასე...... ვინც თვლით რომ ჯანმედის წევრი ხართ და "რა ასრის ეს?" თემაც თქვენთვის ახლოსაა და რეგულარულად იღებთ მონაწილეობას განხილვებში, გთხოვთ დააფიქსიროთ თქვენი აზრი. შემთხვევითმა ან ჯანმედში გავლით მყოფმა იუზერებმა თავი შეიკავეთ smile.gif)

უჰ, რამდენი ვწერე. დააფასეთ ახლა შრომა smile.gif)

Posted by: Thandrus 24 Jan 2010, 21:45
LUKA-BRAZI

ლუკა, საინტერესო მოსაზრებაა ნამდვილად. მაგრამ არამგონია საბაზისო კითხვების განხილვამ ასე გაამართლოს. ამ თემის მთავარი ხიბლი ისაა, რომ თავს იმტვრევ დიაგნოზის დასასმელად, უნდა რაღაც გამორიცხო, რაღაც შეამოწმო, რაღაც გეზს მიყვე... და საბაზისოს კითხვები როგორ წარმოგიდგენია მაგ. ფიზიოლოგიაში? რას ნიშნავს ესა თუ ის გრაფიკი? ეს ან იცი, ან არ იცი - challenge-ც ნაკლებია. ჭკვიანური აზრი იქნება საბაზისო საგნების შესახებ თემის გახსნა, მაგრამ არამგონია, რომ აქტიური იყოს. იქაც ქეისებისნაირი სისტემა არა მგონია, რომ გამოდგეს. უფრო იმას დავდებთ ხოლმე, რაც ვერ გავიგეთ და მერე ვიღაც მადლიანი მოვა და აგვიხსნის biggrin.gif ასე რომ, ჩემი აზრი ასეთია: ახალი თემის გახსნას ვემხრობი, მაგრამ პროდუქტიულობას ვერ შეგპირდები.

Posted by: LUKA-BRAZI 24 Jan 2010, 22:32
Thandrus
ჰო, challenge-ს არც მოველით ალბათ, იმიტომ რომ თავის მტვრევა არ მოგიწევს შენი თქმის არ იყოს. ამ შემთხვევაში მართლაც
QUOTE
ან იცი, ან არ იცი

ჰოდა, მაგას ვამბობ ზუსტად: განვამტკიცოთ ცოდნა საბაზისო საგნებში. აი როგორც ქვიზები აიეტში biggrin.gif ქვიზებია ქვია ხო თქვენთან? smile.gif
ვთქვათ კითხვა:
რას აკეთებს p53 გენი? და პასუხების ვარიანტები; ან უვარიანტოდ და პირდაპირ პასუხი უნდა დაწეროს გამომცნობმა.

აქტიურობას რაც შეეხება, მართლაც დიდად აქტიური არ იქნება ალბათ, ძირითადად 4-5 კაცი თუ დავპოსტავთ, მაგრამ თავის საქმეს გააკეთებს მაინც, იმჰო.... რას იტყვი?

სხვები რას ფიქრობენ?

Posted by: anarxisti 25 Jan 2010, 00:01
QUOTE (LUKA-BRAZI @ 24 Jan 2010, 22:32 )
Thandrus
ჰო, challenge-ს არც მოველით ალბათ, იმიტომ რომ თავის მტვრევა არ მოგიწევს შენი თქმის არ იყოს. ამ შემთხვევაში მართლაც
QUOTE
ან იცი, ან არ იცი

ჰოდა, მაგას ვამბობ ზუსტად: განვამტკიცოთ ცოდნა საბაზისო საგნებში. აი როგორც ქვიზები აიეტში biggrin.gif ქვიზებია ქვია ხო თქვენთან? smile.gif
ვთქვათ კითხვა:
რას აკეთებს p53 გენი? და პასუხების ვარიანტები; ან უვარიანტოდ და პირდაპირ პასუხი უნდა დაწეროს გამომცნობმა.

აქტიურობას რაც შეეხება, მართლაც დიდად აქტიური არ იქნება ალბათ, ძირითადად 4-5 კაცი თუ დავპოსტავთ, მაგრამ თავის საქმეს გააკეთებს მაინც, იმჰო.... რას იტყვი?

სხვები რას ფიქრობენ?

საინტერესო იდეაა, უნდა იცადოს...
მაგალითად მე საბაზისო საკითხები აღარ მახსოვს (ახალი ხო არ ვიცი) ..... ბევრი.. და ოკაზია მექნება გავიხსენო, ვისწავლო biggrin.gif

Posted by: LUKA-BRAZI 25 Jan 2010, 00:55
anarxisti
შენ თუ კიდევ რამე გაქვს სასწავლი, ჩვენ რაღა უნდა ვთქვათ? biggrin.gif აი კლინიკურ ქეისებს რომ უფრო ხსირად გვთავაზობდე, ყველანი მოხარულნი ვიქნებოდით smile.gif

Posted by: lgogokhia 25 Jan 2010, 11:28
LUKA-BRAZI
საბაზისო კითხვებისთვის ალბათ უკეთესია ახალი თემა გაიხსნას.. smile.gif

Thandrus
QUOTE
კარგი, გეტყვით მაშინ - ეს ქრომობლასტომიკოზია.

sad.gif ეს არ ვიცოდი, მაგრამ აღარ დამავიწყდება.. ამიტომაა ეს გარჩევები კარგი... პროდუქტიული ჩვენი ცოდნის პირობებში..

Posted by: LUKA-BRAZI 25 Jan 2010, 12:39
lgogokhia
QUOTE
ეს არ ვიცოდი, მაგრამ აღარ დამავიწყდება.. ამიტომაა ეს გარჩევები კარგი... პროდუქტიული ჩვენი ცოდნის პირობებში..

ჰო ზუსტად. ვიაქტუროთ ხალხო რა.. smile.gif LG, შენც დადე ქეისები ხოლმე..... როცა ვირუსებთან და ფაგებთან საქმეს მორჩები ხოლმე smile.gif

lgogokhia
ე.ი. S1-ის კითხვებს შენც ემხრობი ჰო ? smile.gif
* * *
სხვები რას ფიქრობთ?...............

Posted by: Blind_Torture_Kill 25 Jan 2010, 15:54

............................................... ვემხრობი

Posted by: E-36 25 Jan 2010, 19:19
+1
მეც ვემხრობი ასევე ლუკა smile.gif

Posted by: Romina 25 Jan 2010, 19:35
+1
კარგი იდეაა და ვემხრობი, თუმცა კლინიკური ხასიათის შეკითხვებზე მსჯელობა უფრო სახალისოა smile.gif

Posted by: LUKA-BRAZI 27 Jan 2010, 19:56
QUOTE
Cousteau 35 vano_t 33

QUOTE
mika9 22 niniaa 16 mtvareuli 13

თქვენც თქვით თქვენი აზრი და გავხსნი ახალ თემას და ეგ იქნება smile.gif

Posted by: mtvareuli 28 Jan 2010, 01:17
LUKA-BRAZI
QUOTE
თქვენც თქვით თქვენი აზრი

მოიქეცი როგორც საჭიროდ ჩათვლი

Posted by: Solveig 28 Jan 2010, 01:47
LUKA-BRAZI
QUOTE
რას აკეთებს p53 გენი? და პასუხების ვარიანტები; ან უვარიანტოდ და პირდაპირ პასუხი უნდა დაწეროს გამომცნობმა.

არ ვიცი, ჩემი ხმა მიიღება თუ არა, მაგრამ მე არ გირჩევდით smile.gif

ერთი მარტივი მიზეზის გამო-საბაზისო საგნების სწავლა ყველას შეგიძლიათ-წიგნები-გაქვთ, უცხო ენები იცით, ტვინიც გაქვთ smile.gif

კლინიკურ შემთხვევებს მიეძალეთ, იმიტომ, რომ აქ პრაქტიკოსი ექიმებიც შემოდიან და მათგან გაცილებით ბევრს ისწავლით კონკრეტულად სამკურნალო საქმეში და დიფერენციალურ დიაგნოსტიკაშიც გაიწაფებით, ვიდრე, ვთქვათ, აქ რომ ლენინჯერის ბიოქიმიის, ალბერტსის მოლბიოლოგიის და ჯენევეის იმუნოლოგიის რჩევა დაიწყოთ.

ექიმად დასპეციალება უკვე ავტომატურად ნიშნავს, რომ არაკლინიკური საგნები გავლილი უნდა გქონდეს..ტუ მაინცდამაინც მაგაშიც გინდათ ვარჯიში, მეორე თემა გახსენით, არევა არ ივარგებს smile.gif

ასეთია ჩემი აზრი.

Posted by: mtvareuli 28 Jan 2010, 02:25
Solveig

QUOTE
საბაზისო საგნების სწავლა ყველას შეგიძლიათ-წიგნები-გაქვთ, უცხო ენები იცით, ტვინიც გაქვთ

კლინიკურ შემთხვევებს მიეძალეთ, იმიტომ, რომ აქ პრაქტიკოსი ექიმებიც შემოდიან და მათგან გაცილებით ბევრს ისწავლით კონკრეტულად სამკურნალო საქმეში და დიფერენციალურ დიაგნოსტიკაშიც გაიწაფებით, ვიდრე, ვთქვათ, აქ რომ ლენინჯერის ბიოქიმიის, ალბერტსის მოლბიოლოგიის და ჯენევეის იმუნოლოგიის რჩევა დაიწყოთ

გეთანხმები

და +მაგას

როცა კითხვა ამგვარად იქნება დასმული
QUOTE
რას აკეთებს p53 გენი?

ამის დასერჩვა უპრობლემოდ შეიძლება წიგნებშიც და გუგლშიც. თემას აზარტი და მუღამი არ ექნება

მე ვთვლი რომ საერთოდ არაა საჭირო ამ საკითხების ფორუმზე გარჩევა, ანუ სპეციალურად

თორე თუ სიტყვა მოიტანს რამე საინტერესო ფაქტს ან მოვლენას, რომელიც თეორიულში შედის, რათქმაუნდა ვახსენებთ


Posted by: Cousteau 28 Jan 2010, 13:57
user posted image
ზა

QUOTE
მოიქეცი როგორც საჭიროდ ჩათვლი

+ 1

Posted by: LUKA-BRAZI 28 Jan 2010, 14:17
Solveig
ეს საგნები გავლილი გვაქვს რა თქმა უნდა. მაგრამ რადგანაც რამდენიმე ადამიანი აქ S1 გამოცდისთვის ვემზადებით, ვიფიქრე, ერთამანეთის შემოწმება მაგ გამოცდის მასალებში, ისევ ჩვენ წაგვადგებოდა.... ერთგვარი ქვიზი, რომელიც ინტერნეტშიც ბევრია, მაგრამ აქაც რომ იყოს, უფრო სახალისო იქნებოდა....... რა ვიცი, ვნახოთ......

lgogokhia
Cousteau
Blind_Torture_Kill
Thandrus
რომ ემხრობით, კარგია, მაგრამ იაქტიურებთ რო? user.gif ამ თემაშიც არ აქტიურობს არავინ........ user.gif

Posted by: donvaso 28 Jan 2010, 21:29
ცალკე თემის გახსნა არ უნდა იყოს პრობლემა(მითუმეტეს შენთვის biggrin.gif) ხოდა გახსენი და ნახავ გაამართლებს თუ არა... wink.gif
ეს თემა კიდევ ისევე იყოს როგორც არის... smile.gif

Posted by: Cousteau 10 Feb 2010, 03:58
ცალკე თემის გახსნა მინდოდა ამ კითხვისთვის, ანუ იმ თემი ლუკას რო დიდი ხანი უნდა, მარა თუ მოუნდება თვითონ გახსნის იმედია ; )

მაშასადამე კითხვა მდგომარეობს შემდეგში:

ფეოქრომოციტომა არის თირკმელზედა ჯირკვალის სიმსივნე, ცნობილია რომ თირკმელზედა ჯირკვალი, ნორეპინეფრინთან შედარებით (20%), უპირატესად ასინთეზირებს ეპინეფრინს (ადრენალინს 80%) ხოლო ეს ფეოქრომოციტომა კი უპირატესად ასინთეზირებს ნორეპინეფრინს (ნორადრენალინს) რატომ? spy.gif


Posted by: LUKA-BRAZI 10 Feb 2010, 15:08
Cousteau
მე არ ვუპასუხებ gigi.gif

Posted by: Romina 15 Feb 2010, 21:29
Cousteau

რა საინტერესოა, არ ვიცი smile.gif იმიტომ ხომ არა რომ ფეოქრომოციტომის შემთხვევაში, განსხვავებით ჯანმრთელი თირკმელზედა ჯირკვლისგან, კატექოლამინების გამოყოფა ნერვული იმპულსით (ანუ ორგანიზმის საჭიროების გამო) არ სტიმულირდება და შესაბამისად ნორადრენალინი აღარ გარდაიქმნება ადრენალინად?

ეხლა რასაც დავწერ კითხვის ხასიათი არ აქვს, უფრო ინფორმაციასავით არის. რამდენი რამე უნდა იცოდე ექიმმა, კარგი პრაქტიკოსი რომ იყო sad.gif 60 წლის ქალმა მიმართა ექიმს მარჯვენა მხრის სახსრის და წინამხრის არეში ტკივილით, მოძრაობის შეზღუდვით. გასინჯვით და გამოკვლევით (მხრის სახსრის რენტგენი, ექოსკოპია), დაისვა პერიართრიტის დიაგნოზი და დაეწყო მკურნალობა. გარდა ტკივილისა, პაციენტი აღნიშნავდა რომ ამავე ხელში გრძნობდა ძალის დაქვეითებას და უჭირდა ასოების გამოყვანა. ესეც ჩაითვალა ფესვობრივ სიმპტომად და არ უმოქმედია დასმულ დიაგნოზზე. მკურნალობის ფონზე (არასტეროიდებს + ადგილობრივი ინექციები) მდგომარეობა გაუმჯობესდა ტკივილის მხრივ, მაგრამ ხელში სისუსტე დარჩა და დაემატა მარჯვენა ქვედა კიდურის არეში სისუსტე, პაციენტი თითქოს მიათრევდა ფეხს. მაგრამ ეს ძლივს შესამჩნევი სიმპტომი იყო. საბოლოოდ ბევრი რომ არ გავაგრძელო ერთმა ჭკვიანმა ნევროლოგმა დასვა დიაგნოზი: პარკინსონიზმი - 1 სტადია. საქმე ის იყო რომ პარკინსონიზმისთვის დამახასიათებელმა კუნთების რიგიდობამ გამოიწვია კუნთების მიმაგრების ადგილების დაზიანება და პერიართრიტის სურათი. პარკინსონიზმის ერთადერთ გამოვლინება თვეების განმავლობაში ეს ტკივილი იყო და დააბნია ექიმი smile.gif

Posted by: Thandrus 15 Feb 2010, 23:09
Cousteau

QUOTE
ფეოქრომოციტომა არის თირკმელზედა ჯირკვალის სიმსივნე, ცნობილია რომ თირკმელზედა ჯირკვალი, ნორეპინეფრინთან შედარებით (20%), უპირატესად ასინთეზირებს ეპინეფრინს (ადრენალინს 80%) ხოლო ეს ფეოქრომოციტომა კი უპირატესად ასინთეზირებს ნორეპინეფრინს (ნორადრენალინს) რატომ?


არ ვიცი... შეიძლება, როგორც რომინამ თქვა, იმის გამოა, მაგრამ დარწმუნებული არ ვარ. ჰარისონში და რობინსშიც ვერ ვიპოვე ამის ახსნა (შეიძლება მე ვეძებდი ცუდად) eMedicine-ში ეწერა ამ ფაქტზე, მაგრამ ახსნილი არ იყო, როგორ და რატომ... ასე რომ, თუ პასუხი გაქვს, გაგვანათლე smile.gif

Romina

QUOTE
ეხლა რასაც დავწერ კითხვის ხასიათი არ აქვს, უფრო ინფორმაციასავით არის. რამდენი რამე უნდა იცოდე ექიმმა, კარგი პრაქტიკოსი რომ იყო60 წლის ქალმა მიმართა ექიმს მარჯვენა მხრის სახსრის და წინამხრის არეში ტკივილით, მოძრაობის შეზღუდვით. გასინჯვით და გამოკვლევით (მხრის სახსრის რენტგენი, ექოსკოპია), დაისვა პერიართრიტის დიაგნოზი და დაეწყო მკურნალობა. გარდა ტკივილისა, პაციენტი აღნიშნავდა რომ ამავე ხელში გრძნობდა ძალის დაქვეითებას და უჭირდა ასოების გამოყვანა. ესეც ჩაითვალა ფესვობრივ სიმპტომად და არ უმოქმედია დასმულ დიაგნოზზე. მკურნალობის ფონზე (არასტეროიდებს + ადგილობრივი ინექციები) მდგომარეობა გაუმჯობესდა ტკივილის მხრივ, მაგრამ ხელში სისუსტე დარჩა და დაემატა მარჯვენა ქვედა კიდურის არეში სისუსტე, პაციენტი თითქოს მიათრევდა ფეხს. მაგრამ ეს ძლივს შესამჩნევი სიმპტომი იყო. საბოლოოდ ბევრი რომ არ გავაგრძელო ერთმა ჭკვიანმა ნევროლოგმა დასვა დიაგნოზი: პარკინსონიზმი - 1 სტადია. საქმე ის იყო რომ პარკინსონიზმისთვის დამახასიათებელმა კუნთების რიგიდობამ გამოიწვია კუნთების მიმაგრების ადგილების დაზიანება და პერიართრიტის სურათი. პარკინსონიზმის ერთადერთ გამოვლინება თვეების განმავლობაში ეს ტკივილი იყო და დააბნია ექიმი


საღოლ ნევროლოგს smile.gif

Posted by: Romina 15 Feb 2010, 23:13
Cousteau
QUOTE
ასე რომ, თუ პასუხი გაქვს, გაგვანათლე


ხო, გაგვანათლე რა. გვაინტერესებს smile.gif

Posted by: E-36 16 Feb 2010, 11:16
Romina
QUOTE
ეხლა რასაც დავწერ კითხვის ხასიათი არ აქვს, უფრო ინფორმაციასავით არის. რამდენი რამე უნდა იცოდე ექიმმა, კარგი პრაქტიკოსი რომ იყო60 წლის ქალმა მიმართა ექიმს მარჯვენა მხრის სახსრის და წინამხრის არეში ტკივილით, მოძრაობის შეზღუდვით. გასინჯვით და გამოკვლევით (მხრის სახსრის რენტგენი, ექოსკოპია), დაისვა პერიართრიტის დიაგნოზი და დაეწყო მკურნალობა. გარდა ტკივილისა, პაციენტი აღნიშნავდა რომ ამავე ხელში გრძნობდა ძალის დაქვეითებას და უჭირდა ასოების გამოყვანა. ესეც ჩაითვალა ფესვობრივ სიმპტომად და არ უმოქმედია დასმულ დიაგნოზზე. მკურნალობის ფონზე (არასტეროიდებს + ადგილობრივი ინექციები) მდგომარეობა გაუმჯობესდა ტკივილის მხრივ, მაგრამ ხელში სისუსტე დარჩა და დაემატა მარჯვენა ქვედა კიდურის არეში სისუსტე, პაციენტი თითქოს მიათრევდა ფეხს. მაგრამ ეს ძლივს შესამჩნევი სიმპტომი იყო. საბოლოოდ ბევრი რომ არ გავაგრძელო ერთმა ჭკვიანმა ნევროლოგმა დასვა დიაგნოზი: პარკინსონიზმი - 1 სტადია. საქმე ის იყო რომ პარკინსონიზმისთვის დამახასიათებელმა კუნთების რიგიდობამ გამოიწვია კუნთების მიმაგრების ადგილების დაზიანება და პერიართრიტის სურათი. პარკინსონიზმის ერთადერთ გამოვლინება თვეების განმავლობაში ეს ტკივილი იყო და დააბნია ექიმი


up.gif up.gif up.gif

Posted by: Cousteau 19 Feb 2010, 17:55
Answer:

თირკმელზედა ჯირკვალი ასინთერზირებს ეპინეფრინს ნორეპინეფრინიდან, ამას ორი რამე ჭირდება: PNMT- ფენილეთანოლამინ-ნ-მეთილ ტრანსფერაზა (ვცდილობ ეს სახელი დავიმახსოვრო : D ) და კორტიზოლი, კორტიზოლის მიწოდება კორტექსიდან ხდება, ფეოქრომოციტომის დროს, სიმსივნეს აქვს PNMT მაგრამ ვერ იღებს კორტიზოლს, ამიტომაც ასინთეზირებს ნორეპინეფრინს : )

Posted by: Romina 19 Feb 2010, 18:31
Cousteau


smile.gif ძალიან საინტერესოა, მადლობა.
მაგრამ სიმსივნეც ხომ ასინთეზირებს ადრენალინს, შეიძლება უფრო იშვიათად და უფრო მცირე რაოდენობით მაგრამ მაინც?

Posted by: Thandrus 19 Feb 2010, 18:55
Cousteau

ვაჰ... პირველად მესმის, მართალი რომ გითხრა... შეგიძლია წყარო მიმითითო, სად ამოიკითხე? user.gif რამდენიმე წიგნში ვნახე კატექოლამინების სინთეზი და ფეოქრომოციტომა და არაფერი ეწერა ამაზე (ერთი კია - ფიზიოლოგიის წიგნებში არ მინახავს) ჰოდა დამაინტერესა ძალიან. smile.gif

Posted by: Cousteau 19 Feb 2010, 19:10
QUOTE (Thandrus @ 19 Feb 2010, 18:55 )
ერთი კია - ფიზიოლოგიის წიგნებში არ მინახავს

Linda S. Costanzo, Physiology, 3d edition, Chapter 2 : )

Romina yes.gif

Posted by: LUKA-BRAZI 19 Feb 2010, 19:50
Thandrus
აჰა, ბევრი რომ აღარ იწვალო smile.gif :
Adrenal Medulla
Body_ID: HC002010
The adrenal medulla is a specialized ganglion in the sympathetic division of the autonomic nervous system. The cell bodies of its preganglionic neurons are located in the thoracic spinal cord. The axons of these preganglionic neurons travel in the greater splanchnic nerve to the adrenal medulla, where they synapse on chromaffin cells and release ACh, which activates nicotinic receptors. When activated, the chromaffin cells of the adrenal medulla secrete catecholamines (epinephrine and norepinephrine) into the general circulation. In contrast with sympathetic postganglionic neurons, which release only norepinephrine, the adrenal medulla secretes mainly epinephrine (80%) and a small amount of norepinephrine (20%). The reason for this difference is the presence of phenylethanolamine-N-methyltransferase (PNMT) in the adrenal medulla, but not in sympathetic postganglionic adrenergic neurons (see Fig. 1-18). PNMT catalyzes the conversion of norepinephrine to epinephrine, a step that, interestingly, requires cortisol from the nearby adrenal cortex; cortisol is supplied to the adrenal medulla in venous effluent from the adrenal cortex.
Body_ID: P002023
A tumor of the adrenal medulla, or pheochromocytoma, may be located on or near the adrenal medulla, or at a distant (ectopic) location in the body (Box 2-1). Unlike the normal adrenal medulla, which secretes mainly epinephrine, a pheochromocytoma secretes mainly norepinephrine, which is explained by the fact that the tumor is too far from the adrenal cortex to receive the cortisol that is required by PNMT.

Posted by: idallgo 25 Feb 2010, 19:46
ეგ საკითხი ფეოქრომოციტომაზე ანატომიის წიგნშიც შემხვდა High Yield Gross Anatomy-ში. ოღონდ მასე დაწვრილებით არ წერია.

Posted by: LUKA-BRAZI 25 Feb 2010, 20:50
idallgo
medical student? smile.gif

Posted by: BadbadGirl 26 Feb 2010, 00:53
აუ ხალხებო, დამეხმარეთ რა რას გულისხმობენ Brainstem ის მაკროსკოპულ სტუქტურებში?

ვერ გავიგე რა უნდათ,

დიდი მადლობა წინასწარ.

Posted by: Thandrus 26 Feb 2010, 01:09
BadbadGirl

ჰმ... ალბათ იმ სტრუქტურებს, რომელიც brainstem-ში შედის: medulla oblongata-ს (მოგრძო ტვინი), pons-ს (ხიდი) და midbrain-ს (შუა ტვინი)

LUKA-BRAZI

QUOTE
interestingly, requires cortisol from the nearby adrenal cortex;


უყურე შენ კონსტანცოს biggrin.gif ეს არ ვიცოდი, გმადლობთ smile.gif

Posted by: LUKA-BRAZI 26 Feb 2010, 01:18
Thandrus
QUOTE
უყურე შენ კონსტანცოს

biggrin.gif
მაშ, მაშ! smile.gif)
QUOTE
ჰმ... ალბათ იმ სტრუქტურებს, რომელიც brainstem-ში შედის: medulla oblongata-ს (მოგრძო ტვინი), pons-ს (ხიდი) და midbrain-ს (შუა ტვინი)

yes.gif

Posted by: BadbadGirl 26 Feb 2010, 01:20
Thandrus
მეც მასე ვფიქრობდი დიდი მადლობა
და კიდე მეორე კითხვა
რა მიკროსკოპულ სტრუქტური მდებარეობს CNS ის თეთრ და ნაცრისფერი სუბსტანციაში?
(აზრზე არ ვარ ქართულად/ინგლისურად როგორაა ეს სიტყვები-იმპულსებს რომ ატარებს აფერენტულს და ეფერენტულს ვიტომ მაგ გზებს გულისხმობენ თეთრ სუბსტანციაში, და მაგალითად Nucleus caudatus, nucleus ruber ნაცრისფერში? თუ უბრალოდ დენდრიტებსა და აქსონებზე აქვთ საუბარი?)

Posted by: LUKA-BRAZI 26 Feb 2010, 01:24
BadbadGirl
თეთრი ნივთიერება შედგება აქსონებისგან, რომლებიც ქმნიან იმპულესების გამტარ გზებს (აფერენტულ, ეფერენტულ და კომუნიკაციურ და ასოციაციურს და ა.შ.). რუხი ნივთიერება არის მხოლოდ დენდრიტები და უფრო მეტად ნეირონის სხეულები. ეგენი ქმნიან თავის ტვინის ქერქს და ქერქქვეშა ბირთვებს, ასევე ზურგის ტვინის რუხ ნივთიერებას (პეპელას რომ გავს smile.gif).

Posted by: BadbadGirl 26 Feb 2010, 01:28
LUKA-BRAZI
ხო მაგრამ რას გულისხმობენ? აი ეგ ვერ გავიგე? რა ვუპასუხო კითხვაზე აქსონები - დენდრიტი და სხეული
თუ გზები და ბირთვები რომლებიც მათგან იქმნება?

შვედურიდან რომ გადმოვთარგმნო, სიტყვასიტყვით აი ესეა კითხვა - რა მიკროსკოპულ სტრუქტური მდებარეობს CNS ის თეთრ და ნაცრისფერი სუბსტანციაში? პასუხი უნდა იყოს მოკლე და კონკრეტული sad.gif

პ.ს. ისე, მგონი მართლა ნეირონის აგებულებაზეა საუბარი, თორემ კორტექს ცერებრი როგორ არის მიკროსკოპული სტრუქტურა?

Posted by: LUKA-BRAZI 26 Feb 2010, 01:35
BadbadGirl
QUOTE
მიკროსკოპულ სტრუქტური

ეს ვერ შევამჩნიე. თუ მიკროსკოპულზეა, მაშინ იქნება: თეთრ ნივთიერებას ქმნიან აქსონები, რუხს ნეირონის სხეულები და დენდრიტები.

Posted by: BadbadGirl 26 Feb 2010, 01:36
LUKA-BRAZI
Thandrus
ორივეს მადლობა smile.gif

აწი რაღა მიჭირს ორ კითხვაზე ქე მაქვს პასუხი biggrin.gif biggrin.gif

Posted by: LUKA-BRAZI 26 Feb 2010, 01:38
BadbadGirl
არაფერს.... და შვედურად რატომ აკეთებ ტესტებს? smile.gif)

Posted by: BadbadGirl 26 Feb 2010, 01:40
LUKA-BRAZI
იმიტომ რომ შვედურად ვსწავლობ biggrin.gif biggrin.gif

კიდე დავსვავ ჩაინიკურ კითხვებს გამოცდა თუ ვერ ჩავაბარე. ნეიროანატომია მეორადი საგანია და მიფუჩეჩებას ვაპირებ biggrin.gif

კიდევ ერთი კითხვა რომელსაც ძალიან საინტერესო პასუხი აქვს და ვერ ვხვდები (არა მარტო მე საბედნიეროდ) რატომ.
კიხვა მდგომარეობი შემდეგში- სად მთავრდება Cerebellum -ის ეფერენტული გზა?

და პასუხი იცი რა ჩაგვაწერინეს? Motor cortex via Thalamus.
ახლა გამაგებინე, ეფერენტული გზა როგორ შეიძლება დამთავრდეს Motor cortex ზე. ან შეცდომააა, ამ მართლა ვერაფერი გავუგე ამ გზებს.

Posted by: LUKA-BRAZI 26 Feb 2010, 02:12
QUOTE
სად მთავრდება Cerebellum -ის ეფერენტული გზა?

ეფერენტული თუ აფერენტული?

ყოველ შემთხვევაში, Motor cortex არის ნებითი მოძრაობების მაკონტროლებელი უბანი. თალამუსი კი უმეტესობა აფერენტულ და ეფერენტული იმპულსების გამტარი სტრუქტურაა და რადგანაც ნათხემი მონაწილეობს მოძრაობის კოორდინაციაში, არ არის გასაკვირი ამ სამი სტრუქტურის ურთიერთკავშირი.

Posted by: BadbadGirl 26 Feb 2010, 02:20
LUKA-BRAZI
ეფერენტული.
აფერენტული რომ ყოფილიყო არ გამიკვირდებოდა.

Posted by: LUKA-BRAZI 26 Feb 2010, 02:25
BadbadGirl
მოტორული ეფერენტული გზა როგორც მახსოვს ზურგის ტვინის წინა რქებში მთვრდება, მაგრამ მგონი რაღაც გამომრჩა. Thandrus, Cousteau, დაგვეხმარეთ smile.gif

Posted by: BadbadGirl 26 Feb 2010, 14:48
როგორც ველოდი იყო ტესტში კითხვა ცერებელუმის ეფფერენტულის შესახებ.
იმიტომ ველოდი რომ ძალიან ჩამჭრელი კითხვაა, ეფფერენტული ეგრევე გაფირებინებს იმაზე რომ ომპულსი გარეთ გამოდის, თუმცა ამ შემხვევაშI ასე არაა.

ხოდა დავწერე როგორცაა და ფრჩხილებშიც მიუწერე, ძალიან მეუცნაურება მაგრამ ჩვენმა ლექტორმა ასე თქვა თქო biggrin.gif biggrin.gif

Posted by: Solveig 26 Feb 2010, 16:56
BadbadGirl
ფიზიოლოგიის სახელმძღვანელო არ გაქვს???
როგორ მეცადინეობ?
* * *
BadbadGirl
QUOTE
Brainstem ის მაკროსკოპულ სტუქტურებში?

ეგ არის ტვინის ღერო, რომელშიც შედის მოგრძო ტვინი (medulla oblongata) ხიდი (pons), შუა ტვინი (mesencephalon). და ზოგჯერ ნათხემსაც (cerebellum) და შუამდებარე ტვინსაც (diencephalon) მაგას მიათვლიან ხოლმე...

QUOTE
რა მიკროსკოპულ სტრუქტური მდებარეობს CNS ის თეთრ და ნაცრისფერი სუბსტანციაში?

ჩემი აზრით, იგულისხმება აქსონები (თეთრი) და სხეულები და დენდრიტები (ნაცრისფერი)...რადგან თავის ტვინის ბაზალური ბირთვები მიკრო კი არა, მაკროსკოპული სტრუქტურებია, ტვინის განაჭერზე ჩანს შეუიარაღებელი თვალითაც.LUKA-BRAZI სავსებით მართალია.


QUOTE
სად მთავრდება Cerebellum -ის ეფერენტული გზა?

ეფერენტული ზოგადად გულისხმობს გამომავალს...არ არის აუცილებელი, მაინცდამაინც პერიფერიაზე მთავრდებოდეს ეფერენტული გზა.

და ქერქში რომ მთავრდება, ამაშიც რა არის გასაკვირი smile.gif

იოსელიანის ფიზიოლოგიის სახელმძღვანელო მაინც არ გაქვს? ფსიქოლოგები მაგით სწავლობდით, როგორც მახსოვს...ბუციკო ჩხარტიშვილი არ გასწავლიდა?

Posted by: Thandrus 26 Feb 2010, 17:25
BadbadGirl

ცერებელუმის ეფერენტულ გზებში ალბათ გულისხმობენ Cerebellar output pathways-ს, ანუ იმ გზებს რომელიც ცერებელუმიდან გამოდიან და თავის ტვინის სხვა სტრუქტურებში მიდიან.

როგორც ვიცით, ცერებელუმი (ნათხემი) სამ ზონად შეიძლება რომ დავყოთ: 1. ლატერალური ნახევარსფერო (რომელიც მოტორულ დაგეგემარებაში მონაწილეობს) 2. შუა ნახევარსფერო (რომლის ფუნქცია კიდურების დისტალური ნაწილების კოორდინაციაა) და 3. ვერმისი / ფლოკულონოდულარული ნაწილი (რომლებიც კიდურების პროქსიმალური ნაწილების და ტანის მოძრაობას აკროორდინირებენ, აგრეთვე არეგულირებენ ბალანსს და ვესტიბულო-ოკულარულ რეფლექსებს).

ახლა რაც შეეხება ცერებელუმიდან გამომავალ გზებს: ცერებელუმში არსებობს ოთხი "ღრმა ბირთვი" (deep nuclei) ესენია dentate, emboliform, globose და fastigial ბირთვები (Don't Eat Greasy Foods - კარგი მნემონიკი დასამახსოვრებლად), რომლებშიც ნათხემის ამათუიმ ნაწილიდან გამომავალი გზები შედის და შემდეგ გრძელდება. ლატერალური ნახევარსფეროს გზები dentate nucleus-ში მიდიან. აქიდან გამომავალი ნეირონები superior cerebellar peduncle-ს გავლით (რომელიც შუა ტვინის დონეზე დეკუზაციას აკეთებს - ანუ ცენტრის მეორე მხარეს გადადის) მიდიან თალამუსის ვენტროლატერალურ ბირთვში. საერთოდ, ამ ბირთვში შემავალ გზებს thalamic fasciculus-ს უწოდებენ, და მის შემადგენლობაში ცერებელუმიდან წამოსული ნეირონების გარდა ბაზალური განგლიებიდან წამოსული ნეირონებიც არიან. ვენტროლატერალური ბირთვიდან პროექციები კორტექსში (მოტორული, პრემოტორული კორტექსები, დამხმარე მოტორული არე. პარიეტალური წილი) მიდის, რომელიც თავის მხრივ, კორტიკოსპინალურ ტრაქტს არეგულირებს. ასე ხდება მოძრაობის დაგეგმვა. ამას გარდა, ლატერალური ნახევარსფეროდან ბოჭკოების რაღაც ნაწილი თალამუსის გავლით პრეფრონტალურ კორტექსშიც მიდის (რასაც ფიქრობენ, რომ კოგნიტიური ფუნქცია აქვს) და კიდევ dentate nucleus-იდან red nucleus-ის პარვოცელულარულ ნაწილში მიდის, რაც თავის მხრივ, red nucleus-ის ამ ნაწილიდან გამომავალი გზების გათვალისწინებით, ცერებელარულ წრეს (cerebellar circuitry) ქმნის.

შუა ნახევარსფეროდან გამომავალი გზა interposed nuclei-ში მიდის. მერე როგორც ლატერალური ნახევასფეროს გზა. ისე ესეც superior cerebellar peduncle-ს გავლით თალამუსის ვენტროლატერალურ ბირთვში და იქიდან მოტორულ კორტექსში მიდის, მოტორული კორტექსი კი კორტიკოსპინალურ ტრაქტს აკონტროლებს. ამასთან ერთად, interposed nuclei-დან ნეირონები კონტრალატერალური red nucleus-ის მაგნოცელულარულ ნაწილში მიდის, საიდანაც შემდეგ რუბროსპინალური ტრაქტი (კორტიკოსპინალურთან ერთად, რუბროსპინალური ტრაქტიც მოტორული ტრაქტია) ეშვება. ამითი კიდურების დისტალური ნაწილების მოძრაობა კოოდრინირდება.

როგორც ვთქვი, ვერმისი ტანის კუნთების კოორდინაციას აკონტროლებს. ამისათვის, მისი გზები ძირითადად მედიალურ მოტორულ ტრაქტებთან (ანტერიორ კორტიკოსპინალური, რეტიკულოსპინალური, ტექტოსპირალური, ტექტოსპინალური ტრაქტები) მიდის. ფლოკულონოდულარული წილი (და ვერმისის ქვედა ნაწილი) კი თვალების კოორდინირებულ მოძრაობებს აკონტროლებს, ამის გამო მისი ეფერენტული გზები MLF-თან (median longitudinal fasciculus - ამ სტრუქტურაზე დავწერე რამდენიმე ხნის წინ ამ თემაში) მიდის, რომელიც ვესტიბულურ ბირთვებს ექტრაოკულარული კუნთების მოძრაობის მაკონტროლებელ ბირთვებთან აერთებს. ვერმისიდან გამომავალი გზა fastigial nuclei-ში მიდის. ამ ბირთვებიდან, uncinate fasciculus-სა (ეს ფასციკულუსი superior cerebellar peduncle-სთან ერთად მიდის) თალამუსის ვენტროლატერალური ბირთვის გავლით კორტექსს გადაეცემა, რომელიც ანტერიორ კორტიკოსპინალურ ტრაქტს გააკონტროლებს. იგივე გზის ეფერენტები მიდიან ტექტუმის არეში, საიდანაც შემდეგ ტექტოსპინალური ტრაქტი იწყება. Fastigial-იდან გამოსული ნეირონები კიდევ inferior cerebellar peduncle-სთან არსებულ juxtarestiform body-ს გავლით იფსილატერალურ რეტიკულარულ ფორმაციაში (საიდანაც რეტიკულოსპინალური ტრაქტი იწყება) და ვესტიბულარულ ბირთვებში (ვესტიბულოსპინალური ტრაქტების დასაწყისი) მიდიან. ვესტიბულარული ბირთვებიდან ნეირონები ნაწილი ცერებელუმში ბრუნდება (ამ წრეს ბალანსის ფუნქცია აქვს) ნაწილი კი MLF-ში მიდის (ექსტრაოკულარული მოძრაობა).

საინტერესოა, რომ fastigial nucleus-იდან ნეირონების მცირე ნაწილი ზურგის ტვინის ზედა ცერვიკალურ ნაწილზეც პროეცირდება (რაც გამონაკლისია, იმ წესის, რომ ცერებელუმიდან ზურგის ტვინში პირდაპირ გზა არ მიდის).

ახლა კიდევ, შეიძლება მთავარი: ცერებელუმი აკონტროლებს იფსილატერალური მხარის მოძრაობებს (ანუ, მარჯვენა ნაწილი მარჯვენა მხარეს, მარცხენა - მარცხენას) ეს განპირობებულია იმით, რომ ცერებელუმის ეფერენტები შუა ტვინში დეკუზაციას აკეთებენ. ამ ეფერენტებით საბოლოო ჯამში გაკონტროლებული კორტიკოსპინალური ტრაქტი კი ჩამოსვლისას ზურგისა და თავის ტვინის საზღვართან კიდევ ერთ დეკუზაციას აკეთებს.

ბევრი და ცოტა აბდაუბდად ვწერე, მაგრამ იმედია ძალიან მშრალად არ გამომივიდა smile.gif

Posted by: BadbadGirl 26 Feb 2010, 17:45
Thandrus
ვაიმე დიდი მადლობა.
მიუხედავად იმისა რომ დიდი იმედი მაქვს გამოცდა ჩავაბარე, მაინც ვინახავ შენ პოსტს. რა იცი რაში გამოგვადგება მე და ჩემ ამხანაგებს biggrin.gif :

Solveig
ამ წიგნით ვსწავლობდით ( იმედია არ შემეტენა რომელიმე გამოცდა, თორემ მომიწევს გადმოღება და თავიდან კითხვა biggrin.gif ) - http://www.pearson.ch/1471/9780321491800/Human-Anatomy-International-Edition.aspx
ფიზიოლოგიას ორშაბათიდან ვიწყებთ.

Posted by: idallgo 26 Feb 2010, 20:03
QUOTE
idallgo
medical student?


არა დავამთავრე, კაი ხანია.

QUOTE
ბუციკო ჩხარტიშვილი არ გასწავლიდა?


მე მასწავლიდა ფიზიოლოგიას.

Posted by: Freedoctor 28 Feb 2010, 06:44
ლოლ მეც ბუციკო მასწავლიდა ფიზიოლოგიას tongue.gif

Posted by: Ni-L 28 Feb 2010, 12:54
idallgo
Freedoctor
TSU-ში სწავლობდით?

Posted by: BadbadGirl 28 Feb 2010, 14:54
ხალხებო Lumbar puncture -ის გაკეთების შემდეგ, რამდენ ხანში შეიძლება წამოდგომა?

Posted by: Cousteau 28 Feb 2010, 17:13
QUOTE (BadbadGirl @ 28 Feb 2010, 14:54 )
ხალხებო Lumbar puncture -ის გაკეთების შემდეგ, რამდენ ხანში შეიძლება წამოდგომა?

ვიკიზე 2 საათიო

მე მეტი ვიცოდი mo.gif

Posted by: Thandrus 5 Mar 2010, 01:03
აბა ერთი ქეისი smile.gif მგონი რთული არ უნდა იყოს, მაგრამ მაინც საინტერესოა smile.gif

16 წლის გოგო (ევროპის ერთ-ერთი განვითარებადი სახელმწიფო) შემოდის ED-ში უცბათ გაჩენილი მუცლის ძლიერი ტკივილის გამო, რომელიც როგორც თვითონ ამბობს, "მეგობართან მაგრად ჩახუტების" შემდეგ გაჩნდა. ტკივილი ED-ში მოსვლამდე 2 საათის წინ გაჩნდა. ის ამ ტკივილს აღწერს როგორც მწვავეს (sharp), მუდმივს, მარჯვენა ზედა კვადრანტში ყველაზე მეტად ინტენსიურს და მარჯვენა მხარში გადამცემს. პაციენტი იხსენებს მცირე, ყრუ მუცლისმიერ დისკომფორტსა და მუცლის პროგრესულ სისავსეს ბოლო 2 თვის განმავლობაში. თუმცა მას ამ სიმპტომების გამო ექიმისთვის არ მიუმართავს. პაციენტს აღენიშნება ტანზე დიფუზური გამონაყარი, რომელიც მისი თქმით, დაახლოებით მუცლისმიერ დისკომფორტთან ერთად გაუჩნდა. მის ისტორიაში არ არის ალერგია საკვებზე და მას ED-ში მოსვლამდე ახალი ტიპის საკვები არ მიუღია. იგი უარყოფს ყოველგვარი სიცხის, გულისრევის და არანორმალური ნაწლავური მოქმედების ქონას. მისი კანის ფერი დროთა განმავლობაში არ შეცვლილა. ბოლო ორი კვირის განმავლობაში პაციენტი, მუცლის დისკომფორტის გამო, აცეტამინოფენს იღებდა, თუმცა სხვა რამ წამალი მას არ მიუღია. მას არ აქვს რაიმე ქრონიკული დაავადება ან გადატანილი ოპერაცია. საოჯახო ისტორია უმნიშვნელოა. შინაური ცხოველებიდან, მას სახლში ორი ძაღლი ჰყავს, რომელსაც ის უვლის.

ფიზიკური შემოწმებისას, პაციენტი აშკარა დისკომფორტშია. ტემპერატურა - 37.3 'C, წნევა - 110/70, პულსი - 110. მისი კანი ფერმკრთალია, სიყვითლის გარეშე, მაგრამ მას აქვს ურტიკარიული გამონაყარი, რომელიც ყველაზე მეტად ტანზე და კიდურების პროქსიმალურ ნაწილშია გამოხატული. პაციენტი კარგად ნაკვები და განვითარებულია. სუნთქვისას მკერდის მოძრაობა სიმეტრიულია. გულის ტონები ნორმალურია. რეგულარული რითმით და შუილების გარეშე. პალპაციისას, მარჯვენა ზედა კვადრანტში, ღვიძლის კიდის ზემოდან გადამფარავი მკვრივი მასა ისინჯება. მთელი ზედა მუცელი მკვრივი და მგრძნობიარეა, განსაკუთრებით მარჯვენა სუბკოსტალური არე.

ლაბორატორიული ტესტების შედეგები:

WBC - 18.6 × 103/µL (18.6 × 109/L)
40% ნეიტროფილები (0.40)
22% ლიმფოციტები (0.22)
8% მონოციტები (0.08)
21% ეოზინოფილები (0.21)

(ნორმები: WBC - 4.5-11 × 103/µL; ნეიტროფილები - 40-70%; ლიმფოციტები - 22-44%; მონოციტები - 4-11%; ეოზინოფილები - 0-8%).

ბილირუბინი (მთლიანი) - 1.98 mg/dL (ნორმა - 0.3-1.0 mg/dL).
AST - 101 U/L, ALT - 04.7 U/L.
ჰემატოკრიტი და ფირფიტები - ნორმა

მუცლის რენტგენზე ცანს არასპეციპიკური ნაწლავის გაზის სურათი (nonspecific bowel gas pattern) და პმევმოპერიტონიუმის არარსებობა. ultrasound-ი გაკეთდა ნაღვლის ბუშტის შესაძლო დაავადების გამოსაკვლევად; მასზე ჩანს დიდი ჰიპოექოგენური ზონა ღვიძლში, უსწორმასწორო კიდეებით და ღვიძლის გარშემო თავისუფალი სითხის მცირედი რაოდენობით, შემდეგ გაკეთდა მუცლის CT, რომლის სურათებსაც მოვიყვან:

http://img63.imageshack.us/i/vvvnd.jpg/

http://img717.imageshack.us/i/vvv2.jpg/

ახლა დიაგნოზი თქვენზეა smile.gif

Posted by: anarxisti 5 Mar 2010, 01:26
ექინოკოკური ცისტის გასკდომა.

ძაღლი სახლში,
ალერგიული რეაქცია, ექინოკოკური ცისტის გასკდომის სპეციფიური ნიშანი.

+/-
ბიოლოგია,
რადიოლოგიური სურათი

Posted by: Thandrus 5 Mar 2010, 01:55
anarxisti

რათქმაუნდა up.gif

მოკლედ, შანსი აღარ დატოვეთ biggrin.gif

ახლა სხვამ დადეთ და მე ვიმტვრევ თავს... smile.gif

Posted by: BadbadGirl 9 Mar 2010, 22:59
ისევ გაწუხებთ ჩემი ჩაინიკური შეკითხვებით biggrin.gif, ამჯერად კითხვები ნეიროფიზიოლოგიიდანაა smile.gif
მაშ ასე თქვენი დახმარება მჭირდება შემდეგ კითხვასთან დაკავშირებით smile.gif
რა ემართება პაციენტს რომელსაც აქვს ტრამვა მარცხენა პარიეტალურ ასოციუაციურ კორტექსზე? რა ქვია ამ სინდრომს და რა სიმპტომები ახასიათებს?

მადლობა წინასწარ smile.gif

Posted by: Solveig 9 Mar 2010, 23:15
QUOTE
მარცხენა პარიეტალურ ასოციუაციურ კორტექსზე?

http://en.wikipedia.org/wiki/Gerstmann's_syndrome

ვიკი გამოიყენე ხოლმე smile.gif)

Posted by: Thandrus 9 Mar 2010, 23:52
BadbadGirl

მარცხენაში ალბათ დომინანტურ მხარეს გულისხმობ ხომ? (ეს სჯობს რომ დააზუსტო ხოლმე, რადგანაც ადამიანების მცირე ნაწილს დომინანტური მარჯვენა მხარე აქვს). თუ დომინანტურ პარიეტალურ ასოციაციურ კორტექსზე კითხულობ, მაშინ მისი დაზიანება გერცმანის სინდრომს იწვევს. ამ სინდრომში რამდენიმე საინტერესო სიმპტომი ერთიანდება:

1) მარტივ არითმეტიკულ მოქმედებებთან დაკავშირებული პრობლემები
2) ე.წ. Right-Left Confusion - პაციენტი ვერ არჩევს სხეულის მარჯვენა და მარცხენა მხარეებს
3) ე.წ. "თითების აგნოზია" - პაციენტი ვერ ასახელებს თითების სახელებს (ცერა, საჩვენებელი, შუა და ა.შ.)
4) აგრაფია - პაციენტს არ შეუძლია დაწეროს ნაკარნახები ან თავისი ნათქვამი

არადომინანტური (უხშირესად მარჯვენა) პარიეტალური ასოციაციური კორტექსის დაზიანება იწვევს კონტრალატერალურ ჰემინეგლექტს ამ დროს პაციენტი სხეულის დაზიანების საწინააღმდეგო მხარეს საერთოდ არ აქცევს ყურადღებას (ყველაზე მძიმე შემთხევებში ტკივილიც კი უყურადღებოდ რჩება).

საინტერესო დაავადებაა ბალინტის სინდრომი, რომელსაც პოსტერიორული პარიეტალური (ზოგჯერ პარიეტო-ოქციპიტალური) კორტექსის ბილატერალური დაზიანება იწვევს. ამ სინდრომში ჩვეულებრივ, სამი სიმპტომია:

1. სიმულტანაგნოზია - ეს წამყვანი სიმპტომია. ეს ნიშნავს, რომ პაციენტს თავისი მხედველობის არის მხოლოდ ნაწილ-ნაწილ აღქმა შეუძლია. მაგალითად, ასეთ პაციენტს დიდი სურათი რომ ანახო, ის მის ნაწილ-ნაწილ აღქმას დაიწყებს. ეს ნაწილები ერთი მეორეს არ მიყვება, ამიტომაც სურათის სრული აღქმა ფაქტიურად, ვერ ხორციელდება.

2. ოპტიკური ატაქსია- ამ დროს პაციენტს უჭირს დისტანციაზე მდგარი საგნისათვის ხელის დადება თვალის კონტროლის ქვეშ. ნათხემისებრი ატაქსიებისაგან ეს იმით განსხვავდება, რომ აქ პროპრიოცეპციური და smooth მოძრაობები ნორმაშია (მაგალითად, ამ საგნის აღების შემდეგ, დახუჭული თვალებითაც, პაციენტი ამ საგანს როგორც უნდა, ისე დააატრიალებს)

3. ოკულარული აპრაქსია - პაციენტს უჭირს მისი მხედველობის არის პერიფერიაზე მყოფ საგნებზე თვალის საკადებით (თვალის სწრაფი მოძრაობებით) გაყოლება. ამისათვის მას ხშირად თავის გვერდზე მიბრუნება სჭირდება.

ამასთან ერთად, შეიძლება აღინიშნოს ჰემინეგლექტი, აფაზია და მხედველობის არის ქვედა კვადრანტში ხედვის პრობლემები.

ესაა მოკლედ...

Posted by: BadbadGirl 9 Mar 2010, 23:58
Thandrus
დიდიიიიიიი და ვებერთელა მადლობა.
რას გულისხმობენ არ ვიცი, ძველ ტესტში ამსეა მოცემული შეკითხვა smile.gif

კიდე შეგაწუხებ ჩაინიკური შეკითხვებით smile.gif

Solveig
აქ რომ მიხსნიან ბევრად მიადვილდება გაგება smile.gif
თორემ ლექციაზეც კი ახსნენს მაგრამ რა დამრჩა თავში? biggrin.gif არაფერი
* * *
შემდეგი ძლაიან მარტივი შეკითხვები:

1. დაასახელეთ NMDA რეცეპტორს ორი თვისება რომელიც სინაფსის მოქმედებას ახანგრძლივებს. ერთი თვისება ისაა რომ ეს რეცეპტორი ატარებს კალციუმის იონებს, მაგრამ მერე თვისება რომელია?

2. როგორ ანსხვავებს შიდა ყური მაღალ და დაბალ სიხშირეებს? (პასუხი უნდა იყოს ძალიან მოკლე და მე ეს მოკლე არ ვერ ვნახე )

3. ზუსტად რა სახის სენსორულ ინფორმაციებს!!! აგზავნის Utriculus და Sacculus -ტვინისაკენ? ერთი ვიცი რომ ესააა თავის მდგომარეობა. მაგრამ დანარჩენი რა სახის ინფორმაციაა?


Posted by: Freedoctor 11 Mar 2010, 20:08
Thandrus


კაი წიგნია ბლუმენფილდი smile.gif

Posted by: Thandrus 11 Mar 2010, 22:12
BadbadGirl

უკაცრავად, ახლა ვნახე მხოლოდ შენი ბოლო პოსტი smile.gif

QUOTE
1. დაასახელეთ NMDA რეცეპტორს ორი თვისება რომელიც სინაფსის მოქმედებას ახანგრძლივებს. ერთი თვისება ისაა რომ ეს რეცეპტორი ატარებს კალციუმის იონებს, მაგრამ მერე თვისება რომელია?


ჰმ... ალბათ იმაზე ამბობ, რომ არხში მაგნიუმის იონია გაჭედილი, ხო? მოკლედ გეტყვი მაშინ. გლუტამინერგული სამი ტიპის იონოტროპული რეცეპტორი არსებობს CNS-ში, AMPA, კაინატის და NMDA. AMPA და NMDA ფართოდაა CNS-ში გავრცლებული, კაინატის რეცეპტორები კი ზოგიერთ სტრუქტურაში (როგორც მახსოვს, ჰიპოკამპუსში, ნათხემში და მგონი ზურგის ტვინში), AMPA-ს და კაინატს მოსვენების პოტენციალზე შეუძლიათ არხის გახსნა, NMDA-ს არხი კი (არხის მაგნიუმის იონით ბლოკირების გამო) მხოლოდ მაშინ იხსნება, როდესაც სინაფსი დეპოლარიზირებულია. როდესაც სუმაციის პრინციპით, ნეირონი საკმარისად დეპოლარიზდება, NMDA რეცეპტორის არხი გაიხსნება და ბევრი კალციუმი შემოვა, რაც უჯრედში კალციუმ დამოკიდებულ კინაზებს გაააქტივებს, AMPA-ს და NMDA-ს რეცეპტორების გენების ექპრესიას გააძლიერებს და საბოლოო შედეგად long-term potentiation-ს მივიღებთ.

QUOTE
2. როგორ ანსხვავებს შიდა ყური მაღალ და დაბალ სიხშირეებს? (პასუხი უნდა იყოს ძალიან მოკლე და მე ეს მოკლე არ ვერ ვნახე )


მოკლედ, ეს ყური მაგარი დაღრანკული ორგანოა და ტვინს მაგრად აგიბნევს biggrin.gif ასე რომ, მხოლოდ იმას გეტყვი, რომ კოხლეაში არის ე.წ. hair cell-ები, რომლებსაც ზემოდან კიდევ ცილიები აქვთ, რომელიც მექანორეცეპტორების ფუნქციას ასრულებენ. ამ hair cell-ებს და მათ საყრდენ უჯრედებს ერთად კორტის ორგანო (Organ of Corti) ჰქვიათ. ჰოდა, ეს hair cell-ები ამ კორტის ორგანოს მთელ სიგრძეზე ტონოტროპულად არიან განლაგებული, ანუ, მაღალი სიხშირეები ააქტივებენ ოვალურ სარკმელთან ახლოს მყოფ hair cell-ებს და დაბალი სიხშირეები კი კოხლეის წვეროსთან მყოფებს. მოკლე პასუხში ალბათ ამ ტონოტროპულ განლაგებას გულისხმობენ smile.gif

QUOTE
3. ზუსტად რა სახის სენსორულ ინფორმაციებს!!! აგზავნის Utriculus და Sacculus -ტვინისაკენ? ერთი ვიცი რომ ესააა თავის მდგომარეობა. მაგრამ დანარჩენი რა სახის ინფორმაციაა?


თავის მდგომარეობას (უფრო სწორედ გადახრას) და წრფივ აჩქარებას.

Freedoctor

QUOTE
კაი წიგნია ბლუმენფილდი


yes.gif ნამდვილად, საუკეთესოა ნეიროანატომიის textbook-ებიდან რაც მინახავს. რასაც ჰქვია, გამიღვიძა ამ საგნისადმი ინტერესი (არადა ჭირივით მეჯავრებოდა სასწავლებელში Snell-ის გავლის შემდეგ gigi.gif ). იმფორმატიულად, საკმაოდ დატვირთულია და მაინც ძალიან სახალისოდ იკითხება, ვითომ არც იღლები...

Posted by: BadbadGirl 11 Mar 2010, 22:55
Thandrus
მადლობააააა :* smile.gif
ახლა გამაგებინეთ ოკუპაციურ თერაპევტს რაში სჭირდება ეს ყველაფერი და პირობას ვიძლევი სერიოზულად მოვეკიდო საგანს.

Freedoctor
სამწუხაროდ "დრო არ გვაქვს" კარგი წიგნებისათვის. მთელი ფიზიოლოგიის კურსს 5 კვირაში გავდივართ. შესაბამისად ამ საგნის სერიოზულად სწავლაზე პრეტენზია არც მაქვს, არც მსურს. მინდა გამოცდა ჩავაბარო და დავივიწყო მუდამ და მარადჟამს biggrin.gif ისე გეთანხმები რომ ძალიან საინტერესო საგანია, მაგრამ როცა გადიხარ ადამიანურ ტემპში. თუმცა ახლავე დავძებნი მაგ წიგნს ბიბლიოთეკის კატალოგში. ჰმ , ვერ ვნახულობ ზისტად რა ქვია წიგნსა და ავტორს?

კიდევ ერთი კითხვა - what is the relationship between contraction velocity and muscle tension? ქართულად მიპასუხეთ ფლიზ smile.gif მადლობა წინასწარ smile.gif



Posted by: Thandrus 11 Mar 2010, 23:50
BadbadGirl

QUOTE
ახლა გამაგებინეთ ოკუპაციურ თერაპევტს რაში სჭირდება ეს ყველაფერი და პირობას ვიძლევი სერიოზულად მოვეკიდო საგანს.


ჩემი აზრით, ოკუპაციური თერაპია საკმაოდ უკავშირდება ფსიქოლოგიას, ფსიქოლოგია კი თავის მხრივ, ფსიქიატრიასა და ნევროლოგიას biggrin.gif

QUOTE
კიდევ ერთი კითხვა - what is the relationship between contraction velocity and muscle tension?


აუ ეს ფიზიოლოგია კარგად აღარ მახსოვს sad.gif არ გადამიხედავს კარგახანია არაფრისთვის... ასე რომ, არ მინდა ჩემი პასუხით შეცდომაში შეხვიდე.

QUOTE
სამწუხაროდ "დრო არ გვაქვს" კარგი წიგნებისათვის. მთელი ფიზიოლოგიის კურსს 5 კვირაში გავდივართ.


როგორი სისტემა გაქვთ, კურაციული? ამ ხუთი კვირის განმავლობაში მხოლოდ ფიზიოლოგიას გადიხართ? ნეიროანატომიას რაც შეეხება, ყოველდღე რომ იმეცადინო, ალბათ 7-8 კვირაში შეიძლება მორჩე (ნუ, ყველაფერი 5-ზე გეცოდინებაო ვერ დაგპირდები, მაგრამ აზრზე ნამდვილად მოხვალ).

QUOTE
ი. თუმცა ახლავე დავძებნი მაგ წიგნს ბიბლიოთეკის კატალოგში. ჰმ , ვერ ვნახულობ ზისტად რა ქვია წიგნსა და ავტორს?


ნეიროანატომიის წიგნზე თუ ამბობ, Neuroanatomy Through Clinical Cases ჰქვია და ავტორი Hal Blumenfeld-ია.

Posted by: BadbadGirl 12 Mar 2010, 00:13
Thandrus
კი მხოლოდ ფიზიოლოგიას გავდივარ, და მართლა ყოველდღე ვმეცადინეობ და ძალიან ბევრს მაგრამ ალბათ ის რომ შვედური ენა არ ვიცი ასე კარგად თავის როლს თამაშობს. არ ვიცი რაში უფრო ბევრი დრო მეხარჯება ამ ყველაფრის თარგმანა/გააზრებაში თუ სწავლა/დამახსოვრებაში. თან ძალიან ჩქარი კურსია და მიჭირს მართლა.

ნეიროანატომია 7-8 კვირაში კი არა 2 კვირაში გავიარეთ. ნეიროფიზიოლოგიისთვისაც მხოლოდ ორი კვირა გვაქვს. მერე ზოგადი ფიზიოლოგია 3 კვირა და იმედია დამთავრდება ამით ჩვენი წამება.

ჩემ კითხვას კი რაც შეეხება, მართლა ვერ ვხვდები. ხანდახან მგონია რომ უკუკავშირში არიან, მაგრამ თან მგონია რომ ვცდები.

biggrin.gif რა კარგად ვარ. ნეიროანატომიის კი არა ფიზიოლოგიის რამე კარგი, სტუდენტზე ორიენტირებული წიგნი მინდოდა smile.gif
* * *
რაცხა ვიპოვე მგონია


Striated muscles develop their maximum isometric tensions at their resting lengths and develop only smaller tensions at lengths greater or less than resting length. The velocity of shortening of a muscle depends upon its load, the greater the load, the lower the velocity. Expressed another way, the greater the velocity of shortening, the smaller is the load that can be lifted by the muscle.

The force of muscle contraction can be graded by changing the frequency of discharge in active motor units and by changing the number of active motor units, resulting in forces graded between the tension of a twitch in the smallest motor unit to the tension of a fused tetanus in all motor units of the muscle.

Fast muscle is distinct from slow muscle in its faster twitch contractions, higher maximum tetanic frequencies, lower myoglobin content, lower blood flow, easier fatiguability, and innervation by larger axons that discharge intermittently at higher frequency. A motor unit contains either fast or slow muscle fibers, but a muscle is usually a mixture of both fast and slow fibers.

Posted by: Thandrus 12 Mar 2010, 00:49
BadbadGirl

რავიცი.... ბევრი წიგნია... 5-კვირიანი ფიზიოლოგიისათვის არც ვიცი რა გირჩიო ნორმალური. Guyton-ის ფიზიოლოგიის წიგნი ყველაზე ადვილია, მაგრამ დაახლოებით 3+ -ოსნის დონეზე წერია რა... ერთსადაიმავეს იმეორებს და გიღეჭავს. მაგითი შეიძლება მოასწრო. Ganong-ი არის იმფორმატიულად ძალიან დახუნძლული წიგნი, მაგრამ თუ ფიზიოლოგიასთან შეხება არ გქონია, ძალიან გაგაწვალებს. "სპოლუსლოვა" უნდა გაიგო ყველაფერი... Constanzo-ზე ლუკა ბრაზის ჰკითხე, მე არ მქონია შეხება... ისე აქებენ. დანარჩენი რავიცი, NMS-ი შეიძლება გამოგადგეს გასამეორებლად.

Posted by: BadbadGirl 12 Mar 2010, 01:04
Thandrus
დავსერჩავ ჯერ მაგ ავტორებს, ჩავიწერ წიგნის სახელებს და მერე ვნახავ ბიბლიოთეკაში, რომელიც გამიადვილდება იმას წამოვიღებ.

საბედნიეროდ http://www.sahlgrenska.gu.se/english/?languageId=100001&contentId=-1&disableRedirect=true&returnUrl=http%3A%2F%2Fwww.sahlgrenska.gu.se%2F არ არსებობს სამიანი, ოთხიანი, და ა.შ. ახალი კანონის მიხედვით სტუდენტს ორი სახის შეფასება ენიჭება, დადებითი და უარყოფითი. დადებითისათვის მეორეხარისხოვან საგნებში (ფიზიოლოგია, ანატომია და ა.შ. ) 65% უნდა იყოს სწორი პასუხი.

ისე, მთლად ავაჩეთე აქაურობა, არადა სხვაგან სად დავსვა კითხვა?

Posted by: BadbadGirl 16 Mar 2010, 23:29
მგონი ექპერიმენტს ატარებენ ჩვენზე, ან მაგრად გვეკაიფებიან.
გამაგებინეთ ერთ დღეში შესაძლებელის სისხლის მიმოქცევის და გულის ფიზიოლოგიის გავლა და მერე მოსწრება და სწავლა? იმიტომ რომ ახლა თუ არ მოვასწარით, ხვალ უკვე სხვა რამე იქნება სასწავლი და სულ "ჩავფლავდებით"

Posted by: Thandrus 16 Mar 2010, 23:57
BadbadGirl

ჰო ეგ ცუდია... რომელი წიგნით გადაწყვიტე მეცადინეობა?

Posted by: BadbadGirl 17 Mar 2010, 00:57
Thandrus
შვედური წიგნია, მერე კიდე კომპენდიუმებს გვირიგებენ და კიდევ დამატებითი ლიტერატურის სახით გვირჩიეს Guyton A, Hill J. Textbook of Human Physiology, 11:th edition. ეს წიგნი კარგია მაგრამ მხოლოდ მისი ზომა გააგიჟებს ნორმალურ ადამიანს biggrin.gif biggrin.gif

Posted by: vano_t 17 Mar 2010, 08:33
ერთ generic case-ს დავდებ. კლინიკური თვალსაზრისით მნიშვნელოვანია და უფრო clinical skills მოითხოვს, ვიდრე რაღაც არაჩვეულებრივი დიაგნოზების გაკეთების უნარს.

65 წლის ავადმყოფი, წარსულში მწეველი, მოდის კლინიკაში ჩვეულებრივი ვიზიტით (follow-up ვიზიტით). ავადმყოფს აქვს end-stage COPD (ფილტვების ქრონიკული ობსტრუქციული დაავადების ბოლო სტადია). ავადმყოფის წამლებია ინჰალაციური კორტიკოსტეროიდები, იპრატროპიუმი, ალბუტეროლი, ფოსამაქსი. ავადმყოფი ამბობს, რომ მისი ბაზისური მდგომარეობა (baseline condition) არ შეცვლილა. აქვს ხველა ნახველით. არც ხველის სიხშირე არ შეცვლილა და არც ხასითი. ნახველიც იგივე ფერისაა. არ აქვს სიცხე. სუნთქვის უკმარისობის ხარისხი არ შეცვლილა. გამოკვლევისას ტემპერატურა ქონდა 36,5 გრადუსი. პულსი 120 და რეგულარული, პულსური ოქსიმეტრია აჩვენებს 70%-ს ჟანგბადის გარეშე, წნევა აქვს 90/60. გამოკვლევისას სუნთავა დაქვეითებულია ორივე მხარეს, მაგრამ არ არის შეცვლილი.

რა უნდა გაუკეთდეს ავადმყოფს კლინიკაში და რაზე იფიქრებთ? (ავადმყოფი თქვენია, რომელსაც დიდი ხანი იცნობთ)

Posted by: LUKA-BRAZI 17 Mar 2010, 13:51
vano_t
ფილტვისმიერი გული? გულის ექოსკოპიას გავუკეთებდი.. EF-ს გავიგებდით....

QUOTE
(ავადმყოფი თქვენია, რომელსაც დიდი ხანი იცნობთ)

აქ რაღაც clue იმალება ჰო ვანო? biggrin.gif

Posted by: utilizatori 17 Mar 2010, 14:21
QUOTE
პულსური ოქსიმეტრია აჩვენებს 70%-ს ჟანგბადის გარეშე,


და ეს პაციენტი გონებაზეა?
ჟანგაბდი უნდა მისცე პირველ რიგში და თუ არ აიწია სატურაციამ უნდა დააინტუბირო კიდეც
ეს სადმე წაიკითხე თუ რეალური შემთხვევაა
სატურაცია 70 % და გონებაზე იყო პაციენტი?

ვაბშე რენტგენიც გადასაღებია
მიუხედავად იმისა რომ სუნთქვა ორივე მხარეს ტარდება
მაინც შესაძლოა პნევმოთორაქსი იყოს

QUOTE
(ავადმყოფი თქვენია, რომელსაც დიდი ხანი იცნობთ)

ეგრე რომ იყოს მეცოდინებოდა რა მექნა smile.gif

Posted by: vano_t 17 Mar 2010, 21:35
utilizatori
QUOTE
და ეს პაციენტი გონებაზეა?
ავადმყოფი ამბობს, რომ მისი ბაზისური მდგომარეობა (baseline condition) არ შეცვლილა.

QUOTE
ჟანგაბდი უნდა მისცე პირველ რიგში და თუ არ აიწია სატურაციამ უნდა დააინტუბირო კიდეც
ეს სადმე წაიკითხე თუ რეალური შემთხვევაა
სატურაცია 70 % და გონებაზე იყო პაციენტი?
რეალური შემთხვევებია. არ ჭირდება არაფერი ინტუბაცია.

QUOTE
ვაბშე რენტგენიც გადასაღებია
მიუხედავად იმისა რომ სუნთქვა ორივე მხარეს ტარდება
მაინც შესაძლოა პნევმოთორაქსი იყოს
ავადმყოფი ამბობს, რომ მისი ბაზისური მდგომარეობა (baseline condition) არ შეცვლილა.

აქვს ხველა ნახველით. არც ხველის სიხშირე არ შეცვლილა და არც ხასითი. ნახველიც იგივე ფერისაა.

არ აქვს სიცხე.

სუნთქვის უკმარისობის ხარისხი არ შეცვლილა.

ავადმყოფის ქრონიკული მდგომარეობა არ შეცვლილა.

LUKA-BRAZI
QUOTE
ფილტვისმიერი გული? გულის ექოსკოპიას გავუკეთებდი.. EF-ს გავიგებდით....
წაიკითხე რაც უტილიზატორს დავუწერე ლუკა.

QUOTE
აქ რაღაც clue იმალება ჰო ვანო?
clue ის არის, რომ იცი ავადმყოფი. იცი, რომ მდგომარეობა არ გამწვავებულა; გაქვს მისი chart (ანკეტა ჩვენებურად მემგონი) და რასაც გინდა ნახავ შიგნით smile.gif

Posted by: Thandrus 17 Mar 2010, 21:41
vano_t

მწეველია? ისე, ორივე ფილტვი აქვს ამ კაცს? tongue.gif ან ფილტვის სტრუქტურული პრობლემები?

Posted by: vano_t 17 Mar 2010, 21:53
Thandrus
QUOTE
მწეველია? ისე, ორივე ფილტვი აქვს ამ კაცს? tongue.gif ან ფილტვის სტრუქტურული პრობლემები?

ყოფილი მწეველია, ორივე ფილტვი აქვს და აქვს end-stage COPD (ფილტვების ქრონიკული ობსტრუქციული დაავადების ბოლო სტადია).

Posted by: Cousteau 17 Mar 2010, 22:40
QUOTE
როგორ ანსხვავებს შიდა ყური მაღალ და დაბალ სიხშირეებს? (პასუხი უნდა იყოს ძალიან მოკლე და მე ეს მოკლე არ ვერ ვნახე )

ძალიან მოკლედ: ბაზილაურლი მემბრანის სხვადასხვა ნაწილები ირხევა სხვადასხვა სიხშირეებზე

+ მგონი (ეს ზუსტად არ მახსოვს,შეიძლება ვცდები) Hair Cell-ები, სპეციფიურებია სხვადასხვა სიხშირისათვის

QUOTE
5-კვირიანი ფიზიოლოგიისათვის არც ვიცი რა გირჩიო ნორმალური


Costanzo's Physiology, 400 გვერდია, ცოტა წერია მაგრამ საკმარისი (მგონი)

Guyton-ს თუ მოასწრებ (1000 გვერდი) ხო მთლად კარგი : D

PS გაიტონი უფრო ადვილი წასაკითხია yes.gif

Posted by: Cinderella88 18 Mar 2010, 21:31
ძალიან მაინტერესებს რა დაავადებაა, რომელიც აღწერილია სახარებაში როგორც დემონებით შეპყრობილი.
კაცი როა შიშველი რო დარბის ტყეში და ბალახს ჭამს, და ფლობს უსაზღვრო ძალას,
რკინის ჯაჭვებსაც კი რომ გლეჯს?

ვიცი მორწმუნენი ხსნიან რომ დემონები ყავს ჩასახლებულიო,
მაგრამ მეცნიერული და მედიცინურო ნორმალური ახსნა არ არსებობს?

დავიჯერო ყველა ექიმსაც სჯერა რომ ადამიანს დემონმები უსახლდებიან?

იმიტომ მაინტერესებს რომ ზუსტად მსგავსი სიმპტომები ქონდა ჩემი მეზობლის ბიძას თურმე.
დროდადრო შეტევები ემართებოდა, ტანზე იხდიდა და ტყეში გარბოდა, სადაც ბალახს ჭამდა.
თან ამ შეტევების დროს ფლობდა უსაზღვრო ძალას ცალი ხელით შეეძლო აეწია 200 კილოიანი ქვა, და დამესხვრია რკინის ჯაჭვებიო.
ეს დაახლოებით 20 წლის წინ იყო, მაშინ ექიმებმა ვერაფერი გაუგეს,
და ოჯახმა ჩათვალა დემონებით არის შეპყრობილიო, და მკითხავები დაჰყავდათ თურმე.

მაინტერესებს დღეს დღეობით თუ აქვს მედიცინას ამაზე პასუხი?
დაადგინდა რა დაავადებაა? თუ კვლავ დემონებს აბრალებთ?

რასთან გვაქვს საქმე? რა დაავადება ეს?

rolleyes.gif

Posted by: BadbadGirl 18 Mar 2010, 21:49
QUOTE
COPD (ფილტვების ქრონიკული ობსტრუქციული დაავადებია

დღეს "აგვიხსნეს" რესპირაციაში. ვერაფერი გავიგე biggrin.gif biggrin.gif

Posted by: lgogokhia 19 Mar 2010, 17:14
Cinderella88
QUOTE
რასთან გვაქვს საქმე? რა დაავადება ეს?

შიზოფრენია..

Posted by: Blind_Torture_Kill 19 Mar 2010, 23:34
QUOTE
რა უნდა გაუკეთდეს ავადმყოფს კლინიკაში და რაზე იფიქრებთ? (ავადმყოფი თქვენია, რომელსაც დიდი ხანი იცნობთ)


ტრანსპლანტაცია ყველაზე კარგი გამოსავალი

utilizatori
QUOTE
სატურაცია 70 % და გონებაზე იყო პაციენტი?


საკომპენსაციოდ პოლიციტემია ექნებოდა და მაგიტო იქნებოდა ესე

Posted by: vano_t 19 Mar 2010, 23:46
მოკლედ რომ ვთქვათ, არაფრის გაკეთება არ უნდა ამ ავდმყოფს. ავადმყოფს აქვს ქრონიკული კომპენსირებული მგომარეობა და ამას ვერაფერ უშველი. ასეთი ავადმყოფები home O2-ზე არიან. ხანდახან იძრობენ ჟანგბადს როცა გარეთ გადიან. ტაქიკარდიაც და შედარებით დაბალი წნევაც ამ ავადმყოფში არაფერს წარმოადგენს განსაკუთრებულს. COPD-ს ბოლო სტადიის დროს ხშირია დაბალი ჟანგბადის კონცენტრაცია და ორგანიზმი ეგუება ამას ნელნელა (ნელნელა ვითარდება კონცენტრაციის ვარდნა).

QUOTE
ტრანსპლანტაცია ყველაზე კარგი გამოსავალი
რამოდენიმე პაციენტი მყავს მძიმე ქრონიკული COPD-თ, რომელთაც მისცეს ეგ option და არცერთმა არ აირჩია. საქმე იმაშია, რომ ფილტვების გადანერგვის შემდეგ საშინელი შეზღუდვები აქვთ ამ ავადმყოფებს და quality of life არის ძალიან დაბალი. გარდა ამისა, გადანერგვის შემდეგ გართულებებიც დიდია და სიცოცხლის ხანგრძლივობაც დაბალი.

Posted by: Blind_Torture_Kill 19 Mar 2010, 23:47
QUOTE
პულსი 120 და რეგულარული, პულსური ოქსიმეტრია აჩვენებს 70%-ს ჟანგბადის გარეშე, წნევა აქვს 90/60


წნევა ესე რატო აქვს ?

Posted by: vano_t 20 Mar 2010, 01:00
Blind_Torture_Kill
QUOTE
QUOTE
პულსი 120 და რეგულარული, პულსური ოქსიმეტრია აჩვენებს 70%-ს ჟანგბადის გარეშე, წნევა აქვს 90/60


წნევა ესე რატო აქვს ?

მასეთ წნევებს ატარებს მთელი ცხოვრება. არ აწუხებს. ამიტომ ავღნიშნე, რომ ავადმყოფს დიდი ხანი ვიცნობთ. chart-ში რომ ჩავხედოთ, 10 წლის წინაც დაბალი წნევები ქონდა, რაც მისთვის ნორმალურია.

Posted by: utilizatori 20 Mar 2010, 15:40
vano_t

აბა თუ არაფერი შეცვლილა
არც წნევა, არც პულის და არც ზოგადი მდგომარეობა
ჩავიწერ მონაცემებს და გავუშვებ სახლში
მხარზე მოვუთათუნებ კიდევაც biggrin.gif

Posted by: BadbadGirl 21 Mar 2010, 16:04
ჩემი მორიგი ჩაინიკური შეკითხვვა smile.gif წინასწარ მადლობა პასუხისათვის smile.gif

ამოსუნთქვისას ფილტვები სრულიად რომ თავისუფლდებოდეს ჟანგბადისაგან, რა ეფექტს მივირებდით და რატომ?

Posted by: person 21 Mar 2010, 22:32
BadbadGirl
QUOTE
ამოსუნთქვისას ფილტვები სრულიად რომ თავისუფლდებოდეს ჟანგბადისაგან, რა ეფექტს მივირებდით და რატომ?

სრულად რო თავისუფლდებოდეს ალვეოლების ელასტიურობა ძალიან მაღალი იქნებოდა, ყოველ ჩასუნთქვას დიდი ძალა დაჭირდებოდა და საბოლოოდ დისტრეს სინდრომს მივიღებდით, იმას რასაც ჩვილებში ვხვდებით .
აქვე მეც ვიკითხავ, ან ვითხოვ რა ვიცი. თუ ვინმემ იცით მექანიზმი, რითიც ინსულინის რაოდენობა SHBG -ზე მოქმედებს. უფრო ამის გრაფიკული გამოსახულება მაინტერესებს და ვინმეს თუ გაქვთ მომაწოდეთ გთხოვთ.

Posted by: anarxisti 22 Mar 2010, 19:07
პლაცებოს თემაში დავდე და არავინ მიპასუხა..
ამ თემას ხო ჰქვია "რა არის ეს?" smile.gif
ხოდა იყოს აქაც..
არადა, საკმაოდ ცნობილი და ეფექტური ყოფილა...ერთ დროს. დიდი ხალხი იყენებდა თურმე.
Aqua simplex (60 g) - Illa repetita (40 g) - Eadem stillata (10 g) - protoxyde d'hydrogène (0,30 g) - Nil aliud (1,25 g)
5 წვეთი ჭამის წინ.


ხოდა რა წამალია ასეთი და რისთვის გამოიყენებოდა??

Posted by: LUKA-BRAZI 22 Mar 2010, 20:03
anarxisti
biggrin.gif

სერ ჯოზეფ ოლიფი, ნაპოლეონ მესამის ექიმი.. ამ "წამალს" სადაც მხოლოდ წყალია და სხვა არაფერი
Subsidence Aqua (water fountain)
Illa repetita (the same repeated)
Same dripping (the same distilled)
Nitrous Hydrogen ( H 2O : chemical formula of water )
Nil aliud (nothing else)
იყენებდა იმპოტენციის და ფრიგიდულობის სამკურნალოდ smile.gif)

Posted by: Thandrus 22 Mar 2010, 20:10
LUKA-BRAZI

QUOTE
იყენებდა იმპოტენციის და ფრიგიდულობის სამკურნალოდ )


მერე, შედეგები? tongue.gif

Posted by: anarxisti 22 Mar 2010, 20:10
LUKA-BRAZI
up.gif
ისე, მოგეცადა ცოტა biggrin.gif

Thandrus
QUOTE
მერე, შედეგები?


შედეგები... ფანტასტიური ყოფილა...
მარა, როდესაც ერთ ლათინურის მცოდნეს უნახავს რეცეპტი... ეგრევე დაითხოვეს... ტყუილისათვის.

Posted by: vano_t 22 Mar 2010, 21:22
anarxisti
QUOTE
შედეგები... ფანტასტიური ყოფილა...
მარა, როდესაც ერთ ლათინურის მცოდნეს უნახავს რეცეპტი... ეგრევე დაითხოვეს... ტყუილისათვის.

დაითხოვეს და დაუვარდათ კიდეც ხო? (ანუ, დაითხოვეს და დაეთხოვათ) biggrin.gif

Posted by: Solveig 22 Mar 2010, 21:35
QUOTE
Aqua simplex

QUOTE
protoxyde d'hydrogène

biggrin.gif მაგარი იყო.

დაახლოებით მაგ ყალიბის ტყუილით ორმა კაცმა ამ რამდენიმე წლის წინ დიჰიდროგენმონოქსიდის "სკანდალი" მოაწყო, რომელიც კულუარული ინფორმაციით, საქართველოში მართლა დაიჯერეს biggrin.gif

Posted by: BadbadGirl 22 Mar 2010, 22:29
ხალხებო, მოკლეთ (3-4 პუნქტი) შეგიძლიათ მითხათ რა არის B-cell და T-cell ფუნქცია? smile.gif

person
:* გაიხარე smile.gif


ერთი კარგი ამბავი, ნეიროანატომიის გამოცდა (სამი კვირის უკან რომ მქონდა ისა biggrin.gif ) ძალიან კარგად ჩავაბარე, ტვინი რომ ამოგჭამეთ ჩემი შტერული კითხვებით, როგორც ჩანს ძალიანაც შედეგიანი გამოდგა smile.gif ახლაც ნება მიბოძეთ კიდევ შეგაწუხოთ ჩაინიკურ შეკითხვებით smile.gif

Posted by: Solveig 22 Mar 2010, 22:40
BadbadGirl
QUOTE
B-cell და T-cell ფუნქცია?

T-cell-ის არის სიმსივნური და ვირუსით ინფიცირებული უჯრედების მოკვლა (cytotoxic CD8+ T cells), B უჯრედებისა და მაკროფაგების აქტივაცია (Helper CD4+ T cells), სუპრესორული მოქმედება (CD4+ Regulatory T cells).

მაგრამ ეს ძალიან უტრირებულია...ზოგადად, T cell immunity არის იმუნური სისტემის მთავარი მამოძრავებელი ძალა...

B-cell-ის არის ანტისხეულების წარმოება (antibody production)

ისე, უმჯობესია, ცალკე თემა გახსნა. yes.gif ეს თემა სხვა მიზნისათვისაა.

ისედაც აპირებდნენ ლუკა და მეგობრები მსგავსი თეორიული განათლებისათვის განკუთვნილი თემის გახსნას.

Posted by: BadbadGirl 22 Mar 2010, 23:07
Solveig
მეც მაგას ვამბობ რომ სხვა მიზნისთვისაა, მაგრამ სხვაც რომ არაა?
იმედია გახსნიან ლუკა და მეგობრები საჭირო თემას.

მადლობა smile.gif

რაც ჩამომიწერე T helper cell ფუქნქციებია, და ზოგადად ყველანაირ T -cell ზე ვრცელდება ეგ ფუნქციები?

Posted by: Solveig 22 Mar 2010, 23:30
BadbadGirl
QUOTE
რაც ჩამომიწერე T helper cell ფუქნქციებია, და ზოგადად ყველანაირ T -cell ზე ვრცელდება ეგ ფუნქციები?

აუ, მოკლედ, როგორ გითხრა, რა sad.gif(

ჰელპერები ადრე ზოგადად ერქვა CD4+, მაგრამ ბოლო 20 წელიწადში აღმოჩნდა, რომ ამ უკანასკნელების რამდენიმე ქვეტიპია ციტოკინების პროფილის და ფუნქციის მიხედვით და ეს ფუნქცია სულაც არ გულისხმობს ჰეპლერობას (ანუ. იგივე B უჯრედების დახმარებას ანტისხეულების წარმოებაში). კერძოდ, აღწერილია :

Th1-მონაწილეობს ციტოტოქსიკურ რეაქციებში (ააქტიურებს მაკროფაგებს, რომლებიც კლავენ)
Th2 მონაწილეობს B უჯრედების აქტივაციაში, ალერგიულ და ანტიპარაზიტულ რეაქციებში
Tregs-რეგულატორული T უჯრედები, რომლებიც არიან CD4+CD25+Foxp3+ ან Foxp3-,,,და თრგუნავენ იმუნურ პასუხს ან უჯრედული კონტაქტით (Foxp3+) ან ციტოკინების სეკრეციით (Foxp3+/-, ანუ Tr1/Tr3 , რომლებიც იყენებენ IL-10 ან TGF beta).

ამის გარდა აღწერეს კიდევ ეგრეთწოდებული Th17 უჯრედები, რომლებიც აუტოიმუნური დაავადებების დროსაა ინტენსიურად წარმოდგენილი...

და ყველა ამ ქვეტიპს გააჩნია შესაბამისი ბირთვული ფაქტორი, ანუ გენი, რომლის აქტივაცია იწვევს წინამორბედი CD4+T უჯრედის დიფერენცირებას ამა თუ იმ მიმართულებით..რატომ მიეწოდება ინფორმაცია კონკრეტულ ბირთვულ ფაქტორს და არა მეორეს, ეს ჯერ ბოლომდე არაა ცნობილი, დიდი როლი უნდა ჰქონდეს ციტოკინებით მიწოდებულ სიგნალს, განსაკუთრებით ანტიგენის წარმდგენი უჯრედების მიერ გამოყოფილ ციტოკინებს...ასევე, აღწერილია პლასტიკურობაც, ანუ ერთი ტიპის CD4+T უჯრედის გარდაქმნა მეორედ.

Posted by: BadbadGirl 22 Mar 2010, 23:32
Solveig
აუ რა მაგარიააა.
მაგ ყველაფერს დავიმახსოვრებ აკრგად და დავწერ კითხვა თუ მოვიდა biggrin.gif
ძალიან გაუკვირდებ მასწავლებელს biggrin.gif

Posted by: BadbadGirl 30 Mar 2010, 15:48
მოვედიიიი ჩემი ჩაინუკური შეკითხვით smile.gif
შარდის ანალიზში, კრეატინინის დონე რისი მაჩვენებელია?
* * *
უფ, გადავირიე რა.
კაცო ახლა კი ვხვები რა შუაშია ეს კრეატინინი, მაგრამ ტვინის განძრევა რომ მეზარება, მაგი რისი ნიშანია ნეტა?

Posted by: Thandrus 14 Apr 2010, 01:49
აბა ერთ ქრესტომათიულ ქეისს დავდებ... მგონი გამოცნობა არ უნდა გაგიჭირდეთ smile.gif

15 წლის გოგონა მოიყვანეს საავადმყოფოში ფარინგიტის და ვაგინიტის 2 დღის ისტორიით, რომელიც ასოცირებული იყო ღებინებასთან და წყლოვან ფაღარათთან. მას ჰქონდა სიცხე, დაბალი წნევა და დიფუზური ერითემული გამონაყარი მთელს ტანზე. ლაბორატორიულმა ტესტებმა აჩვენეს აციდოზი, ოლიგურია, დისემინირებული ინტრავასკულარული კოაგულაცია და მძიმე თრომბოციტოპენია. რენტგენნზე გამოჩნდა ფილზებში ორმხრივი ინფილტრატები. ის გადაიყვანეს ICU-ში სადაც იგი დასტაბილიზირდა და ნელ-ნელა 17 დღის განმავლობაში მისი მდგომარეობა გამოსწორდა. ICU-ში ყოფნის მესამე დღეს მას სახის, ტანის და კიდურების კანის აქერცვლა დაეწყო, 14-ე დღისათვის კი ხელის და ფეხისგულების კანიც აიქერცლა.

დასვით ზუსტი დიაგნოზი და დაწერეთ მისი ეტიოლოგია!

Posted by: mtvareuli 14 Apr 2010, 01:54
Thandrus

ტოქსიური შოკის სინდრომი mo.gif


Posted by: Thandrus 14 Apr 2010, 03:10
mtvareuli

აჰააა chups.gif

ახლა ბარემ გამომწვევიც თქვი ზუსტად, ვინაიდან ეს მნიშვნელოვანია smile.gif

Posted by: basa-ttt 14 Apr 2010, 11:13
Thandrus
ქუნთრუშა ან სტრეპტოკოკით გამოწვეული სეფსისი?

აქერცვლა ქუნთრუშასია

Posted by: person 14 Apr 2010, 12:11
Thandrus
საფენების ხმარების გამო
ოქროსფერი სტაფილოკოკებით გამოწვეული

Posted by: Thandrus 14 Apr 2010, 19:19
basa-ttt

QUOTE
ქუნთრუშა ან სტრეპტოკოკით გამოწვეული სეფსისი?

აქერცვლა ქუნთრუშასია


სტრეპტოკოკულმა ინფექციამ (ტოკსიკურმა შოკმა) შეიძლება მსგავსი სიმპტომები გამოიწვიოს, მაგრამ რაღაც განსხვავებები არის - კერძოდ, სტრეპტოკოკული ტოქსიკური შოკის დროს ყოველთვის არის სისხლში ბაქტერემია და ყველაზე ხშირად ვითარდება ღრმა კანქვეშა ინფექციების (მაგ. მანეკროზირებელი ფასციიტის) შემდეგ.

person

QUOTE
საფენების ხმარების გამო ოქროსფერი სტაფილოკოკებით გამოწვეული


აბსოლუტურად მართალია! up.gif

ამერიკაში, 80-90-იან წლებში ტოკსიკური შოკის სინდრომი სწორედ მენსტრუაციის დროს ჰიპერშემწოვი ტამპონების ხმარებასთან იყო კავშირში. ტამპონებში S. aureus-ი კარგად მრავლდება და ტოქსიკური შოკის ტოქსინს გამოყოფს. ამიტომ, თუ ადამიანს ვაგინიტი აქვს და ასეთი სიმპტომები - გამომწვევი სტაფილოკოკია. მარტივი ქეისი იყო, მაგრამ ამ ნიუანსზე მინდოდა, რომ ყურადღება მიგექციათ smile.gif



Posted by: mtvareuli 14 Apr 2010, 23:09
Thandrus
QUOTE
ამერიკაში, 80-90-იან წლებში ტოკსიკური შოკის სინდრომი სწორედ მენსტრუაციის დროს ჰიპერშემწოვი ტამპონების ხმარებასთან იყო კავშირში. ტამპონებში S. aureus-ი კარგად მრავლდება და ტოქსიკური შოკის ტოქსინს გამოყოფს.

ახლა რამე შეიცვალა ტამპონებში თუ ისევ არსებობს ეგ საფრთხე?


Posted by: Thandrus 14 Apr 2010, 23:32
mtvareuli

როგორც ვიცი, როდესაც დაადგინეს, რომ ტოქსიკური შოკის ბევრი შემთხვევა ჰიპერმააბსორბილებელი ტამპონების გამოყენებას უკავშირდებოდა, ასეთი ტამპონები ამოიღეს გაყიდვიდან და უფრო დაბალი აბსობციის მქონეთი ჩაანაცვლეს. თან როგორც ამბობენ ამერიკაში შემცირდა ყველა სახის ტამპონების მოხმარება და ამან დაავადების სიხშირის კლება გამოიწვია. ამჟამად ამერიკაში წლის განმავლობაში მხოლოდ 150-მდე შემთხვევა გვხვდება.

Posted by: MAIN KAMPF 17 Apr 2010, 12:49
http://www.radikal.ru

რა ხდება აქ ვთქვათ

Posted by: donvaso 17 Apr 2010, 17:08
MAIN KAMPF
გული აქვს მარჯვნივ? spy.gif
თუ აორტის ანევრიზმაა? spy.gif
თუ არცერთი cry.gif

Posted by: Tornike Alashvili 17 Apr 2010, 18:55
ვერტიკალური მდებარეობაა გულის
ფოლაქებია -ალბათ ჩაკირული ლიმფური კვანძები
პათოლოგია? არ ვიცი , ალბათ გადატანილი ტუბერკულიოზი
თუ საერთოდ ფოლაქები არაა გახსნილი საროჩკიდან

Posted by: Blind_Torture_Kill 17 Apr 2010, 23:27
ლავიწის ძვლებს შორის ასიმეტრია
ხერხემლის სვეტიც დეფორმირებული ჩანს
ოვალური ფორმის სტრუქტურებიდან ერთ-ერთი ღილს გავს

ტრამვაა ?



Posted by: Thandrus 17 Apr 2010, 23:38
Blind_Torture_Kill

QUOTE
ტრამვაა ?


რენტგენის ცნობა დიდად არ მეხერხება, მაგრამ მეც ტრავმა მგონია.

Posted by: MAIN KAMPF 18 Apr 2010, 00:35
QUOTE

თუ საერთოდ ფოლაქები არაა გახსნილი საროჩკიდან

ღილები ნამდვილად არის, მაგაზე ყურადღებას არ ვითხოვდი
biggrin.gif ეს არის ტიპიური სურათი ტუბერკულოზის yes.gif

Posted by: Thandrus 18 Apr 2010, 01:01
MAIN KAMPF

აჰ... და გაღუნული ხერხემალი ვითომ პოტის დაავადების გამო? user.gif

მაგრამ ფილტვებზე რომ ვერ ვხედავ ვერაფერს?

Posted by: MAIN KAMPF 18 Apr 2010, 12:23
QUOTE
აჰ... და გაღუნული ხერხემალი ვითომ პოტის დაავადების გამო?

ავადმყოფი დგას არასწორად. ამ ავადმყოფს აქვს AIDS-clinical stage 4, TB. უმრავლესობას შიდსიანებს აქვთ TB (ამ ქვეყანაში სადაც მე ვიმყოფები სამხ.აფრიკა ყველაზე გავრცელებული დაავადებაა)
yes.gif

Posted by: Thandrus 18 Apr 2010, 12:37
MAIN KAMPF

შეგიძლია მითხრა, რა არის ამ სურათზე ტუბერკულოზისთვის სპეციფიკური?

QUOTE
სამხ. აფრიკა


ვაჰ, სამხრეთ აფრიკაში ხარ? biggrin.gif მსოფლიო ჩემპიონატზე დასწრებას აპირებ? smile.gif

Posted by: LUKA-BRAZI 18 Apr 2010, 13:13
QUOTE
ვიმყოფები სამხ.აფრიკა

ვაა, ბანანები, ანანასები, ოკეანე, პლაჟები, პალმები.... smile.gif

Posted by: Cousteau 18 Apr 2010, 15:20
QUOTE (MAIN KAMPF @ 18 Apr 2010, 12:23 )
ამ ქვეყანაში სადაც მე ვიმყოფები სამხ.აფრიკა

მაგათ იციან ფიურერი რო გყავს ავატარზე? gigi.gif

Posted by: MAIN KAMPF 18 Apr 2010, 17:31
QUOTE
შეგიძლია მითხრა, რა არის ამ სურათზე ტუბერკულოზისთვის სპეციფიკური?

ე.წ. consolidation, შუასაყარში გადიდებული lymph nodes და კავერნოზული უბანი, მე მონიშვნა არ ვიცი როგორ კეთდება სურათზე.
QUOTE
მაგათ იციან ფიურერი რო გყავს ავატარზე

biggrin.gif მანდელაზე მაინც არ გავცვლი

Posted by: LUKA-BRAZI 18 Apr 2010, 19:03
MAIN KAMPF
ჩემი მონიშვნა სწორია?

Posted by: MAIN KAMPF 19 Apr 2010, 17:44
ახალი ქეისი აფრიკიდან, სურათი მობილურით არის გადაღებული თუმცა წინასთან შედარებით ადვილია. გარდა ამისა ამის შემდეგ უკვე დავდებ რადიოლოგის მიერ დაწერილ დასკვნას. როგორც წინა შემთხვევა პაციენტი შიდსიანია.
http://www.radikal.ru

Posted by: donvaso 19 Apr 2010, 18:21
MAIN KAMPF
მარჯვნივ აქვს ლიქვორი, გულიც ცოტა დიდია... მარტივად ესაა.... biggrin.gif

ხო, თან ქალია... biggrin.gif biggrin.gif

Posted by: MAIN KAMPF 19 Apr 2010, 18:44
biggrin.gif biggrin.gif სხვები რას იტყვიან მოგვიანებით დავდებ პასუხს

Posted by: donvaso 19 Apr 2010, 19:14
MAIN KAMPF
ფესვები გაფართოებულია, ბრონქოექტაზებიც არის მგონი, ოდნავ სქოლიოზიც .... ეს ცოტა რთულად.... biggrin.gif biggrin.gif biggrin.gif

Posted by: MAIN KAMPF 19 Apr 2010, 23:26
donvaso
yes.gif სწორად ახსენი
http://www.radikal.ru
streaky opacitie თუ შეგიძლია მონიშნო

Posted by: Thandrus 20 Apr 2010, 15:18
MAIN KAMPF

რა მაინტერებს... მართლა 10-დან 3 შიდსიანია სამხრეთ აფრიკაში? user.gif

Posted by: MAIN KAMPF 20 Apr 2010, 16:52
QUOTE
რა მაინტერებს... მართლა 10-დან 3 შიდსიანია სამხრეთ აფრიკაში?

სამიდან ერთი რათქმა უნდა შავ მოსახლეობას ეხება

vik.gif 15 მილიონი დაავადებული სრული მოსახლეობა 55 მილიონი. boli.gif

Posted by: Blind_Torture_Kill 20 Apr 2010, 23:58
რამდენი ხანია აქ არაფერი არ დამიდია no.gif

აბა ეს რა არის

Posted by: Thandrus 21 Apr 2010, 09:04
Blind_Torture_Kill

არნოლდ-კიარის ტიპის (ტიპი I) მალფორმაციაა, რამაც ცოტა ქვემოთ სირინგომიელია გამოიწვია.

Posted by: Blind_Torture_Kill 21 Apr 2010, 11:49
Thandrus

QUOTE
არნოლდ-კიარის ტიპის (ტიპი I) მალფორმაციაა, რამაც ცოტა ქვემოთ სირინგომიელია გამოიწვია.



კარგია

Posted by: Thandrus 21 Apr 2010, 16:42
აბა მაშინ მეც გავაგრძელებ თემას... რისი ტიპიური სურათია ეს?

Posted by: Blind_Torture_Kill 21 Apr 2010, 19:36
Thandrus

პარკუჭები გაფართოებულია ?

Posted by: Thandrus 21 Apr 2010, 19:56
Blind_Torture_Kill

კი, გადიდებულია

Posted by: Blind_Torture_Kill 21 Apr 2010, 20:15

QUOTE
კი, გადიდებულია



ჰიდროცეფალია

Posted by: Thandrus 21 Apr 2010, 20:18
Blind_Torture_Kill

no.gif

Posted by: Blind_Torture_Kill 21 Apr 2010, 20:19

.....................................................................
შეგიძლია ტვინის სურათი დადო ?

Posted by: Thandrus 21 Apr 2010, 20:30
Blind_Torture_Kill

არა. ეს ქრესტომათიული, ტიპიური სურათია. წარმოიდგინე, რას შეეძლო ლატერალური პარკუჭების ასე გადიდება?

იგივე კვეთაში ნორმალური თავის ტვინის სურათი უნდა გამოიყურებოდეს ასე (იხ. სურათი):

Posted by: Blind_Torture_Kill 21 Apr 2010, 20:33
რახან ტვინის სურათი არ გვაქ
ვვარაუდობ რომ ტვინის მასის შემცირებას

Posted by: Thandrus 21 Apr 2010, 20:38
Blind_Torture_Kill

ნუ, ტვინის მასა თავისთავად შემცირებული იქნება. მაგრამ უფრო ზუსტი დიაგნოზის დასმაც შეიძლება. ნახე, ლატერალური პარკუჭების ლატერალურად ნორმაში რომ გამობურცული რაღაცაა, ის ამ ავადმყოფის სურათში საერთოდ არაა (ამიტომ ჩანს გადიდებული და მრგვალი პარკუჭი). ჰოდა ახლა გაიხსენე, მანდ რა სტრუქტურა უნდა იყოს წესით და პასუხსაც მიაგნებ smile.gif

Posted by: Blind_Torture_Kill 21 Apr 2010, 20:41
QUOTE
ნუ, ტვინის მასა თავისთავად შემცირებული იქნება. მაგრამ უფრო ზუსტი დიაგნოზის დასმაც შეიძლება. ნახე, ლატერალური პარკუჭების ლატერალურად ნორმაში რომ გამობურცული რაღაცაა, ის ამ ავადმყოფის სურათში საერთოდ არაა (ამიტომ ჩანს გადიდებული და მრგვალი პარკუჭი). ჰოდა ახლა გაიხსენე, მანდ რა სტრუქტურა უნდა იყოს წესით და პასუხსაც მიაგნებ


ჰანტინგტონი ?

Posted by: Thandrus 21 Apr 2010, 20:52
Blind_Torture_Kill

აჰა bis.gif

მანდ წესით კაუდატუსის თავი და პუტამენი უნდა იყოს რისი დეგენერაციაც ჰანტინგტონის დროს გვხვდება.

Posted by: Blind_Torture_Kill 21 Apr 2010, 21:26
Thandrus

თავიდან ალზჰაიმერი მეგონა ჰაიდროცეფალუს ექს ვაკუო

Posted by: MAIN KAMPF 10 May 2010, 17:05
ტროპიკული მედიცინის მოყვარულთ სამხრეთაფრიკული ქეისი. ძალიან საინტერესო შემთხვევა რასაც სხვაგან ალბათ ვერ ნახავ. ეს ავადმყოფი დღეს დილით ვნახე დაახლოებით ორ საათში მოკვდა. თუ ვინმეს ნანახი აქვს ეს დაავადება უცებ მიხვდება. ერთი იმას ვიტყვი რომ როგორც წინა შემთხვევები პაციენტი შიდსიანია. დავწერ მისი მკურნალობის მეთოდსაც, იქნება ვინმეს გამოადგეს(ბევრი აბარებს გამოცდას საზღვარგარეთ) smile.gif gigi.gif
http://radikal.ru/F/s43.radikal.ru/i101/1005/6f/5e2dad89be69.jpg.html

Posted by: Thandrus 10 May 2010, 17:46
MAIN KAMPF

ვააჰ... ეს კანი არის აქერცლილი, თუ ჰემორაგიებია? user.gif

Posted by: irakli222 10 May 2010, 19:27
პემფიგუსი
==============================================

Posted by: lgogokhia 10 May 2010, 22:34
MAIN KAMPF
დარწმუნებული არ ვარ ნამდვილად, მაგრამ პირველი აზრი რაც მომივიდა არის სტაფილოკოკური scalded skin syndrome.... მაგრამ შიდსთან რამე კავშირში არის თუ არა სპეციფიკურად არ ვიცი..

Posted by: Thandrus 10 May 2010, 23:19
irakli222

QUOTE
პემფიგუსი


საინტერესოა ნამდვილად... როგორც ვიცი, ზოგიერთი ავტოიმუნური ფენომენი ახასიათებს შიდსი-ს, პემფიგუსი არ მახსოვს, იქ რომ ყოფილიყო. თუმცა გუგლში რომ აკრიფავ "Aids" და "Pemphigus"-ს, კი გიგდებს რაღაც შედეგებს tongue.gif

lgogokhia

QUOTE
დარწმუნებული არ ვარ ნამდვილად, მაგრამ პირველი აზრი რაც მომივიდა არის სტაფილოკოკური scalded skin syndrome.... მაგრამ შიდსთან რამე კავშირში არის თუ არა სპეციფიკურად არ ვიცი..


თუ აქერცლილად ჩავთვლით იმ კანს, მაშინ ეგეც აზრია, მაგრამ ეს ბავშვების სინდრომია უფრო.

ჩემი აზრით, შეიძლება ტოქსიკური შოკის სინდრომი იყოს (უფრო სტრეპტოკოკული, ვიდრე სტაფილოკოკური).

ერთი კია, MAIN KAMPF-მა გვითხრა, ამას ტროპიკებში თუ შევხვდებითო, ჰოდა რამე ეგზოტიკური ინფექცია ხომ არაა? biggrin.gif

Posted by: lgogokhia 10 May 2010, 23:27
Thandrus
მე მგონი ეს არის toxic epidermal necrolysis გამოწვეული ანტივირუსული და/ან სულფონამიდური პრეპარატებით...

QUOTE
თუ აქერცლილად ჩავთვლით იმ კანს, მაშინ ეგეც აზრია,

მე "აქერცლილად" მივიღე ეგ დაზიანებები.. SSSS ნამდვილად ბავშვების დაავადებაა, მაგრამ იმუნოკომპრომიზებულ მოზრდილებშიც შესაძლებელია იყოს იშვიათად (emedicine), მაგრამ to be honest პასუხი რომ დავწერე ეგ არ გამითვალისწინებია smile.gif ასე რომ ჩემი პასუხი არის TEN და ვნახოთ რა არის ნამდვილი პასუხი

Posted by: Blind_Torture_Kill 11 May 2010, 19:48
ესეთი რაღაც ჯერ არ მინახია
თუ დამწვრობას არ ჩავთვლით

............................................................

Posted by: lgogokhia 11 May 2010, 20:18
QUOTE
თუ დამწვრობას არ ჩავთვლით

ისე, TEN და SSSS მართვა დამწვრობის მსგავსად ხდება.. ლოგიკურადაც და რეალურადაც.. რათქმაუნდა პათოგენეზურად ემატება გარკვეული დეტალები.. არ გამოჩნდა კითხვის ავტორი?

Posted by: Blind_Torture_Kill 11 May 2010, 20:29
QUOTE
ისე, TEN და SSSS მართვა დამწვრობის მსგავსად ხდება.. ლოგიკურადაც და რეალურადაც.. რათქმაუნდა პათოგენეზურად ემატება გარკვეული დეტალები.. არ გამოჩნდა კითხვის ავტორი?


კითხვის ავტორი არ გამოჩენილა მარა დაემატა ჩემი ქეისი smile.gif
რა არის სურათზე

Posted by: Thandrus 11 May 2010, 20:53
Blind_Torture_Kill

პმევმოთორაქსია?

გული რატომ არ ჩანს? user.gif

Posted by: lgogokhia 11 May 2010, 21:12
Blind_Torture_Kill
QUOTE
დაემატა ჩემი ქეისი

not my business smile.gif მაგრამ მაინც სურათი რატომ არ დიდდება? ვიცი, ალბათ მასეც უნდა გამოვიცნო, მაგრამ again, not my business lol.gif

Posted by: Blind_Torture_Kill 11 May 2010, 21:29
QUOTE
not my businessმაგრამ მაინც სურათი რატომ არ დიდდება? ვიცი, ალბათ მასეც უნდა გამოვიცნო, მაგრამ again, not my business


მეტი ვერ ვნახე სხვა smile.gif

Thandrus

დაველოდები პასუხებს და მერე დავდებ სწორ პასუხს

Posted by: donvaso 11 May 2010, 21:54
Blind_Torture_Kill
გულში ჰაერია? eek.gif
(პნევმო+კარდია biggrin.gif biggrin.gif biggrin.gif )

Posted by: Blind_Torture_Kill 11 May 2010, 22:50
donvaso

ჰაეროვანი ემბოლიაა რომელიც მთლიანად გულის ღრუებში ისახება

Posted by: Thandrus 11 May 2010, 23:03
Blind_Torture_Kill

wow.gif და მაგხელა მოცულობის ჰაერი გულამდე როგორ მივიდა?

საინტერესო რამე იყო, ისე up.gif

Posted by: Blind_Torture_Kill 12 May 2010, 00:00
Thandrus

მყვინთავის გულია კეისონით

(რაც ჰაერის ბუშტუკები წარმოიშვება ან მარჯვენა გულში ანდ ფილტვის სისხლძარღვებში ჩაიჭედება)

მარა ამ შემთხვევაში ფილტვის სისხლძარღვებიც გადალახა და მოხვდა მარცხენა გულშიც

Posted by: Thandrus 12 May 2010, 00:13
Blind_Torture_Kill

კარგი იყო smile.gif

აბა, ეს რა არის:

მარცხენა სურათზე ბავშვი ჩვეუებრივადაა, მარჯვენაზე კი გაღიმებას ცდილობს - სურათი გადაღებულია ოპერაციამდე

http://img263.imageshack.us/i/36385399.jpg/

Uploaded with http://imageshack.us

ეს სურათი კი - ოპერაციის შემდეგ

http://img688.imageshack.us/i/88636157.jpg/

Uploaded with http://imageshack.us

დასვით დიაგნოზი!

Posted by: mtvareuli 12 May 2010, 02:58
Thandrus
QUOTE
დასვით დიაგნოზი!

მობიუსის სინდრომი
* * *


პ.ს. სინდრომის გარდა ძალიან საინტერესოა მობიუსის ლენტი fig.gif ბავშვობაში ვერთობოდი ხოლმე chups.gif


Posted by: Cousteau 12 May 2010, 04:26
პატარა მარტივი ქეისი user.gif

50 წლის ქალი ნელნელა დაეწყო მხედველობის პრობლემა,გეუბნებათ რომ ეხლა უკვე მარცხენა თვალით თითქმის ვერაფერს ვერ ხედავს დღისით. ფოტოფობია არ აქვს, ნეიროლოგიური გამოკვლევა ჯერ არ ჩატარებულა,


დასაბნევად + კარნახისთვის:
დიდი ხანია ჭირს დაახლოებით ეგეთი რაღაც
ამას მკურნალობს

and the diagnosis is...

Posted by: Thandrus 12 May 2010, 16:31
mtvareuli

QUOTE
მობიუსის სინდრომი


ყოჩაღ! bis.gif

QUOTE
პ.ს. სინდრომის გარდა ძალიან საინტერესოა მობიუსის ლენტი


კი... მეც მიყვარს ასეთი უცნაური რაღაცეები... კიდევ გერმანელი მუსიკოსი Dieter Moebius-ია საინტერესო კაცი, ელექტრონული მუსიკის ერთერთი ფუძემდებელი smile.gif



* * *
Cousteau

QUOTE
დასაბნევად + კარნახისთვის:


დავიბენი ცოტა, ხო იცი... ტიკებს მკურნალობს?

ძალიან ზოგადად გიწერია; რამდენიმე რამემ შეიძლება Day Blindness-ი რამდენიმე რამემ შეიძლება გამოიწვიოს მაგრამ ამ ქალის ისტორიას, მითუმეტეს ცალ თვალში რომ აქვს, ვერ მივუსადაგე ვერაფერი.

რა წამალს იღებს?

Posted by: MAIN KAMPF 12 May 2010, 17:09
აქ რა ამბავი ყოფილა biggrin.gif უბრალოდ ორი დღეა ჰოსპიტალში შევყოვნდი ვერ შემოვედი. რაც შეეხება ჩემს დადებულ სურათს. ეს არის : steven johnson syndrome---მის აღწერას არ დავდებ ინფორმაციას თვითონ ნახავთ ყოჩაღები ხართ smile.gif ეს მეორედ ვნახე აქ და ადვილი დასამახსოვრებელია. დავწერ მის მკურნალობასაც შეიძლება ვინმეს დააინტერესოს ან სადმე გამოგადგეთ biggrin.gif
Prednisone 15 mg po b.d
Bactoban ointment b.d to open area
Terramycin ointment b.d eyes
Chloramex bd lips
Glycothemol mouthwash gargle t.d.

P.M. რენტგენის სურათზე გულის კონტურები ჩანს გულის საზღვრები გადიდებული არაა, სავარაუდოდ პნევმოკარდიაა, შესაძლებელია შუასაყრიდან მოხვდა. ტამპონადა რომ ყოფილიყო საზღვრები გადიდდებოდა.

Posted by: Thandrus 12 May 2010, 17:19
MAIN KAMPF

ნუ, სწორი ყოფილა ესეიგი lgogokhia, Stevens-Johnson-ი და TEN რამ გაყო smile.gif

QUOTE
P.M. რენტგენის სურათზე გულის კონტურები ჩანს გულის საზღვრები გადიდებული არაა, სავარაუდოდ პნევმოკარდიაა, შესაძლებელია შუასაყრიდან მოხვდა. ტამპონადა რომ ყოფილიყო საზღვრები გადიდდებოდა.


ჰო... ასეთი რამე მართლა პირველად ვნახე.

Posted by: LUKA-BRAZI 12 May 2010, 17:37
ყოჩაღ, კარგია რომ ამ თემას პატრონობთ და შეძლებისდაგავარად აქტიურობთ. up.gif up.gif up.gif

მეც შემოგიერთდებით "მალე".

:სახელმწიფოგამოცდებსმოვრჩებიდამერე:

Posted by: Cousteau 12 May 2010, 19:18
QUOTE (Thandrus @ 12 May 2010, 16:31 )


რა წამალს იღებს?

მაგას რათქმაუნდა არ გეყტვი : D

დღისით ვერ ხედავს დავამატებ დღის სინათლეზე ვერ ხედავსს
არც ფოტოავერსია არ აქვს და არც ფოტოფობია

რთულად უყურებ მარტივ ქეისს user.gif

Posted by: lgogokhia 12 May 2010, 19:18
MAIN KAMPF
მე აშკარად ვთვლი რომ ეს უფრო TEN არის ვიდრე სტივენს-ჯონსონის სინდრომი. ეს ორი მხოლოდ დაზიანების სიფართით განსხვავდება ერთმანეთისგან, თუ არის სხეულის ზედაპირზე 10%-ზე ნაკლები მაშინ თვლიან როგორც სტივენს ჯონსონის სინდრომი და ლეტალობა ამ შემთხვევაში 1-5%ია და TEN არის მაშინ როდესაც კანის detachment ვრცელდება სხეულის ზედაპირის 30%-ზე მეტზე, რომელიც უფრო შეესაბამება სინამდვილეს ამ შემთხვევაში და რა შემთხვევაშიც ლეტალობა გაცილებით მაღალია. ყოველ შემთხვევაში სურათზე ასე ჩანს.. ამას რათქმაუნდა მკურნალობის თვალსაზრისით მნიშვნელობა არ აქვს, მაგრამ ნუ დიაგნოზს ალბათ როგორც TEN გავაფორმებდი smile.gif please, find attached article wink.gif http://rapidshare.com/files/386475141/SJS_TEN.pdf.html


Posted by: MAIN KAMPF 12 May 2010, 20:16
QUOTE
მე აშკარად ვთვლი რომ ეს უფრო TEN არის ვიდრე სტივენს-ჯონსონის სინდრომი.


no.gif
არა და რატომ, ჯერ ერთი ვთვლი რომ აქაურ სპეციალისტებს უნდა ვენდოთ იმიტომ რომ მათ მეტი გამოცდილება აქვთ და შეუძლიათ დიფერენცირება გაუკეთონ ამ დაავადებას. toxic epidermal necrolysis და erythema multiforme ერთმანეთის მსგავსია, მხოლოდ ამ პირველის შემთხვევაში დაავადება უმეტესად გამოწვეულია წამლების მიღების გამო. ჩემს მოყვანილ შემთხვევაში პაციენტს არანაირი წამლის მიღება არ დაფიქსირებულა, მე ხაზი გავუსვი რომ ავადმყოფი იყო შიდსიანი ამ ავადმყოფებს თითქმის უმრავლესობას აქვს ან გადატანილი აქვს მრავალი ინფექც ან სოკ დაავადება. ჩვეულებრივ ამ ავადმყოფშიაც ისევე როგორც სხვა შიდსიანებში ერთ-ერთი მიზეზი ჰერპესია.
QUOTE
please, find attached article  http://rapidshare.com/files/386475141/SJS_TEN.pdf.html
ეს ვერ გავხსენი , მაგრამ შეგიძლია მოიძიო შემდეგი ლიტერატურა
The merck manual(IN COMMITMENT TO THE HEALTH OF ALL SOUTH AFRICANS)
smile.gif

Posted by: lgogokhia 12 May 2010, 20:38
Cousteau
QUOTE

50 წლის ქალი ნელნელა დაეწყო მხედველობის პრობლემა,გეუბნებათ რომ ეხლა უკვე მარცხენა თვალით თითქმის ვერაფერს ვერ ხედავს დღისით. ფოტოფობია არ აქვს, ნეიროლოგიური გამოკვლევა ჯერ არ ჩატარებულა,

ამირან გულში მღეროდა-ს პონტში ქეისია ეს მე მგონი smile.gif შეიძლება ნეიროლეპტიკების/ანტიდეპრესანტების (ტურეტის სინდრომისთვის) გვერდითი ეფექტიდან დაწყებული(უფრო სავარაუდოდ, რადგან ამბობ
QUOTE
მაგას რათქმაუნდა არ გეყტვი : D
) ნეიროფიბრომატოზით ან ჰანტინგტონის დაავადებით დამთავრებული, რადგან ის ზოგადი სიმპტომები (ტიკები, age of onset, no photophobia ) რომელიც შენ დაწერე შეიძლება ახასიათებდეს ბევრ რამეს.
QUOTE
ვერ ხედავს დღისით
ამით რამის თქმა გინდოდა? დღისით ვერ ხედავს მაგრამ ღამე კი? ანუ ჰემერალოპია (არ ვიცი რამდენად ხშირად გამოიყენება ეს ტერმინი ნევროლოგიაში) თუ მონოკულარული სიბრმავე?


* * *
MAIN KAMPF
yes.gif პირველ რიგში SJS და TEN-ს შორის რომ მხოლოდ დაზიანების ფართობშია განსხვავება ამაში შევთანხმდეთ smile.gif უამრავ ლიტერატურას მოგიყვან ამ საკითხზე და აი შენი მერკიდანაც http://www.merck.com/mmpe/sec10/ch117/ch117i.html?qt=TEN&alt=sh ანუ იმის თქმა რომ ერთი წამლის გვერდითი ეფექტია და მეორე ნაკლებად, არ შეესაბამება სიმართლეს. მეორე - ის რომ პაციენტი არ ღებულობდა წამლებს არ გულისხმობს რომ ან SJS არის ან TEN, როგორც აღნიშნე HIV+ ავადმყოფებში შეიძლება სხვადასხვა ოპორტუნისტული ინფექციებით იყოს გამოწვეული (თუნდაც HSV, ან სოკოვანი ინფექციები ან მიკოპლაზმა) და მესამე
QUOTE
აქაურ სპეციალისტებს უნდა ვენდოთ იმიტომ რომ მათ მეტი გამოცდილება აქვთ და შეუძლიათ დიფერენცირება გაუკეთონ ამ დაავადებას
რათქმაუნდა ვენდოთ უნდა, მაგრამ ეს ორი დიაგნოზის დიფერენცირება კლინიკურად იმდენად არ არის მნიშვნელოვანი, რომ ხშირად უგულებელყოფენ ხოლმე და გარდა ამისა ეჭვის შეტანა შეცდომა არ არის smile.gif ასე რომ სუფთა მათემატიკური გაგებით მაინც არ დაგეთანხმები და ალბათ მაინც TEN არის, რადგან დაზიანების ზედაპირის ფართობი 30%-ზე მეტია..

იმ სტატიას კი აგერ სხვა საიტზე ავააპლოუდე და შეგიძლია აქედან სცადო http://www.megaupload.com/?d=EDPVBAL7 დიდი მნიშვნელობა არ აქვს რათქმაუნდა ამ ყველაფერს და მე მოვრჩი ამით ამ თემაზე ლაპარაკს, მაგრამ just for the sake of argument wink.gif

Posted by: Cousteau 12 May 2010, 22:05
QUOTE (lgogokhia @ 12 May 2010, 20:38 )
ამით რამის თქმა გინდოდა? დღისით ვერ ხედავს მაგრამ ღამე კი? ანუ ჰემერალოპია (არ ვიცი რამდენად ხშირად გამოიყენება ეს ტერმინი ნევროლოგიაში) თუ მონოკულარული სიბრმავე?



yes.gif

საღამოს უკეთესად ხედავს ვიდრე დილით yes.gif

Posted by: mtvareuli 12 May 2010, 22:26
Cousteau

ამ ტიკების სამკურნალო რომელიმე პრეპარატი იწვევს კოლბების დაზიანებას?


Posted by: MAIN KAMPF 12 May 2010, 22:33


QUOTE
ფართობი 30%-ზე მეტია

ეს არ არის მაჩვენებელი დიაგნოზის დასმის. დიაგნოზი ისმევა კლინიკური მაჩვენებლების ერთობლიობის შესაბამისად. ვენდოთ თუ არა კლინიკის გამოცდილებას რა თქმა უნდა ვენდოთ სადაც ასეთი შემთხვევა საკმაოდ ხშირია. ბანალურად არც აპენდიციტის დიაგნოზი ისმევა მხოლოდ ილეოცეკალურ არეში ტკივილით.



smile.gif

Posted by: Cousteau 12 May 2010, 23:03
QUOTE (mtvareuli @ 12 May 2010, 22:26 )
Cousteau

ამ ტიკების სამკურნალო რომელიმე პრეპარატი იწვევს კოლბების დაზიანებას?

ნწუ, ამ ტიკების სამკურნალოდ თუ ჰალოპერიდოლს აძლევდნენ ის იწვევს კატარაქტის გაჩენას, ცენტრალური dense კატარაქტის დროს გუგა ვიწროვდება და კატარაქტის ირგვლივ შუქი თითქმის საერთოდ არ შედის თვალში, საღამოს (ანუ როცა განათება არ არის ძლიერი,) მეტი სხივები შედის და პაციენტი უკეთესად ხეადვს ვიდრე დილით

Posted by: mtvareuli 12 May 2010, 23:07
Cousteau

აჰა, გასაგებია biggrin.gif მე სხვა მიმართულებით წავედი user.gif

Posted by: lgogokhia 13 May 2010, 00:13
MAIN KAMPF
QUOTE
ეს არ არის მაჩვენებელი დიაგნოზის დასმის.

რათქმაუნდა არ არის.. მაგრამ SJS-ს და TEN-ს შორის მაგაშია მარტო განსხვავება.. smile.gifCousteau
Cousteau
QUOTE
ნწუ, ამ ტიკების სამკურნალოდ თუ ჰალოპერიდოლს აძლევდნენ ის იწვევს კატარაქტის გაჩენას, ცენტრალური dense კატარაქტის დროს გუგა ვიწროვდება და კატარაქტის ირგვლივ შუქი თითქმის საერთოდ არ შედის თვალში, საღამოს (ანუ როცა განათება არ არის ძლიერი,) მეტი სხივები შედის და პაციენტი უკეთესად ხეადვს ვიდრე დილით

smile.gif არ იყო კლინიკური შემთხვევა ნამდვილად, არამედ უფრო მოგონილი. ჰალოპერიდოლის და სხვა ნეიროლეპტიკების გვერდითი ეფექტები ძირთადად ექსტრაპირამიდული მოვლენები, NMS, tardive dyskinesia და ა.შ. მაგრამ კატარაქტა ნამდვილად არ მახსოვდა და არც ისე ხშირია როგორც ვხედავ, არა? ანუ ყოველ შემთხვევაში მსგავსი კლინიკური პრეზენტაციით..

Posted by: Cousteau 13 May 2010, 00:32
მაგიტო დავწერე ზემოთ რომ ტიკები გინდ ქონია და გინდ არა : ))) ანუ ცენტრალურ დენს კატარაქტაზე იყო კითხვა : )))

QUOTE
ნეიროლეპტიკების გვერდითი ეფექტები ძირთადად ექსტრაპირამიდული მოვლენები, NMS, tardive dyskinesia და ა.შ

yes.gif yes.gif yes.gif

PS, ყველაზე ცივილური თემაა ფორუმზე (ნუ გარდა იმისა რომ ყველაზე საინტერესოა და ყველაზე კომპეტენტური)

Posted by: Thandrus 13 May 2010, 03:53
Cousteau

ჰეჰ, ეგრეც ვიფიქრე gigi.gif

აბა ერთი ქრესტომათიული შემთხვევა:

45 წლის უსახლკარო კაცი მოდის ED-ში სიცხით და ხველების ოთხდღიანი ისტორიით. ნახველი არის სქელი, სისხლიანი ფლეგმა. კაცი. სუნთქვის და ხველებისას. მკერდის მარჯვენამხრივ ტკივილს უჩივის. ის უარყოფს რაიმე ადრინდელ დაავადებას და აღარ ახსოვს ბოლოს როდის იყო ექიმთან. ის არ ეწევა და ნარკოტიკებს არ იღებს, მაგრამ ვისკის დალევის შანსს ხელიდან არ უშვებს. შემოწმებისას, ის ჭუჭყიანი, დაუვარცხნელი და ცუდად ნაკვები ჩანს. ტემპერატურა 38,9 C. პულსი 105, და სუნთქვის სიხშირე 30 / წთ. ფილტვის შემოწმებისას ისმის სუნთქვის შემცირებული ხმიანობა და crackle-ები მარჯვენა ქვემო, პოსტერიორულ ნაწილში. ლაბორატორული კვლევა აჩვენებს თეთრი უჯრედების მომატებას. რენტგენზე ჩანს მარჯვენა ქვედა წილში მკვივი ინფილტრატი აბსცესით. ნახველი აღებულია კულტურისთვის და გრემის წესით შეღებვისთვის, რადგანაც ეჭვი ბაქტერიულ ეტიოლოგიაზეა.

ჰოდა, დასვით დიაგნოზი და დაასახელეთ გამომწვევი smile.gif

Posted by: irakli222 13 May 2010, 09:47
მოაბსცედირე პნევმონია, გამომწვევი ოქროსფერი სტაფილოკოკი

Posted by: lgogokhia 13 May 2010, 10:20
Thandrus
QUOTE
აბა ერთი ქრესტომათიული შემთხვევა:

შესაძლებელია პნევმოკოკური პნევმონია, რადგან ის ზოგადად პნევმონიის ყველაზე ხშირი გამომწვევია, თუმცა ნახველი ამ შემთხვევაში არის ჟანგისფერი (rusty). ალკოჰოლიკებში კლებსიელა უფრო დამახასიათებელია, მითუმეტეს ამბობ რომ
QUOTE
ნახველი არის სქელი, სისხლიანი ფლეგმა
, რომელიც შესაძლოა არა სისხლი, არამედ კლებსიელას წითელი პიგმენტია.. ამიტომ ჩემი მოკრძალებული პასუხი იქნება პნევმონია/ფილტვის აბსცესი გამოწვეული Klebsiella pneumoniae-თი. biggrin.gif გრამით შეღებვისას რა ნახეს? gigi.gif P.S. ამ შეკითხვას რომელია უფრო ხშირი გამომწვევი ჭკუიდან გადავყავარ!!!

Posted by: anarxisti 13 May 2010, 11:05
QUOTE
მაგრამ ვისკის დალევის შანსს ხელიდან არ უშვებს.

ღვიძლის ციროზის გართულება, პნევმოპატია უნდა იყოს. სულაც ბანალური პნევმოკოკით დაინფიცირებული.

Posted by: Thandrus 13 May 2010, 14:23
lgogokhia

QUOTE
ამიტომ ჩემი მოკრძალებული პასუხი იქნება პნევმონია/ფილტვის აბსცესი გამოწვეული Klebsiella pneumoniae-თი.


აბსოლუტურად მართალი ხარ - გრემით შეღებვისას დიდი, გრემ-ნეგატიური ჩხირები ნახეს, შემდეგი ტესტებით დადგინდა რომ, K. pneumonie-ს შეესაბამებოდა.

საერთოდ, მართლები ხართ ყველანი, ისტორია ძალიან შემოკლებულად იყო მოცემული, ალკოჰოლიკ და უჭმელ-უსმელ კაცში ბევრმა რამემ შეიძლება მოგცეს პნევმონია, მაგრამ როგორც ვთქვი, შემთხვევა წიგნიდან ამოღებული იყო, ასე რომ ხელმოსაჭიდი სიტყვები, ალკოჰოლიზმი, აბსცესი და სისხლიანი ფლეგმა ყველაზე მეტად კლებსიელას გავარუდებინებს yes.gif

Posted by: Blind_Torture_Kill 13 May 2010, 22:40
აბა რა მიკროორგანიზმია
მახსოვს მიკრობიოლოგიას რომ გავდიოდით ამის კულტურა გავაკეთე და ჩემ შვილებს ვეძახდი smile.gif

Posted by: lgogokhia 13 May 2010, 22:55
Thandrus
კლასიკური UშMLE სტეპ 1 შეკითხვა იყო smile.gif

აბა ახალი ქეისი: 71 წლის ქალი შემოიყვანეს სისუსტით, სომნოლენციით, confusion და slurred speech-ით. ოჯახის გადმოცემით მისი სიმპტომები დაიწყო 3 თვის წინ მდგომარეობის თანდათანი გაუარესებით. პაციენტი არ არის შეშუპებული და არ არის დეჰიდრატაციის ან ჰიპერჰიდრატაციის ნიშნები. გამოკითხვით, პაციენტი ბოლო 3 წლის განმავლობაში მკურნალობს რამიპრილით და მეტოპროლოლით `intermittent ატრიალური ფიბრილაციის და ჰიპერტონიული დაავადების გამო. წნევა/სიცხე ტრალი ვალი ნორმალური ყველაფერი.. ჰოდა აგერ ხელში გყავთ ეს ავადმყოფი და რას უზამთ, ანუ რა გამოკვლევებს გააკეთებთ და როგორ წახვალთ დიფ-დიაგნოზისკენ.. smile.gif
* * *
Blind_Torture_Kill
QUOTE
აბა რა მიკროორგანიზმია

შეიძლება Fusobacterium, თან რომ ამბობ შენი შვილებიო (ნერწყვი??

Posted by: Blind_Torture_Kill 13 May 2010, 23:33
lgogokhia

აუტოფსია smile.gif
მაგის გარეშე მე დიაგნოზს ვერ ვსვავ biggrin.gif


სისხლის საერთო
ელექტროლიტები
ჯერ

QUOTE
შეიძლება Fusobacterium, თან რომ ამბობ შენი შვილებიო (ნერწყვი??

არა ეგ არ არის smile.gif

ეს გრამ დადებითია

Posted by: Thandrus 14 May 2010, 00:31
Blind_Torture_Kill

ჩემი აზრით, Bacillus Antracis-ია.

შენ საიდან დათესე? biggrin.gif

Posted by: lgogokhia 14 May 2010, 00:42
Blind_Torture_Kill
QUOTE
აუტოფსია
no.gif biggrin.gif

QUOTE
სისხლის საერთო

არაფერი მნიშვენლოვანი
QUOTE
ელექტროლიტები

Na - 107mM/L
პლაზმის ოსმოლარობა - 227mM/kg
შარდის Na - 86 mM/L
შარდის ოსმოლარობა - 495 mM/kg
დანარჩენი უმნიშვნელო..

მეტი რა გინდა?

QUOTE
ეს გრამ დადებითია

boli.gif ძალიან ცუდად შეღებილი რომ არ იყოს, ძალიან მარტივი ამოსაცნობია smile.gif მაგრამ დარწმუნებული ვარ ამ ბაქტერიის კულტურას შენ ვერ გააკეთებდი და რაღაც გეშლება აშკარად smile.gif

* * *
Thandrus
QUOTE
ჩემი აზრით, Bacillus Antracis-ია.

ნამდვილად..

Posted by: Thandrus 14 May 2010, 00:50
lgogokhia

ჰმ... სინტერესოა. როგორც ახლა ვნახე, რამიპრილსაც და მეტოპროლოლსაც (მითუმეტეს ხანგრძლივად გამოყენების შემთხვევაში) საკმაოდ დიდი ბუკეტი აქვს adverse effect-ების, რომელშიც ეს სიმპტომებიც შეიძლება შევიდეს. მაგრამ ასე მარტივად ალბათ არაა აქ საქმე smile.gif და ატრიალურ ფიბრილაციას ამ წამლებით მკურნალობენ? user.gif თრომბოზის რისკი არ უნდა შეამცირონ მანდ? მოკლედ რავიცი - რადგან ნევროლოგიური სიმპტომებით მოვიდა პაციენტი, ჯერ MRI-ს ვუკეთებ biggrin.gif

Posted by: lgogokhia 14 May 2010, 01:28
Thandrus
ოკ, გეტყვი MRI-ს პასუხს, მაგრამ დაგამადლებ, იმიტომ რომ არ გეკადრება შენ უყურადღებობა და რაღაც გამოგრჩა რომელიც არ უნდა გამოგრჩენოდა იმ ინფორმაციით რაც მოგეცი biggrin.gif თავის ტვინის MRI-ზე არაფერი საინტერესო არ გამოჩნდა wink.gif

Posted by: Thandrus 14 May 2010, 02:27
lgogokhia

როცა მე ვწერდი, ლაბ-ანალიზები არ გეწერა. smile.gif

ანალიზებში ჰიპონატრემია და ჰიპოოსმოლალობა ჩანს, რასაც ამ სიმპტომების გამოწვევა შეუძლია. წესით ნორმალურად ამ დროს, შარდის ოსმოლალობა ძალიან დაბალი უნდა იყოს, თუ წყლის გამოყოფის დეფექტი არ არის. აქ კი დაბალი არ არის; ასეთი რამე შეიძლება SIADH-ის დროს იყოს.


Posted by: lgogokhia 14 May 2010, 07:26
Thandrus
QUOTE
SIADH

yes.gif 2kiss.gif გააგრძელე.. საბოლოო დიაგნოზი რათქმაუნდა სხვაა.. რას უტარებ საბოლოო დიაგნოზისთვის? hint: კლასიკური შემთხვევა არ არის
* * *
P.S.
QUOTE
როცა მე ვწერდი, ლაბ-ანალიზები არ გეწერა.
თუ არ წაგიკითხავს MRI-მდე არ უნდა მოგეთხოვა რუტინული ანალიზები? doh!! biggrin.gif
* * *
P.P.S.
QUOTE
Creutzfeldt–Jakob disease ?

ეხლა ვკითხულობდი ამ თემის პირველ გვერდებს და რაღაც გამახსენდა.. თქვენ იცით თუ არა რომ famous პიოფაგი მზადდება Brain-Heart Infusion broth-ზე?? lol.gif დანარჩენი თქვენი ლოღიკისთვის მომინდია.. lol.gif lol.gif ეს ისე კონფიდენციალური ინფო biggrin.gif

Posted by: Blind_Torture_Kill 14 May 2010, 09:21
Thandrus
QUOTE
ჩემი აზრით, Bacillus Antracis-ია.

შენ საიდან დათესე?

ეგაა


QUOTE
ძალიან ცუდად შეღებილი რომ არ იყოს, ძალიან მარტივი ამოსაცნობიამაგრამ დარწმუნებული ვარ ამ ბაქტერიის კულტურას შენ ვერ გააკეთებდი და რაღაც გეშლება აშკარად


შეღებილი ცუდია მართალი ხარ

ჩვენ თსსუ-ს მიკრობიოლოგიის კათედრაზე მაგისი კულტურა გავაკეთე
და კაი ჩასუქებული კოლონიებიც გამოვზარდე

დასუსტებული ფორმები ქონდათ და იმით ვაკეთებდით

Posted by: Thandrus 14 May 2010, 17:22
lgogokhia

QUOTE
გააგრძელე.. საბოლოო დიაგნოზი რათქმაუნდა სხვაა.. რას უტარებ საბოლოო დიაგნოზისთვის? hint: კლასიკური შემთხვევა არ არის


თუ SIADH-ს მივყვებით, ალბათ კორტიზოლი უნდა გავზომოთ, რადგან ACTH-ს გამოყოფის პრობლემებმაც შეიძლება ასეთი რამე მოგვცეს. თუ ნორმაშია, მოგვიწევს მაშინ ექტოპიური AVP-ს გამომყოფი სიმსივნის ძებნა...
* * *
QUOTE
თუ არ წაგიკითხავს MRI-მდე არ უნდა მოგეთხოვა რუტინული ანალიზები? doh!!


gigi.gif

QUOTE
ეხლა ვკითხულობდი ამ თემის პირველ გვერდებს და რაღაც გამახსენდა.. თქვენ იცით თუ არა რომ famous პიოფაგი მზადდება Brain-Heart Infusion broth-ზე??დანარჩენი თქვენი ლოღიკისთვის მომინდია.. ეს ისე კონფიდენციალური ინფო


პიოფაგი რა არის - პიოგენური ბაქტერიების საწინააღმდეგო ფაგი? ნუ, გააჩნია, ვისი ტვინ-გულია biggrin.gif თუ ძროხის არაა. CJD-ს რისკი არ უნდა იყოს. ისე, ფაგებით მართლა მკურნალობენ ინფექციებს? წიგნებში არ წერია ხოლმე ამაზე არაფერი...

Posted by: lgogokhia 14 May 2010, 20:12
Thandrus
QUOTE
კორტიზოლი

14 ug/dL

QUOTE
პიოფაგი რა არის - პიოგენური ბაქტერიების საწინააღმდეგო ფაგი?

yes.gif

QUOTE
ნუ, გააჩნია, ვისი ტვინ-გულია

http://en.wikipedia.org/wiki/Brain_heart_infusion_broth

QUOTE
ისე, ფაგებით მართლა მკურნალობენ ინფექციებს?

no.gif maxati.gif vis.gif biggrin.gif

Posted by: Thandrus 15 May 2010, 00:36
lgogokhia

QUOTE
14 ug/dL


ნორმაა ესეც user.gif

კარგი, სანამ სიმსივნეზე წავალთ... გულისრევა / ღებინება ხომ არ ჰქონია პაციენტს (ამას შეუძლია ასეთი კლინიკური სურათის მოცემა, თან ახლა ვნახე, მეტოპროლოლის გვერდით ეფექტებში შედის)?

Posted by: lgogokhia 15 May 2010, 01:56
Thandrus
QUOTE
გულისრევა / ღებინება ხომ არ ჰქონია პაციენტს

no.gif მოკლედ, იხილეთ MRI და სიმსივნის ჰისტოლოგიური სურათი smile.gif

http://www.radikal.ru

http://www.radikal.ru

Posted by: Thandrus 15 May 2010, 03:59
lgogokhia

QUOTE
მოკლედ, იხილეთ MRI და სიმსივნის ჰისტოლოგიური სურათი


ხო, ნუ სიმსივნე გასაგები იყო.

რომელი სიმსივნეა, სამწუხაროდ ვერ ვცნობ sad.gif სურათი აშკარად T2-ის დონეზეა გაკეთებული მაგრამ რა უნდა იყო ის... ჰისტოლოგიურად ვერ ვამსგავსებ ამ დაჯღანულ უჯრედს რამე ნორმალურს (Goblet Cell-ი ხომ არ არის?)...

მოკლედ, Goblet Cell Carcinoid-ი ხომ არ არის? biggrin.gif

Posted by: lgogokhia 15 May 2010, 05:27
Thandrus
QUOTE
მოკლედ, Goblet Cell Carcinoid-ი ხომ არ არის?

no.gif დაველოდოთ აბა ხალხს, ვინც ჰისტოლოგიაში ან MRI-ში ერკვევა wink.gif და მართალი რომ გითხრა არანაირი წარმოდგენა არ მაქვს მანდ სურათებზე რაც ხდება baby.gif უბრალოდ, ნამდვილად ამ პაციენტისგანაა და ნამდვილად დიაგნოზი მხოლოდ ამით დაისვა biggrin.gif ჩემს საქმეს მე მოვრჩი, "გავაგზავნე" გამოკვლევაზე და რასაც დამიწერენ იმას წავიკითხავ.. მაგრამ ვაი რომ NBME არ ფიქრობს მთლად მასე biggrin.gif

Posted by: Monardo 15 May 2010, 18:35
პრაქტიკული შემთხვევა

წარმოიდგინეთ, რომ არ გაქვთ ლაბორატორია ან სხვა თანამედროვე კველვის აპარატურა.

10 თვის ბავშვს ღამით აუწია სიცხემ მაქსიმუმ 39° გრადუსამდე და დილით განავალის ფერი "გაღიავდა" (მანამდე ყავისფერი იყო). პირველივე ეჭვი A ჰეპატიტზე გვქონდა, მაგრამ ქოლესტაზის სხვა არცერთი ნიშანი არ იყო (სიყვითლე არ აღინიშნებოდა). მეორე დღეს ტემპერატურა დარეგულირდა, მაგრამ ბავშვს აღენიშნებოდა ზოგადი სისუსტე და ადვილად დაღლადობა. განავალი კიდევ უფრო გაღიავდა. მესამე დღეს ყველაფერი ანალოგიური, მაგრამ განავალი კიდევ უფრო გაღიავდა (ღია კანის ფერი, "ბეჟი" გახდა) და შეიძინა ძალიან მძაფრი აყროლებული სუნი.

მეოთხე დღეს, ზუსტად იდენტური სიმპტომატიკა განუვითარდა ბავშვის მამას (ერთდღიანი სიცხე, განავლის გაღიავება და ანალოგიური სუნი).

ბავშვს რათქმაუნდა თავისი სიმპტომატიკის აღწერა არ შეეძლო, მაგრამ რადგან პირველი სამი დღე ერთნაირი იყო, ვივარაუდეთ, რომ რასაც მამა გრძნობდა, ისეთივე ჩივილები ჰქონდა ბავშვს. ბავშვის ინტენსიური ტირილიც ადასტურებს, რომ ორივე პაციენტს აღენიშნებოდა მოვლითი ტკივილები მუცელში და მეტეორიზმი.

ჩვენ სავარაუდო დიაგნოზი დავსვით მხოლოდ ამ კლინიკის მიხედვით და მაინტერესებს თუ დაემთხვევა თქვენს დიაგნოზს. თუ შეიძლება მხოლოდ დიაგნოზი კი არა ლოგიკური ჯაჭვიც დაწეროთ.

Posted by: Thandrus 15 May 2010, 21:19
Monardo

ისტორიის შეკრება შეიძლება? რამე საინტერესო იყო ისტორიაში (მაგ. მიღებული საჭმელი.)?

ისე, მე კუჭ-ნაწლავის ტრაქტის პარაზიტულ ინფექციაზე მივიტანდი ეჭვს (მაგ. G. lamblia-ს ინფექცია).

Posted by: lgogokhia 15 May 2010, 21:26
პირველ რიგში არის ინფექციური გასაგები მიზეზების გამო. ვინაიდან არ ქონდა ინფექციური ჰეპატიტის სხვა ნიშნები, ამიტომ დროებით გადავდოთ გვერდზე (არ გამოვრიცხავ არაფერს). მუცლის ტკივილი და ერთდღიანი ცხელება კამპილობაქტერიისთვის არის დამახასიათებელი, მაგრამ განავლის აქოლიურობა და სუნი რამდენად, არ ვიცი.. განავლის აქოლიურობა, სუნი და ცხელებამ შეიძლება გიარდიაზიც გვაფიქრებინოს, მაგრამ ასე მწვავედ დაწყება და მოკლე პროდრომი რამდენადაა დამახასიათებელი არ ვიცი.. სხვას ვერ ვფიქრობ, ღვიძლი პალპაციით როგორი იყო? მამა-შვილს ხომ არ აქვს მგზავრობის ისტორია? ენდემური რა არის იმ მხარეში?

Posted by: Monardo 15 May 2010, 23:16
ძალიან სასიამოვნოა იმის მოსმენა, რომ შენი დიაგნოზი ემთხვევა სხვის დიაგნოზს.

სწორედ Giardia lamblia დავასახელეთ მიზეზად.

მათ განავალს უფრო სტეატორეა ჰქვია, ვიდრე აქოლიური, მითუმეტეს, რომ ქოლესტაზი არ იყო

Posted by: Professor Xachikian 16 May 2010, 05:34
lgogokhia
QUOTE
ჩემს საქმეს მე მოვრჩი, "გავაგზავნე" გამოკვლევაზე და რასაც დამიწერენ იმას წავიკითხავ


მართალია. მთავარია დაისვა SIADH დიაგნოზი. სიმსივნური კერა ნაპოვნია. ეხლა საბოლოო ვერდიქტი ბიოფსიაზე და პათანატომის დასკვნაზეა. ასე ხდება რეალურ პრაქტიკაში

თუმცა, ამ შემთხვევაში რადგან გამოცნობაზეა საქმე, გამოვთქვავ ჩემს ვარაუდს:

მოგნი, ანატომიურად სისმივნე ფარისებრ ჯირკვალში უნდა იყოს. გავაკეთებდი ფარისებრი ჯირკვლის ულტრასაუნდს და ულტრასაუნდით დამიზნებით ბიოფსიას (Fine needle aspiration - FNA - under guidance).

მდებარეობის და ჰისტოლოგიური პრეპარატის მიხედვით, ალბათ ფარისებრი ჯირკვლის პაპილარული სიმსივნეა გამოსახული.
აქვე უნდა აღვნიშნო, რომ ფარისებრი ჯირკვლის პაპილარულ სიმსივნეს SIADH არ ახასიათებს, და ეს ალბათ ერთ-ერთი ძალზე იშვიათი შემთხვევაა.

Posted by: Thandrus 16 May 2010, 07:43
Professor Xachikian

გამარჯობა! როგორ ხარ? რატომ აღარ ჩანხარ აქ? smile.gif

Posted by: lgogokhia 16 May 2010, 12:22
Professor Xachikian
QUOTE
მდებარეობის და ჰისტოლოგიური პრეპარატის მიხედვით, ალბათ ფარისებრი ჯირკვლის პაპილარული სიმსივნეა გამოსახული

yes.gif yes.gif აბსოლუტურად!! smile.gif

QUOTE
აქვე უნდა აღვნიშნო, რომ ფარისებრი ჯირკვლის პაპილარულ სიმსივნეს SIADH არ ახასიათებს, და ეს ალბათ ერთ-ერთი ძალზე იშვიათი შემთხვევაა.

კი მე თავიდანვე ავღნიშნე რომ არ იყო კლასიკური შემთხვევა smile.gif პაციენტს ჩაუტარდა ძალიან დეტალური კვლევები სიმსივნის აღმოსაჩენად, მუცლის, გულმკერდის ღრუს CT გამოკვლევით არ გამოვლინდა საეჭვო წარმონაქმნი. მხოლოდ
QUOTE
ფარისებრი ჯირკვლის ულტრასაუნდს და ულტრასაუნდით დამიზნებით ბიოფსიას (Fine needle aspiration - FNA - under guidance).
გამოვლინდა პაპილარული კიბო. ჩაუტარდა ტოტალური თიროიდექტომია შემდგომი სტანდარტული მკურნალობით. ოპერაციიდან რამდენიმე დღეში ელექტროლიტები ნორმაში იყო და მენტალური სტატუსიც გაუმჯობესდა. ამიტომ SIADH-ს მიზეზად ჩაითვალა ფარისებრი ჯირკვლის პაპილარული სიმსივნე, რაც თავისთავად უიშვიათესი შემთხვევაა smile.gif ეს პრინციპში მარტივი ქეისი იყო, თუმცა იშვიათი და დამაბნეველი, და შესაბამისად საინტერესო. Thandrusს თქმის არ იყოს ამ ქეისის ამოხსნისას უამრავი რაღაცის ნახვა მომიხდა, რაც ამ ყველაფრის აზრია ალბათ smile.gif

P.S. ეს არის აღწერილი შემთხვევები რომელსაც განვიხილავთ postdoc meeting-ებზე და არა საკუთარი.. ეს ისე ქოფირაითისთვის biggrin.gif

* * *
Monardo
QUOTE
მათ განავალს უფრო სტეატორეა ჰქვია, ვიდრე აქოლიური, მითუმეტეს, რომ ქოლესტაზი არ იყო

შენი აღწერილობა უფრო აქოლიურობად ჩავთვალე, ვიდრე სტეატორეად smile.gif სტეატორეა და foul smelling stools არის გიარდიაზის alarm ყველა ვარიანტში, თუ კლინიკაც შეესაბამება მაშინ დიდი ალბათობით შეგიძლია დიაგნოზის დადგენა, რომელიც ამ შემთხვევაშიც მოხდა.. თუმცა იქ სადაც არანაირი გამოკვლევის საშუალება არ მქონდა, ალბათ მეტრონიდაზოლიც არ მექნებოდა, ამიტომ მინიმუმ მაგისთვის გავაგზავნიდი სადმე სადაც "ბარემ" გამოკვლევებსაც ჩავუტარებდი baby.gif ვერ დადგებოდა ექიმი ჩემგან შუა საუკუნეებში არა? smile.gif

Posted by: MAIN KAMPF 19 May 2010, 18:25
მარტივი რამეა გახსენება ყოველთვის საჭიროა smile.gif რა ხდება აქ chups.gif
[URL=http://radikal.ru/F/s41.radikal.ru/i091/1005/37/1b18bb283ff4.jpg.html][IMG]http://s41.radikal.ru/i091/1005/37/1b18bb283ff4t
* * *
არ ჩაიტვირთა mad.gif http://www.radikal.ru

Posted by: Thandrus 19 May 2010, 18:34
MAIN KAMPF

ეს ასეთი უცნაური ჩრდილი რატომაა? user.gif

Posted by: donvaso 19 May 2010, 19:54
MAIN KAMPF
მარჯვნივ,პლევრაშია ჰაერი, ჰილტვი ისე საცოდავაა ჩაფუშული რომ......
ხო ლიქვორიცაა ცოტა.... sad.gif



Posted by: Blind_Torture_Kill 19 May 2010, 20:53


QUOTE
მარჯვნივ,პლევრაშია ჰაერი, ჰილტვი ისე საცოდავაა ჩაფუშული რომ...... ხო ლიქვორიცაა ცოტა....

მარჯვენამხრივი პნევმოთორაქსი და შეხორცებებიც არის მგონი

Posted by: MAIN KAMPF 19 May 2010, 23:30
დიახ პნევმოთორაქსია biggrin.gif .........

Posted by: lgogokhia 21 May 2010, 21:31
მოკლედ, ეხლა კიდევ ახალი საინტერესო ქეისი smile.gif

33 წლის ჩინელ/ამერიკელი ქალი შემოვიდა მუდმივი/მოპულსირე თავის ტკივილით და 150/100 წნევით მიუხედავად ანტიჰიპერტენზიული მკურნალობისა. 19 წლის ასაკში იგივე სიმპტომებით მიმართა კლინიკას, როდესაც აღმოჩნდა, რომ მისი წნევა იყო 200/140. ბიოქიმიური გამოკვლევები იყო ნორმალური, გარდა ერთადერთი ეპიზოდისა, როდესაც კალიუმი იყო 3.2. ექოკარდიოგრაფია ნორმალური იყო, აორტის კოარქტაციის ნიშნების გარეშე. მაშინ დაესვა ესენციური ჰიპერტენზიის დიაგნოზი და დაენიშნა ატენოლოლი და ვალსარტანი, რომელმაც მისი წნევები დაარეგულირა 130-145/100-105მდე. 1 წლის წინ გათხოვდა რის გამოც გადაწყვიტა ანტიჰიპერტენზიული პრეპარატების შეცვლა ორსულობისთვის მოსამზადებლად, თუმცა ამის შემდეგ მისი არტერიული წნევა უკიდურესად ძნელი სამართავი გახდა. დღეისათვის ავადმყოფს ეძლევა ამლოდიპინი, ატენოლოლი და მეთილდოპა.

smile.gif

Posted by: Cousteau 21 May 2010, 22:20
QUOTE (lgogokhia @ 21 May 2010, 21:31 )
მოკლედ, ეხლა კიდევ ახალი საინტერესო ქეისი smile.gif

33 წლის ჩინელ/ამერიკელი ქალი შემოვიდა მუდმივი/მოპულსირე თავის ტკივილით და 150/100 წნევით მიუხედავად ანტიჰიპერტენზიული მკურნალობისა. 19 წლის ასაკში იგივე სიმპტომებით მიმართა კლინიკას, როდესაც აღმოჩნდა, რომ მისი წნევა იყო 200/140. ბიოქიმიური გამოკვლევები იყო ნორმალური, გარდა ერთადერთი ეპიზოდისა, როდესაც კალიუმი იყო 3.2. ექოკარდიოგრაფია ნორმალური იყო, აორტის კოარქტაციის ნიშნების გარეშე. მაშინ დაესვა ესენციური ჰიპერტენზიის დიაგნოზი და დაენიშნა ატენოლოლი და ვალსარტანი, რომელმაც მისი წნევები დაარეგულირა 130-145/100-105მდე. 1 წლის წინ გათხოვდა რის გამოც გადაწყვიტა ანტიჰიპერტენზიული პრეპარატების შეცვლა ორსულობისთვის მოსამზადებლად, თუმცა ამის შემდეგ მისი არტერიული წნევა უკიდურესად ძნელი სამართავი გახდა. დღეისათვის ავადმყოფს ეძლევა ამლოდიპინი, ატენოლოლი და მეთილდოპა.

smile.gif

24 საათიანი შარდის ანალიზი ხო არ ჩატარებია კატექოლამინებზე როდესმე?

სხვა საერთოდ არაფერი არ აწუხებს?

Posted by: lgogokhia 21 May 2010, 22:43
Cousteau
QUOTE
24 საათიანი შარდის ანალიზი ხო არ ჩატარებია კატექოლამინებზე როდესმე?

საერთო კატექოლამინები შარდში (N+NE) - 65 ug/24hr
QUOTE
სხვა საერთოდ არაფერი არ აწუხებს?

არაფერს აღნიშნავს. კონკრეტულად თუ რამე შეკითხვა გაქვს გიპასუხებ smile.gif

Posted by: MAIN KAMPF 21 May 2010, 23:06
შეკითხვის არსი რაშია როგორ ვუმკურნალოთ პაციენტს თუ რა გამოკვლევები ჩავუტაროთ? აქ გაუგებარია არის თუ არა ორსულად, რახანც მეთილდოპას ღებულობს სავარაუდოდ ორსულია, თუ ორსულია რომელ ტრიმესტრშია, თუ არის დამატებითი ანალიზები. რახანც კალიუმი გვაქვს გამოყოფილი ამ შემთხვევაში გავაკეთებდი ანალიზს ალდოსტერონზე ასევე თირკმლების ექოსკოპია არტერიის დოპლერით. ესენციური ჰიპერტენზია რამდენადაა აქ !თუ დადებითი პასუხი იქნა შეიძლება ჰიპერალდოსტერინიზმზე ვილაპარაკოთ. ჰო რა ვიცი ესაა ჩემი აზრი biggrin.gif

Posted by: Blind_Torture_Kill 21 May 2010, 23:38
lgogokhia


QUOTE
თუ დადებითი პასუხი იქნა შეიძლება ჰიპერალდოსტერინიზმზე ვილაპარაკოთ. ჰო რა ვიცი ესაა ჩემი აზრი


მეც მაგ აზრზე ვარ

Posted by: lgogokhia 22 May 2010, 00:16
MAIN KAMPF
QUOTE
შეკითხვის არსი

QUOTE
როგორ ვუმკურნალოთ პაციენტს თუ რა გამოკვლევები ჩავუტაროთ?
"თუ"-ს ნაცვლად "და" ჩასვი და შემოატრიალე smile.gif
QUOTE
აქ გაუგებარია არის თუ არა ორსულად

არ არის.
QUOTE
ალდოსტერონზე

=57 ng/dL
QUOTE
თირკმლების ექოსკოპია არტერიის დოპლერით

არ გამოვლინდა ჰემოდინამიკურად მნიშვნელოვანი სტენოზი თირკმლის არტერიებში. დანარჩენი ნორმალური..

QUOTE
ჰო რა ვიცი ესაა ჩემი აზრი

მიღებულია შენი კომენტარი biggrin.gif

P.S. ექოსკოპიაზე უნდა გამეკეთებინა Ctrl+B მაგრამ ვერ ვაკეთებ smile.gif

Posted by: Blind_Torture_Kill 22 May 2010, 01:57
lgogokhia
რაოდენობებს ნორმა მიუთითე ხოლმე

მომატებულია ალდოსტერონი
თირკმელზედა ჯირკვლები რა მდგომარეობაშია

Posted by: lgogokhia 22 May 2010, 05:19
Blind_Torture_Kill
QUOTE
რაოდენობებს ნორმა მიუთითე ხოლმე

no.gif მაგის მოძებნაზე იმდენი ვნახე საინტერესო, რომ ვერ დაგიკარგავდი opportunity-ს smile.gif

QUOTE
თირკმელზედა ჯირკვლები რა მდგომარეობაშია


QUOTE
ექოსკოპია არტერიის დოპლერით
რამე მნიშვნელოვანი არ გამოვლინდა..

რომ დავწერო ყველაფერი აზრი არ აქვს, რადგან მარტივია, მაგრამ tricky და იშვიათი, როგორიც ზემოთ ქეისი იყო. ამიტომ მოდით ასე გავაკეთოთ რომ "გამოიკვლიეთ" რეალურ სიტუაციაში და მე გეტყვით გამოკვლევის პასუხებს.. ერთ რეალურ შენიშვნას მივცემ რამდენიმეს, ჩემი ჩათვლით.. იმის გამო რომ რამდენიმე არ ვართ კლინიცისტები, საკითხს ვუდგებით ძალიან ქრესტომათიულად, ანუ პირდაპირ ვეძებთ იმ hint-ებს, რომელიც ამოცანას მარტივად ამოხსნის.. განსხვავებით კლინიცისტებისგან (არ ჩამოვთვლი ყველას, მაგრამ ბოლო რამდენიმე პოსტის მიხედვით მაგალითად MAIN KAMPF ან Professor Xachikian), რომელთაც დიფ-დიაგნოსტიკის და გამოკვლევის ჩატარების თანმიმდევრობის ცოდნის გამო ამოცანას მარტივად ხსნიან smile.gif ანუ კლინიკური აზროვნების უნარი უნდა დავხვეწოთ, რომელიც მნიშვნელოვანია CS-ისთვის, თუ ვინმე დარდობს მაგაზე.. (მაგალიტად მე).. იმედია და დარწმუნებული ვარ ასეთი მათხოჯი ქეისები არ იქნება მანდ, "ოცნებების რექვიემში", მაგრამ მსგავსი სიტუაციები უფრო ეფექტურად "გვასწავლის" ბევრ რამეს.. მე კი არ გასწავლით რამეს, უბრალოდ გიზიარებთ იმ შენიშვნებს რომელსაც მე მაძლევს ჩემზე ჭკვიანი ხალხი wink.gif

Posted by: LUKA-BRAZI 22 May 2010, 12:27
lgogokhia
QUOTE
რომ დავწერო ყველაფერი აზრი არ აქვს, რადგან მარტივია, მაგრამ tricky და იშვიათი, როგორიც ზემოთ ქეისი იყო. ამიტომ მოდით ასე გავაკეთოთ რომ "გამოიკვლიეთ" რეალურ სიტუაციაში და მე გეტყვით გამოკვლევის პასუხებს.. ერთ რეალურ შენიშვნას მივცემ რამდენიმეს, ჩემი ჩათვლით.. იმის გამო რომ რამდენიმე არ ვართ კლინიცისტები, საკითხს ვუდგებით ძალიან ქრესტომათიულად, ანუ პირდაპირ ვეძებთ იმ hint-ებს, რომელიც ამოცანას მარტივად ამოხსნის.. განსხვავებით კლინიცისტებისგან (არ ჩამოვთვლი ყველას, მაგრამ ბოლო რამდენიმე პოსტის მიხედვით მაგალითად MAIN KAMPF ან Professor Xachikian), რომელთაც დიფ-დიაგნოსტიკის და გამოკვლევის ჩატარების თანმიმდევრობის ცოდნის გამო ამოცანას მარტივად ხსნიან  ანუ კლინიკური აზროვნების უნარი უნდა დავხვეწოთ, რომელიც მნიშვნელოვანია CS-ისთვის, თუ ვინმე დარდობს მაგაზე.. (მაგალიტად მე).. იმედია და დარწმუნებული ვარ ასეთი მათხოჯი ქეისები არ იქნება მანდ, "ოცნებების რექვიემში", მაგრამ მსგავსი სიტუაციები უფრო ეფექტურად "გვასწავლის" ბევრ რამეს.. მე კი არ გასწავლით რამეს, უბრალოდ გიზიარებთ იმ შენიშვნებს რომელსაც მე მაძლევს ჩემზე ჭკვიანი ხალხი

ეუფ biggrin.gif

vano_t -ს დამიძახეთ! smile.gif)

არ შეიძლება ორსულთა ჰიპერტენზია იყოს? თანაც ისედაც ქონდა წნევები და ამას დაემატა ორსულთა ტრანზიტორული ჰიპერტენზია.. ჰა? smile.gif)


* * *
QUOTE
P.S. ექოსკოპიაზე უნდა გამეკეთებინა Ctrl+B მაგრამ ვერ ვაკეთებ

მაგას ვერც იზამ, [b][/b] ამ ტეგებს შორის უნდა მოაქციო ის სიტყვა, რომლის გამუქებაც გინდა smile.gif

Posted by: Thandrus 22 May 2010, 13:35
LUKA-BRAZI

QUOTE
არ შეიძლება ორსულთა ჰიპერტენზია იყოს?


არ არის ორსულადო...


* * *
lgogokhia

კარგი, მაშინ რენინი უნდა გავზომოთ ყველა ვარიანტში და მერე შედეგის მიხედვით ვიმოქმედოთ...

Posted by: lgogokhia 22 May 2010, 20:44
LUKA-BRAZI
QUOTE
vano_t -ს დამიძახეთ!

აშკარად!! biggrin.gif biggrin.gif

QUOTE
არ შეიძლება ორსულთა ჰიპერტენზია იყოს?

ვა რე ორსულ'ო, ვაარეეეეეე !!! biggrin.gif biggrin.gif

Thandrus
QUOTE
არგი, მაშინ რენინი უნდა გავზომოთ ყველა ვარიანტში და მერე შედეგის მიხედვით ვიმოქმედოთ...

yes.gif
რენინის აქტივობა ნორმალური ნატრიუმის დიეტის პირობებში პირაღმა დაწოლილი (ანუ supine biggrin.gif ) მომატებული იყო ორივე მხარეს (6.4 ng/mL
მარჯვენა თირკმლის ვენაში 7.6 ng/dl მარცხენა თირკმლის ვენა) და ქვემო ღრუ ვენაში (6.3 ng/ml), ასევე random რენინის აქტოვობა პლაზმაში (38ნგ/მლ)... tongue.gif

P.S. ისე random როგორაა ქართულად?

Posted by: Sophist 22 May 2010, 21:10
ბისეპტოლმა ნაგვლის ბუშტის ან ღვიძლის გაღიზიანება იცის?

Posted by: MAIN KAMPF 22 May 2010, 21:18
QUOTE
ბისეპტოლმა ნაგვლის ბუშტის ან ღვიძლის გაღიზიანება იცის?

ეს არის ამ ქეისის პასუხი
lol.gif

Posted by: Thandrus 23 May 2010, 10:22
Sophist

QUOTE
ბისეპტოლმა ნაგვლის ბუშტის ან ღვიძლის გაღიზიანება იცის?


"გაღიზიანებაში" რას გულისხმობ, არ ვიცი. ისე, ყოფილა შემთხვევები, რომ ბისეპტოლის მიცემის შემდეგ ღვიძლის ფერმენტები მომატებულა ან სიყვითლე წარმოშობილა.

lgogokhia

QUOTE
რენინის აქტივობა

QUOTE
მომატებული იყო


ანუ, ისევ დავბრუნდით, საიდანაც დავიწყეთ - მეორად ჰიპერალდოსტერონიმზე. ნუ ჰიპერალდოსტერონიზმი + ჰიპერტენზია, შეიძლება თირკმლის არტერიებშია რამე პრობლემა... გამოვიკვლიოთ.,

Posted by: Blind_Torture_Kill 23 May 2010, 10:49
რენინის წარმომმქემნელი რაიმე სიმსივნე იქნება

QUOTE
QUOTEბისეპტოლმა ნაგვლის ბუშტის ან ღვიძლის გაღიზიანება იცის?

ეს არის ამ ქეისის პასუხი


lol.gif lol.gif lol.gif lol.gif lol.gif

Posted by: Thandrus 23 May 2010, 11:05
Blind_Torture_Kill

QUOTE
რენინის წარმომმქემნელი რაიმე სიმსივნე იქნება


ჰო, ეგეც შეიძლება იყოს, მაგრამ რადგანაც ეგ იშვიათი რამეა, ამიტომ თირკმლის არტერიის პრობლემების გამორიცხვა მინდა...

Posted by: Blind_Torture_Kill 23 May 2010, 11:21
QUOTE
QUOTEთირკმლების ექოსკოპია არტერიის დოპლერით

არ გამოვლინდა ჰემოდინამიკურად მნიშვნელოვანი სტენოზი თირკმლის არტერიებში. დანარჩენი ნორმალური..



Posted by: Thandrus 23 May 2010, 11:27
Blind_Torture_Kill

უუუპს, გამომრჩენია...

მაშინ ალბათ გეთანხმები... მაგრამ ისიც გასათვალისწინებელია, რომ ამ ქალს ძალიან დიდი ხნის განმავლობაში აქვს ეს წნევა. შეიძლება სიმსივნემ ამდენი ხნის განმავლობაში მხოლოდ ჰიპერალდოსტერონიზმი მოგცეს (მართალია, პატარა სიმსივნეებს შეუძლიათ ამის გაკეთება, მაგრამ 16 წელი მაინც ძალიან დიდი დროა...)?

Posted by: MAIN KAMPF 23 May 2010, 13:45
QUOTE
ვა რე ორსულ'ო, ვაარეეეეეე

სორექ წიე, ქოთქვი მუშარე საქმე biggrin.gif

Posted by: lgogokhia 24 May 2010, 21:48
MRA (magnetic resonance angiogram) გამოავლინა 6X6მმ ზომის კისტა მარცხენა თირკმლის ზედა პოლუსზე (ექსოკოპიამ რატომ ვერ დაინახა არ ვიცი, ვერ ხედავს მაგ ზომის რაღაცეებს?).
QUOTE
რენინის წარმომმქემნელი რაიმე სიმსივნე
დ ჩაითვალა და გაუკეთდა ლაპარასკოპიული ნაწილობრივი ნეფრექტომია. მდგომარეობის შემდგომი გაუმჯობესებით

MAIN KAMPF
QUOTE
სორექ წიე, ქოთქვი მუშარე საქმე 

პათომორფოლოგიურად კი აღმოჩნდა "იუქსტაგლ'ომერულარულ'ი უჯრედიში "უსაშველო ლახალა" " biggrin.gif biggrin.gif

Posted by: MAIN KAMPF 24 May 2010, 23:12
QUOTE
გამოავლინა 6X6მმ ზომის კისტა მარცხენა თირკმლის ზედა პოლუსზე

პოლიკისტოზს შეუძლია გამოიწვიოს ჰიპერტენზია, მაგრამ ასე მცირე ზომის კისტას? არ ვიცი.თუ არის ინფორმაცია მეორე თირკმელზეც ხომ არ არის რაიმე მსგავსი წარმონაქმნი, ასევე თუ არის კიდევ დამატებითი ტესტები რაც აქ არ დაწერილა. რამდენი ხნის დაკვირვებაა პაციენტზე. პლიიიზ

Posted by: lgogokhia 24 May 2010, 23:35
MAIN KAMPF
მასალის პათომორფოლოგიური გამოკვლევით გამოვლინდა იუქსტაგლომერულური უჯრედის სიმსივნე, რომელზეც ოპერაციის წინაც გამოთქვა ვარაუდი. რენინის აქტოვობები სელექციურად ორივე ვენაში იყო გაზომილი შესაბამისი მედიკამენტური მომზადების შემდეგ და მარცხენა მხარეს ყველა შემთხვევაში უფრო მომატებული იყო ვიდრე მარჯვენაში. ასევე მომატებული იყო ალდოსტერონის ექსკრეცია შარდში, ამიტომ ოპერაციისწინა დიაგნოზი იყო აშკარა - რენინმაპროდუცირებელი სიმსივნე. ოპერაციიდან მეორე დღეს რენინის აქტივობა undetectable იყო და ალდოსტერონიც ნორმაში ჩამოვიდა. აქ უბრალოდ ის იყო საეჭვო, რომ რატომ ამ 18 წლის განმავლობაში არ დადგინდა ეს და რატომ დატანჯეს ეს ქალი.. პოლიკისტოზი არ იყო, არამედ ერთი contrast enhancing მასა მარცხენა თირკმლის ზედა პოლუსზე.

Posted by: Thandrus 4 Jun 2010, 03:51
აბა ერთი ქეისი წიგნიდან (ჩემი ადაპტაციით gigi.gif )

8 წლის. მწვავე მიელოიდური ლეიკემიით (AML) ჩინელ დაავადებულ ჩინელ გოგოს, ამ ავადმყოფობის მკურნალობის დროს, განმეორებადი სოკოვანი და ბაქტერიული ინფექციების ხანგრძლივი და რთული ისტორია აქვს. ინფეციებში შედიოდა ფილტვის ასპერგილოზი და სხვადასხვა დროს სხვადასხვა გრამ-პოზიტიური და ნეგატიური ბაქტერიებით გამოწვეული სეპტიცემია. მეროპენემით და ამიკაცინით მკურნალობის დროს ამ გოგოს განუვითარდა სეპტიცემია და ეს ბაქტერია (რომელიც უნდა დაასახელოთ) ამოითესა. რომელიც ტრიმეთოპრიმ-სულფამეტოქსაზოლზე სენსიტიური იყო.. რამდენიმე დღის შემდეგ, მას განუვითარდა მტკივნეული, ერითემული, კვანძოვანი კანის დაზიანებები. კანის ბიოფსიითაც ეს ბაქტერია ამოითესა. ინტრავენული ტრიმეთოპრიმ-სულფამეტოქსაზოლით მკურნალობის შედეგად კანის დაზიანებები ნელ-ნელა განიკურნა და ბაქტერემია გაქრა.

ჰოდა, ახლა დაასახელეთ ბაქტერია, რომელიც ყველაზე მეტი შანსია რომ ამ ყველაფრის გამომწვევი იყოს smile.gif

Posted by: Roller777 4 Jun 2010, 05:26
ვააა.... ჰაუსის ჯგუფის მსჯელობას მაგონებს ეს თემა biggrin.gif

13-ის როლში რომელი ხართ, გამოდი სიყვარული უნდა აგიხსნა love.gif

Posted by: idallgo 4 Jun 2010, 18:54
QUOTE
8 წლის. მწვავე მიელოიდური ლეიკემიით (AML) ჩინელ დაავადებულ ჩინელ გოგოს, ამ ავადმყოფობის მკურნალობის დროს, განმეორებადი სოკოვანი და ბაქტერიული ინფექციების ხანგრძლივი და რთული ისტორია აქვს. ინფეციებში შედიოდა ფილტვის ასპერგილოზი და სხვადასხვა დროს სხვადასხვა გრამ-პოზიტიური და ნეგატიური ბაქტერიებით გამოწვეული სეპტიცემია. მეროპენემით და ამიკაცინით მკურნალობის დროს ამ გოგოს განუვითარდა სეპტიცემია და ეს ბაქტერია (რომელიც უნდა დაასახელოთ) ამოითესა. რომელიც ტრიმეთოპრიმ-სულფამეტოქსაზოლზე სენსიტიური იყო.. რამდენიმე დღის შემდეგ, მას განუვითარდა მტკივნეული, ერითემული, კვანძოვანი კანის დაზიანებები. კანის ბიოფსიითაც ეს ბაქტერია ამოითესა. ინტრავენული ტრიმეთოპრიმ-სულფამეტოქსაზოლით მკურნალობის შედეგად კანის დაზიანებები ნელ-ნელა განიკურნა და ბაქტერემია გაქრა.

ჰოდა, ახლა დაასახელეთ ბაქტერია, რომელიც ყველაზე მეტი შანსია რომ ამ ყველაფრის გამომწვევი იყოს 


Stenotrophomonas Maltofilia ამინოგლიკოზიდებზე და კარბაპენემებზე რეზისტენტულია და ტრიმეთოპრიმ-სულფამეტოქსაზოლზე სენსიტიურია, ასევე კანის ნოდულარულ დაზიანებებსაც იწვევს.

Posted by: Thandrus 4 Jun 2010, 20:29
idallgo

QUOTE
Stenotrophomonas Maltofilia ამინოგლიკოზიდებზე და კარბაპენემებზე რეზისტენტულია და ტრიმეთოპრიმ-სულფამეტოქსაზოლზე სენსიტიურია, ასევე კანის ნოდულარულ დაზიანებებსაც იწვევს


SUPER!

ზუსტად გააკეთეთ ის აქცენტები, რაზეც მინდოდა, რომ ყურადღება მიგექციათ smile.gif

Posted by: Archer 5 Jun 2010, 05:05
თქვენა და რა მაინტერესებს ასე ყველა თემაზე რომ მსჯელობთ აქ ექიმები... ყველაფერი საიდან იცით? smile.gif მესმის თავის სპეციალობაში იყოს პროფესიონალი კაცი, მაგრამ ყველაფერში? smile.gif არ არის ბუნებრივი. ჰაუსი მარტო კინოშია smile.gif

Posted by: MAIN KAMPF 5 Jun 2010, 17:06
QUOTE
თქვენა და რა მაინტერესებს ასე ყველა თემაზე რომ მსჯელობთ აქ ექიმები... ყველაფერი საიდან იცით?  მესმის თავის სპეციალობაში იყოს პროფესიონალი კაცი, მაგრამ ყველაფერში?  არ არის ბუნებრივი. ჰაუსი მარტო კინოშია 

yes.gif
პრინციპში მართალია ეს პატიოსანი ადამიანი, იუესემელის ან პლაბის კითხვარებიდან ვეცადოთ ნაკლებად იყოს დასმული შეკითხვები ანდა ცალკე თემა გაკეთდეს არც ეგ იქნება ურიგო. აქ ისეთი შეკითხვები ისმევა რაც პრაქტიკაში ძალიან იშვიათად შეიძლება შეგვხვდეს. როგორც ერთი აქაური პროფესორი ამბობს ''რენდიუ ოსლერი'' უფრო მეტად საგამოცდო ბილეთებშია ვიდრე პრაქტიკაში. მე ვცდილობ დავდო ისეთი შემთხვევები რაც შემხვდა ასევე ვცდილობ ფოტო სურათებიც მოვაყოლო. ამ შეკითხვის ავტორს ვეტყვი რომ მართალია ყველაფერი არ შეიძლება იცოდე, რაც არ ვიცი წიგნიდან ვცდილობ დავამატო. ჰაუზი აქ არავინაა მაგრამ ამ თემაში მონაწილე ექიმები საკმარისზე კარგად არიან მომზადებულები yes.gif

Posted by: Cousteau 5 Jun 2010, 18:23
QUOTE (Archer @ 5 Jun 2010, 05:05 )
თქვენა და რა მაინტერესებს ასე ყველა თემაზე რომ მსჯელობთ აქ ექიმები... ყველაფერი საიდან იცით? smile.gif მესმის თავის სპეციალობაში იყოს პროფესიონალი კაცი, მაგრამ ყველაფერში? smile.gif არ არის ბუნებრივი. ჰაუსი მარტო კინოშია smile.gif

აქ რა კითხვებიც იდეაბა ნებისმიერმა ექიმმა უნდა გასცეს პასუხი, რეალურად არანაირი სუპერგანსაკუთრებული ქეისები აქ არ იდება (ნუ ძალიან ძალიან იშვიათად იდება, ალბათ კითხვების 5-10% მხოლოდ)
ჩემდა სამწუხაროდ ვაღიარებ მაგას, რაც აქ იდება ყველაფერს ვერ ვცემ პასუხს ნამდვილად

QUOTE
რაც არ ვიცი წიგნიდან ვცდილობ დავამატო

უნიში, სულ 2-3 მართლა კარგი ექიმი მასწავლიდა, აქედან ორმა ეგეთი რაღაც მითხრა, ერთმა: უნის რო დაამთავრებ, თუ გეცოდინება დასმული კითხვის პასუხი ზუსტად სად უნდა მოძებნო წიგნში მაშინ ტყულად არ გისწავლია.

მეორემ კიდე (ძალიან ძლიერი ინფექციონისტია, ისეთი რო მშურდა კიდეც biggrin.gif) ''სტაფილოკოკის'' დონის ქეისებზე წიგნში არ ვიხედებიო, დანარჩენ შემთხვევებში პაციენტთან სულ წიგნით შევდივარო. არადა ამ კაცმა რამდენიც იცის თავის საქმეში მაგდენს როცა ვისწავლი თავი ნახევარღმერთი მეგონება gigi.gif

ნუ მორალი: წიგნში ჩახედვა არ ტეხავს (იმჰო)


Posted by: Thandrus 5 Jun 2010, 20:10
Archer

QUOTE
თქვენა და რა მაინტერესებს ასე ყველა თემაზე რომ მსჯელობთ აქ ექიმები... ყველაფერი საიდან იცით?მესმის თავის სპეციალობაში იყოს პროფესიონალი კაცი, მაგრამ ყველაფერში?არ არის ბუნებრივი. ჰაუსი მარტო კინოშია


კუსტოს პასუხს ვეთანხმები - კითხვების უმრავლესობა არ არის ისეთი, რომ მაინცადამაინც ვიწრო და ღრმა სპეციალისტი დასჭირდეს. აქ უფრო მეტ როლს მოსაზრება და სხარტი აზროვნება თამაშობს, ვიდრე ყველაფრის დეტალურად ცოდნა (ნუ რათქმაუნდა, ექიმმა ძალიან ბევრი უნდა იცოდეს ამის გარეშეც). რასაც ჩვენ თვითონ პასუხს ვერ ვცემთ, სხვადასხვა რესურსებსაც (წიგნებს, ინტერნეტს) თავისუფლად ვიყენებთ. ვინაიდან აქ არ არის შეჯბრება ვინ ვის სჯობს (მხოლოდ ჯანსაღი აზარტი ქეისის ამოხსნის), წიგნში ჩახედვა ყველანაირად მისასალმებელია; ჩვენ ყველა აქ ბევრ რამეს ვსწავლობთ ერთმანეთისგან. მართლაც, ამ თემისგან უფრო მეტი სარგებელი მივიღე, ვიდრე წელს გავლილი რამდენიმე კურაციისგან smile.gif

MAIN KAMPF

QUOTE
პრინციპში მართალია ეს პატიოსანი ადამიანი, იუესემელის ან პლაბის კითხვარებიდან ვეცადოთ ნაკლებად იყოს დასმული შეკითხვები ანდა ცალკე თემა გაკეთდეს არც ეგ იქნება ურიგო. აქ ისეთი შეკითხვები ისმევა რაც პრაქტიკაში ძალიან იშვიათად შეიძლება შეგვხვდეს. როგორც ერთი აქაური პროფესორი ამბობს ''რენდიუ ოსლერი'' უფრო მეტად საგამოცდო ბილეთებშია ვიდრე პრაქტიკაში. მე ვცდილობ დავდო ისეთი შემთხვევები რაც შემხვდა ასევე ვცდილობ ფოტო სურათებიც მოვაყოლო. ამ შეკითხვის ავტორს ვეტყვი რომ მართალია ყველაფერი არ შეიძლება იცოდე, რაც არ ვიცი წიგნიდან ვცდილობ დავამატო. ჰაუზი აქ არავინაა მაგრამ ამ თემაში მონაწილე ექიმები საკმარისზე კარგად არიან მომზადებულები


რატომ, ერთი მეორეს სულაც არ უშლის. თან მრავალფეროვნებაცაა და თემატურათაც თემას შეესაბამება.

Cousteau

QUOTE
უნიში, სულ 2-3 მართლა კარგი ექიმი მასწავლიდა, აქედან ორმა ეგეთი რაღაც მითხრა, ერთმა: უნის რო დაამთავრებ, თუ გეცოდინება დასმული კითხვის პასუხი ზუსტად სად უნდა მოძებნო წიგნში მაშინ ტყულად არ გისწავლია.


ეს გვითხრა ჩვენმა საყვარელმა დეკანმა ინსტიტუტის პირველ გაკვეთილზე biggrin.gif

QUOTE
ნუ მორალი: წიგნში ჩახედვა არ ტეხავს (იმჰო)


ჩემი აზრით, ნებისმიერ კეთილსინდისიერ ექიმს მაგიდაზე მისი სპეციალობის (და ალბათ საერთო შინაგანი მედიცინის) უახლესი დეტალური სახელმძღვანელოები უნდა ედოს. ეს მხოლოდ რამდენიმე ექიმთან მინახავს და ის ექიმები ძალიან დავაფასე.

Posted by: Archer 5 Jun 2010, 20:17
ხო რა ვიცი არნოლდ-კიარის პათოლოგია ნევროლოგის და ნეიროქირურგის გარდა სხვას რომ ეცოდინება ქების ღირსია უბრალოდ smile.gif

Posted by: Thandrus 5 Jun 2010, 20:18
Archer

მეორე კურსზე გავდივართ ამას, პათოლოგიაში biggrin.gif

Posted by: Archer 5 Jun 2010, 20:23
Thandrus
იცი ემბრიოლოგიაში რამდენი რამე მაქვს ნასწავლი და სულ ხუთებზე, მაგრამ გინეკოლოგიამდე რომ მივედი კი აღარ მახსოვდა...

Posted by: Cousteau 5 Jun 2010, 21:07
QUOTE (Archer @ 5 Jun 2010, 20:23 )
მაგრამ გინეკოლოგიამდე რომ მივედი კი აღარ მახსოვდა...

გინეკოლოგიამდე რო მივედი ცხოვრება აღარ მინდოდა gigi.gif

whats this?
user posted image

Posted by: Archer 6 Jun 2010, 17:41
Cousteau
ეგ ჩემი აზრით არის გენეტიკური დაავადება, რომლის ერთ-ერთი სიმპტომია ლიპომა სკლერაზე. )

Posted by: Cousteau 7 Jun 2010, 09:08
Congenital Palpebral Coloboma როგორც წესი დაკავშირებული Dermoid წიმწივნესთან

Posted by: LUKA-BRAZI 7 Jun 2010, 11:18
wassup niggas?! biggrin.gif

მე ვთვლი რომ მნიშვნელობა არ აქვს ქეისებს, ძნელი იქნება თუ ადვილი, სპეციფიკური თუ ზოგადი და ა.შ. მთავარია იყოს მსჯელობა, ჯანსაღი დისკუსია..... მახსოვს ერთი პერიოდი ღამეებს რომ ვათენებდით გამოცნობაში ხოლმე smile.gif მაშინ vano_t-ც ხშირად პოსტავდა და Guardian-იც ფორუმზე იყო......


Posted by: Thandrus 7 Jun 2010, 19:05
Cousteau

QUOTE
Congenital Palpebral Coloboma როგორც წესი დაკავშირებული Dermoid წიმწივნესთან


ვაჰ... კოლობომაა ეგ? user.gif მე მარტო ირისზე მეგონა რომ კოლობომა არსებობდა... ვიკიზე ვნახე, თურმე თვალის სხვა ნაწილებშიც შეიძლება რომ იყოს. კარგი იყო - ჩემი ტვინის ყულაბაში ერთი ოცთეთრიანი ჩავარდა biggrin.gif

LUKA-BRAZI

QUOTE
მე ვთვლი რომ მნიშვნელობა არ აქვს ქეისებს, ძნელი იქნება თუ ადვილი, სპეციფიკური თუ ზოგადი და ა.შ. მთავარია იყოს მსჯელობა, ჯანსაღი დისკუსია..... მახსოვს ერთი პერიოდი ღამეებს რომ ვათენებდით გამოცნობაში ხოლმემაშინ vano_t-ც ხშირად პოსტავდა და Guardian-იც ფორუმზე იყო......


ეს თემა უფრო საინტერესო ხდება მაშინ, როდესაც კლინიცისტები დებენ თავის ქეისებს. ასე უფრო ინტერესდები, რადგან იცი, რომ არსად წიგნში ეს ქეისი ამოკითხული არ გექნება biggrin.gif სტუდენტები რა ქეისებსაც ვდებთ, უმეტესობა ქრესტომათიულია და ამიტომ, ხშირ შემთხვევაში, გამოცდილი ექიმებისთვის პრობლემას არ წარმოადგენს. თუმცა, მე ვთვლი, რომ სჯობს ხშირად დავდოთ ასეთი პატარა ქეისები, რომ თემა არ ჩაიძიროს.

* * *
მე დავდებ ერთს:

ხუთი თვის ბავშვი მოიყვანეს საავადმყოფოში სიცხის, ერთი კვირის განმავლობაში ღებინების და სამი დღის განმავლობაში კრუნჩხვების ისტორიით. შემოწმებისას, ბავშვის სიცე იყო 38,5'C, ჰქონდა გამუდმებული კრუნჩხვები და მისი თვალის გუგები რეაქტიული იყო. მისი ცნობა თანდათან უარესდებოდა და საბოლოოდ ბავშვი კომაში ჩავარდა. გულსისძარღვთა, სასუნთქი სისტემის, და მუცლის შემოწმებას არაფერი განსაკუთრებული არ გამოუვლენია. CT-მ აჩვენა ტვინის ბაზალური ცისტერნების აცლა (effacement) და თხელი დაბალი სიმკვრივის მასა ბილატერალურად ფრონტალურ გირუსებზე, რაც სუბდურულ ჰემატომაზ ან ეფუზიაზე მიანიშნებდა.

ისტორიაში საინტერესო იყო ის, რომ მშობლები შვილს ხელოვნური ჭის წყლით აბანავებდნენ.

ლაბორატორიული ანალიზების შედეგები

ლეიკოციტების მთლიანი რიცხვი - 7,300/mm3, ნეიტროფილები - 71%, ლიმფოციტები 26%, ეოზინოფილები 3%
ჰემოგლობინი - 10.2 g/dl
პერიფერიულ ნაცხში პარაზიტები არ გამოვლენილა.
ელექტროლიტები: ნატრიუმი - 133 mmol/liter, კალიუმი 4.4 mmol/liter, ქლორი 95 mmol/liter, ბიკარბონატი 18.6 mmol/liter, ფოსფორი - 4.7 mg/dl, BUN - 4 mg/dl, შრატში კალციუმის დონე 9.2 mg%,, გლიკემია - 120 mg/dl.

ცერებროსპინალური სითხე მღვრიე და ჰემორაგიული იყო. CSF გლუკოზის დონე - 10 mg/dl. ცილების დონე - 361 mg/dl,, უჯრედების რიცხვი 30 cells/mm3, ნეიტროფილები 80%, ლიმფოციტები - 20%

ჭის წყალში, რომელსაც ბავშვის დასაბანად იყენებდნენ, აღმოაჩინეს გამომწვევი.

მას დაეწყო მკურნალობა ცეფტრიაქსონით (100მგ/კგ) და ინტრავენული ამფოტერიცინ B-თი, მაგრამ ექიმების ცდისდა მიუხედავად, ბავშვი მიღების მეორე დღეს გარდაიცვალა.

დასვით დიაგნოზი და დაასახელეთ გამომწვევი!

თუ გაგიჭირდებათ, შემდეგ CSF-ში ნაპოვნი ორგანიზმის სურათსაც დავდებ.

Posted by: Blind_Torture_Kill 7 Jun 2010, 21:40
Thandrus

დადე სურათიც

Posted by: grigolich 7 Jun 2010, 21:55
42 წლის ქალს უკვე 10 წელია აქვს გულის უკმარისობის მოვლენები, ბოლო ერთი წელია მდგომარეობა მკვეთრად გაუარესდა,თან დაერთო ტკივილი გულმკერდის მიდამოში, რომელიც არ უკავშირდებოდა ფიზიკურ დატვირტვას. ასევე პერიოდულად, განსაკუთრებით საკვების მიღების შემდეგ, უჩიოდა ტკივილს მუცელში. ექოკარდიოგრაფიულად რაიმე პათოლოგია (მათ შორის სარქვლოვანი), არ გამოვლინდა. ყავს ორი შვილი, მშობიარობამ და ორსულობამ ჩაიარა გართულებების გარეშე. ერჩია კორონაროგრაფია, თუმცა პროცედურის ჩატარება ვერ მოხდა, რადგან ვერც ბარძაყის და ვერც რადიალური არტერიებიდან ვერ ჩატარდა კათეტერიზაცია, არტერიებში კონტრასტის შეშვებისას ჩანდა რომ ოკლუზირებული იყო 100%-ით.

Posted by: LUKA-BRAZI 7 Jun 2010, 21:59
grigolich
QUOTE
არტერიებში კონტრასტის შეშვებისას ჩანდა რომ ოკლუზირებული იყო 100%-ით.

რა იყო ოკლუზირებული 100%-ით?! კორონარები? smile.gif ალბათ ბარძაყის არტერია და რადიალური, მაგრამ ბარძაყის არტერიაზე კათეტერიზაცია საკმაოდ პროქსიმალურად კეთდება, და მაგის ზემოთ რომ კიდევ ოკლუზირებული იყოს, კიდურს რა კვებავს მაშინ? რა კოლატერალები უნდა ქონდეს?! მართლა 100% იყო?

Posted by: grigolich 7 Jun 2010, 22:03
კორონარებამდე ვერ ავიდა კატეტერი, და არც კონტრასტი არ გატარდა მუცლის აორტის დონეზე იკო ოკლუზია, ხო, კოლატერალები ალბატ იკო, ტორე ისე სავარაუდოდ ვერ იცოცხლებდა 10 წელი:
)

Posted by: Archer 7 Jun 2010, 22:05
ააა. მაშინ ეგ ტკივილები ნაწლავების იშემიით იქნება გამოწვეული

Posted by: LUKA-BRAZI 7 Jun 2010, 22:09
grigolich
მუცლის აორტის 100%-იანი ოკლუზია ჯერ ერთი არ გამიგია, მეორეც, მუცლის აორტა ერთადერთი გზა არის არტერიული სისხლისთვის მუცლის ორგანოების ვისცერული პერფუზიისთვის, კიდურებისთვის და ა.შ. ნუ არ ვიცი..... მსხვილი არტერიის სრული ოკლუზია ფატალურია........... არათუ აორტის......

Posted by: grigolich 7 Jun 2010, 22:10
კი, ნაწლავების იშემიაც არის, და რა იწვევა ამ ყველაფერს ერთად

Posted by: Archer 7 Jun 2010, 22:12
grigolich
100%-იანი ოკლუზია აორტის მართლა ფანტასტიკის სფეროა რა ვიცი ლუკა ბრაზის არ იყოს. smile.gif
ათეროსკლეროზი პირველ რიგში
LUKA-BRAZI
QUOTE
მსხვილი არტერიის სრული ოკლუზია ფატალურია

ჩემი ამბიდან გეტყვი რომ კისრის მსხვილი არტერიებიდან (წყვილი საძილე არტერია და წყვილი ხერხემლის არტერია) 3-ის სრული ოკლუზია ნელ-ნელა შეთავსებადია სიცოცხლესთან. ერთიც ყოფნის რომ მოამარაგოს ტვინი სისხლით თუ დრო მიეცა კოლატერალების გასახსნელად

Posted by: grigolich 7 Jun 2010, 22:15
თუ მწვავეა ოკლუზია, მაშინ ნამდვილად ფატალურიაsmile.gif თუმცა, მაგალითად თუ ლერიშის სინდრომს გავიხსენებთ, ზოგჯერ თითქმის 100% არის ხოლმე თეზოს არტერიები ოკლუზირებული, მაგრამ ადამიანს სიარული შეუძლია
* * *
42 წლის ქალს ნაკლებსავარაუდოა ისეთი ათეროსკლეროზი კონდეს, რომ ყველა მაგისტრალური სისხლძარღვი ჩაიკეტოს

Posted by: LUKA-BRAZI 7 Jun 2010, 22:21
Archer
ვილიზიის წრე მეც ვიცი როგორ მუშაობს biggrin.gif მაგრამ მუცლის აორტა?! ეგ ნაღდად პირველად გავიგე....... smile.gif რამ უნდა დაახშოს მაგხელა სანათური თუნდაც ქრონიკულად? ამდენი ათეროსკლეროზული ფოლაქი და ქილომიკრონების დალექვა....... მაგაზე ადრე, სხვა არტერიებს მოსპობდა........ ალბათ კომპრესია იყო... მაგრამ ეგეც წარმოუდგენელია... ნუ რა ვიცი smile.gif)
* * *
წავედი, ტესტები მაქვს სამეცადინო........... მოვალ მოგვიანებით...

Posted by: donvaso 7 Jun 2010, 22:25
grigolich
არასპეციფიკური აორტო-არტერიიტი?
კომპრესიას ხომ არ განიცდიდა? (ეს დაუწერიათ უკვე biggrin.gif )

Posted by: Blind_Torture_Kill 7 Jun 2010, 22:26
QUOTE
ჩემი ამბიდან გეტყვი რომ კისრის მსხვილი არტერიებიდან (წყვილი საძილე არტერია და წყვილი ხერხემლის არტერია) 3-ის სრული ოკლუზია ნელ-ნელა შეთავსებადია სიცოცხლესთან.


ეს მართლა ესეა ?

Posted by: Archer 7 Jun 2010, 22:27
grigolich
გენიტიკური წინასწარგანწყობა რომ არის ათეროსკლეროზის მაგ ასაკი შესაფერისია ფატალური გართულებებისთვისაც. ლიპიდოგრამა გააკეთეთ ყოველი შემთხვევისთვის

LUKA-BRAZI
ვილიზიის წრეს ვაჟაზე ასწავლიან smile.gif ქართულად ვილისის წრეა. ან უილისი. რუსულად вилизиев круг და ქართველებმა გადმოთარგმნისას კალკირება გაუკეთეს. biggrin.gif კი მოუსპია კორონარული არტერიები და რა ვიცი რამ მოსპო მაინცდამაინც smile.gif

Posted by: grigolich 7 Jun 2010, 22:31
ათეროსკლეროზისადმი მასეთი ხარისხის გენეტიკური დაავადება ანწყობა 42 წლამდე არ აცოცხლებდაsmile.gif თან X ქრომოსომასთან არის შეჭიდული და ქალებს ნაკლებად...

Posted by: Archer 7 Jun 2010, 22:31
Blind_Torture_Kill
ჩემი თვალით მაქვს ნანახი ეგეთი ნაინსულტარი პაციენტები. რა თქმა უნდა ცუდად აქვთ საქმე. დისცირკულატორული ენცეფალოპათია გაშვებულია (იგივე სისხლძარღვოვანი დემენცია) და სულ იმის საშიშროების წინაშე დგას რომ განმეორდება ინსულტი ან მეთხე არტერიის ბოლომდე დახშობაც მოხდება. იმიტომ რომ სამი თუ დაეხშო ე.ი. მეოთხეშიცაა ცვლილებები

Posted by: grigolich 7 Jun 2010, 22:35
ქალს 10 წელია ნელნელა უვიწრივდებოდა სისხლძარღვები, და როგორც საბოლოოდ გაირკვა კორონარები საერთოდ არ იყო ჩართული, უფრო დიდი კალიბრის სისხლძარღვები

Posted by: Archer 7 Jun 2010, 22:37
grigolich
ტაკაიასუ თუ გგონია მაშინ რევმატოლოგის კონსულტაცია
ისე თუ გული ნორმაშია უკმარისობა საიდან აქვს?

Posted by: grigolich 7 Jun 2010, 22:44
უკმარისობა იკიდან იყო რომ პოსტდატვირთვა იყო საშინლად გაზრდილიsmile.gif
კი, ტაკაიასუ ვივარაუდე და დაუდგინდა კიდეც

Posted by: Archer 7 Jun 2010, 22:48
grigolich
მერე ექოკარდიოგრაფიაზე არ გამოჩნდა ჰიპერტროფირებული კედლები გულის?

Posted by: Thandrus 7 Jun 2010, 23:57
აუჰ, აქ მთელი ამბები მოგისწრიათ ჩემს არყოფნაში biggrin.gif

ისე, წესად გვაქვს, რომ სანამ ერთი ქეისი არ დასრულება მეორეს არ ვდებთ ხოლმე, რომ არ აირიოს ერთმანეთში.

რადგან ეს კარდიოლოგიური ქეისი მორჩა, გადმოვიტან აქეთ ჩემს შემთხვევას.

---------------
ხუთი თვის ბავშვი მოიყვანეს საავადმყოფოში სიცხის, ერთი კვირის განმავლობაში ღებინების და სამი დღის განმავლობაში კრუნჩხვების ისტორიით. შემოწმებისას, ბავშვის სიცე იყო 38,5'C, ჰქონდა გამუდმებული კრუნჩხვები და მისი თვალის გუგები რეაქტიული იყო. მისი ცნობა თანდათან უარესდებოდა და საბოლოოდ ბავშვი კომაში ჩავარდა. გულსისძარღვთა, სასუნთქი სისტემის, და მუცლის შემოწმებას არაფერი განსაკუთრებული არ გამოუვლენია. CT-მ აჩვენა ტვინის ბაზალური ცისტერნების აცლა (effacement) და თხელი დაბალი სიმკვრივის მასა ბილატერალურად ფრონტალურ გირუსებზე, რაც სუბდურულ ჰემატომაზ ან ეფუზიაზე მიანიშნებდა.

ისტორიაში საინტერესო იყო ის, რომ მშობლები შვილს ხელოვნური ჭის წყლით აბანავებდნენ.

ლაბორატორიული ანალიზების შედეგები

ლეიკოციტების მთლიანი რიცხვი - 7,300/mm3, ნეიტროფილები - 71%, ლიმფოციტები 26%, ეოზინოფილები 3%
ჰემოგლობინი - 10.2 g/dl
პერიფერიულ ნაცხში პარაზიტები არ გამოვლენილა.
ელექტროლიტები: ნატრიუმი - 133 mmol/liter, კალიუმი 4.4 mmol/liter, ქლორი 95 mmol/liter, ბიკარბონატი 18.6 mmol/liter, ფოსფორი - 4.7 mg/dl, BUN - 4 mg/dl, შრატში კალციუმის დონე 9.2 mg%,, გლიკემია - 120 mg/dl.

ცერებროსპინალური სითხე მღვრიე და ჰემორაგიული იყო. CSF გლუკოზის დონე - 10 mg/dl. ცილების დონე - 361 mg/dl,, უჯრედების რიცხვი 30 cells/mm3, ნეიტროფილები 80%, ლიმფოციტები - 20%

ჭის წყალში, რომელსაც ბავშვის დასაბანად იყენებდნენ, აღმოაჩინეს გამომწვევი.

მას დაეწყო მკურნალობა ცეფტრიაქსონით (100მგ/კგ) და ინტრავენული ამფოტერიცინ B-თი, მაგრამ ექიმების ცდისდა მიუხედავად, ბავშვი მიღების მეორე დღეს გარდაიცვალა.

დასვით დიაგნოზი და დაასახელეთ გამომწვევი!

სურათსაც ვდებ.

Posted by: Cousteau 8 Jun 2010, 00:09
QUOTE (Thandrus @ 7 Jun 2010, 19:05 )
Cousteau

QUOTE
Congenital Palpebral Coloboma როგორც წესი დაკავშირებული Dermoid წიმწივნესთან


ვაჰ... კოლობომაა ეგ? user.gif მე მარტო ირისზე მეგონა რომ კოლობომა არსებობდა... ვიკიზე ვნახე, თურმე თვალის სხვა ნაწილებშიც შეიძლება რომ იყოს. კარგი იყო - ჩემი ტვინის ყულაბაში ერთი ოცთეთრიანი ჩავარდა biggrin.gif

LUKA-BRAZI

QUOTE
მე ვთვლი რომ მნიშვნელობა არ აქვს ქეისებს, ძნელი იქნება თუ ადვილი, სპეციფიკური თუ ზოგადი და ა.შ. მთავარია იყოს მსჯელობა, ჯანსაღი დისკუსია..... მახსოვს ერთი პერიოდი ღამეებს რომ ვათენებდით გამოცნობაში ხოლმემაშინ vano_t-ც ხშირად პოსტავდა და Guardian-იც ფორუმზე იყო......


ეს თემა უფრო საინტერესო ხდება მაშინ, როდესაც კლინიცისტები დებენ თავის ქეისებს. ასე უფრო ინტერესდები, რადგან იცი, რომ არსად წიგნში ეს ქეისი ამოკითხული არ გექნება biggrin.gif სტუდენტები რა ქეისებსაც ვდებთ, უმეტესობა ქრესტომათიულია და ამიტომ, ხშირ შემთხვევაში, გამოცდილი ექიმებისთვის პრობლემას არ წარმოადგენს. თუმცა, მე ვთვლი, რომ სჯობს ხშირად დავდოთ ასეთი პატარა ქეისები, რომ თემა არ ჩაიძიროს.

* * *
მე დავდებ ერთს:

ხუთი თვის ბავშვი მოიყვანეს საავადმყოფოში სიცხის, ერთი კვირის განმავლობაში ღებინების და სამი დღის განმავლობაში კრუნჩხვების ისტორიით. შემოწმებისას, ბავშვის სიცე იყო 38,5'C, ჰქონდა გამუდმებული კრუნჩხვები და მისი თვალის გუგები რეაქტიული იყო. მისი ცნობა თანდათან უარესდებოდა და საბოლოოდ ბავშვი კომაში ჩავარდა. გულსისძარღვთა, სასუნთქი სისტემის, და მუცლის შემოწმებას არაფერი განსაკუთრებული არ გამოუვლენია. CT-მ აჩვენა ტვინის ბაზალური ცისტერნების აცლა (effacement) და თხელი დაბალი სიმკვრივის მასა ბილატერალურად ფრონტალურ გირუსებზე, რაც სუბდურულ ჰემატომაზ ან ეფუზიაზე მიანიშნებდა.

ისტორიაში საინტერესო იყო ის, რომ მშობლები შვილს ხელოვნური ჭის წყლით აბანავებდნენ.

ლაბორატორიული ანალიზების შედეგები

ლეიკოციტების მთლიანი რიცხვი - 7,300/mm3, ნეიტროფილები - 71%, ლიმფოციტები 26%, ეოზინოფილები 3%
ჰემოგლობინი - 10.2 g/dl
პერიფერიულ ნაცხში პარაზიტები არ გამოვლენილა.
ელექტროლიტები: ნატრიუმი - 133 mmol/liter, კალიუმი 4.4 mmol/liter, ქლორი 95 mmol/liter, ბიკარბონატი 18.6 mmol/liter, ფოსფორი - 4.7 mg/dl, BUN - 4 mg/dl, შრატში კალციუმის დონე 9.2 mg%,, გლიკემია - 120 mg/dl.

ცერებროსპინალური სითხე მღვრიე და ჰემორაგიული იყო. CSF გლუკოზის დონე - 10 mg/dl. ცილების დონე - 361 mg/dl,, უჯრედების რიცხვი 30 cells/mm3, ნეიტროფილები 80%, ლიმფოციტები - 20%

ჭის წყალში, რომელსაც ბავშვის დასაბანად იყენებდნენ, აღმოაჩინეს გამომწვევი.

მას დაეწყო მკურნალობა ცეფტრიაქსონით (100მგ/კგ) და ინტრავენული ამფოტერიცინ B-თი, მაგრამ ექიმების ცდისდა მიუხედავად, ბავშვი მიღების მეორე დღეს გარდაიცვალა.

დასვით დიაგნოზი და დაასახელეთ გამომწვევი!

თუ გაგიჭირდებათ, შემდეგ CSF-ში ნაპოვნი ორგანიზმის სურათსაც დავდებ.

Primary Amoebic Meningoencephalitis

გამომწვევი: Naegleria Fowleri

ალბათ

Posted by: Thandrus 8 Jun 2010, 00:15
Cousteau

QUOTE
Primary Amoebic Meningoencephalitis

გამომწვევი: Naegleria Fowleri

ალბათ


ცხრიანში შეაგდე რა biggrin.gif საინტერესო ამ შემთხვევაში ის იყო, რომ კლასიკურად, ნეგლერიით გამოწვეული მენინგოენცეფალიტი ბასეინებში, ტბებში და სხვა ასეთ დაგუბებულ ადგილებში ცურვის შემდეგ ვითარდება. ამ შემთხვევაში ეს არ იყო - ბავშვს უბრალოდ ჭის წყლით ბანდნენ. ამიტომაც იყო ეს ქეისი, ჩემი აზრით, საინტერესო.

ნეგლერიის მენინგოენცეფალიტის დროს გასაკვირია, მაგრამ ერთადერთი ეფექტურთან მიახლოებული წამალი Ampho B არის (სოკოს საწინააღმეგო აგენტი), თუმცა წარმატებული გამოსავლის შანსი ძალიან მცირეა.


Posted by: Blind_Torture_Kill 8 Jun 2010, 01:21
Thandrus

კაი ქეისი იყო

Posted by: Thandrus 8 Jun 2010, 16:35
სანამ ერთი ერთი დღით დაგტოვებდეთ, ერთი "რა? სად? როდის?"-ის ტიპის კითხვას დავსვამ biggrin.gif

ამ დაავადებას ორი ფაზა ახასიათებს, პირველი ცხელების, რომლის სიკვდილიანობა მკურნალობის გარეშე საკმაოდ დიდია, მეორე კი ქრონიკული - კანის, სადაც ერითემული ნოდულები წლების განმავლობაში შეიძლება დარჩეს. ეს ორი ფაზა ადრე სხვადასხვა დაავადებები ეგონათ. მეცხრამეტე საუკუნის ბოლოს, ერთ-ერთმა სამედიცინო სტუდენტმა, რომ დაემტკიცებინა ამ ორი დაავადების მიზეზ-შედეგობრივი კავშირი, ავადმყოფის ნოდულიდან ასპირატი აიღო და ვენაში ინექცია გაიკეთა. რამდენიმე დღის შემდეგ სტუდენტს განუვითარდა ცხელება და ის გარდაიცვალა. ამით დამტკიცდა, რომ ცხელება და ქრონიკული ნოდულები ერთი დაავადების ორი ფაზა იყო. შემდგომში ამ დაავადებას ამ სტუდენტის სახელი უწოდეს.

დამისახელეთ სტუდენტის გვარი, დაავადების გამომწვევი პათოგენი და ამ ორი ცალკეული ფაზის სახელი, რომელიც ზოგჯერ ახლაც გამოიყენება ხოლმე.

biggrin.gif

Posted by: MAIN KAMPF 8 Jun 2010, 18:27
QUOTE
CT-მ აჩვენა ტვინის ბაზალური ცისტერნების აცლა (effacement) და თხელი დაბალი სიმკვრივის მასა ბილატერალურად ფრონტალურ გირუსებზე, რაც სუბდურულ ჰემატომაზ ან ეფუზიაზე მიანიშნებდა.

biggrin.gif
დღეს განყოფილებაში ასეთი სურათი დამხვდა. ფეხბურთი იწყება bis.gif http://www.radikal.ru
ბაფანა..ბაფანა sa.gif

Posted by: Sephora 9 Jun 2010, 13:02
არ არის არავინ აქ? რაღაც მაინტერესებს.

Posted by: MAIN KAMPF 9 Jun 2010, 16:55
QUOTE
არ არის არავინ აქ? რაღაც მაინტერესებს.

ბრძანეთ
chups.gif

Posted by: lgogokhia 11 Jun 2010, 10:43
Thandrus
QUOTE
ამ დაავადებას ორი ფაზა ახასიათებს, პირველი ცხელების, რომლის სიკვდილიანობა მკურნალობის გარეშე საკმაოდ დიდია, მეორე კი ქრონიკული - კანის, სადაც ერითემული ნოდულები წლების განმავლობაში შეიძლება დარჩეს. ეს ორი ფაზა ადრე სხვადასხვა დაავადებები ეგონათ. მეცხრამეტე საუკუნის ბოლოს, ერთ-ერთმა სამედიცინო სტუდენტმა, რომ დაემტკიცებინა ამ ორი დაავადების მიზეზ-შედეგობრივი კავშირი, ავადმყოფის ნოდულიდან ასპირატი აიღო და ვენაში ინექცია გაიკეთა. რამდენიმე დღის შემდეგ სტუდენტს განუვითარდა ცხელება და ის გარდაიცვალა. ამით დამტკიცდა, რომ ცხელება და ქრონიკული ნოდულები ერთი დაავადების ორი ფაზა იყო. შემდგომში ამ დაავადებას ამ სტუდენტის სახელი უწოდეს.
დამისახელეთ სტუდენტის გვარი, დაავადების გამომწვევი პათოგენი და ამ ორი ცალკეული ფაზის სახელი, რომელიც ზოგჯერ ახლაც გამოიყენება ხოლმე.

მთელი დღე ვფიქრობ ამ შეკითხვაზე და გადამრია biggrin.gif biggrin.gif ჯერ მეგონა ჰენსენის დაავადება ანუ ლეპრა, მაგრამ ჰენსენი მსგავს სიტუაციაში არ გარდაცვლილა როგორც ვნახულობ. არადა ვიცი ზუსტად რომ მსგავსი სცენარი წამიკითხავს ძალიან ადრე, მანამ სანამ ჯერ კიდევ სკოლაში ვიყავი და წიგნს ერქვა "მე თუ ვინმე სხვა". მხოლოდ ამის გახსენება შევძელი.. დადე პასუხი ან ვინმემ უპასუხეთ რა პლზ smile.gif

ნეგლერიის ქეისმა ჰაუსის ეპიზოდი გამახსენა.. ხომ გაქვთ ნანახი ეგ ეპიზოდი? smile.gif

Posted by: Archer 11 Jun 2010, 17:49
Thandrus
QUOTE
სანამ ერთი ერთი დღით დაგტოვებდეთ, ერთი "რა? სად? როდის?"-ის ტიპის კითხვას დავსვამ biggrin.gif

ამ დაავადებას ორი ფაზა ახასიათებს, პირველი ცხელების, რომლის სიკვდილიანობა მკურნალობის გარეშე საკმაოდ დიდია, მეორე კი ქრონიკული - კანის, სადაც ერითემული ნოდულები წლების განმავლობაში შეიძლება დარჩეს. ეს ორი ფაზა ადრე სხვადასხვა დაავადებები ეგონათ. მეცხრამეტე საუკუნის ბოლოს, ერთ-ერთმა სამედიცინო სტუდენტმა, რომ დაემტკიცებინა ამ ორი დაავადების მიზეზ-შედეგობრივი კავშირი, ავადმყოფის ნოდულიდან ასპირატი აიღო და ვენაში ინექცია გაიკეთა. რამდენიმე დღის შემდეგ სტუდენტს განუვითარდა ცხელება და ის გარდაიცვალა. ამით დამტკიცდა, რომ ცხელება და ქრონიკული ნოდულები ერთი დაავადების ორი ფაზა იყო. შემდგომში ამ დაავადებას ამ სტუდენტის სახელი უწოდეს.

პარტახტიან ტიფზე ვიცი გამომწვევის გამოყოფის დროს დაიღუპა პროვაცეკი და გამომწვევს პროვაცეკის რიკეტსია უწოდეს. გამონაყარი ახასიათებს, მაგრამ სტუდენტი არ იყო ნამდვილად smile.gif

Posted by: lgogokhia 11 Jun 2010, 22:02
Thandrus
QUOTE
დამისახელეთ სტუდენტის გვარი, დაავადების გამომწვევი პათოგენი და ამ ორი ცალკეული ფაზის სახელი, რომელიც ზოგჯერ ახლაც გამოიყენება ხოლმე.

DAMN THAT!!! lol.gif lol.gif lol.gif პასუხი არის Carrion's disease ანუ Oroya fever, რომლის გამომწვევი არის Bartonella bacilliformis..


Posted by: Thandrus 11 Jun 2010, 22:50
lgogokhia

bis.gif bis.gif bis.gif

ვიცოდი, რომ ბოლოსდაბოლოს გამოიცნობით, პასუხიც იმიტომ არ დავწერე biggrin.gif

რათქმაუნდა.

პერუელმა სტუდენტმა Daniel Alcides Carrión-მა გადაწყვიტა დაემტკიცებინა კავშირი ოროიას ცხელებასა (Oroya Fever) და კანის ქრონიკულ დაავადება პერუულ მეჭეჭს (Verruga Peruana) შორის. ორივე ეს დაავადება პერუში, ეკვადორში და კოლუმბიაშია ენდემური. ამისათვის მან აიღო ბავშვის შუბლის მეჭეჭიდან ასპირატი და თავის სისხლში გააკეთა ინოკულაცია. ამან მასში ოროიას ცხელება გამოიწვია. როგორც ვთქვი, ანტიბიოტიკებით მკურნალობის გარეშე ამ დაავადებას საკმაო სიკვიდლიანობა აქვს, ამიტომ კარიონიც გარდაიცვალა. ის 28 წლის იყო. ცოტა ხნის შემდეგ, Oroya Fever-ი და Verruga Peruana გაერთიანდა და დაერქვა Carrión's Disease. ახლა კარიონი პერუს ეროვნულ გმირად არის გამოცხადებული. ეკვადორში კი მისი სახელობის სტადიონი არსებობს biggrin.gif

გამომწვევი კი მართლაც B. bacilliformis-ია.

Posted by: Blind_Torture_Kill 12 Jun 2010, 00:15
lgogokhia
Thandrus

თვქენ დაბადებიდან სამედიცინო ლიტერატურას ეცნობოდით თუ როგორაა
up.gif

Posted by: Thandrus 12 Jun 2010, 01:00
Blind_Torture_Kill

QUOTE
თვქენ დაბადებიდან სამედიცინო ლიტერატურას ეცნობოდით თუ როგორაა


gigi.gif

ამას წინათ ვკითხულობდი მიკრობიოლოგიის წიგნიდან Bartonella-ს ინფექციებზე და იქიდან დამებადა ეს შეკითხვა biggrin.gif

Posted by: Thandrus 12 Jun 2010, 13:49
ოფტოპიკები ფორმალინით ინაქტივირდა.

ახლა, ქეისს ველოდები yes.gif

Posted by: Thandrus 21 Jun 2010, 15:45
tatusia

როდესაც ჰასუხებს გაიგებ,

http://forum.ge/?showtopic=34130720&f=&st=15&#entry20274758

ამ თემაში დაწერე აუცილებლად smile.gif

Posted by: patriciia 21 Jun 2010, 16:22
Thandrus
QUOTE
ოფტოპიკები ფორმალინით ინაქტივირდა.

მე რომ დავწერე ყბიკს ტკივილზე, ისიც დააფორმალინეთ?

Posted by: Thandrus 21 Jun 2010, 17:50
patriciia

ესეც იმ ტემაში გადავიტანე smile.gif რამდენიმე პოსტი ორივე პრობლემას ეყოფოდა და შუაზე ვერ გავყოფდი.

ჰე, ხალხო, რამე ქეისი მოიტათ რა...
* * *
konkretula

თქვენი პოსტი გადატანილია ამ თემაში:

http://forum.ge/?showtopic=33737196&f=&st=750

აქ ასეთი კითხვები არ ისმევა. აქ კლინიკურ შემთხვევებს ვდებთ და განვიხილავთ.

Posted by: Dr Pepper 4 Jul 2010, 03:43
რომ დაეცით აქ ყველა ქადაგად biggrin.gif
აბა ჰა
რომელი ფერმენტების კლასი აკატალიზებს ATF-ის ან NTF-ის პიროფოსფატური ბმის ჰიდროლიზის შედეგად წარმოქმნილ ენერგიას?


Posted by: Thandrus 4 Jul 2010, 05:53
Dr Pepper

QUOTE
რომელი ფერმენტების კლასი აკატალიზებს ATF-ის ან NTF-ის პიროფოსფატური ბმის ჰიდროლიზის შედეგად წარმოქმნილ ენერგიას?


კითხვას ვერ მიგიხვდი - გვეკითხები, რომელი ფერმენტების ხლიჩავს ATP-ს ADP-დ და ფოსფატად?

ATP-აზებს გულისხმობ ალბათ.

Posted by: Dr Pepper 4 Jul 2010, 14:02
არაკაცო ფერმენტების რომელი კლასი საჭიროებს კატალიზითვის ATP-ის ან NTP-ის პიროფოსფატური ბმის ჰიდროლიზის შედეგად გამოყოფილ ენერგიას... კითხვას ამაზე მაგრად ვერ დასვამ biggrin.gif
დაუკვირდი .
არა ATF-აზები არაფერშუაშია

* * *
ანუ გახლიჩვის შედეგად გამოყოფილ ენერგიას რომელი ფერმენტების კლასი საჭიროებს?

Posted by: Thandrus 4 Jul 2010, 14:56
Dr Pepper

QUOTE
ანუ გახლიჩვის შედეგად გამოყოფილ ენერგიას რომელი ფერმენტების კლასი საჭიროებს?


აჰ, ახლა გასაგებია... ლიგაზები და ტრანსფერაზები.

Posted by: Dr Pepper 4 Jul 2010, 15:11
საღოლ biggrin.gif წოწ
მარამ ტრანსფერაზების კლასი ნამდვილად არ საჭიროებენ აი ლიგაზები 100%

სისხლში ეპინეფრინის დონის გაზრდა აინჰიბირებს გლიკოლიზს ღვიძლში, რითია განპირობებული საპირისპირო ეფექტი მიოკარდიუმში?

Posted by: LUKA-BRAZI 4 Jul 2010, 15:20
Dr Pepper
QUOTE
სისხლში ეპინეფრინის დონის გაზრდა აინჰიბირებს გლიკოლიზს ღვიძლში, რითია განპირობებული საპირისპირო ეფექტი მიოკარდიუმში?

ეპინეფრინის დადებით ქრონოტროპულ ეფექტთან? smile.gif)

Posted by: Dr Pepper 4 Jul 2010, 15:34
ხო ნუ თავისთავად smile.gif
შეკუმშვის გახშირება იჩითება აბა რაა tongue.gif
მაგრამ ამ კითხვას არსებითი მნიშვნელობა არ ქონდა ისე tongue.gif
* * *
Thandrus
ხო
ტრანსფერაზების კლასი ძირითადად აკატალიზებენ
ამა თუ იმ ჯგუფის გადატანას ერთი ნაერთიდან მეორეზე

* * *
კაი რახან შემოიხედეთ და გახვედით მე გიპასუხებთ :P
სისხლში ეპინეფრინის დონის გაზრდა აინჰიბირებს გლიკოლიზს ღვიძლში, ეს იმითია განპირობებული რომ მიოკარდის ბიფუნქციური ფოსფოფრუქტოკინაზა-2-ის იზოფერმენტი, ზრდის ფრუქტოზა-2,6-ბისფოსფატის დონეს :)

LUKA-BRAZI
გულის მუშაობის ექსტრაკარდიალური ნერვული რეგულაცია ხორციელდება მოგრძო ტვინში არსებული ცთომ
* * *
აუ რატომ არ მაწერინებს ბოლოდე..? იმიტომ მეზიზღება ეს ფორუმი,ჯერ მილიონი უაზრო შემოდის და მერე მილიარდი შეზღუდვა გაქვს...მაპათიეთ ოფისთვის

Posted by: LUKA-BRAZI 4 Jul 2010, 23:04
Dr Pepper
QUOTE
იმიტომ მეზიზღება ეს ფორუმი

აბა, აბა! biggrin.gif

რატომ არ გაწერინებს? წერის დროს ტავისით პოსტავს თუ რა?

ისე საინტერესოა შენი კითხვები. ბიოქიმიკოსი ხარ? smile.gif

QUOTE
გულის მუშაობის ექსტრაკარდიალური ნერვული რეგულაცია ხორციელდება მოგრძო ტვინში არსებული ცთომ

ჰო, ბატონი ვაგუსი თვალყურს ადევნებს სინუსის კვანძს... smile.gif))

Posted by: MAIN KAMPF 5 Jul 2010, 01:05
QUOTE
რომელი ფერმენტების კლასი აკატალიზებს ATF-ის ან NTF-ის პიროფოსფატური ბმის ჰიდროლიზის შედეგად წარმოქმნილ ენერგიას?

wow.gif მდაა რატომ აწვალებთ ინტელექტს. საინტერესოა პრაქტიკაში რამდენად გამოსაყენებელია. დადევით'' ჟიზნენნი'' კეისები biggrin.gif

Posted by: LUKA-BRAZI 5 Jul 2010, 01:22
MAIN KAMPF
შენ სპეციალობით ქირურგი ხარ ჰო? ემანდ რადიოლოგი არ აღმოჩნდე თორემ ამ ქეისს ეგრევე გამოიცნობ smile.gif)

ქეისის კითხვაა: "რა არის ეს?" biggrin.gif

user posted image

user posted image


* * *
სურათზე თუ "save as"-ს დააწკაპუნებთ, სურათის სახელით შეიძლება გუგლით გამოცნობა, ასე რომ "save as"-ით "ჟულიკობას" არ ვართ! biggrin.gif

Posted by: Dr Pepper 5 Jul 2010, 01:53
აუ ადრე მაქვს სადღაც ნანახი მსგავსი რაღაც ტოო... ტუბის ბაზარია ხოო?
ოღონდ არ მახსოვს რადა როგორ
დაწერო იქნება პასუხი smile.gif
* * *
LUKA-BRAZI
გულის მუშაობის ექსტრაკარდიალური ნერვული რეგულაცია ხორციელდება მოგრძო ტვინში არსებული ცთომილი ნერვის ბირთვებით და ზურგის ტვინის გულმკერდის ზედა ხუთი სეგმენტის სიმპათიკური ნერვებით. სიმპათიკური ნერვული ბოჭკოებით გულისკენ მიმავალი იმპულსები იწვევენ გულის შეკუმშვების გახშირებას. ესაა დადებითი ქრონოტროპული მოქმედება, ზრდიან ამ შეკუმშვების ძალას ანუ ხდება დადებითი ინოტროპული მოქმედება და ახდენენ მიოკარდიუმის აგზნებადობას ანუ საქმე გვაქვს დადებით ბათმოტროპულ ეფექტთან, და ბოლოს ისინი ზრდიან აგზნების გატარების სიჩქარეს ეს არის უკვე დადებითი დრომოტროპული ეფექტი. აღნიშნული ეფექტები დაკავშირებულნი არიან სიმპათიკური ნერული სისტემის დაბოლოებებიდან გამომუშავებული ნორეპინეფრინის მიერ გულის β-ადრენორეცეპტორების აგზნებასთან. გულში ადრენომიმეტიკების შეყვანა იწვევს გულის მუშაობის ზუსტად ისეთი სახის ცვლილებებს, როგორც სიმპათიკური ნერვების გაღიზიანება. wink.gif
ამის თქმას ვლამობდი...

Posted by: MAIN KAMPF 5 Jul 2010, 02:08
[quote]სურათზე თუ "save as"-ს დააწკაპუნებთ, სურათის სახელით შეიძლება გუგლით გამოცნობა, ასე რომ "save as"-ით "ჟულიკობას" არ ვართ!


ღმერთმა დამიფაროს მაიმუნობისაგან biggrin.gif ორივე ფილტვის ზემო წილები რაღაცა ფინთს biggrin.gif უკავია. ქვემოთ ვერაფერს ცუდს ვერ ვხედავ, გულის საზღვრები გაფართოებული მეჩვენება,ტრაქეა ოდნავ მარჯვნივაა გადაწეული, ეს შავი რგოლები სავარაუდოდ ბულებია biggrin.gif თუ მეტყვი პასუხს კარგი იქნება ძილს ვაპირებ biggrin.gif

Posted by: Dr Pepper 5 Jul 2010, 02:21
მეც დავსამ ერთ კითხვას smile.gif
ალკოჰოლდეჰიდროგენაზა (ოქსიდორედუქტაზები) გადრდაქმნის მეთანოლს (CH3OH) ტოქსიურ ნაერთში - ფორმალდეჰიდში (CH2O). პაციენტებს რომლებსაც მიღებული აქვთ მეთანოლის მომწამვლელი რაოდენობა, უნიშნავენ ეთანოლს (CH3CH2OH)
რატომ? biggrin.gif დაასაბუთეთ რატომ იძლევა ექიმი ასეთ დანიშნულებას... tongue.gif

Posted by: badu 5 Jul 2010, 02:35
Dr Pepper

ethanol acts as a competitive inhibitor and occupies methanol binding sites on alcohol dehydrogenase, preventing methanol's conversion into its toxic metabolite. წავა? biggrin.gif
* * *
LUKA-BRAZI

ბრონქიექტაზიო ვიტყოდი მაგრამ არ გავს. უცნაურია. შეიძლება მეჩვენება მაგრამ ფილტვის პარენქიმას არ გავს. it does not make sense anatomically either sad.gif

Posted by: Dr Pepper 5 Jul 2010, 02:52
badu
აუცილებლად მართალი ხარ smile.gif ეთანოლი მეთანოლის სტრუქტურული ანალოგია და მოსაოდნელია ეფექტური კონკურენცია მეთანოლთან ფერმენტის აქტიური ცენტრის დასაკავებლად smile.gif
პროსტა ნუ გუგლავთ რა biggrin.gif


* * *
რომელი ფერმენტის დეფიციტითაა გამოწვეული გირკეს დაავადება?

Posted by: Thandrus 5 Jul 2010, 03:55
აუჰ, აქ რა ამბები ყოფილა, სახლიდან ვერ გავა რა კაცი gigi.gif

Dr Pepper

როგორც ვიცით, გლიკოლიზში "1-კინაზის" აქტიურობისთვის "2-ფოსფატია" საჭირო. ეპინეფრინის პოსტ-რეცეპტორული სიგნალები საბოლოო ჯამში ამ ენზიმის ფოსფორილირება "2-კინაზას" აქტიობას ამცირებენ. შედეგად ფოსფატაზური ნაწილი აქტიურდება "2-ფოსფატი" აღარ წარმოიქმნება და გლიკოლიზი ჩერდება. გულში სხვა ამ ენზიმის სხვა ტიპია. ეპინეფრინის სიგნალინგი მგონი იგივეა (თუ სწორად მახსოვს), მაგრამ ფოსფორილაციის შემდეგ ღვიძლისგან განსხვავებით, გულის ენზიმი მეტ აქტიობას იძენს. "2" იზრდება და გლიკოლიზი მიდის biggrin.gif

LUKA-BRAZI

ვუპასუხო შენს სურათს? gigi.gif

* * *
Dr Pepper

QUOTE
ეთანოლი მეთანოლის სტრუქტურული ანალოგია და მოსაოდნელია ეფექტური კონკურენცია მეთანოლთან ფერმენტის აქტიური ცენტრის დასაკავებლად
პროსტა ნუ გუგლავთ რა


ამას ფომეპიზოლი არ სჯობს? wink.gif

QUOTE
რომელი ფერმენტის დეფიციტითაა გამოწვეული გირკეს დაავადება?


კლასიკური ფონ გირკე გლუკოზა-6 ფოსფატაზის დეფიციტია.

Posted by: Dr Pepper 5 Jul 2010, 14:23
Thandrus
+1 საღოლ smile.gif


აბა ცხელი კითხვა მოიდა biggrin.gif

თქვენი პაციენტია 6 თვის პაწუა გოგონა biggrin.gif მას აღენიშნება რესპირაციული და საჭმლის მომნელებელი სისტემის მხრივ პრობლემები. ექოკარდიოგრაფიულად აღენიშნება კარდიომეგალია. ლიზოსომებში ალფა-გლუკოზიდაზას აქტივობა დაქვეითებულია. სინათლის მიკროსკოპით ვლინდება დიდი რაოდენობით გლიკოგენის შემცველი ვაკუოლები. რა ჯანდაბა დიაგნოზს დაუსმამდით ბოვშვს? biggrin.gif

Posted by: LUKA-BRAZI 5 Jul 2010, 15:51
Dr Pepper
შენს ამ თემაში გამოჩენას ყველანი მივესალმებით, მაგრამ აქ ერთი წესია, ქეისების ქრონოლოგიას ვიცავთ, თორემ მერე დიდი აბდაუბდა გამოდის ხოლმე. ოკ ნიგგა? biggrin.gif ისე ახლოს კი ხარ პასუხთან, მაგრამ პასუხი მთლად ის არ არის რაც გგონია wink.gif

პ.ს. გვითხარი შენი პროფესია.... არა, თუ http://en.wikipedia.org/wiki/MI5-ის თანამშრომელი ხარ, ნუ გვეტყვი, შარში არ ამოყო თავი biggrin.gif

badu
ბადუს გაუმარჯოს.. როგორაა საქმეები? smile.gif მართალი ხარ, ბრონქოექტაზია არ არის, არც პარენქიმაა smile.gif დააკვირდი სურათს, რამის ფარგლით დახაზულ წრეწილს გავს smile.gif

Thandrus
გახსოვს ლულა-ქაბაბის კითხვა? smile.gif)))))))

MAIN KAMPF
მართალია, ფინთი პონტია ზემო წილებში. კიდევ ცადე აბა smile.gif


Posted by: irakli222 5 Jul 2010, 16:03
LUKA-BRAZI

მემგონი უკვე იყო მსგავსი კითხვა.
კავერნოზული ტუბერკულიოზის მკურნალობის დროს პლევრის ღრუში შეყავდათ უცხო სხეულები (ამ შემთხვევაში ბურთები) რომელთა ზეწოლის ხარჯზე იხურება კავერნა.

Posted by: Thandrus 5 Jul 2010, 16:04
Dr Pepper

QUOTE
თქვენი პაციენტია 6 თვის პაწუა გოგონა. მას აღენიშნება რესპირაციული და საჭმლის მომნელებელი სისტემის მხრივ პრობლემები. ექოკარდიოგრაფიულად აღენიშნება კარდიომეგალია. ლიზოსომებში ალფა-გლუკოზიდაზას აქტივობა დაქვეითებულია. სინათლის მიკროსკოპით ვლინდება დიდი რაოდენობით გლიკოგენის შემცველი ვაკუოლები. რა ჯანდაბა დიაგნოზს დაუსმამდით ბოვშვს?


ალფა გლუკოზიდაზა ანუ მჟავური მალტაზა, ხომ? ეს და სიმპტომები პომპეს დაავადებაზე მიუთითებენ, რომელიც, სამწუხაროდ, არ იკურნება.

QUOTE
გახსოვს ლულა-ქაბაბის კითხვა?


აჰა biggrin.gif

Posted by: MAIN KAMPF 5 Jul 2010, 16:16
QUOTE
მართალია, ფინთი პონტია ზემო წილებში. კიდევ ცადე აბ

no.gif რატომ უნდა ვცადო ე.ი.ვერ გამოვიცანი. მაგრამ სურათის წაკითხვით ბულას გავს
QUOTE
მემგონი უკვე იყო მსგავსი კითხვა.
კავერნოზული ტუბერკულიოზის მკურნალობის დროს პლევრის ღრუში შეყავდათ უცხო სხეულები (ამ შემთხვევაში ბურთები) რომელთა ზეწოლის ხარჯზე იხურება კავერნა
თუ ეს მართალია ბულა და ბურთი რამ გაყო
biggrin.gif ასეთი მკურნალობის მეთოდზე არ გამიგია boli.gif

Posted by: LUKA-BRAZI 5 Jul 2010, 16:36
მოკლედ.... აღნიშნულ სურათზე გამოსახულია ფილტვი პლომბაჟის მერე smile.gif

http://ajrccm.atsjournals.org/cgi/content/full/157/5/1609

Posted by: Dr Pepper 5 Jul 2010, 17:40
LUKA-BRAZI
ყო ნეგგა ყო biggrin.gif
okey bro
გათვალისწინებული ბუძიტ smile.gif
მე ვარ სამედიცინოს ერთერთი სტუდენტთაგანი და ამავედროს SWAT-ის გენერალური დირექტორი biggrin.gif
ხო ისა სამკუნალო პონტში რომ იყო ტუბზე მეც ასე მახსოვდა smile.gif)))))

Thandrus
+1
პომპეს დაავადებაა საღოლ biggrin.gif
* * *
ისა ეხლა რომ დავსვა შეიძლება? biggrin.gif

Posted by: LUKA-BRAZI 5 Jul 2010, 18:19
Dr Pepper
კი შეიძლება smile.gif

Posted by: Thandrus 5 Jul 2010, 18:49
მეც დაგისვამ ახლა შეკითხვას biggrin.gif

მოჰყავთ რამდენიმე თვის ბავშვი, ამის მაგვარი (იხ. სურათი) სახის დისმორფიით. ბავშვი განვითარებით თავის ასაკს ჩამორჩება, აქვს დაქვეითებული სმენა, ხშირად უვითარდება კრუნჩხვები, აქვს ჰეპატომეგალია, თირკმლის ფუნქცია მოშლილია, GI ტრაქტიდან სისხლდენაა; კიდევ საკმაოდ ბევრი სიმპტომია, რომლის ჩამოთვლაც დიდ დროს წაგვართმევს და არაა საჭირო.

უკეთებ ბიოქიმიურ ლაბორატორიულ ანალიზს და ხედავ, რომ:

ქსოვილებში პლაზმალოგენების დონე მკვეთრად დაკლებულია
დაგროვებულია very long chain fatty acid-ები (მაგ. C24, C26)
phytanic acid-ის დაჟანგვა ნორმალურად ვერ ხდება და ამის გამო ამის დონე მომატებულია
ნაღვლის მჟავების წარმოქმნაც შემცირებულია

კიდევ არის ლაბორატორიული ანომალიები, მაგრამ დიაგნოზისთვის ესენიც საკმარისია.

მოკლედ, დასვით დიაგნოზი, ჩამომიყალიბეთ დაავადების პათოგენეზი და პროგნოზი!

Posted by: LUKA-BRAZI 5 Jul 2010, 20:28
Thandrus
დი ჯორჯის სინდრომი? baby.gif

Posted by: Thandrus 5 Jul 2010, 21:30
LUKA-BRAZI

QUOTE
დი ჯორჯის სინდრომი?


არა, აქ ეს არაა. ზოგიერთი სიმპტომი კი აქვს ამ ორ დაავადებას ერთნაირი (ნუ ეს "ერთნაირი", როგორც წიგნებში წერია, რეალობაში ალბათ განხსხვავდება), მაგრამ ამ ბავშვებს, დი ჯორჯისთვის დამახასიათებელი, მაგალითად, გულის სტრუქტურული ანომალიები, თიმუსის აპლაზია და მგლის ხახა არ აქვთ.

თან სახე, როგორც პედიატრები ამბობენ, ეგრევე ამ დაავადებაზე გაფიქრებინებს biggrin.gif პლუს, ანალიზებშიც ბევრი hint-ია...

Posted by: Blind_Torture_Kill 5 Jul 2010, 21:54
QUOTE
მოჰყავთ რამდენიმე თვის ბავშვი, ამის მაგვარი (იხ. სურათი) სახის დისმორფიით. ბავშვი განვითარებით თავის ასაკს ჩამორჩება, აქვს დაქვეითებული სმენა, ხშირად უვითარდება კრუნჩხვები, აქვს ჰეპატომეგალია, თირკმლის ფუნქცია მოშლილია, GI ტრაქტიდან სისხლდენაა; კიდევ საკმაოდ ბევრი სიმპტომია, რომლის ჩამოთვლაც დიდ დროს წაგვართმევს და არაა საჭირო.

უკეთებ ბიოქიმიურ ლაბორატორიულ ანალიზს და ხედავ, რომ:

ქსოვილებში პლაზმალოგენების დონე მკვეთრად დაკლებულია
დაგროვებულია very long chain fatty acid-ები (მაგ. C24, C26)
phytanic acid-ის დაჟანგვა ნორმალურად ვერ ხდება და ამის გამო ამის დონე მომატებულია
ნაღვლის მჟავების წარმოქმნაც შემცირებულია


zellweger syndrome

Posted by: Thandrus 5 Jul 2010, 22:10
Blind_Torture_Kill

აჰა,

ახლა პათოგენეზი და პროგნოზიც wink.gif

Posted by: Blind_Torture_Kill 5 Jul 2010, 22:28
Thandrus

http://en.wikipedia.org/wiki/Zellweger_syndrome

Posted by: Thandrus 5 Jul 2010, 23:07
Blind_Torture_Kill

კაი ჰო : mo.gif

აბა, ეილერს-დანლოს სინდრომის რომელი ტიპია ყველაზე ცუდ პროგნოზთან ასოცირებული და რატომ? biggrin.gif

Posted by: Blind_Torture_Kill 5 Jul 2010, 23:21
Thandrus


შემაერთებელქსოვილოვანი პათოლოგიაა და აორტის განშრევებად ანევრიზმას იწვევს აი ესაა რაც გავიხსენე ამაზე smile.gif

Posted by: Veshapo_O 5 Jul 2010, 23:22
გამარჯობათ, არ ვიცი ვის ვკითხო ეს და ამიტო გადავწყვიტე ფორუმზე მეკითხა. მოკლედ რა ხდება : დგეს უკვე მეორე შემთხვევა იყო როდესაც ყურიდან მქონდა სისხლდენა sad.gif ვიგრძენი რო ყური დამეხშო და როცა ყური გამოვიწმინდე ბამბა იყო მთლად სისხლით სველი. თუკი ვინმემ რამე შეგიძლიათ მითხრათ არ დაიზაროთ. წინასწარ დიდი მადლობა



P.S. თუკი ზუსტად არ იცით გთხოვთ ტყუილად გულს ნუ გამიხეთქავთ smile.gif

Posted by: badu 6 Jul 2010, 01:15
აი ცხელ-ცხელი ქეისი თქვენ. ეს ბავშვი 1 თვის წინ ვნახე და ქეის რეპორტი დავწერე. ინგლისურადაა, ქართულად გადათარგმნა მეზარება, პარდონ sad.gif

P.S. 9 mo female
C.C. “weakness”

HPI: This patient is a 9-month-old female who presented to ER for 3-week history of weakness, lethargy and decreased PO intake as per her mother. She initially visited her PMD a day earlier who sent her to ER for “abnormal lab value.” Mom claims that the child has been less playful and cheerful lately. She is usually a really happy baby, but last few weeks she has been sleeping more and has not been moving around as much. She also has decreased PO intake. Normally the baby would drink 5oz of Similac Q3 hours, but lately she has been drinking 2oz Q3-4 hours. Urine output has not changed significantly. She urinates more than 5 times a day. Patient sometimes gags and spits up but does not vomit. She has no recent history of fevers, chills, coughs, rhinorrhea, nasal congestion, diarrhea, or rashes. No sick contacts at home, no recent travel history.

Mother also mentioned that the patient’s clothes have become looser. She felt that that baby has lost some weight. The patient was weighed at PMD’s office and turned out to have had lost 4 lbs since last visit which was 1 month prior. The PMD ordered basic labs, and a day later called the mother and told her to take the child to ER due to an “abnormal lab value”.

PMH: No significant PMH, no hospitalizations, no surgeries.

Allergies: NKDA

Medications: none

Birth Hx: Born at 39 weeks gestation via C/S for non-reassuring FHRT. Birth weight – 5lbs: Small for Gestational Age. Spent 4 days in NICU for hypoglycemia and presumed sepsis. She was also noted to have torticollis, which resolved and echocardiogram showed small anterior restrictive VSD.

Developmental Hx:
Pt is lagging developmentally at all levels.
Gross Motor (3-4 mo level): She is unable to sit unsupported, does not roll front to back. Can hold up his head.
Fine motor (4-5 mo): grasps with both hands together. Does not transfer hand to hand.
Language (6 mo level): She babbles but does not say any words. Does not understand commands.
Social (4-5 mo level): Does not have stranger anxiety. Extremely social and happy around strangers.


Family Hx: Grandfather has HTN, DM, and heart disease. Mother had unknown electrolyte disturbance and was hospitalized 4 times. Uncle was hospitalized as a child for 2 years for paralysis (details can’t be recalled).
Only child. Mother is 32, healthy. Father is 34, healthy.

Social Hx: lives with parents. Parents have no marital problems. Financially stable. No one smokes or drinks alcohol at home. No drugs. No medications are kept at home. 1 healthy family dog.

Review of systems:
- General: weakness, fatigue, weight loss, and change in appetite. No chills, fever, sweats.
- Neuromuscular: no tremors, no seizures, no syncope.
- Cadiovascular: no CP, no SOB, no cyanosis, no murmur, no HTN.
- Respiratory: no URI, no cough, no wheezing, no asthma, no pneumonia, no otitis media.
- GI: no nausea, no vomiting, no diarrhea, no constipation, no rectal bleeding, no abdominal pain, no jaundice.
- GU: no polyuria, no change in urine color, no recent UTI.

Vitals: BP: 101/64 HR: 136 RR 37 T 97.8 Wt: 5.8kg (<5%) Ht: 25.5 in (<5%)
Head circumference 43cm (25%)

Physical Exam:
General: NAD, awake, alert, smiling, very friendly. Looks smaller than stated age. Has tendency to arch his back.
Skin: no jaundice, no rash, no cyanosis, no pallor.
Head: NCAT, low nasal bridge, upturning nose, elfin facies.
Eyes: EOMI, bilateral red reflex present, no icterus, bright blue color
Ears, Nose, Throat: clear TM b/l, throat clear, ears normal set but somewhat enlarged.
Neck: supple, lymph nodes not palpable.
Thorax: symmetric expansion, good air entry, LCAB
CVS: +S1S2, RRR, Grade 2/6 holosystolic SEM best heard at lower sternal border.
Abdomen: Soft, NTND, bowel sounds present, no palpable masses, no organomegaly.
Extr: FROMx4, 2+ pulses, cap refill <2sec,
Neuro: grossly intact CN 2-12, hypotonic extremities, does not hold her head up, no focal deficits.

Pertinent labs:
CBC: 11.9> 11.1/32.7<370
Na 143 K 3.9 Cl 112 Bicarb 25 BUN 15 Cr 0.3 Glu 92
Ca2+ >16mg/dL Phos 4.2 Mg 1.8 Intact PTH <3 TSH 4.55 Free T4 1.08
Total Protein 7.2 Albumin 4.8 Tbili 0.2 Dbili 0.0 SGPT/OT 15/29 Alk Phos 192
Albumin 4.8
Venous Blood Gas: 7.40/42/41/26
VitD 25-OH wnl Vit A - wnl
U/A – negative UCx – negative
Random urine Cr = 23.4 Random urine Ca = 17.0 Ca/Creatinine= 0.7 (nl 0.2)


Pertinent Studies:
- EKG: Normal rate and rhythm, no PR, QRS, or QT abnormalities, nonspecific T wave abnormalities present. Possible RVH or biventricular hypertrophy.
- CXR: Lungs clear, no masses. Visualized osseus structures intact.
- Abd XR – normal, no abnormal calcifications
- Hand & Wrist XR – normal osseous structures, no subperiosteal bone resorption, no evidence of osteitis fibrosa cystica. Normal.
- Renal U/S – bilateral medullary neprhocalcinosis.
- Echocardiogram: small PFO with L→R shunting, small mid-muscular VSD with restrictive L→R shunting, mild supravalvular aortic stenosis.



Posted by: Thandrus 6 Jul 2010, 02:53
Veshapo_O

ეს თემა ასეთი შეკითხვებისთვის არ არის; მაგრამ რადგანაც პრობლემა გაქვს და აქ შედარებით მეტი ექიმი იხედება, 1 ხვალ საღამომდე დავტოვებ და შემდეგ მოვუხერხებ რამეს.

აქიდან დავიწყოთ: რამდენი იყო სისხლი ბამბაზე? რაიმე სახის თავის ტრავმა ხომ არ გქონია? ყური ხომ არ გაქვს გაჭრილი? გტკივა? როგორ გესმის ყურში?
* * *
badu

QUOTE
low nasal bridge, upturning nose, elfin facies.


QUOTE
Extremely social and happy around strangers.


Williams Syndrome? user.gif

Posted by: badu 6 Jul 2010, 22:25
Thandrus

კიიიიი, ვა საღოლ, ბულს აიო რომ იტყვიან, ეგრევე მოარტყი. ისე ელფინ ფეისის წაშლა დამავიწყდა, მაგრამ მაინც, very impressive.
კიდევ 2 კითხვა მაქვს:
1) სახის და სოციალური განვითარების გარდა, რა ნიშნებს ხედავ ამ პაციენტში რაც ვილიამსის სინდრომისთვის დამახასიათებელია?
2) რატომ მოხვდა ეს ბავშვი საავადმყოფოში, ანუ რამ გამოიწვია this particular admission?

Posted by: Veshapo_O 7 Jul 2010, 00:06
სად დავსვა ეს კითხვა რომელიც გუშინ დავსვი აქ რო პასუხები მივიღო, მიმასწავლეთ არ დაიზაროთ

Posted by: Thandrus 7 Jul 2010, 01:17
Veshapo_O

გახსენი ახალი თემა და "სისხლდენა ყურიდან" დაარქვი. ყველაზე უკეთესი ეს იქნება. და საერთოდ, ერთხელ ექიმთან ვიზიტს გირჩევ, რადგანაც შორიდან ძნელია ასე, რამის თქმა.
* * *
badu

QUOTE
1) სახის და სოციალური განვითარების გარდა, რა ნიშნებს ხედავ ამ პაციენტში რაც ვილიამსის სინდრომისთვის დამახასიათებელია?


ნუ, ამათ გარდა, რაც თვალში მომხვდა, ესაა აქსიალური კუნთების ჰიპოტონია (რის გამოც ბავშვი ვერ ჯდება), წონაში ვერმატება და კიდევ, ზოგადი ჩამორჩენა, პლუს VSD-ც შევამჩნიე ახლა.

QUOTE
2) რატომ მოხვდა ეს ბავშვი საავადმყოფოში, ანუ რამ გამოიწვია this particular admission?


თუ ბავშვი მოიყვანეს "abnormal lab results"-ის გამო, ესეიგი, T4-ის ნაკლებობამ და ჰიპოკალცემიამ მოაყვანინათ (რომელიც ხშირად აქვთ ამ სინდრომიანებს)

Posted by: MAIN KAMPF 7 Jul 2010, 01:53
QUOTE
T4 1.08
,
QUOTE
Total Protein 7.2 Albumin 4.8

QUOTE
Venous Blood Gas: 7.40/42/41/26

QUOTE
mild supravalvular aortic stenosis

QUOTE
Renal U/S – bilateral medullary neprhocalcinosis.

ეს დაავადება არ მახსოვს yes.gif ანალიზები რა თქმა უნდა რაც ზემოთ დავწერე შეცვლილია. ვერ გავიგე კალციუმის რაოდენობა თუ დამიწერ კარგი იქნება ამ უკანასკნელს ვუკავშირებ bilateral medullary neprhocalcinosis. დაბალია T4აქედან შეიძლება აიხსნას
QUOTE
weakness, lethargy
საინტერესოა ]mild supravalvular aortic stenosis[თა
ნდაყოლილია დაავადება სავარაუდოდ.თუმცა აქ ყველაფერს ხსნის elfin facies. biggrin.gif როგორც გაირკვა. არის კიდევ სხვა ცვლილებები მაგრამ არა მნიშვნელოვანი. ეხლა ვნახავ ამ დაავადებაზე ინფორმაციას biggrin.gif
* * *
ყურიდან სისხლდენას რაც შეეხება. მრავალი მიზეზი შეიძლება იყოს , უფრო ხშირი მიზეზია ტრავმა:მექანიკური ან ბაროტრავმა უცხო სხეულმაც იცის, ნახე ექიმი საჭიროა ყურის დათვალიერება smile.gif

Posted by: Dr Pepper 7 Jul 2010, 02:58
badu
მე იმან დამაბნია რომ
ანდრენ ბიერსინგ უილიამსის სინდრომი ძირითადად მამრებშია :|

Posted by: badu 7 Jul 2010, 05:06
Thandrus
QUOTE
თუ ბავშვი მოიყვანეს "abnormal lab results"-ის გამო, ესეიგი, T4-ის ნაკლებობამ და ჰიპოკალცემიამ მოაყვანინათ (რომელიც ხშირად აქვთ ამ სინდრომიანებს)


no.gif
დისმორფული სახე და ჰიპოკალცემია უფრო დი ჯორჯის სინდრომს ახასიათებს. თაიროიდის ფუნქციას არა უშავს, TSH ნორმის ფარგლებშია.

MAIN KAMPF
QUOTE
ვერ გავიგე კალციუმის რაოდენობა თუ დამიწერ კარგი იქნება ამ უკანასკნელს ვუკავშირებ bilateral medullary neprhocalcinosis


კალციუმი 16მგ/დლ-ზე მაღალია. ნორმა სადღაც 8-10ია. სწორედ დააკავშირე ეს ორი ერთმანეთთან.

QUOTE
mild supravalvular aortic stenosis


supravalvular aortic stenosis is a very specific finding in williams syndrome. So whenever you see this finding in a child, you have to always think about williams syndrome. it also may be associated with sporadic deletion of elastin gene only. in fact, deletion in williams syndrome includes elastin gene. children with williams syndrome also have focal large and medium arterial narrowing elsewhere. (ინგლისურისთვის ბოდიშს ვიხდი, ესე უფრო მეადვილება)

Dr. Pepper

ვილიამს სინდრომი დაკავშირებულია მეშვიდე ქრომოსომაზე რამოდენიმე გენის წაშლასთან (მათ შორის, ელასტინის). ასე რომ არა მგონია მარტო მამაკაცებში არსებობდეს ეს დაავადება. წესით თანასწორი უნდა იყოს. ამ გოგოს ნამდვილად დაუდგინდა ვილიამსი. FISH ანალიზით გაკეთდა დიაგნოზი.


საბოლოოდ რას ფიქრობთ, რატომ მოხვდა ეს საწყალი ბავშვი PICU-ში?
EKG-მ რატომ დაგვაინტერესა?
და როგორ ვუმკურნალოთ?

Posted by: Thandrus 7 Jul 2010, 05:51
badu

QUOTE
ჰიპოკალცემია


ფუ, რათქმაუნდა პიპერკალცემიას ვგულისხმობდი... Lapsus manus smile.gif

QUOTE
საბოლოოდ რას ფიქრობთ, რატომ მოხვდა ეს საწყალი ბავშვი PICU-ში?


ვააჰ, არ არის საკმარისი მიზეზები? biggrin.gif

კიდევ ერთი ვარიანტი: ალბათ აორტის სტენოზის ოპერაციულ შესწორებას განიხილავდნენ. ECG-ც ალბათ იმიტომ გადაიღეს, რომ შეემოწმებინათ გულის ფუნქცია.

QUOTE
და როგორ ვუმკურნალოთ?


ეს არ ვიცი, სიმართლე გითხრა, ზუსტად. როგორც ვიცი, აორტული სტენოზის დროს აგრესიული ქირურგიული ჩარევაა საჭირო, მაგრამ ამ ბავშვში ეს რამდენადაა, არ ვიცი (moderate რომ წერია...). დიეტაც უნდა შედგეს ბავშვობიდან და ზედმეტი კალციუმის ან ვიტამინ D-ს შემცველ პროდუქტებს თავი უნდა ავარიდოთ.

Posted by: Dr Pepper 7 Jul 2010, 13:07
badu
მე ვიცი უილიამსის სინდრომი არის თანდაყოლილი ანომალიების ნაკრები. რომელიც მიმდინარეობს VII ქრომოსომაზე 26ის გენის ამოშლლით. ასევე ჩდება მუცლის წინა კედლის ქვედა ნაწილის კუნთების სრული ან ნაწილობრივი დეფექტი, ასევე გვხვდება ორმხრივი კრიპტორქიზმი, ჰიდრონეფროზი, თირკმელების დისპლაზია. შეიძლება იყოს შეკავშირებული კუჭ ნაწლავის, გულის და ჩონჩხის ანომალიებთან, და რომ გადაიცემა აოტოსომურ – დომინანტური გზით

Posted by: MAIN KAMPF 7 Jul 2010, 13:53
QUOTE
თაიროიდის ფუნქციას არა უშავს

F4 დაკლებულია , რა პარამეტრებით იზომება თქვენთან.
QUOTE
საბოლოოდ რას ფიქრობთ, რატომ მოხვდა ეს საწყალი ბავშვი PICU-ში?
რაც ჩამოთვალე საკმარისია
biggrin.gif წავიკითხე რომ ასევე დამახასიათებელი სახის გარდა ამ სინდრომის მქონე ადამიანებს ახასიათებთ ცისფერი თვალები მააშ biggrin.gif

Posted by: badu 7 Jul 2010, 16:15
MAIN KAMPF
QUOTE
წავიკითხე რომ ასევე დამახასიათებელი სახის გარდა ამ სინდრომის მქონე ადამიანებს ახასიათებთ ცისფერი თვალები მააშ


კიი მართალია. მაგის დაწერაც დამავიწყდა, ამ ბავშვს ძალიან ღია ცისფერი თვალები ქონდა. ეს ბავშვები გვანან ლეპრიქონებს. აქვს დიდი პირი, დიდი ტუჩები, მაღლა აპრეხილი ცხვირი, და ასე შ. ძაან საყვარლები არიან smile.gif

QUOTE
F4 დაკლებულია , რა პარამეტრებით იზომება თქვენთან.

ნორმაა 0.7-1.9 ნგ/დლ, ასე რომ მაგ შემთხვევაში ნორმალურია თაიროიდის ფუნქცია. აქ სხვათა შორის F4-ს დიდ ყურადღებას აღარ აქცევენ. მთავარი TSH-ია. ნორმალ 5-ზე დაბალია.

რაც შეეხება თუ რატომ მოხვდა ეს ბავშვი PIჩU-ში. მაგას მხოლოდ და მხოლოდ ერთი ახსნა აქვს. Occam's razor-ი ხომ გაგიგიათ. ანუ ერთი პათოლოგია უნდა ხსნიდეს ყველა სიმპტომს.

Thandrus
QUOTE
კიდევ ერთი ვარიანტი: ალბათ აორტის სტენოზის ოპერაციულ შესწორებას განიხილავდნენ. ECG-ც ალბათ იმიტომ გადაიღეს, რომ შეემოწმებინათ გულის ფუნქცია.

მიუხედავად იმისა რომ ამ ბავშვს მართლაც აქვს გულის დეფექტები, გულის მუშაობა ჯერჯერობით დამაკმაყოფილებელია. არ აეღნიშნება cyanosis, no signs of heart failure (no pulmonary edema, no hepatomegaly, no ankle edema). გულზე ოპერაცია მოიცდის.

დავუბრუნდეთ Occam's razor-ს. მთავარი რაც ხდება ამ პაციენტში არის ჰიპერკალცემია. კალციუმი ძალიან(!!!) მაღალია. ნორმა 10-ზე დაბალია. ამ ბავშვს ქონდა 16-ზე მაღალი. ესეთი რამე ძალიან იშვიათად ხდება და ეგრე ICU-შია გადაყვანა საჭირო. EKG-ც მაგიტომ იყო საჭირო. high calcium is associated with short QT on EKG. Its usually unsignificant, however severly high calcium (which was the case in this patient) can lead to suptraventricular and ventricular arrhythmias. So in this case, hypercalcemia is responsible for her decreased appetite, decreased weight gain, fatigue, weakness, generalized hypotonia. It caused medullary nephrocalcinosis in her kidneys. She was gaining weight fine up until 2 months ago, so we can not just think of williams syndrome as the cause. Something else must've happened that caused all these symptoms.
Hypercalcemia is common in the first year of life in Williams Syndrome patients. It is associated with decreased release of calcitonin (this hormone lowers calcium normally) and altered vitamin D receptors. Hypercalcemia usually goes away after the first year of life and does not really come back later on.

Clinical pearl - დისმორფული სახე და ჰიპოკელცემია - დი ჯორჯი... დისმორფული სახე და ჰიპერკალცემია - ვილიამსი. პედიატრიის გამოცდებზე იყო ეს შეკითხვა.


მკურნალობაზე რომ გკითხეთ, ვგულისმობდი ჰიპერკალცემიის მკურნალობას smile.gif

Posted by: Thandrus 7 Jul 2010, 17:48
badu

QUOTE
მკურნალობაზე რომ გკითხეთ, ვგულისმობდი ჰიპერკალცემიის მკურნალობას


ჰიპერკალცემიიანი ბავშვი (თან ასეთი დიდი ჰიპერკალცემიით) დეჰიდრირებული იქნება, ამიტომ, ფიზიოლოგიური ხსნარი უნდა გავუკეთოთ, რომ სისხლის მოცულობა აღდგეს, თან ამას პლუს loop დიურეტიკები,
* * *
ამ ქეისის მერე კიდევ დადე რაიმე საინტერესო, please smile.gif

Posted by: grigolich 13 Jul 2010, 21:39
ზელვეგერი როგორც მახსოვს პეროქსიზომების მუტაციის შედეგად ვიტარდება, ტუ ვცდები?

Posted by: mtvareuli 14 Jul 2010, 21:45
Thandrus
QUOTE
loop დიურეტიკები

მარყუჟოვანი დიურეტიკები


Posted by: Thandrus 15 Jul 2010, 03:44
grigolich

QUOTE
ზელვეგერი როგორც მახსოვს პეროქსიზომების მუტაციის შედეგად ვიტარდება, ტუ ვცდები?


სწორად გახსოვს, არაფუნქციური პეროქსისომებია. დეფექტი ძირითადად PEX გენების პროდუქტებშია (პეროქსინები), რომლებსაც რიბოსომებზე დასინთეზებული ფერმენტების პეროქსისომაში ტრანსპორტი ევალებათ. ამ სატრანპორტო ცილების მუტაციების შედეგად გამოწვეულ დაავადებებს პეროქსისომული ბიოგენეზის დაავადებები ეწოდებათ (peroxisomal biogenesis disorders).

ეს ჯგუფი 4 დაავადებას მოიცავს (ნელსონის პედიატრიას თუ დავუჯერებთ):

ზელვეგერის სინდრომი
ახალშობილთა ადრენოლეიკოდისტროფია
ახალშობილთა რეფსუმის დაავადება
რიზომელური ქონდროდისპლაზია პუნქტატა

პეროქსისომული დაავადებების მეორე კლასიც არსებობს, რომელშიც თვითონ პეროქსისომული რომელიმე ენზიმია მუტირებული (ოღონდ ამ დაავადებების სია აღარ მახსოვს biggrin.gif )
* * *
mtvareuli

QUOTE
მარყუჟოვანი დიურეტიკები


Pardon, mademoiselle gigi.gif

Posted by: mtvareuli 15 Jul 2010, 04:22
Thandrus

"თლა" ქართული ვარიანტი თუ გინდა - მარყუჟოვანი შარდმდენები biggrin.gif


mo.gif

Posted by: Thandrus 15 Jul 2010, 04:37
mtvareuli

შენ, ძველ ქართულად როგორ არის, ეგეც გეცოდინება biggrin.gif

Posted by: inesa1936 15 Jul 2010, 13:01
QUOTE
გამარჯობათ, არ ვიცი ვის ვკითხო ეს და ამიტო გადავწყვიტე ფორუმზე მეკითხა. მოკლედ რა ხდება : დგეს უკვე მეორე შემთხვევა იყო როდესაც ყურიდან მქონდა სისხლდენა  ვიგრძენი რო ყური დამეხშო და როცა ყური გამოვიწმინდე ბამბა იყო მთლად სისხლით სველი. თუკი ვინმემ რამე შეგიძლიათ მითხრათ არ დაიზაროთ. წინასწარ დიდი მადლობა

მე ოტო-რინო არ ვარ, მაგრამ შემიძლია დაგამშვიდოთ: ზუსტად ასე მქონდა მეც - სისიხლიანი ბამბა, თავის ყურის დაგუბებით და აღმომაჩნდა - ყურის წმენდის დროს დაფის აპკის მთლიანობა დამირღვევია ( სუფთა ტრავმულია) და დამინიშნეს ბევრი მედიკამენტი - სიმართლე გითხრა არ მომიკლავს თავი, ერთადერთი: ერიდე ყურში წყლის მოხვედრას! თავის დაბანის ან მსგავსი მანიპულაციისას ვაზელინიანი ან რაიმე კრემიანი ბამბა ჩაიდე ყურში! ყურის ტკივილისას არაფერი არ ჩაიწვეთო ყურში - გახეთქილ აპკზე, ეს მტკივნეულია! მოკლედ, აი 1 თვე გავიდა და მოხდა რეგენერაცია ( ალბათ, თორემ მე მეტი ექიმთან არ ვყოფილვარ!), ახლა ოდნავ ყლაპვის დროს რაღაც დამატებითი ხმა მესმის, ალბათ ესეც აღდგება! იდენტური არაფერი არსებობს, მაგრამ დაახლოებით ვფიქრობ იგივე გჭირს - ეს იმისთვის რომ არ ინერვიულო! და ისე, მაინც ჩაახედე ერთ კარგ ოტო0 რინოს ყურში! მალე გამოჯანმრთელებას გისურვებ!

Posted by: BadbadGirl 15 Jul 2010, 17:30
ის თემა, ჩაინიკურ შეკითხვებს რომ ვსვავდით ვერ ვიპოვე და რა ვქნა? არადა რამოდენიმე შეკითხვა მაქვს.

Posted by: MAIN KAMPF 15 Jul 2010, 18:02
რა არის ეს biggrin.gif [URL=http://radikal.ru/F/s59.radikal.ru/i165/1007/bb/fceb9369222f.jpg.html][IMG]http://s59.radikal.ru/i165/1007/bb/fceb9369222ft
* * *
http://www.radikal.ruპირველი ფოტო არ გამიხსნა თავიდან ვაკოპირებ

Posted by: utilizatori 15 Jul 2010, 18:19
MAIN KAMPF

ისეთი შთაბეჭდილება მრჩევა რომ რაღაცეები აწყვია გულმკერდზე smile.gif

Posted by: Blind_Torture_Kill 15 Jul 2010, 18:24
MAIN KAMPF

რამე მოკლე ინფო დაამატე

ვინაა
რატომ მოვიდა ექიმთან-რა აწუხებს

Posted by: MAIN KAMPF 15 Jul 2010, 20:32
ეს ავადმყოფი დღეს დილით დამხვდა განყოფილებაში. სუნთქვის უკმარისობით, მარცხენა მხარეს ყურადღება არ მიაქციოთ ჟანგბადის მიმწოდებელი მილია biggrin.gif მეტი რა დავამატო კრეატინინი არ გჭირდებათ lol.gif

Posted by: Thandrus 16 Jul 2010, 02:48
MAIN KAMPF

სიმსივნეა? მაგალითად ადენოკარცინომა.

Posted by: MAIN KAMPF 16 Jul 2010, 12:05
სარძევე ჯირკვლის იმპლანტია biggrin.gif chups.gif

Posted by: Thandrus 16 Jul 2010, 12:42
MAIN KAMPF

QUOTE
სარძევე ჯირკვლის იმპლანტია


ააჰ, გასაგებია, მარჯვენა მხარეს biggrin.gif მაგრამ მაჯვნივ რა არის, სტერნოკლავიკულარიდან რამდენიმე ნეკნით ქვემოთ? user.gif

Posted by: LUKA-BRAZI 16 Jul 2010, 12:46
Thandrus
QUOTE
მაგრამ მაჯვნივ რა არის, სტერნოკლავიკულარიდან რამდენიმე ნეკნით ქვემოთ?

მგონი ეკგ-ს ელექტროდის ტრანსმიტერი smile.gif მაგრამ დანარჩენი 3 სად გაქრა? მოიძრო? smile.gif)

Posted by: Thandrus 16 Jul 2010, 14:15
BadbadGirl

QUOTE
ის თემა, ჩაინიკურ შეკითხვებს რომ ვსვავდით ვერ ვიპოვე და რა ვქნა? არადა რამოდენიმე შეკითხვა მაქვს.


ისაა, ის biggrin.gif დასვი, ცუდო გოგო gigi.gif

Posted by: SweetGirl 16 Jul 2010, 16:04
მე რა პრობლემა მაქვს ე.ი


მტკივა მარცხენა ფეხის კუნთი. წვივის გარეთა თითქმის უკანა მხარეს.შარშან სექტემბერში ვიგრძენი ჩანთა ხელში რომ მეჭირა და მივიქნევდი მოვირტყი. ვიფიქრე დაჟეჟილი მაქვსთქო. კაი 2 კვირის მრეც რომ მომიხვდა და ისევ მეტკინა გავიფიქრე აქამდე ხომ არ მეტკიებოდა დაჯეჟილიო და დავაკვირდი. ვიზუალურად არაფერი მეტყობა მარა ხელს თუ იმ მტკივნეულ ტოჩკაში მოვარტყავ საკმაუოდ მტკივა. ხან დატვირთვაზეც მტკივა უკვე. ან წინ რომ გადავიწევი და გაიჭიმება ფეხი, ან ცავიმუხლები და ბარძაყით დავაწვები იმ კუნთს, ან ფეხმორთხმით თუ დავჯექი...მერე 1 პერიოდი გამიარა, საერთოდ გადამავიწყდა და ეხლა ისევ ამტკივდა. მგონი მეორე მხარესაც მეწყება

რა ჯანდაბაა ეხლა ეს?

Posted by: Thandrus 16 Jul 2010, 17:17
SweetGirl

ეს თემა ასეთი კითხვებისთვის არაა განკუთვნილი - აქ კლინიკური შემთხვევები იდება და შემდეგ განვიხილავთ.

თუმცა, იყოს ცოტა ხნით შენი პოსტი, ვინაიდან აქ შედარებით მეტი ექიმი შემოდის.

ჩემი აზრი თუ გაინტერესებს, შეიძლება ეს ნერვის ქრონიკული გაღიზიანების / მსუბუქი ანთების შედეგი იყოს. მაგალითად ფეხის უკანა ნაწილზე ძვლოვანი წანაზარდით (რაც ხშირია ხოლმე), ამიტომ გირჩევდი, თუ ეს ძალიან გაწუხებს, რენტგენი გადაგეღო.

კარგი იქნება კითხვას ამ თემაში თუ დასვამ, არის იქ მცოდნე ხალხი, დაგეხმარებიან:

http://forum.ge/?showtopic=33722586&st=525&#entry20707354

Posted by: MAIN KAMPF 16 Jul 2010, 21:16
QUOTE
ისაა, ის  დასვი, ცუდო გოგო 

lol.gif lol.gif lol.gif

Posted by: @@@@ 31 Jul 2010, 18:06
შეკითხვა მაქ ერთი smile.gif
რატომ "აღიზიანებს" ზოგიერთი საკვები ნაღვლის ბუშტა? და რაში გამოიხატება ეს სიტყვა "გაღიზიანება"? დავუშვათ, ნიგზინას ან შემწვარ ღორის ხორცს რომ შეჭამს ადამიანი ეს ხელს უშლის ნაღვლის ბუშტიდან სითხის გამოსვლას თუ როგორ ხდება?

Posted by: Thandrus 31 Jul 2010, 20:59
@@@@

QUOTE
რატომ "აღიზიანებს" ზოგიერთი საკვები ნაღვლის ბუშტა? და რაში გამოიხატება ეს სიტყვა "გაღიზიანება"? დავუშვათ, ნიგზინას ან შემწვარ ღორის ხორცს რომ შეჭამს ადამიანი ეს ხელს უშლის ნაღვლის ბუშტიდან სითხის გამოსვლას თუ როგორ ხდება?


რამდენიმე მექანიზმია. პირველი ისაა, რომ ცხიმიანი საკვების მოსანელებლად ნაღვლის ღვიძლიდან სანაღვლე გზების გავლით ნაწლავში ნაღველი უნდა ჩავიდეს, სადაც შემდეგ ის თავის საქმეს გააკეთებს. ბევრი ცხიმის მიღებისას ეს ყველაფერი უფრო აქტიურად უნდა მოხდეს, რაც ნიშნავს სანაღვლე გზების გავლით მეტი ნაღველის დინებას და ამ გზების ამომფენი შრის მექანიკურ გაღიზიანებას.

მეორე ფაქტორია ის, რომ ზოგადად ცხიმიანი საკვები სისხლში ქოლესტეროლის მომატებას უწყობს ხელს. გარდაქმნილი ქოლესტეროლი ერთი მხრივ, საჭიროა ნაღვლის მჟავების წარმოსაქმნელად, მაგრამ როდესაც სისხლში ქოლესტეროლი ზედმეტია, მისი ეს პროდუქტები ნაღვლის ბუშტში და სანაღვლე გზების კედლებზე კენჭების სახით ილექება - ესეც გაღიზიანების მეორე ახსნა smile.gif

QUOTE
და რაში გამოიხატება ეს სიტყვა "გაღიზიანება"?


მართალი გითხრა, ეს "ტერმინი" ცოტათი ბუნდივანია. ეს შეიძლება ნიშნავდეს ყველაფერს სტეატორეიდან (ცხიმოვანი განავალი - ვინაიდან ნაღველის სათანადოდ ვერგამოყოფის შემთხვევაში ცხიმის მონელება ცუდად ხდება) მწვავე კენჭოვანი ქოლეცისტიტისთვის დამახასიათებელ ტკივილამდე (როდესაც კენჭი გზებში იჭედება, მექანიკურად აღიზიანებს / აზიანებს კედელს და იწვევს ანთებით რეაქცის).

Posted by: Sophist 31 Jul 2010, 22:31
ოთხია დღეა ისეთი შეგრძნება მაქვს თითქოს ყელში რაღაც გაჩხერილი მაქვს... თუმცა ტკივილს არ ვგრძნობ.. კატა მყავს სახლში და ნეტავი მისი ბეწვის ამბავი ხო არ არის? ექიმობებო რას ფიქრობთ?

Posted by: BadbadGirl 1 Aug 2010, 20:19
აუ ხალხებო, გთხოვთ დამეხმარეთ რა.
MCP, PIP and DIP სახსრების ყველაზე მნიშვნელოვანი ექსტენსორი და ფლექსორი რომელი კუნთებია?

ასევე კიდევ ერთი კითხვაა რომელსაც ვერ ვპასუხობ - დაასახელეთ ერთი მნიშვნელოვანი კუნთი ბარძაყის აბ-ადდუქციისათვის, ფლექს-ექსტენციისათვის?

მადლობა წინასწარ.

ვიცი არასწორ ადგილას არის კითხვა, მაგრამ სადაც სწორი იქნებოდა ის ადგილი ვერ ვიპოვე.


Posted by: giorgi6088 2 Aug 2010, 00:05
მეგობარმა მითხრა რომ დაახლოებით თვე იქნება ქავილი აწუხებს ქვემოთ ოგონდ მარტო კვერცხებზე რომ ვუტხარი ანალიზები გაეკეთებინდა მითხრა რომ გაიკეთა სრული ანალიზები 2 თვის წინ და სუფთა იყო მას შემდეგ კი არანაირი კავშირი ქონია თქვენ რას მირჩევთ როგორ მოიქცეს?

Posted by: alex_b 3 Aug 2010, 22:26
ბ ჰეპატიტის თემა რატომღაც დუმს და იქნებ აქ მაინც მითხრათ პასუხი შემდეგ პოსტზე:

მოკლეთ ვცხოვრობ ესპანეთში, და გადავწყვითე სისხლის გაღება,
როცა სისხლი ამიღეს გამომკითხეს და რაკი არაფერი არ მჭირდა მოკლეთ ამომსისხლეს

ერთი თვის მერე წერილი მომივიდა, რომ ჭირდებოდათ ჩემი სისხლის ხელახალი გამოკვლევა, რაღაც ეჭვების გამო.
როცა მივედი ამიღეს სისხლი და ექიმს ვკითხე რისი ეჭვი ქონდათ, და მიპასუხა, რომ სავარაუდოდ ჰეპატიტი B გაქვსო, მაგრამ ჯერ ეს ეჭვია და მოკლეთ შეგატყობინებთო.
მერე მომივიდა წერილი, სადაც დადასტურებული იყო, რომ ნამდვილად მქონდა ჰეპატიტი B და იყო რეკომენდაცია ჩემს ოჯახის ექიმთან მიმეტანა ეს წერილი. ამ წერილში ასე ეწერა

HBaAg : REACTIVO
PCR VHB: POSITIVO

ექიმთან მივიტანე, დამინიშნა ხელახლა ანალიზები, რომლის მიხედვითაც იყო შემდეგი სურათი:

Hepatitis B, HBsAg (EIA) dil.suero POSITIVO
Hepatitis B, anti-HBc (EIA) dil.suero POSITIVO
Hepatitis B, anti HBv IgM (IEA) dil.suero Negativo
Hepatitis B, HBeAg (EIA) dil.suero Positivo
Hepatitis B, anti-HBe (EIA) dil.suero Positivo

ხოდა არ ვიცი რა ეუცნაურა ექიმს, მაგრამ მითხრა რომ ეს ცოტა უცნაური პასუხიაო, და მოდი თავიდან გაიკეთე ანალიზებიო. დაახლოებით სამი კვირის მერე ამიღეს სისხლი, რის მიხედვითაც იყო შემდეგი სურათი:

Hepatitis B, HBsAg (EIA) dil.suero POSITIVO
Hepatitis B, anti-HBc (EIA) dil.suero POSITIVO
Hepatitis B, anti HBv IgM (IEA) dil.suero Negativo
Hepatitis B, HBeAg (EIA) dil.suero Negativo
Hepatitis B, anti-HBe (EIA) dil.suero Positivo

მოკლეთ ამ ჩამონათვალიდან მეოთხე მაჩვენებელი პოზიტივიდან ნეგატივად გარდაიქმნა სამი კვირის განმავლობაში. თან ექიმი რაღაცას იქექებოდა წიგნში, რამაც ცოტა არ იყოს ნდობა შემისუსტა... მაგრამ მითხრა ექიმმა რომ კარგად ხარო, და ვირუსმა დაასრულა გამრავლება, და აღარ გაქვსო. იტოკში გამოკეთდიო.

იქნებ ვინმემ ვისაც ამ ლათინური აბცდ-ების გესმით მოკლეთ გამარკვიოთ, რა ხდება, მართლა მაგრა უნდა მიხაროდეს? თუ როგორაა საქმე.

წინასწარ დიდი მადლობა
Warning:
მგონი რომელიღაც თემაში გუპასუხათ Masked-მა, ნახეთ აბა

Posted by: >gorski*JDM*lover> 6 Aug 2010, 01:27
ნაოპერაციები ცხვირი(ვთქვათ1წლის შემდეგ)
უფრო გამძლეა ტრავმის მიმართ თუ ზავაცკოიbiggrin.gif

ეს შეკითხვა ქვედა ყბის მოტეხილობიდან გამომდინარე დავსვი>ძვლოვანი კორძის ეფექტი

Posted by: Cousteau 10 Aug 2010, 17:41
user posted image

user posted image

რა ჭირს დეიდას?

Posted by: Blind_Torture_Kill 10 Aug 2010, 21:59
Cousteau
დეიდა ცუდადაა ............................................................................

Posted by: LUKA-BRAZI 11 Aug 2010, 10:14
Cousteau
დერმატომიოზიტი ან სკლეროდერმია?

Blind_Torture_Kill
QUOTE
დეიდა ცუდადაა ............................................................................

Ultimate diagnosis biggrin.gif

Posted by: Cousteau 14 Aug 2010, 13:28
QUOTE (LUKA-BRAZI @ 11 Aug 2010, 10:14 )
Cousteau
დერმატომიოზიტი ან სკლეროდერმია?


you tell me : )

QUOTE
Blind_Torture_Kill
QUOTE
დეიდა ცუდადაა ............................................................................
Ultimate diagnosis biggrin.gif

gigi.gif
დეიდას პრობლემები აქვს gigi.gif

P.S. მეცადინეობა ხო არ გინდათ ვინმეს?

Posted by: LUKA-BRAZI 14 Aug 2010, 13:47
Cousteau
QUOTE
P.S. მეცადინეობა ხო არ გინდათ ვინმეს?

PHD-ის? smile.gif
* * *
პ.ს.
QUOTE
სკლეროდერმია

ალბათ baby.gif

Posted by: Cousteau 14 Aug 2010, 14:05
QUOTE
Cousteau
QUOTE
P.S. მეცადინეობა ხო არ გინდათ ვინმეს?
PHD-ის? smile.gif * * *

Phd-სთვის რაც გჭირდება იმის

QUOTE
პ.ს.
QUOTE
სკლეროდერმია
ალბათ baby.gif


ამ კონკრეტულ დეიდას სკლეროდერმა არ აწუხებს, თუ დააკვირდები რაც აწუხებს იასამისფერია და რაც მაგას აწუხებს იმისათვის ტიპიურია ასეთი იასამისფერი გამონაყარი სახეზე

Posted by: Thandrus 14 Aug 2010, 14:27
Cousteau

ასე, ერთი შეხედვით, დერმატომიოზიტი მგონია user.gif

Posted by: LUKA-BRAZI 14 Aug 2010, 14:37
ჰო, დერმატომიოზიტისთვის ყოფილა იასამნისფერი ლაქები და გამონაყარები დამახასიათებელი

Posted by: Cousteau 14 Aug 2010, 14:45
QUOTE (Thandrus @ 14 Aug 2010, 14:27 )
Cousteau

ასე, ერთი შეხედვით, დერმატომიოზიტი მგონია user.gif

yes.gif

Heliotrope rash
Gottron's Papules

Posted by: bohiko 15 Aug 2010, 22:31
ფეხით სიარულის,მეცადინეობის და ა.შ. დროს თავბრუსხვევა რას ნიშნავს ?


სისხლის ანალიზი ნორმაშია, კარდიოგრამა, გულის ultrasoundic.

გემოგლობინი ნორმაშია, არც ჩიყვი არ არის. მოკლედ უკვე აღარ ვიცი .....

Posted by: MAIN KAMPF 15 Aug 2010, 23:58
როდის იწყება , რამდენი ხანი გრძელდება, პირველად შეამჩნიე თუ არა , რაიმეს ხომ არ უკავშირებ , სხვას ოჯახში თუ აქვს ასეთი პრობლემა....მიზეზი მრავალი შეიძლება იყოს პირველ რიგში ნევროპათოლოგს გაესინჯე და მისგან მიიღე რჩევა

Posted by: bohiko 16 Aug 2010, 02:59
QUOTE (MAIN KAMPF @ 15 Aug 2010, 23:58 )
როდის იწყება , რამდენი ხანი გრძელდება, პირველად შეამჩნიე თუ არა , რაიმეს ხომ არ უკავშირებ , სხვას ოჯახში თუ აქვს ასეთი პრობლემა....მიზეზი მრავალი შეიძლება იყოს პირველ რიგში ნევროპათოლოგს გაესინჯე და მისგან მიიღე რჩევა

ნევროპათოლოგს გავესინჯე საქმეც მაგაშია და მითხრა არაფერი გჭირსო.


განსაკუთრებით თუ დღის განმავლობაში ბევრი ვიმოძრავე მაშინ მაქვს აი რასაც ქვია გაბრუება არ მიორდება თვალებში, უბრალოდ ვბრუვდები .აი რომ უნდა ჩაგეძინოს, ვითენთები, თითქოს გული მეპარება.


შიშები მაქვს მაგრამ ეს შიშებიც ზუსტად ამ გაბრუებამ გამოიწვია , სულ მეშინია ან ეხლა წამივა გული ან ეხლა თქო. არა, სხვას ოჯახში არავის არ აქვს ესეთი პრობლემა, თან არ ვარ საქართველოში და ეს უფრო მაშინებს sad.gif(((((


პ.ს. იწყება ნებისმიერ დროს და გრძელდება რამდენი ხანიც უნდა.

Posted by: პეტრუჩი 16 Aug 2010, 13:23
ეს კაი თემა გავაძრე აქ მეც შემოვალ ხოლმე smile.gif
Thandrus
მიხვდი ალბათ ხო ვინც ვარ smile.gif ვნახოთ ერთი როგორ ფორმაში ვარ 3 თვიანი შესვენების შემდეგ

Posted by: bohiko 17 Aug 2010, 03:12
QUOTE (bohiko @ 16 Aug 2010, 02:59 )
QUOTE (MAIN KAMPF @ 15 Aug 2010, 23:58 )
როდის იწყება , რამდენი ხანი გრძელდება, პირველად შეამჩნიე თუ არა , რაიმეს ხომ არ უკავშირებ , სხვას ოჯახში თუ აქვს ასეთი პრობლემა....მიზეზი მრავალი შეიძლება იყოს პირველ რიგში ნევროპათოლოგს გაესინჯე და მისგან მიიღე რჩევა

ნევროპათოლოგს გავესინჯე საქმეც მაგაშია და მითხრა არაფერი გჭირსო.


განსაკუთრებით თუ დღის განმავლობაში ბევრი ვიმოძრავე მაშინ მაქვს აი რასაც ქვია გაბრუება არ მიორდება თვალებში, უბრალოდ ვბრუვდები .აი რომ უნდა ჩაგეძინოს, ვითენთები, თითქოს გული მეპარება.


შიშები მაქვს მაგრამ ეს შიშებიც ზუსტად ამ გაბრუებამ გამოიწვია , სულ მეშინია ან ეხლა წამივა გული ან ეხლა თქო. არა, სხვას ოჯახში არავის არ აქვს ესეთი პრობლემა, თან არ ვარ საქართველოში და ეს უფრო მაშინებს sad.gif(((((


პ.ს. იწყება ნებისმიერ დროს და გრძელდება რამდენი ხანიც უნდა.

???????





________________________

Posted by: svani67 17 Aug 2010, 10:19
alex_b
ჰეპატტის თემაში მგონი გიპასუხეთ

ვირუსი გქონდა, მაგრამ ორგანიზმი მოერია, როგორც ხშირად ხდება ბ ჰეპატიტის დროს (ც ჰეპატიტისგან განსხავებით) და ახლა აღარ გაქვს.

ეგ პასუხი მაგას ნიშნავს

ბ ჰეპატიტი იშვიათად გადადის ქრონიკულში; 90 % ორგანიზმი ერევა ვირუსს. ც ჰეპატიტის დროს სამწუხაროდ ასეთი კარგი შედეგები ვერ არის

Posted by: bohiko 19 Aug 2010, 04:23
QUOTE (bohiko @ 16 Aug 2010, 02:59 )
QUOTE (MAIN KAMPF @ 15 Aug 2010, 23:58 )
როდის იწყება , რამდენი ხანი გრძელდება, პირველად შეამჩნიე თუ არა , რაიმეს ხომ არ უკავშირებ , სხვას ოჯახში თუ აქვს ასეთი პრობლემა....მიზეზი მრავალი შეიძლება იყოს პირველ რიგში ნევროპათოლოგს გაესინჯე და მისგან მიიღე რჩევა

ნევროპათოლოგს გავესინჯე საქმეც მაგაშია და მითხრა არაფერი გჭირსო.


განსაკუთრებით თუ დღის განმავლობაში ბევრი ვიმოძრავე მაშინ მაქვს აი რასაც ქვია გაბრუება არ მიორდება თვალებში, უბრალოდ ვბრუვდები .აი რომ უნდა ჩაგეძინოს, ვითენთები, თითქოს გული მეპარება.


შიშები მაქვს მაგრამ ეს შიშებიც ზუსტად ამ გაბრუებამ გამოიწვია , სულ მეშინია ან ეხლა წამივა გული ან ეხლა თქო. არა, სხვას ოჯახში არავის არ აქვს ესეთი პრობლემა, თან არ ვარ საქართველოში და ეს უფრო მაშინებს sad.gif(((((


პ.ს. იწყება ნებისმიერ დროს და გრძელდება რამდენი ხანიც უნდა.

??????




_____________________________

Posted by: BadbadGirl 19 Aug 2010, 18:46
ხალხო, Moment arm რა არის? როგორ ითვლება და რა მნიშვნელობა აქვს მას?

გადავაქოთე ჩემი ანატომიის წიგნები და ვერსად ვიპოვე.

წინასწარ მადლობა.

Posted by: Thandrus 19 Aug 2010, 19:31
BadbadGirl

QUOTE
ხალხო, Moment arm რა არის? როგორ ითვლება და რა მნიშვნელობა აქვს მას?


ანატომიაში რატომ უნდა იყოს user.gif ბერკეტის მომენტია მგონი, ქართულად - ფიზიკის წიგნებში ნახე, სადაც ზოგადად ძალის მომენტია განხილული.

აი, ვიკიზე არის სტატიები რუსულად და ინგლისურად:

http://ru.wikipedia.org/wiki/Крутящий_момент
http://en.wikipedia.org/wiki/Torque

მგონი წერია, რაც გაინტერესებს smile.gif

Posted by: BadbadGirl 19 Aug 2010, 20:00
Thandrus
ნუ მკითხავ რატომ უნდა იყოს!
რაღაცისთვის და ეს რაღაც არის სწორი პასუხი ტესტის ერთ ერთ კითხვაზე. იდეაში კი ალბათ მოძრაობის ძალის გასარკვევად?!

ანატომიას მესამედ ვაბარებ სმაილიკი.

მადლობა.
* * *
მგონი მეც ვიპოვე რაღაც

Joint Moment Arm

Although muscles produce linear forces, motions at joints are all rotary. The rotary torque is the product of the linear force and the moment arm or mechanical advantage of the muscle about the joint's center of rotation. Mechanically, this is the distance from the muscle's line of action to the joint's center of rotation.

Determination of joint moment arm requires an understanding of the anatomy and movement (kinematics) of the joint of interest. For example, some joints can be considered to rotate about a fixed point. A good example of such a joint is the elbow. At the elbow joint, where the humerus and ulna articulate, the resulting rotation occurs primarily about a fixed point, referred to as the center of rotation. In the case of the elbow joint, this center of rotation is relatively constant throughout the joint range of motion. However, in other joints (for example the knee) the center of rotation moves in space as the knee joint rotates because the articulating surfaces are not perfect circles. In the case of the knee, it is not appropriate to discuss a single center of rotation--rather we must speak of a center of rotation corresponding to a particular joint angle, or, using the terminology of joint kinematics, we must speak of the instant center of rotation (ICR), that is, the center of rotation at any "instant" in time or space. Download the PostScript

Having defined a joint ICR, the moment arm is defined as the perpendicular distance from line of force application to the axis of rotation. This is illustrated for a simulated elbow joint. In A, the elbow joint is almost fully extended. Let the angle, q, between the brachialis muscle and the ulna be relatively small, e.g., q=20°. Let the distance between the brachialis insertion site and the elbow instant center be 5 cm. In this case, the perpendicular distance between the line of force application and the elbow ICR is shown by the dotted line in A and is equivalent to 5 cm x sin(20°) = 1.7 cm. Thus because the joint is nearly fully extended, this presents an unfavorable mechanical advantage to the muscle--the moment arm is relatively small. Much of the force generated by the muscle will simply compress the joint, not rotate it. Contrast this situation with the conditions shown in B, where the joint has now been flexed so that q=50°. Now, the moment arm equals 5 cm x sin(50°) = 4.3 cm. We see that for a simple hinge joint (a joint with a fixed ICR), the maximum moment arm is attained at q=90°. If we plotted moment arm vs. joint angle for this simple hinge joint, we would obtain a simple sine function that has a maximum of 5 cm occurring at q=90°. Such a curve can be generated for any joint. In general, the experimental curves are not quite as simple as the one here.

Posted by: BadbadGirl 2 Sep 2010, 01:10
სად ჩასულა ეს თემა ტიტუ smile.gif
ხალხო დამეხმარეთ კონსულტაცით.
ახლა დავიწყეთ ინტერნალ მედიცინაზე გადარბენა ხოდა მეც უნდა გადავირბინო რომ ჩავაბარო.
არის თუ არა სწორი რომ მედიკამენტი ტრომბილი გამოყენება გულის ინფაქტისა და Stabil Angina pectoris დროს? რა პრეპარატი უნდა გამოვიყენოთ არასტაბულური Stabil Angina pectoris და Atrial fibrillation დროს?
რა სახის მედიკამენტები უნდა გამოვიყენოთ ტვინში სიხლჩაქცევის დროს - ნიტრო პრეპარატები, ბეტაბლოკარე?

მადლობა.


* * *
დამავიწყდა კიდე დამეწერა პრეპარატი ვარანი როდის გამოიყენება?

პ.ს. ვიკიპედიაზე და ნეტში მეც მოვძებნი მაგრამ იმდენი წერია კითხვის დრო არ ამქვს. ამიტომ გთხოვთ თქვენ რომ გამცეთ პასუხი.

Posted by: MAIN KAMPF 2 Sep 2010, 04:00
რამე ''პუტიოვი''დადეთ alk.gifსაინტერესო

Posted by: Blind_Torture_Kill 2 Sep 2010, 11:41
57 year-old man presented to the emergency room with a chief complaint of "I have a severe headache and fever". The patient had been seen one week prior to presentation by his local physician with the complaints of headache and low grade fever. Two days prior to presentation he was seen in the emergency room at an outside hospital complaining of worsening of his headache and fever. While at that hospital, he was seen by the infectious disease service and a lumbar puncture was performed showing 169.7 mg/dl of protein, 38 mg/dl of glucose, and 1,464 white cells/ mm3. The cerebrospinal spinal fluid differential was noted for 100 neutrophils but no bacteria.

Past Medical History:

1. Bursitis of Left shoulder for one year treated by hydrocortisone injections, once every three months by an outside physician prior to presentation.
2. Orthotopic Liver Transplant in 1992 for cirrhosis secondary to alcohol use.
3. Cadaveric Renal Transplant in 1994 for chronic renal failure secondary to glomerulonephritis.
4. Hypertension.

Physical Examination:

Physical examination showed a tired, middle aged, man in no apparent distress. His temperature was 38.5 degrees C with stable vital signs. His left shoulder was tender to palpation and movement.

X-ray of left shoulder

თუ მოითხოვთ ბიოფსიური მასალის მორფოლოგიურ სურათსაც დავდებ

ნექსთ სტეპ ინ მენეჯმენთ ოფ დის პეიშენთ

Posted by: MAIN KAMPF 2 Sep 2010, 12:52
ეს სურAთი დაგენანა ნორმალურად რომ გეჩვენებინა მთლიანად biggrin.gif მანდ ჩანს ძვლის სტრუქტურის რღვევაა.ტუბერკულოზური მენინგიტი ხომ არაა.
QUOTE
ნექსთ სტეპ ინ მენეჯმენთ ოფ დის პეიშენთ
ეს LP გრამ ლიტრებში მიჩვენე პლიზ.
QUOTE
His left shoulder was tender to palpation and movement.
რომ არ ყოფილიყო გამიკვირდებოდა
biggrin.gif

Posted by: Blind_Torture_Kill 2 Sep 2010, 13:45
MAIN KAMPF
QUOTE
ეს სურAთი დაგენანა ნორმალურად რომ გეჩვენებინა მთლიანადმანდ ჩანს ძვლის სტრუქტურის რღვევაა.ტუბერკულოზური მენინგიტი ხომ არაა.
ტუბით გამოწვეული მენინგიტი არაა

ესე პათოლოგიური ლოკუსი უფრო კარგად ჩანს ?

QUOTE
ეს LP გრამ ლიტრებში მიჩვენე პლიზ.

ნამორიგეველი ვარ და გადაბმულად ერთი საათიც არ მიძინია პირველ რიგში გამოვიძინებ smile.gif და ისიც მერე თუ გავაკეთე smile.gif

Posted by: MAIN KAMPF 2 Sep 2010, 15:20
მე კიდევ ნაბახუსევი ვარ alk.gif
QUOTE
ეს LP გრამ ლიტრებში მიჩვენე პლიზ.

ამას რომ გააკეთებ მერე დაზუსტებით ვეცდები პასუხი გითხრა

Posted by: irakli222 3 Sep 2010, 00:41
ხალხო ქართულად წერეთ რააა

MAIN KAMPF

ცილა 1,697 გრ/ლ
გლუკოზა 0,38 გრ/ლ

Posted by: MAIN KAMPF 3 Sep 2010, 01:16
თავის კომპიუტერულ გამოკვლევას გავაკეთებდი. ასე მგონია ძვლის რღვევაა, რაც შეიძლება გამოწვეული იყოს კარცინომატოზული მენინგიტით(მეტასტაზი?)

Posted by: kobru1975 4 Sep 2010, 13:42
кандидозиа ег албат схва ар вици ра унда икос


Posted by: Blind_Torture_Kill 4 Sep 2010, 14:24
აი ბიოფსიური მასალის მიკროსკოპული სურათი

http://img198.imageshack.us/i/33443353.gif/

Uploaded with http://imageshack.us
* * *
QUOTE
თავის კომპიუტერულ გამოკვლევას გავაკეთებდი. ასე მგონია ძვლის რღვევაა, რაც შეიძლება გამოწვეული იყოს კარცინომატოზული მენინგიტით(მეტასტაზი?)


თავის კატემ ისეთი არაფერი აჩვენა

Posted by: Cousteau 6 Sep 2010, 23:53
დავით დაწერე რა პასუხი

Posted by: Blind_Torture_Kill 7 Sep 2010, 19:17
იდოს პაწა ხანიც და მერე დავდებ პასუხს
მანამდე თუ რამე კითხვები იქნება გიპასუხებთ

Posted by: MAIN KAMPF 8 Sep 2010, 18:19
QUOTE
იდოს პაწა ხანიც და მერე დავდებ პასუხს

რაღა უნდა იყოს უკვე დამავიწყდა რაზე იყო საუბარი მეზარება თავიდან წაკითხვა biggrin.gif

Posted by: Blind_Torture_Kill 8 Sep 2010, 18:33
QUOTE
QUOTE
იდოს პაწა ხანიც და მერე დავდებ პასუხს


რაღა უნდა იყოს უკვე დამავიწყდა რაზე იყო საუბარი მეზარება თავიდან წაკითხვა 



კრიპტოკოკული ინფექციია

Posted by: MAIN KAMPF 8 Sep 2010, 19:12
QUOTE
კრიპტოკოკული ინფექციია

კრიპტოკოკული მენინგიტი შესაბამისად

Posted by: BadbadGirl 22 Sep 2010, 18:01
აუ ხალხებო, აი ამ კითხვაზე მიპასუხეეეთ ფლიიზზზზ - Recall the main neurotransmitter system for antipsychotic drugs and the two most important for antidepressants.

არის თუ არა პასუხი
1. დოფამინი
2. ნორადრენალინი და სეროტონინი

მადლობა წინასწარ.

კიდევ დღეს ბევრი ვისაუბრეთ ანორექსიაზე, და თუ იცით რატომ არ ხდება ამ დაავადებასთან ბრძოლა ნლპ და ჰიპნოზით? ეს კითხვა სახლში რომ მოვედი ლექციიდან მაშინ გამიჩნდა. user.gif

Posted by: BadbadGirl 29 Sep 2010, 15:51
თუ იცით ALს - პაციენტები გვყავს საქართველოში?
_______________________________

Posted by: baksona1980 22 Jan 2011, 14:39
ფლოქსანის და ცეფამედის კომბინაცია პნევმონიის საკმურნალოდ....როგორი კომბინაციაა smile.gif

Posted by: lgogokhia 22 Jan 2011, 22:52
BadbadGirl
QUOTE
აუ ხალხებო, აი ამ კითხვაზე მიპასუხეეეთ ფლიიზზზზ - Recall the main neurotransmitter system for antipsychotic drugs and the two most important for antidepressants.არის თუ არა პასუხი
1. დოფამინი
2. ნორადრენალინი და სეროტონინი
მადლობა წინასწარ.


გაიხსენე ძირითადი ანტიდეპრესანტები და ის თუ რა ქვია ამ პრეპარატების დიდ ჯგუფს? მაგალითად ესციტალოპრამი, ფლუოქსეტინი, სერტალინი და ა.შ. რა აზრი აქვს აქ რომ მსგავსი მარტივი შეკითხვის პასუხი მიიღო ისე რომ ვერ გაიგო რატომ smile.gif

Posted by: >gorski*JDM*lover> 23 Jan 2011, 02:45
BadbadGirl
QUOTE
დოფამინი

თუ სწორად მახსოვს ნერვული იმპულსების გადაცემაში მონაწილეობს...


Posted by: lgogokhia 23 Jan 2011, 08:38
BadbadGirl
QUOTE
Recall the main neurotransmitter system for antipsychotic drugs and the two most important for antidepressants.

ბოლომდე არ წამიკითხავს შეკითხვა.. რაც ვთქვი და პლუს გაიხსენე შიზოფრენიის თეორიები და მისი სამკურნალო პრეპარატების მოქმედების მექანიზმი, გაიხსენე რატომ არის მათი გვერდითი ეფექტი tardive dyskinesia wink.gif

Posted by: Thandrus 30 Jan 2011, 02:09
აუჰ, რამდენი ხანია აქ არ შემომივლია user.gif

მე კარგი, მაგრამ სხვები მაინც სად ხართ? biggrin.gif

აბა ამას თუ მიხვდებით, რა არის... smile.gif

Posted by: donvaso 30 Jan 2011, 11:07
Thandrus
თვალში რამე პარაზიტია?
თუ კათეტერი?

Posted by: No_LiMiT^ 30 Jan 2011, 11:29
ალბად კათეტერი ..

Posted by: idallgo 30 Jan 2011, 12:18

QUOTE
თუ კათეტერი?


QUOTE
ალბად კათეტერი ..


gigi.gif no.gif


Thandrus

QUOTE
აბა ამას თუ მიხვდებით, რა არის... 



მაგის გამოცნობის ბედნიერებას სხვას მივანიჭებ... smile.gif უბრალოდ ერთ ჰინთს დავამატებ: არის ნემატოდი (ქსოვილის), შედის დაზიანებული კანიდან, ვექტორი არის ბუზი, შეჭრის ადგილას კანქვეშ იწვევს ლოკალიზებულ კანქვეშა ედემას (ე.წ. Calabar Swellings).

...ხო მაგის სახელიც მომწონს . biggrin.gif

Posted by: No_LiMiT^ 30 Jan 2011, 13:32
მასშინ თვალია, ჭია იქნება.. რომელმაც სჰეილება შეშუპება გამოიცვიოს და ალბად ბევრი რაგაც კიდე.. თუ რამე არასწორად ვთქვი მაპატიეთ ჯერ კიდევ მე-3 კურსის სტუდენტი ვარ.. smile.gif

Posted by: Thandrus 31 Jan 2011, 01:42
donvaso

QUOTE
თვალში რამე პარაზიტია?


ჰო, პარაზიტია, პარაზიტი gigi.gif

idallgo

QUOTE
მაგის გამოცნობის ბედნიერებას სხვას მივანიჭებ...უბრალოდ ერთ ჰინთს დავამატებ: არის ნემატოდი (ქსოვილის), შედის დაზიანებული კანიდან, ვექტორი არის ბუზი, შეჭრის ადგილას კანქვეშ იწვევს ლოკალიზებულ კანქვეშა ედემას (ე.წ. Calabar Swellings).


რათქმაუნდა smile.gif

QUOTE
...ხო მაგის სახელიც მომწონს .


აჰა givi.gif

No_LiMiT^

QUOTE
თუ რამე არასწორად ვთქვი მაპატიეთ ჯერ კიდევ მე-3 კურსის სტუდენტი ვარ..


კარგი იქნება, თუ ამ თემაში იაქტიურებ - სტუდენტების აქტიურობით გაცილებით უფრო საინტერესო ხდება ეს თემა smile.gif

ჰოდა ახლა, ამ პატივცემული ჭიის სახელიც დაწერეთ ვინმემ და შემდეგ ქეისზეც გადავიდეთ biggrin.gif

Posted by: Blind_Torture_Kill 31 Jan 2011, 09:58
QUOTE
ჰოდა ახლა, ამ პატივცემული ჭიის სახელიც დაწერეთ ვინმემ და შემდეგ ქეისზეც გადავიდეთ


ფილარიაზი

Posted by: donvaso 16 Apr 2011, 13:28
რატ მიივიწყეთ ეს თემა? biggrin.gif

მოკლედ, ეს პაციენტი(ქალბატონი) იყო ძალიან საინტერესო იმ მხრივ, რომ სანამ კტ-ს გაიკეთებდა წინა დღეს იყო ექოსკოპიაზე და იქ უთხრეს რომ მოცულობითი წარმონაქმნი იყო მუცლის ღრუშI და ურჩიეს კტ-კვლევა. სანამ მივიდოდა ექიმთან(ექოსკოპიაზე) ამ ქალს არანაირი ტკივილი არ აწუხებდა და მხოლოდ ექოსკოპიური და მეორე დღეს კტ-კვლევის შემდეგ ატკივდა "ყრუდ" მუცლის ღრუ.

სურათზე ნახავთ ამ ჩემთვის საოცარი ფაქტის შესახებ:

http://www.radikal.ru
http://www.radikal.ru
http://www.radikal.ru

Posted by: Blind_Torture_Kill 16 Apr 2011, 21:23
donvaso
QUOTE
მოკლედ, ეს პაციენტი(ქალბატონი) იყო ძალიან საინტერესო იმ მხრივ, რომ სანამ კტ-ს გაიკეთებდა წინა დღეს იყო ექოსკოპიაზე და იქ უთხრეს რომ მოცულობითი წარმონაქმნი იყო მუცლის ღრუშI და ურჩიეს კტ-კვლევა. სანამ მივიდოდა ექიმთან(ექოსკოპიაზე) ამ ქალს არანაირი ტკივილი არ აწუხებდა და მხოლოდ ექოსკოპიური და მეორე დღეს კტ-კვლევის შემდეგ ატკივდა "ყრუდ" მუცლის ღრუ.


sacrum ?

Posted by: achikia 9 Apr 2012, 16:05
მოცულობითი წარმონაქმნია მცირე მენჯში საშვილოსნოს CR გავს?

Posted by: GROCK 10 Apr 2012, 11:41
ბავშვობაში ანალური ატრეზია ხომ არ ჰქონდა?

Posted by: mortal 1 May 2012, 12:14
საინტერესო თემაა, რატომ არ აქტიურობთ?

Posted by: orphan 22 Jun 2012, 21:21
ეს თემა აქ მთავრდება თუ სადმე არსებობს სხვა სათაურით?

Posted by: Solveig 22 Jun 2012, 21:30
orphan
მანამდე იყო 2 იგივე თემა. ეს მესამე და უკანასკნელი თავია...

Posted by: tako44 22 Jun 2012, 22:30
Thandrus
მე ამიხსენით ოღონდაც ადამიანურ ენაზე, ეს რა არის? baby.gif sleep.gif

Posted by: orphan 22 Jun 2012, 23:41
პირველ ორს უკვე გადავხედე. გმადლობ..

Posted by: Mali_Mali 24 Jul 2012, 16:05
იქნებ მითხრათ ზუსტად როგორ იქნება ქართულად occult blood in stool

user.gif

უკვე ვიცი mo.gif

Posted by: Mrs_Zum 26 Jul 2012, 19:23
donvaso
რა დიაგნოზი დაესვა ამ პაციენტს? საშვილოსნოს CR-ს არ გავს, თითქოს საკრუმის სიმსივნეა

Posted by: MAIN KAMPF 27 Jul 2012, 23:50
მგონია ნაწლავიდანაა წამოსული...

Posted by: Blind_Torture_Kill 31 Jul 2012, 07:26
სუნთქავს ეს თემა კიდე ?..................
* * *
35 წლის ქალს BMI 42 აწუხებს მუდმივი თავისტკივილები რის გამოც იღებს ტკივილგამაყუჩებლებს, ბოლო პერიოდში ასევე აქვს მხედველობის გაუარესება, CBC-ნორმა, biochemistry-ნორმის ფარგლებში, ოფთალმოსკოპიურად-მხედველობის ნერვის დისკოს შეშუპება,
გაუკეთდა თავის MRI კვლევა


* * *
http://imageshack.us/photo/my-images/217/75699099.jpg/

Uploaded with http://imageshack.us
* * *
რა პათოგნომური ნიშანია სურათზე ?

დიაგნოზი ?

Posted by: agnia 12 Aug 2012, 16:00
Thandrus
QUOTE
ჰოდა ახლა, ამ პატივცემული ჭიის სახელიც დაწერეთ ვინმემ და შემდეგ ქეისზეც გადავიდეთ


ლოა ლოა

Posted by: orphan 12 Aug 2012, 21:13
QUOTE
35 წლის ქალს BMI 42 აწუხებს მუდმივი თავისტკივილები რის გამოც იღებს ტკივილგამაყუჩებლებს, ბოლო პერიოდში ასევე აქვს მხედველობის გაუარესება, CBC-ნორმა, biochemistry-ნორმის ფარგლებში, ოფთალმოსკოპიურად-მხედველობის ნერვის დისკოს შეშუპება,
გაუკეთდა თავის MRI კვლევა


pseudotumor cerebri?
* * *
QUOTE
მოკლედ, ეს პაციენტი(ქალბატონი) იყო ძალიან საინტერესო იმ მხრივ, რომ სანამ კტ-ს გაიკეთებდა წინა დღეს იყო ექოსკოპიაზე და იქ უთხრეს რომ მოცულობითი წარმონაქმნი იყო მუცლის ღრუშI და ურჩიეს კტ-კვლევა. სანამ მივიდოდა ექიმთან(ექოსკოპიაზე) ამ ქალს არანაირი ტკივილი არ აწუხებდა და მხოლოდ ექოსკოპიური და მეორე დღეს კტ-კვლევის შემდეგ ატკივდა "ყრუდ" მუცლის ღრუ.

ABC? Giant cell tumor?
eek.gif
* * *
QUOTE
მოკლედ, ეს პაციენტი(ქალბატონი) იყო ძალიან საინტერესო იმ მხრივ, რომ სანამ კტ-ს გაიკეთებდა წინა დღეს იყო ექოსკოპიაზე და იქ უთხრეს რომ მოცულობითი წარმონაქმნი იყო მუცლის ღრუშI და ურჩიეს კტ-კვლევა. სანამ მივიდოდა ექიმთან(ექოსკოპიაზე) ამ ქალს არანაირი ტკივილი არ აწუხებდა და მხოლოდ ექოსკოპიური და მეორე დღეს კტ-კვლევის შემდეგ ატკივდა "ყრუდ" მუცლის ღრუ.

Giant cell tumor? ABC? spy.gif

Posted by: idallgo 14 Aug 2012, 20:26
QUOTE (Blind_Torture_Kill @ 31 Jul 2012, 07:26 )
სუნთქავს ეს თემა კიდე ?..................
* * *
35 წლის ქალს BMI 42 აწუხებს მუდმივი თავისტკივილები რის გამოც იღებს ტკივილგამაყუჩებლებს, ბოლო პერიოდში ასევე აქვს მხედველობის გაუარესება, CBC-ნორმა, biochemistry-ნორმის ფარგლებში, ოფთალმოსკოპიურად-მხედველობის ნერვის დისკოს შეშუპება,
გაუკეთდა თავის MRI კვლევა


* * *
http://imageshack.us/photo/my-images/217/75699099.jpg/

Uploaded with http://imageshack.us
* * *
რა პათოგნომური ნიშანია სურათზე ?

დიაგნოზი ?

Idiopathic intracranial hypertension biggrin.gif

Posted by: Blind_Torture_Kill 18 Aug 2012, 07:34
MRIზე რა ჩAნს ?
............................................

Posted by: biboia777 18 Aug 2012, 16:36
გამარჯობათ ყველას მოტონეირონზე ვინმეს რამე გსმენიათ?
დედაჩემს უთხრეს რომ ეგ იქნება ალბათო. რაც წავიითხე არ ინკურნებაო და საშინელი რაღაც ყოფილა cry.gif

Posted by: Blind_Torture_Kill 13 Mar 2013, 09:28
46 წლის მამაკაცი ატაქსიით, პერიოდული გულყრებით დაქვეითებული მეხსიერებით რომელიც განუვითარდა ბოლო ერთი წლის მანძილზე და საგრდძნობლად გაუარესდა დღემდე

Posted by: MAIN KAMPF 13 Mar 2013, 19:42
boli.gif I don't know,something wrong

Posted by: Blind_Torture_Kill 14 Mar 2013, 18:26
something is definitely wrong yes.gif

Posted by: Blind_Torture_Kill 16 Mar 2013, 05:28
აღნიშნული პათოლოგია რომელ ქრომოსომას უკავშირდება

A.21
B.17
C.22
D.13
E.3

Posted by: Mrs_Zum 16 Mar 2013, 15:17
Blind_Torture_Kill

QUOTE
B.17


ნეიროფიბრომატოზი I ტიპი
ლიშის კვანძები

Posted by: Blind_Torture_Kill 17 Mar 2013, 00:59
Mrs_Zum
ეგაა

აბა ზედაზე რას იტყვი

Posted by: Mrs_Zum 17 Mar 2013, 02:34
QUOTE (Blind_Torture_Kill @ 13 Mar 2013, 09:28 )
46 წლის მამაკაცი ატაქსიით, პერიოდული გულყრებით დაქვეითებული მეხსიერებით რომელიც განუვითარდა ბოლო ერთი წლის მანძილზე და საგრდძნობლად გაუარესდა დღემდე

პრიონული დაავადება?
ნევროლოგიაში სუსტი ვარ

Posted by: Blind_Torture_Kill 17 Mar 2013, 04:49
Mrs_Zum
yes.gif

ესე სწრაფად პროგრესული ნერვული პათოლოგიის დიფერენციალში პირველ ადგილზე ეგ იქნება სავარაუდოდ

შეგიძლია ინტენსიური სიგნალის კერის ლოკალიზაციაც დაწერო ?

Posted by: teo--teo 15 Mar 2014, 23:15
გამარჯობა, ძალიან გთხოვთ დახმარება და რჩევა მჭირდება... ჩემს ნათესავს აქვს ესეთი პრობლემა, ფეხზე კოჭთან აქვს წანაზარდი რომელიც აღმოჩნდა რომ არის ნერვზე სიმსივნური წარმონაქმნი, ბიოფსიის შედეგი არის შემდეგი : " შესაძლოა მალიგნიზებული შვანომის არსებობა" ძალიან დაბნეულები ვართ სასწრაფოდ ოპერაციაა საჭირო რამდენად იცით ამ დაავადების შესახებ? ან რამდენად საშიშია? ექიმები : კახა თოდუა, გია ციხისელი და ლევან ჭყონია, რომელს გაუწევდით რეკომნდაციას? დიდი მადლობა

Posted by: No_LiMiT^ 16 Mar 2014, 06:01
სურათი როგორ ავტვირთო ?

Posted by: nia4u 27 Mar 2014, 18:44
ნეიროვირუსი რა არის?
ანუ კონკრეტულად რა სჭირს ეკა კვალიაშვილს?

Posted by: Solveig 27 Mar 2014, 19:42
nia4u


ცნება "ნეიროვირუსი" არის ძალიან ზოგადი. იმიტომ, რომ ნერვული სისტემის დაზიანება სხვადასხვა ვირუსებს შეუძლიათ: ჰერპესს, პოლიოვირუსს, ჩუტყვავილას ვირუსს, რეტროვირუსებს............

Posted by: nia4u 27 Mar 2014, 21:47
Solveig
მადლობა smile.gif
მე დავგუგლე და მხოლოდ ვეტერინარიაში ცოფთან მიმართებაში ამომიგდო ეს ტერმინი
და კარგად ვერ გავიგე,
თან გუშინ თვითონაც და ექიმიც რაღაც დაბნეულ-ბუნდოვნად საუბრობდნენ...
ანუ ყველა ვირუსი,რომელიც ნერვულ სისტემას აზიანებს,მოიხსენიება როგორც ნეიროვირუსი?სწორად გავიგე?

Posted by: Solveig 27 Mar 2014, 22:24
nia4u
QUOTE
ყველა ვირუსი,რომელიც ნერვულ სისტემას აზიანებს,მოიხსენიება როგორც ნეიროვირუსი?



როგორც წესი, კონკრეტულ ვირუსზე ლაპარაკობენ და არა რაღაც "ნეიროვირუსზე". არ გამოიყენება ეგ ტერმინი. ამიტომაც ვერ იპოვე ვერაფერი ალბათ smile.gif

ისე, ამ თემას ნუ გავაოფებთ. კარგი თემა იყო ერთ დროს....სამწუხაროა, რომ ეს ხალხი აღარ პოსტავს. ალბათ, ჩააბარეს გამოცდა, ვისაც უნდოდა. smile.gif

Posted by: nia4u 28 Mar 2014, 10:24
Solveig
მადლობა პასუხისთვის smile.gif
გაოფება არ მინდოდა,მაგრამ სხვა შესაფერისი თემა ვერ ვნახე და ახალი თემა აღარ გავხსენი შეგნებულად smile.gif
თუ გინდა წაშალე smile.gif

Posted by: doxaburi 29 Jan 2016, 17:49
lgogokhia
QUOTE
ეხლა ვკითხულობდი ამ თემის პირველ გვერდებს და რაღაც გამახსენდა.. თქვენ იცით თუ არა რომ famous პიოფაგი მზადდება Brain-Heart Infusion broth-ზე??

თუ შეგიძლია მოკვდათათვის გასაგებ ენაზე დაწერე რას ნიშნავს ეს ყოველივე smile.gif

ანუ საშიში პრეპარატია?

ექიმმა დამინიშნა პიფაგი და რო დავგუგლე ერთერთი ეს თემა ამომიგდო და შენი პოსტი ვნახე

Posted by: AIX-500 9 Jun 2016, 11:59
ც პეპტიდი მაქვს მაღალი. 6.9 რას ნიშნავს ეგ?

Powered by Invision Power Board (http://www.invisionboard.com)
© Invision Power Services (http://www.invisionpower.com)